You are on page 1of 160

CONSTITUTIONAL LAW

Professor: Atty. Rene B. Gorospe


Source: Gorospe, Rene B. (2006). CONSTITUTIONAL LAW Notes and Readings on the Bill of Rights, Citizenship and Suffrage (Vol. 1 and 2).
Quezon City: Rex Printing Company, Inc.
Chapter 1
The Fundamental Powers and the Bill of Rights
The Bill of Rights is a charter of liberties for the individual and
a limitation upon the power of the state. The purpose is to
protect the people against arbitrary and discriminatory use of
political power.
Calalang v. Williams
70 Phil. 726 (1940)
Commonwealth Act No. 548 prohibits animal-drawn vehicles from
passing along certain Manila streets during certain hours
Maximo Calalang assails its constitutionality on the ground that it
is an unlawful interference with legitimate business or trade and
abridge the right to personal liberty and freedom of locomotion
Held: No. It was passed in the exercise of the paramount police
power of the state

Police Power
It has been defined as the state authority to enact legislation
that may interfere with personal liberty or property in order to
promote the general welfare
It includes: (1) an imposition or restraint upon liberty or
property, (2) in order to foster the common good
Being what it is, police power cannot stand still. It also has to
adjust to the demands and realities of changing times
It may be delegated to the (1) President and (2) administrative
boards as well as (3) the law-making body of municipal
corporations or local government units. Once delegated, the
agents can exercise only such legislative powers as are
conferred on them by the national lawmaking body
Requisites -- lawful ends through lawful means

Persons and property may be subjected to all kinds of restraints


and burdens, in order to secure the general comfort, health, and
prosperity of the state

United States v. Toribio


15 Phil. 85 (1910)

Liberty should not be made to prevail over authority because then


society will fall into anarchy. Neither should authority be made to
prevail over liberty because then the individual will fall into
slavery

Act No. 1147 regulates the registration, branding and slaughter of


large cattle. The provisions of the said law requires, before large
cattle may be slaughtered or killed for food at the municipal
slaughterhouse, a permit be obtained from the Municipal
Treasurer

The Fundamental Powers


They are inborn in the very fact of statehood and sovereignty.
They are necessary and indispensable as there can be no
effective government without them
They are all exercised primarily by the national legislature
Police Power
Maintenance of a
healthy economic
standard of society
Regulates both
liberty and
property
May be exercised
only by the
government

Property taken is
destroyed
Compensation not
immediate;
sometimes leaving
the reward to be
reaped through his
recognition that he
has done
something for the
public good

Eminent Domain
Taxation
Just
Form of
compensation for
protection and
the property
benefits from the
taken
government
Affects only
property rights
May be
May be exercised
delegated to
only by the
some other
government
entities in the
private sector
Property taken is meant for public use
or purpose
Receipt of market
Immediate and
value of his
apparent in the
property that is
form of
taken
protection and
benefits derived
from the use of
taxes paid

Appellant was convicted of slaughtering an animal without the


requisite permit
Appellant contends the constitutionality on the ground that it
violates the provision the no law shall be enacted which shall
deprive any person of life, liberty, or property without due
process of law
Held: No. The act primarily seeks to protect the large cattle of
the Philippine Islands against theft and to make easy the recovery
and return of such cattle to their proper owners, when lost,
strayed, or stolen
All property is acquired and held under the tacit condition that it
shall not be so used as to injure the equal rights of others or
greatly impair the public rights and interests of the community
Rights of property are subject to such reasonable limitations in
their enjoyment as shall prevent them from being injurious xxx
Confronted by such conditions, there can be no doubt of the right
of the Legislature to adopt reasonable measures for the
preservation of work animals, even to the extent of prohibiting
and penalizing what would, under ordinary conditions, be a
perfectly legitimate and proper exercise of rights of ownership
and control of the private property of the citizen

Police power v. Due process


Due process is the embodiment of the sporting idea of fair
play. The minimum requirements of due process are notice
and hearing which, generally speaking, may not be dispersed
with because they are intended as a safeguard against official
arbitrariness

Previous judicial hearing, however, may be omitted without


violation of due process in view of the nature of the property
involved or the urgency of the need to protect the general
welfare from a clear and present danger

The drift is towards social welfare legislation geared towards


state policies to provide adequate social services, the
promotion of the general welfare, social justice as well as
human dignity and respect for human rights

The protection of the general welfare is the particular function


of the police power which both restrains and is restrained by
due process

Police power is the power to prescribe regulations to promote


health, morals, peace, education, good order or safety and
general welfare of the people

The justification is found in the venerable Latin maxims, Salus


populi est suprema lex and Sic utere tuo ut alienum non laedas,
which call for the subordination of individual interests to the
benefit of the greater number

Police Power v. General Welfare Clause


Police power may be delegated to and exercised by local
government units through the so-called General Welfare Clause

Ynot v. IAC
148 SCRA 569 (1987)

Villacorta v. Bernardo
143 SCRA 480 (1986)

EO 626-A prohibited the interprovincial transportation of carabao


(the poor mans tractor) and carabeef and subjected carabao and
carabeef transported in violation of its provisions to confiscation
and forfeiture, to be distributed to charitable institutions xxx
Whereas, the present conditions demand that the carabaos and
buffaloes be conserved for the benefit of small farmers who rely
on them for energy needs

The Municipal Board of Dagupan City adopted Ordinance No. 22


seeking to regulate the subdivision plans over parcels of land
located therein, which requires, among others, an approval from
the City Engineer and payment of a service fee and a certification
from the City Engineer

Six carabaos transported by Ynot in a pump boat from Masbate to


Iloilo were confiscated by a police station commander of Barotac
Nuevo, Iloilo

Held: Yes. Ordinance No. 22 is null and void being in conflict with
Section 44 of Act 496

Held: Yes. We find that the challenged measure is an invalid


exercise of the police power because the method employed to
conserve the carabaos is not reasonably necessary to the purpose
of the law and, worse, is unduly oppressive. Due process is
violated because the owner of the property confiscated is denied
the right to be heard in his defense and is immediately
condemned and punished

Police power is inherent in the state but not in municipal


corporations. A valid delegation of police power may arise from
express delegation, or be inferred from the mere fact of the
creation of the municipal corporation

An action was brought against its constitutionality

So many excesses are attempted in the name of the police power


xxx

Police Power v. Vices


The power to tax (the power to destroy) cannot be allowed to
defeat an instrumentality or creation of the very entity which
has the inherent power to wield it
What is settled is that the matter of regulating, taxing or
otherwise dealing with gambling is a State concern and hence,
it is the sole prerogative of the State to retain it or delegate it
to local governments

Binay v. Domingo
201 SCRA 508 (1991)
The Municipality of Makati approved Resolution No. 60 ratifying
the ongoing Burial Assistance Program, extending financial
assistance coming from the municipal treasury to bereaved
families with gross family income of less than P2,000.00
The COA disapproved Resolution 60. It held that the resolution
cannot be sustained as a legitimate exercise of the police power
due to a lack of perceptible connection or relation between the
objective sought to be attained and the alleged public safety,
general welfare, etc. of the inhabitants of Makati, and, that the
disbursement of funds was not for a public purpose since it was
for the benefit of only a few individuals and not the whole or
majority of the inhabitants of the Municipality
Held: Yes. The care for the poor is generally recognized as a public
duty. The support for the poor has long been an accepted exercise
of police power in the promotion of the common good.
Resolution No. 60 is a paragon of the continuing program of our
government towards social justice
COA is not attuned to the changing of the times. Public purpose is
not unconstitutional merely because it incidentally benefits a
limited number of persons

Basco v. PAGCOR
197 SCRA 52 (1991)
The PAGCOR was created by virtue of PD 1067-A (and PD 1869)
and was granted a franchise to establish, operate and maintain
gambling casinos on land or water within the territorial
jurisdiction of the Philippines
Petitioners alleged that such law is null and void for being
contrary to morals, public policy and public order; it further
contends that its exemption from paying any tax is violative of the
principle of local autonomy (waiver of right of City of Manila to
impose tax)
Held: No. Gambling in all its forms, unless allowed by law, is
generally prohibited. But the prohibition of gambling does not
mean that the Government cannot regulate it in the exercise of its
police power
PAGCOR has a dual role, to operate and to regulate gambling
casinos. The latter role is governmental, which places it in the
category of an agency or instrumentality of the Government, thus,
exempt form local taxes

Before: Tax Credit (Taxation & Eminent Domain exercised)


After: Tax Deduction (Taxation & Police Power exercised)
Eminent Domain
2|P

LATON

It is an inherent power of the State that enables it to forcibly


acquire private lands intended for public use upon payment of
just compensation to the owner
In times of national emergency, the government temporarily
takes over a public utility imbued with public interest pursuant
to Article XII, Section 17 of the Constitution, it exercises police
power and not its power of eminent domain. Accordingly the
private entity-owner cannot claim any just compensation for
the use of the said business and its properties
The police power being the most active power of the
government and the due process being the broadest limitation
on governmental power, the conflict between this power of
government and the due process of the Constitution is
oftentimes inevitable
City Government of QC v. Ericta
122 SCRA 759 (1983)
The Quezon City Council passed Ordinance No. 6118, S-64
regulating the establishment, maintenance and operation of
private memorial type cemetery or burial ground. Section 9 of the
said ordinance required that at least 6% of the total area of every
memorial park cemetery must be set aside for charity burial,
otherwise, such cemetery will be prohibited from selling memorial
park lots
Himlayang Pilipino contends that Section 9 is not a valid exercise
of police power
Held: Yes. The power to regulate does not include the power to
prohibit. A fortiori, the power to regulate does not include the
power to confiscate
Section 9 is not a mere police regulation but an outright
confiscation. It deprives a person of his private property without
due process of law, nay, even without compensation

Expropriation requires payment of just compensation


This is a power that may be exercised by entities other than the
government itself or its subdivisions and instrumentalities. The
private corporations serving the public, such as public utilities,
may validly be delegated the power.
xxx those engaged in the supply of electricity, water,
telecommunications services and some transportation firms
whose services might require the acquisition of private
property for the efficacious service to the public, may also be
vested with the power of expropriation
Before a municipal corporation may exercise its power of
eminent domain, it must be sanctioned and must not violate
any law
Private lands, for purposes of socialized housing, rank last in the
order of priority for acquisition, and expropriation proceedings
are to be resorted to only after the other modes of acquisition
have been exhausted
Eminent Domain v. Destruction by Necessity
EMINENT DOMAIN
Connotes taking
for use

DESTRUCTION BY NECESSITY
Speaks for itself--condemnation of a
property as a means of self-defense

Entails payment of
just compensation

Primarily exercised
by the government

or self-preservation
The property is precisely destroyed
as a way of promoting the greater
welfare of the populace who might
be endangered or otherwise placed
in harms way
May be exercised by private
individuals

Taxation
The power of the State to impose a charge or burden upon
person, property, or property rights, for the use and support of
the government
Taxation is a destructive power which interferes with the
personal and property rights of the people and takes from them
a portion of their property for the support of the government.
Tax statutes must be construed strictly against the government
and liberally in favor of the taxpayer
Limitations on the Power to tax
The rule of taxation shall be uniform and equitable. The
Congress shall evolve a progressive system of tax
Uniformity means that persons or things of the same class shall
be taxed at the same rate. It requires that all subjects or objects
of taxation, similarly situated, are to be treated alike or put on
equal footing both in privileges and liabilities
Uniformity, however, is not equality, the latter term signifying
that the taxes shall be strictly proportional to the relative value
of the taxable property
It is also an inherent limitation on the power to tax that the
proceeds be for public purpose. They could not be used for
purely private purposes xxx the real purpose of taxation is the
promotion of the common good
Taxation is said to be equitable when its burden falls on those
better able to pay. Taxation is progressive when its rate goes up
depending on the resources of the person affected
It is the strongest of all powers of government
The taxing power has the authority to make a reasonable and
natural classification for purposes of taxation but the
governments act must not be prompted by a spirit of hostility,
or at the very least discrimination that finds no support in
reason
Taxes are the lifeblood of the government and so should be
collected without unnecessary hindrance. However, such
collection should be made in accordance with law xxx
Reyes v. Almanzor
196 SCRA 322 (1991)
Petitioners are owners of parcels of land in Manila which are
leased and occupied as dwelling sites by tenants. RA 6359 was
enacted prohibiting from increasing in monthly rentals of dwelling
units and also disallowing the ejectment of lessees upon the
expiration of the usual legal period (amended by PD 20).
Thereafter, City Assessor of Manila increased tax rates.
Petitioners averred that the reassessments made were excessive,
3|P

LATON

unwarranted, inequitable, confiscatory and unconstitutional


Held: Yes. Public respondents would have this Court completely
ignore the effects of the restrictions of the said law on the market
value of properties within its coverage

Tax exemptions
xxx as broad as the power to tax
Like any other power, it is one that may not be exercised
arbitrarily or whimsically
The Constitution declares outright that: Charitable institutions,
churches and parsonages or covenant appurtenant thereto,
mosques, non-profit cemeteries, and all lands, buildings, and
improvements, actually, directly, and exclusively used for
religious, charitable, or educational purposes shall be exempt
from taxation
No law granting any tax exemption shall be passed without the
concurrence of a majority of all the Members of the Congress
Partnership among Fundamental Powers
Ermita-Malate Hotel and Motel Operators Association, Inc.
v. City Mayor of Manila
20 SCRA 849 (1967)
The Municipal Board of the City of Manila enacted Ordinance No.
4670 regulating the operation of hotels and motels
Petitioners sought to invalidate the ordinance
Held: No. The presumption is all in favor of validity xxx The local
legislative body, by enacting the ordinance, has in effect given
notice that the regulations are essential to the well being of the
people

Negatively put, police power is that inherent and plenary


power in the State which enables it to prohibit all that is hurtful
to the comfort, safety, and welfare of society
There is no controlling and precise definition of due process. It
furnishes though a standard to which governmental action
should conform in order that deprivation of life, liberty or
property, in each appropriate case, be valid

rights accruing to the owner in favor of the farmer-beneficiary, an


exercise of the power of eminent domain

The power being exercised is eminent domain if the property


involved is wholesome and intended for public use. Property
condemned under the police power is noxious or intended for a
noxious purpose which should be destroyed in the interest of
public safety, morals, etc. The confiscation of such property is
not compensable, unlike the taking of property under the
power of expropriation, which requires the payment of just
compensation to the owner
The Bill of Rights
As a Check on Governmental Powers Only
It governs the relationship between the individual and the State
and its agent. The Bill of Rights only tempers governmental
power and protects the individual against any aggression and
unwarranted interference by any department of the
government and its agencies
In the absence of governmental interference, the liberties
guaranteed by the Constitution cannot be invoked against the
State. It concerns not the relation between individuals,
between a private individual and other individuals. What the
Bill of Rights does is to declare some forbidden zones in the
private sphere inaccessible to any power holder
People v. Marti
193 SCRA 57 (1991)
The appellant sought to have (4) gift -wrapped packages sent to a
friend in Zurich, Switzerland through services of a forwarding
company, the Manila Packing and Export Forwarders. Upon
inspection of the box, dried leaves of marijuana were found in the
packages
He questions the admissibility of the marijuana, contending that it
was a product of an illegal search and seizure
Held: No. The evidence sought to be excluded was primarily
discovered and obtained by a private person, acting in his private
capacity and without the intervention and participation of State
authorities

As a Yardstick of Validity and the Standards of Review


Taxation may be made to implement the states police power

The liberty of the citizen may be restrained in the interest of


the public health, or of the public order and safety, or
otherwise within the proper scope of the police power
Association of Small Landowners of the Philippines
v. Secretary of Agrarian Reform
175 SCRA 343 (1989)
PD No. 27 was enacted to provide for the compulsory acquisition
of private lands for distribution among tenant-farmers (CARP)
The constitutionality of such law is challenged raising, among
others, issues of due process and just compensation
Held: No. The taking contemplated is not a mere limitation of the
use of the land. What is required is the surrender of the title to
and the physical possession of the said excess and all beneficial

As a counterweight to the great powers of the government, the


Bill of Rights would pose a constant standard of measurement
to determine the validity of any governmental act which may
limit rights and liberties, or intrude into privacies of persons, or
otherwise impair their freedoms
Determining whether there is sufficient justification for the
governments action depends very much on the level of
scrutiny or the standards of review used
Standards of review--the mere rationality deferential review
standard, the middle-level review or intermediate review or
heightened scrutiny standard, and the strict scrutiny standard
Strict scrutiny--used today to test the validity of laws dealing
with the regulation of speech, gender, or race and facial
challenges are allowed for this purpose; for determining the
quality and the amount of governmental interest brought to
justify the regulation of fundamental freedoms; focus is on the
4|P

LATON

presence of compelling, rather than substantial governmental


interest and on the absence of less restrictive means for
achieving that interest
Deferential review--laws are upheld if they rationally further a
legitimate governmental interest, without the courts seriously
inquiring into the substantiality of such interest and examining
the alternative means by which the objectives could be
achieved; the substantiality of the governmental interests is
seriously looked into and the availability of less restrictive
alternatives are considered
Rational Basis Test has been described as adopting a
deferential attitude towards legislative classifications; it
remains a primary standard for evaluating the constitutionality
of a statute
Strict scrutiny is applied when the challenged statute either (1)
classifies on the basis of inherent suspect characteristic or (2)
infringes fundamental constitutional rights, i.e. the right to
procreation, the right to marry, free speech, etc
Intermediate scrutiny or heightened scrutiny standard is
applied when the challenged statutes classification is based on
either (1) gender or (2) legitimacy; Intensified mean Test, in
which, the court should accept the legislative end, but should
closely scrutinize its relationship to the classification made
What is important to bear in mind is this: To the extent that a
particular liberty interest is considered more important and
more valuable to society, to that extent must the courts utilize
a more demanding and exacting standard with which to
measure that governmental intrusion protected spheres
The Bill of Rights and the 1986 Interregnum
Can the rights and freedoms guaranteed by the Bill of Rights
exist when there is no constitution?
Republic v. Sandiganbayan
407 SCRA 10 (2003)
Major Gen. Josephus Q. Ramas, who was the Commanding
General of the Philippine Army, was investigated by the PCGG
through its AFP Anti-Graft Board for alleged ill-gotten and
unexplained wealth, after which the PCGG filed a complaint for
forfeiture under RA 1379
Elizabeth Dimaano, alleged to be Ramas mistress, was also later
impleaded. The raiding team seized the items detailed in the
seizure receipt together with other items not included in the
search warrant xxx
The Sandiganbayan declared the properties confiscated xxx as
illegally seized and therefore inadmissible xxx
[The Republic] asserts that the revolutionary government
effectively withheld the operation of the 1973 Constitution which
guaranteed private respondents exclusionary right
WON the revolutionary government was bound by the Bill of
Rights of the 1973 Constitution during the interregnum xxx; WON
the protection accorded to the individuals under the xxx
(Covenant) and the xxx (Declaration) remained in effect
during the interregnum
Held: No. The resulting government was indisputably a
revolutionary government bound by no constitution or legal

limitations except treaty obligations that the revolutionary


government, as the de jure government in the Philippines,
assumed under international law
We hold that the Bill of Rights under the 1973 Constitution was
not operative during the interregnum. However, we rule that the
protection accorded to individuals under the Covenant and the
Declaration remained in effect during the interregnum
xxx During the interregnum, a person could not invoke any
exclusionary right under a Bill of Rights because there was neither
a constitution nor a Bill of Rights during the interregnum
To hold that the Bill of Rights xxx remained operative during the
interregnum would render void all sequestration orders issued by
the PCGG before the adoption of the Freedom Constitution
Nevertheless, even during the interregnum the Filipino people
continued to enjoy, under the Covenant and the Declaration,
almost the same rights found in the Bill of Rights xxx
Clearly the raiding team exceeded its authority when it seized
[such] items

Political, Civil, Economic and Other Rights


The Bill of Rights is basically about political and civil rights as
contradistinguished from economic rights which are dealt with
in the constitutional provisions on the national economy and
patrimony, as well as in the provisions on social justice and
human rights
Human rights can be understood to include those that relate to
an individuals social, economic, cultural, political and civil
relations; the universally accepted traits and attributes of an
individual, along with what is generally considered to be his
inherent and inalienable rights, encompassing almost all
aspects of life
What the law guarantees as human right in one country should
be also guaranteed by law in all other countries
Right which inheres in persons from the fact of their
humanity
Civil rights refers to those rights that belong to every citizen
of the state or country including the rights of property,
marriage, equal protection of the law, freedom to contract, etc;
those rights appertaining to a person by virtue of his citizenship
in a state or community or the right of his being a member of
society
Political rights are said to refer to the right to participate,
directly or indirectly, in the establishment or administration of
government, the right to suffrage, the right to hold public
office, etc
Natural rights are those rights that appertain to man in right
of his existence, i.e. the rights to freedom of thought, to
freedom of religious belief, etc
American Bill of Rights -- The First Ten Amendments
The Philippine Bill of Rights has been basically patterned after
the American Bill of Rights which is contained in the first ten
amendments to the United States Constitution

5|P

LATON

Accordingly, in view of the Philippine reliance and reference


every now and then to American case law on certain
constitutional issues relative to the Bill of Rights, parallel
citations to the pertinent provisions of the US Bill of Rights have
to be made occasionally.
International Bill of Rights
While the Bill of Rights as found in the Constitution is the
primary basis for the determination if there is any violation of
the rights of persons, it does not necessarily mean that such is
the sole source of rights that may be recognized
The Court also considers the pertinent international
conventions and declarations in trying to determine if there is a
violation of a persons rights
The most prominent of these is the Universal Declaration of
Human Rights
The individual may still avail of the guarantees provided by the
international instruments and covenants--from the so-called
International Bill of Human Rights to some other declarations
and conventions--to buttress whatever claims to freedom and
liberty that he or she may have
The Bill of Rights, Vigilance and Government as Teacher
A close and literal construction deprives them of half their
efficacy, and leads to gradual depreciation of the right, as if it
consisted more in sound than in substance. It is the duty of the
courts to be watchful for the constitutional rights of the citizen,
and against any stealthy encroachments thereon
The lead should come from the Government itself if ever it
wants the citizens to follow and abide by its commands and
demands
Decency, security, and liberty alike demand that government
officials shall be subjected to the same rules of conduct that are
commands to the citizens. In a government of laws, existence
of the government will be imperiled if it fails to observe the law
scrupulously. Our government is the potent, the omnipresent
teacher
If the government becomes a lawbreaker, it breeds contempt
for law; it invites every man to become a law unto himself; it
invites anarchy
The rights and guarantees may exist. But it takes some
vigilance, some action on the part of the people in order that
those guarantees may come to life and become part of the
human spirit
Additional Cases
(A) The Bill of Rights and the Fundamental Powers
A1. Beltran v. Secretary of Health
476 SCRA 168 (2005)
FACTS: Petitioners comprise the majority of the Board of Directors
of the Philippine Association of Blood Banks, a duly registered
non-stock and non-profit association composed of free standing
blood banks. Public respondent Secretary of Health is being sued
in his capacity as the public official directly involved and charged
with the enforcement and implementation of RA 7719 or the

National Blood Service Act. Section 7 of RA 7719 provides phaseout of Commercial Blood Banks. Petitioners assail the
constitutionality of the said provision on the ground, among
others, that such represents undue delegation if not outright
abdication of the police power of the state.
ISSUE: WON RA 7719 is a valid exercise of police power
HELD: Petitions dismissed. The court upholds the validity of RA
7719.
RATIO: The promotion of public health is a fundamental obligation
of the State. The health of the people is a primordial
governmental concern. RA 7719 was enacted in the exercise of
the States police power in order to promote and preserve public
health and safety.
Police power of the state is validly exercised if (a) the interest of
the public generally, as distinguished from those of a particular
class, requires the interference of the State; and (b) the means
employed are reasonably necessary to the attainment of the
objective sought to be accomplished and not unduly oppressive
upon individuals
Police power is the State authority to enact legislation that may
interfere with personal liberty or property in order to promote
the general welfare.
Thus, persons may be subject to certain kinds of restraints and
burdens in order to secure the general welfare of the State and to
its fundamental aim of government, the rights of the individual
may be subordinated
A2. PHCAP v. Duque III1
535 SCRA 265 (2007)
Health is a legitimate subject matter for regulation by the DOH
(and certain other administrative agencies) in exercise of police
powers delegated to it. The superiority of breastfeeding and
correct information as to infant feeding and nutrition, as in this
case, is infused with public interest and welfare. The DOHs power
under the Milk Code to control information regarding breastmilk
vis--vis breastmilk substitutes is not absolute as the power to
control does not encompass the power to absolutely prohibit the
advertising, marketing, and promotion of breastmilk substitutes.
Implementing rules and regulations imposing labeling
requirements and limitations, as well as a prohibition against
certain health and nutrition claims are inconsistent with the Milk
Code.
Nonetheless, the DOH, in imposing an absolute prohibition on
advertising, promotion, and marketing, the same went beyond its
authority since the same was not within the provisions of the Milk
Code itself.
A3. Carlos Superdrug Corp. v. DSWD
526 SCRA 130 (2007)
FACTS: Petitioners are domestic corporations and proprietors
operating drugstores in the Philippines. Petitioners assail the
constitutionality of Section 4(a) of RA 9257, otherwise known as
the Expanded Senior Citizens Act of 2003. Section 4(a) of RA
9257 grants twenty percent (20%) discount as privileges for the
Senior Citizens. Petitioner contends that said law is
unconstitutional because it constitutes deprivation of private
property.
ISSUE: WON RA 9257 is unconstitutional
HELD: Petition is dismissed.
RATIO: The law is a legitimate exercise of police power which,
1

UST Golden Notes 2010, Political Law

6|P

LATON

similar to the power of eminent domain, has general welfare for


its object.
Accordingly, it has been described as the most essential, insistent
and the least limitable of powers, extending as it does to all the
great public needs. It is the power vested in the legislature by the
constitution to make, ordain, and establish all manner of
wholesome and reasonable laws, statutes, and ordinances, either
with penalties or without, not repugnant to the constitution, as
they shall judge to be for the good and welfare of the
commonwealth, and of the subjects of the same.
For this reason, when the conditions so demand as determined by
the legislature, property rights must bow to the primacy of police
power because property rights, though sheltered by due process,
must yield to general welfare.
A4. BANAT v. COMELEC2
586 SCRA 210 (2009)
The Court therefore strikes down the two percent threshold only
in relation to the distribution of the additional seats as found in
the second clause of Section 11 (b) of RA 7941. The two percent
threshold presents an unwarranted obstacle to the full
implementation of Section 5 (2), Article VI of the Constitution and
prevents the attainment of the broadest possible representation
of party, sectoral or group interests in the House of
Representatives
A5. Mirasol v. DPWH
490 SCRA 318 (2006)
FACTS: Petitioners sought the declaration of nullity of certain
administrative issuances of the DPWH for being inconsistent with
RA 2000, entitled Limited Access Highway Act. Among others, is
AO1 which requires motorcycles shall have an engine
displacement of at least 400cc.
ISSUE: WON said administrative issuances are unconstitutional
HELD: Petition partly granted. It is the DOTC, not the DPWH,
which has authority to regulate, restrict, or prohibit access to
limited access facilities.
We find that AO1 does not impose unreasonable restrictions. It
merely outlines several precautionary measures, to which toll way
users must adhere. These rules were designed to ensure public
safety and the uninhibited flow of traffic within limited access
facilities.
RATIO: The use of public highways by motor vehicles is subject to
regulation as an exercise of the police power of the state. The
police power is far-reaching in scope and is the most essential,
insistent and illimitable of all government powers. The tendency
is to extend rather than to restrict the use of police power. The
sole standard in measuring its exercise is reasonableness.
A6. MMDA v. Viron Transportation Co., Inc.
530 SCRA 341 (2007)
FACTS: PGMA issued EO 179, which provided for the
establishment of a Mass Transport System for Greater Manila.
Pursuant to this EO, the Metro manila Council of the MMDA cited
the need to remove the bus terminals located along major
thoroughfares of Metro Manila. Respondents, provincial bus
operators who had bus terminals that were threatened to be
removed, alleges that EO should be declared unconstitutional and
illegal for transgressing the possessory rights of owners and
operators of public land transportation units over their respective
terminals3
ISSUE: WON EO 179 is a valid exercise of police power
UST Golden Notes 2010, Political Law
3 Stef Macapagal
2

HELD: Petition denied. EO 179 is null and void.


RATIO: MMDA has no police power, let alone legislative power. In
light of the administrative nature of its powers and functions, the
MMDA is devoid of authority to implement the Project as
envisioned by the EO; hence it could not have been validly
designated by the President to undertake the Project. It follows
that the MMDA cannot validly order the elimination of the
respondents terminals
Police power rests primarily with the legislature, such power may
be delegated, as it is in fact increasingly being delegated. By virtue
of a valid delegation, the power may be exercised by the
President and administrative boards as well as by the lawmaking
bodies of municipal corporations or local government under an
express delegation by the LGC of 1991
Measures calculated to promote the safety and convenience of
the people using the thoroughfares by the regulation of vehicular
traffic present a proper subject for the exercise of police power
On Constitutional Law, The true role of Constitutional Law is to
effect an equilibrium between authority and liberty so that rights
are exercised within the framework of the law and the laws are
enacted with due deference to rights.
A7. Yamane v. BA Lepanto Condominium Corporation
474 SCRA 258 (2005)
FACTS: Petitioner City Treasurer of Makati holds respondent, in a
Notice of Assessment, liable to pay the correct business taxes,
fees and charges totaling to P1.6M in which the respondents
protested contending that condominium does not fall under the
definition of a business, thus, they are not liable for such taxes
ISSUE: WON the City Treasurer of Makati may collect business
taxes on condominium corporations
HELD: Petition denied. Accordingly, and with significant degree of
comfort, we hold that condominium corporations are generally
exempt from local business taxation under the LGC, irrespective
of any local ordinance that seeks to declare otherwise.
RATIO: The power of the local government units to impose taxes
within its territorial jurisdiction derives from the Constitution
itself, which recognizes the power of these units to create its
own sources of revenue and to levy taxes, fees, and charges
subject to such guidelines and limitations as the Congress may
provide, consistent with the basic policy of local autonomy.
A8. PPI v. Fertphil Corporation
548 SCRA 485 (2008)
FACTS: Petitioner and private respondent are private corporations
incorporated under Philippine laws. They are both engaged in the
importation and distribution of fertilizers, pesticides and
agricultural chemicals. President Marcos issued LOI 1465 which
provided, among others, for the imposition of a capital recovery
component on the domestic sale of all grades of fertilizers in the
Philippines. Pursuant to the LOI, private respondent paid P10 for
every bag of fertilizer it sold in the domestic market to the
Fertilizer and Pesticide Authority (FPA). After the 1986 Edsa
Revolution, FPA voluntarily stopped the imposition of the P10
levy. Private respondent then demanded from petitioner a refund
of the amounts it paid under LOI 1465
ISSUE: WON the issuance of LOI 1465 is a valid exercise of police
power of the State
HELD: Petition denied. The RTC and the CA did not err in ruling
against the constitutionality of the LOI
7|P

LATON

RATIO: Police power and the power of taxation are inherent


powers of the State. These powers are distinct and have different
tests for validity. Police power is the power of the State to enact
legislation that may interfere with personal liberty or property in
order to promote the general welfare, while the power of taxation
is the power to levy taxes to be used for public purpose. The main
purpose of police power is the regulation of a behavior or
conduct, while taxation is revenue generation. The lawful
subjects and lawful means tests are used to determine the
validity of a law enacted under the police power. The power of
taxation, on the other hand, is circumscribed by inherent and
constitutional limitations.
An inherent limitation on the power of taxation is public purpose.
Taxes are exacted only for a public purpose. They cannot be used
for purely private purposes or for the exclusive benefit of private
persons.
The power to tax exists for the general welfare; hence, implicit in
its power is the limitation that it should be used only for a public
purpose.

ordinary citizens against arbitrary government action, but not


from acts committed by private individuals or entities.
The right to due process guards against unwarranted
encroachment by the state into the fundamental rights of its
citizens and cannot invoked in private controversies involving
private parties.
The discipline of members by a political party does not involve the
right to life, liberty or property within the meaning of the due
process clause.

Chapter 2
Due Process
No person shall be deprived of life, liberty or property without
4
due process of law.
Due Process of Law
Person

A9. Yrasuegui v. PAL


569 SCRA 467 (2008)
FACTS: Petitioner was a former international flight steward of
PAL, herein respondent. Petitioner was dismissed because of his
failure to adhere to the weight standards of the airline company.
Petitioner claims that he was illegally dismissed.

Life, Liberty and Property


Life
Liberty

ISSUE: WON petitioner was discriminated against when he was


dismissed.

Roe v. Wade
410 US 113, 35 L Ed 2d 147, 93 S Ct 705 (1973)

HELD: Petition denied.


RATIO: To make his claim more believable, petitioner invokes the
equal protection clause guaranty of the Constitution. However, in
the absence of governmental interference, the liberties
guaranteed by the Constitution cannot be invoked. Put
differently, the Bill of Rights is not meant to be invoked against
acts of private individuals. Indeed, the US Supreme Court, in
interpreting the 14th Amendment, which is the source of our equal
protection guarantee, is consistent in saying that the equal
protection erects no shield against private conduct, however
discriminatory or wrongful. Private actions, no matter how
egregious, cannot violate the equal protection guarantee.

Caunca v. Salazar
82 Phil. 851, 1 SCUD 177 (1 January 1949)
Lupangco v. CA
160 SCRA 848 (1988)

Property
Ayog v. Cusi, Jr
118 SCRA 492 (1982)

Public Office

A10. Atienza, Jr. v. COMELEC


612 SCRA 761 (2010)
FACTS: Drilon, as president of the LP, announced his partys
withdrawal of support for the administration of PGMA. Petitioner,
LP chairman, and a number of party members denounced Drilons
move. In a party conference, petitioner moved to declare all
positions in the LPs ruling body vacant and elected new officers,
with petitioner as LP president. Drilon filed a petition before the
COMELEC and the latter nullified the elections. Eventually, Roxas
was installed as the new LP president. Petitioners were deemed
resigned for holding the illegal election of LP officers and were
dropped from the roster of LP members.
ISSUE: WON respondents violated petitioners constitutional right
to due process by the latters expulsion from the party.
HELD: Petition denied. The requirements of administrative due
process do not apply to the internal affairs of political parties.
RATIO: The constitutional limitations on the exercise of the states
powers are found in Article III of the Constitution or the Bill of
Rights. The Bill of Rights, which guarantees against the taking of
life, property, or liberty without due process under Section 1 is
generally a limitation on the states powers in relation to the
rights of its citizens. The right to due process is meant to protect

Layno, Sr. v. Sandiganbayan


136 SCRA 536 (1985)

Licenses
Corona v. United Harbor Pilots Association of the Philippines
283 SCRA 31 (1997)

Right-Privilege Dichotomy
RIGHT
The former come under the
protection of the Due Process
Clause

PRIVILEGE
They are just by way of grant
by the State

The American Supreme Court now has rejected the concept


that constitutional rights turn upon whether a governmental
benefit is characterized as a right or as a privilege. . . .
Whether any procedural protections are due depends on the
extent to which an individual will be condemned to suffer
grievous loss
4

CONSTITUTION, Art. III, 1

8|P

LATON

Hierarchy of Rights

It started off originally as simply a guarantee of procedural


fairness

The Due Process Clause protects life, liberty and property


PBMEO v. Philippine Blooming Mills Co., Inc.
51 SCRA 189 (1973)

Due process law was originally used a shorthand expression


for governmental proceedings according to the law of the land
as it existed at the time of those proceedings

PBMEO allegedly informed the respondent Company of the proposed


demonstration. A day before the planned demonstration, the Company
informed the workers that even as their right to demonstrate is
recognized, the normal operations of the Company should not be
unduly prejudiced

The guaranties of due process, though having their roots in


Magna Cartas per legem terrae and considered as procedural
safeguards against executive usurpation and tyranny, have in
this country become bulwarks also against arbitrary legislation

Company filed with the COR a complaint for violation of the CBA,
particularly the No Strike-No Lockout clause

PROCEDURAL
The method or manner by
which the law is enforced

(2) The Bill of Rights is designed to preserve the ideals if liberty, equality
and security against assaults of opportunism, the expediency of the
passing hour, the erosion of small encroachments, and the scorn and
derision of those who have no patience with general principles
Held: Yes. The respondent is the one guilty of unfair labor practices.
Because the refusal on the part of the respondent firm to permit all its
employees and workers to join the mass demonstration against alleged
police abuses and the subsequent separation of the eight (8)
petitioners from the service constituted an unconstitutional restraint
on their freedom of expression, freedom of assembly and freedom of
petition for redress of grievances, the respondent firm committed,
[among others], unfair labor practice for an employer to interfere
with, retrain or coerce employees on the exercise of their rights
guaranteed in Section Three

Basically addressed to those


who adjudicate
Revolves around the right to
be heard
Focuses on rules that are
established in order to ensure
meaningful
adjudications
appurtenant thereto

SUBSTANTIVE
Requires that the law itself,
not merely the procedure by
which the law would be
enforced, is fair, reasonable,
and just
Primarily directed at the
lawmakers
Implicates
fundamental
notions of fairness and justice
Concerns itself with the law,
its
essence,
and
its
concomitant efficacy

In other words, substantive due process looks to whether there


is a sufficient justification for the governments action

Property and property rights can be lost thru prescription; but


human rights are imprescriptible

Procedural Due Process

In the hierarchy of civil liberties, the rights of free expression


and of assembly occupy a preferred position as they are
essential to the preservation and vitality of our civil and
political institutions; and such priority gives these liberties the
sanctity and the sanction not permitting dubious intrusions.

Procedural due process is basically associated with the right to


be notified and heard. This accordingly presupposes that one
has been adequately and meaningfully informed of a case or
matter in which his rights are involved and that jurisdiction has
been validly acquired over him

As heretofore stated, the primacy of human rights over


property rights has been sustained

Jurisdiction is the right to hear and determine, not to


determine without hearing

xxx habeas corpus is the remedy to obtain the release of an


individual, who is convicted by final judgment through a forced
confession, which violated his constitutional right against selfincrimination; or who is denied the right to present evidence in
his defense as a deprivation of his liberty without due process
of law, even after the accused has already served sentence for
twenty-two years

That due process is the equivalent of law of the land which


means the general law; a law which hears before it condemns,
which proceeds upon inquiry and renders judgment only after
trial xxx that every citizen shall hold his life, liberty, property,
and immunities under the protection of the general rules which
govern society

The liberties of any person are the liberties of all of us

Classic procedural due process issues are concerned with what


kind of notice and what form of hearing the government must
provide when it takes a particular action

When freedom of the mind is imperilled by law, it is freedom


that commands a momentum of respect; when property is
imperilled it is the lawmakers judgment that commands
respect. This dual standard may not precisely reverse the
presumption of constitutionality in civil liberties cases, but
obviously it does set up a hierarchy of values within the due
process clause.

The fundamental requirement of due process is the opportunity


to be heard at a meaningful time and in a meaningful manner
It is a well-established doctrine that rules of procedure may be
modified at any time to become effective at once, so long as
the change does not affect vested rights

Related to the hierarchy of rights are the so-called standards


of review discussed earlier in Chapter 1

Additional Cases
(B) Due Process [14]

The Two Faces or Components of Due Process -- Substantive and


Procedural

B1. Republic v. Cagandahan


565 SCRA 72 (2008)
9|P

LATON

People v. Cayat
68 Phil. 12 (1939)
The accused, Cayat, a native of Baguio, Benguet, Mountain Province,
and a member of the non-Christian tribes, was found guilty of violating
Act No. 1639 for having acquired and possessed one bottle of A-1-1 gin,
an intoxicating liquor, which is not a native wine
Cayat challenges the constitutionality of Act 1639 on the grounds that it
is discriminatory and denies equal protection of the laws xxx
HELD: No. Act 1639 satisfies all the requirements:
1. The non-Christian tribes refers, not to religious belief, but to natives
of the Philippine Islands of a low grade of civilization
2. It is designed to insure peace and order in and among the nonChristian tribes xxx free use of highly intoxicating liquors by the nonChristian tribes have often resulted in lawlessness and crimes, thereby
hampering the efforts of the government to raise their standard of life
and civilization
3. It is intended to apply at all times as long as those conditions exist
4. The Act applies equally to all members of the class
Ormoc Sugar Co., Inc. v. Treasurer of Ormoc City
22 SCRA 603 (1968)

Chapter 3
Equal Protection
No person shall be deprived of life, liberty, or property without
due process of law, nor shall any person be denied the equal
5
protection of the laws.
The guarantee is against class legislation, or such legislation
which denies rights to one which are accorded to others, or
inflicts upon one individual a more severe penalty than is
imposed upon another in like case offending
Equality of operation of statutes does not mean indiscriminate
operation on persons merely as such, but on persons according
to the circumstances surrounding them. It guarantees equality,
not identity of rights

The Municipal Board of Ormoc City passed Ordinance No. 4 imposing


on any and all products of centrifugal sugar milled at the Ormoc Sugar
Company Inc., in Ormoc City a municipal tax equivalent to 1% per
export sale to the United States of America and other foreign
countries
Petitioner alleged that the said ordinance is unconstitutional for being
violative of the equal protection clause xxx insofar as Ormoc Sugar Co.,
Inc. is singled out
HELD: Yes. A perusal of the requisites instantly shows that the
questioned ordinance does not meet them, for it taxes only centrifugal
sugar produced and exported by the Ormoc Sugar Company, Inc. and
none other
The taxing ordinance should not be singular and exclusive as to exclude
any subsequently established sugar central, of the same class as
plaintiff, from the coverage of the tax. As it is now, even if later a
similar company is set up, it cannot be subject to the tax because the
ordinance expressly points only to Ormoc Sugar Company, Inc. as the
entity to be levied upon
JM Tuason & Co., Inc. v. Land Tenure Administration
31 SCRA 413 (1970)

The equal protection of the law clause is against undue favor


and individual or class privilege, as well as hostile discrimination
or the oppression of inequality

RA 2616 authorized the expropriation of the Tatalon Estate in Quezon


City jointly owned by herein petitioner, Gregorio Araneta and Company,
Inc. and Florencio Deudor

It does not demand equality among residents; it merely


requires that all persons shall be treated alike, under like
circumstances and conditions both as privileges conferred and
liabilities enforced

Petitioner sought to declare said RA unconstitutional as it is violative of


the equal protection clause since it applies only to the Tatalon Estate
and not to any other lands in Quezon City or elsewhere

Requisites for Valid Classification


1. The classification is based on substantial distinctions which
make real differences
2. It is germane to the purpose of the law
3. It applies not only to present conditions but also to future
conditions which are substantially identical to those of the
present
4. It applies equally to everyone or every member belonging to
the same class

CONSTITUTION, Art. III, 1

HELD: No. The legislature is not required by the Constitution to adhere


to the policy of all or none. Thus, to reiterate, the invocation by the
petitioner of equal protection clause is not attended with success

Gender
The equality of the sexes is something that the Constitution
itself promotes. And this means basically having to treat
women on equal footing with men even as it still maintains a
special solicitude for them
Bradwell v. Illinois
83 US (16 Wall) 130, 21 L Ed 442 (1873)
Mrs. Myra Bradwell, residing in the State of Illinois, applied with the
10 | P

LATON

Supreme Court of that State for a license to practice law. Mrs.


Bradwells application for a license was refused, and it was stated as a
sufficient reason that under the decision of the Supreme Court of
Illinois, the applicant--as a married woman would be bound neither by
her express contracts nor by those implied contracts which is the policy
of the law to create between attorney and client.
HELD: No. The right to control and regulate the granting of license to
practice law in the courts of a State is one of the powers which are not
transferred for its protection to the Federal government, and its
exercise is in no manner governed or controlled by citizenship of the
United States in the party seeking such license
Michael M. v. Superior Court
450 US 464, 67 L Ed 2d 437, 101 S Ct 1200 (1981)
Petitioner, then a 17-year-old male, was charged with violating
Clifornias statutory rape law, which defines unlawful sexual
intercourse as an act of sexual intercourse accomplished with a female
not the wife of the perpetrator, where the female is under the age of
18 years.
Petitioner contends that the law discriminates on the basis of sex,
because only females may be victims and only males may violate the
section
HELD: No. The justification for the statute offered by the State and
accepted by the Supreme Court of California, is that the legislature
sought to prevent illegitimate teenage pregnancies
Geosaert v. Cleary
335 US 464, 93 L Ed 163, 69 S Ct 198 (1948)
A Michigan law forbade any female to act as a bartender unless she be
the wife or daughter of the male owner of a licensed liquor
establishment
HELD: While Michigan may deny to all women opportunities for
bartending, Michigan cannot play favourites among women without
rhyme or reasons.

Stanley v. Illinois
405 US 645, 31L Ed 2d 551, 92 S Ct 1208 (1972)
Under the [challenged] scheme, the children of unmarried fathers,
upon the death of the mother, are declared dependents without any
hearing on parental fitness and without proof of neglect, though such
hearing and proof are required before the State assumes custody of
children of married or divorced parents and unmarried mothers
HELD: Yes. Stanleys claim in the state courts and here us that failure to
afford him a hearing on his parental qualifications while extending it to
other parents denied him equal protection of the laws. We have
concluded that all Illinois parents are constitutionally entitled to a
hearing on their fitness before their children are removed from their
custody. It follows that denying such a hearing to Stanley and those like
him while granting it to other Illinois parents is inescapably contrary to
the Equal Protection Clause

Marriage and Legitimacy


GSIS v. Montesclaros
434 SCRA 441 (2004)
Nicolas Montesclaros, 72-year old widower, married Milagros Orbiso,
then 43 years old. GSIS approved Nicolas application for retirement
granting a lump sum payment of annuity for the first five years and a
monthly annuity thereafter. Nicolas died. Milagros filed with the GSIS a
claim of survivorship pension. GSIS denied the claim because under
Section 18 of PD 1146, the surviving spouse has no right to survivorship
pension if the surviving spouse contracted the marriage with the
pensioner within three years before the pensioner qualified for the
pension
HELD: Yes. We hold that the proviso is unconstitutional because it
violates the due process clause. The proviso is also discriminatory and
denies equal protection of the law. The proviso in question does not
satisfy these requirements. The object of the prohibition is vague.
There is no reasonable connection between the means employed and
the purpose intended (deathbed marriages)
Labine v. Vincent
401 US 532, 28 L Ed 2d 288, 91 S Ct 1971 (1971)

Philippine Association of Service Exporters, Inc. v. Drilon


163 SCRA 386 (1988)
Petitioner, PASEI, a firm engaged principally in the recruitment of
Filipino workers, male and female, for overseas placement, challenges
the constitutional validity of Department Order No. 1 of the DOLE
Guidelines Governing the Temporary Suspension of Deployment of
Filipino Domestic and Household Workers for discrimination against
males or females and for not applying to all Filipino workers but only
to domestic helpers and females with similar skills

Ezra Vincent died intestate, survive only by collateral relations and an


illegitimate minor daughter, whose guardian sued to have her declared
Vincents sole heir
Appellant contends that Louisianas intestate succession laws that bar
an illegitimate child from sharing equally with legitimate children in the
fathers estate constitutes an invidious discrimination violative of the
Due Process and Equal Protection Clauses of the Constitution

HELD: No. As a matter of judicial notice, the Court is well aware of the
unhappy plight that has befallen our female labor force abroad,
especially domestic servants, amid exploitative working conditions
marked by, in not a few cases, physical and personal abuse

HELD: No. To further strengthen and preserve family ties, Louisiana


regulates the disposition of property upon the death of a family man

Tuan Anh Nguyen v. Immigration and Naturalization Service


522 US 53 150 L Ed 2d, 121 S Ct 2053 (2001)

A classification based on age could very well be justified


depending on particular subject matter which may require a
sense of maturity and responsibility, or vigor, health or strength

Tuan Anh Nguyen was born out of wedlock in Vietnem to a Vietnamese


mother and an American Father.
Petitioner claims that 1409--which imposes a set of requirements on
the children of citizen fathers born abroad and out of wedlock to a noncitizen mother that are not imposed under like circumstances when the
citizen parent is the mother--violates equal protection
HELD: No. The challenged classification serves important governmental
objectives and that the discriminatory means employed are
substantially related to the achievement of those objectives

Age

Dumlao v. COMELEC
95 SCRA 392 (1980)
Section4 of BP 52 disqualifies any retired elective provincial, city or
municipal official who has received payment of the retirement benefits
to which he is entitled under the law and who shall have been 65 years
of age at the commencement of the term of office to which he seeks to
be elected to run for the same elective local office from which he has
retired

11 | P

LATON

Petitioner contends that it is discriminatory and contrary to the equal


protection and due process guarantees of the Constitution
HELD: No. The purpose of the law is to allow the emergence of younger
blood in local governments. It is within the competence of the
legislature to prescribe qualifications for one who desires to become a
candidate for office provided they are reasonable, as in this case

Method or Mode of Dying


Vacco v. Quill
521 US 793, 138 L Ed 2d 834, 117 S Ct 2293 (1997)
In New York, as in most States, it is a crime to aid another to commit or
attempt suicide, but patients may refuse even lifesaving medical
treatment. Respondents are physicians who practice in New York
Respondents claim that the distinction between refusing lifesaving
medical treatment and assisted suicide is arbitrary and irrational
HELD: No. The overwhelming majority of the state legislatures have
drawn a clear line between assisting suicide and withdrawing or
permitting the refusal of unwanted lifesaving medical treatment by
prohibiting the former and permitting the latter.

Academic Performance
Academic ratings could very well determine how a student gets
classified or accepted
Tablarin v. Gutierrez
152 SCRA 730 (1987)
The petitioners sought admission into colleges or schools of medicine
for the school year 1987-1988. However, petitioners either did not take
or did not successfully take the National Medical Admission Test
(NMAT) required by the Board of Medical Education which is
administered by private respondent Center for Educational
Measurement (CEM)
Petitioners alleged that it is violative of the Equal Protection Clause by
reason of the yearly changes in the cut-off scores for successful
applicants
HELD: No. We conclude that prescribing the NMAT and requiring
certain minimum scores therein as a condition for admission to medical
schools in the Philippines do not constitute an unconstitutional
imposition

Nationality and Alienage


The Constitution itself has made classifications based on
citizenship, such a in the political field

disputed law was enacted to remedy a real actual threat and danger to
national economy posed by alien domination and control of the retail
business and free citizens and country from such dominance and
control

Office and Employment


Ones office, occupation or employment may also provide basis
for some differences in treatment, such as whether it is in the
public or private sector
It has also been held that elective officials may be treated
differently from appointive officials with respect to the effect of
the filing of their certificates of candidacy, i.e., while the latter
may be deemed resigned, the former may still hold on to their
positions
Nuez v. Sandiganbayan
111 SCRA 433 (1982)
Petitioner, accused before the Sandiganbayan of Estafa through
Falsification of Public and Commercial Documents, assails the validity of
PD 1486, as amended by PD 1606, which created the Sandiganbayan
The Snadiganbayan proceedings violates petitioners right to equal
protection, because--appeal as a matter of right became minimized into
a mere matter of discretion;--appeal likewise was shrunk and limited
only to questions of law, excluding a review of the facts and trial
evidence; and there is only one chance to appeal conviction, by
certiorari to the Supreme Court, instead of the traditional two chances;
while other estafa indictees are entitled to appeal as a matter of right
covering both law and facts and to two appellate courts
HELD: No. The general guarantees of the Bill of Rights, included among
which are the due process of law and equal protection clauses must
give way to a specific provision
International School Alliance of Educators v. Quisumbing
333 SCRA 13 (2000)
The School hires both foreign and local teachers as members of its
faculty, classifying the same into two: (1) foreign-hires and (2) localhires. Foreign-hires are then granted certain benefits not accorded
local-hires.
Petitioner contested the difference in salary rates between foreign and
local-hires
HELD: No. There is no reasonable distinction between the services
rendered by foreign-hires and local-hires. The practice of the School of
according higher salaries to foreign-hires contravenes public policy and,
certainly, does not deserve the sympathy of this Court

Crimes and Punishments


Ichong v. Hernandez
101 Phil. 1155 (1957)
RA No. 1180 An Act to Regulate the Retail Business prohibits persons,
not citizens of the Philippines, and associations, partnerships, or
corporations the capital of which are not wholly owned by citizens of
the Philippines, from engaging directly or indirectly in the retail trade
unless such aliens have actually been engaged in said business on 15
May 1954

Different offenses could not possibly be meted the same


penalty but crimes of the same nature should be treated no
differently
Skinner v. Oklahoma
316 US 535, 86 L Ed 1655, 62 S Ct 1110 (1942)

Petitioner attacks the constitutionality of the Act contending that it


denies to alien residents the equal protection of the laws and deprives
them of their liberty and property without due process of law

Oklahomas Habitual Criminal Sterilization Act provides for the


sterilization, by vasectomy or salpingectomy, of habitual criminals.
Petitioner, convicted of the crimes of stealing chickens, and robbery
with firearms twice, challenged the constitutionality of the Act but the
Oklahoma Supreme Court sustained the Act

HELD: No. The difference in status between citizens and aliens


constitutes a basis for reasonable classification in the exercise of police
power. Resuming what we have set forth above we hold that the

HELD: The equal protection clause does not prevent the legislature
from recognizing degrees of evil
12 | P

LATON

People v. Ching Kuan


74 Phil. 23 (1942)
Ching Kuan was accused of violating section 86 of the Revised
Ordinances of the City of Manila in that he constructed a 297-squaremeter building of strong materials in the district of Tondo without the
proper permit from the city engineer
Petitioner alleged the unconstitutionality of Article 66 of the revised
Penal Code which permits the court to take into consideration the
wealth and means of the culprit in the imposition of fines
HELD: No. It may seem paradoxical, but the truth is that the codal
provision in question, in authorizing the imposition of unequal fines,
aims precisely at equality before the law
People v. Jalosjos
324 SCRA 689 (2000)
the accused-appellant, Romeo F. Jalosjos is a member of Congress,
confined at the national penitentiary while his conviction for statutory
rape on two counts and acts of lasciviousness on six counts is pending
appeal
Petitioner filed a motion asking that he be allowed to fully discharge
the duties of a Congressman, including attendance at legislative
sessions and committee meetings despite his having been convicted in
the first instance of a non-bailable offense. This he primarily anchors on
the mandate of sovereign will
HELD: No. Allowing the accused-appellant to attend congressional
meeting for five (5) days or more in a week will virtually make him a
free man with all the privilege appurtenant to his position
People v. Vera
65 Phil. 56 (1937)
Mariano Cu Unjieng applied for probation. Respondent Judge denied
the application for probation
HELD: Yes. In the case at bar, the resultant inequality may be said to
flow from the unwarranted delegation of legislative power, although
perhaps this is not necessarily the result in every case
One province may appropriate the necessary fund to defray the salary
of a probation officer, while another province may refuse or fail to do
so. In such a case, the Probation Act would be in operation in the
former province but not in the latter. this means that a person
otherwise coming within the purview of the law would be liable to
enjoy the benefits of probation in one province while another person
similarly situated in another province would be denied those same
benefits

This basic individual right sheltered by the Constitution is a


restraint on all three grand departments of our government
and in the subordinate instrumentalities and subdivisions
thereof, and on many constitutional powers, like the police
power, taxation and eminent domain
Central Bank Employees Association, Inc v. BSP
446 SCRA 299 (2004)

In the case at bar, the challenged proviso operates on the basis of the
salary grade or officer-employee status. It is akin to a distinction based
on economic class and status, with the higher grades as recipients of a
benefit specifically withheld from the lower grades

Inverse Equal Protection


All persons or things differently situated should be treated
differently
Philippine Judges Association v. Prado
227 SCRA 703 (1993)
The petitioners, members of the lower courts who feel that their official
functions as judges will be prejudiced by the withdrawal of the franking
privilege of the Judiciary [while retaining the same for the President of
the Philippines, et. al.], assail the constitutionality of Section 35 of RA
7354 as implemented by the Philippine Postal Commission [as
discriminatory]
HELD: Yes. The respondents are in effect saying that the franking
privilege should be extended to only those who do not need it very
much, if at all, but not to those who need it badly.

Equalizing Illegality
What is illegal is plainly against the law and the fact that others
were able to get away with it is no justification to provide equal
chances for others under the Equal Protection Clause
Alunan III v. Mirasol
276 SCRA 501 (1997)
The DILG through then Secretary Rafael M. Alunan III, issued a letterresolution exempting the City of Manila from holding election for the
SK on the ground that the election previously held on 26 May 1990 was
to be considered he first under the newly enacted LGC
Private respondents claimed equal protection clause violation
HELD: No. Even assuming that only barangays in Manila were not
permitted to hold SK elections on December 4, 1992 while the rest of
the 5,000 barangays were allowed even if KB elections had already
been held there before, this fact does not give the youth voters in the
897 Manila barangays ground for complaint because what the other
barangays did was contrary to law. There is no discrimination here

Delegated Discrimination
A violation of the guarantee of equal protection may be seen
on the face of the law itself, or perceived and felt in the manner
in which what pretends to be a just and fair regulation is
actually utilized as a tool to camouflage a discriminatory act
Affirmative Action--A Case of Compensatory Discrimination
Affirmative action is a sort of reverse discrimination in the
sense that minorities have been discriminated against in the
past, are given preference in certain areas, like employment
and layoff, as well as admission to educational institutions

The thrust of the petitioners challenge is that the proviso (on


exemption from the coverage of the Salary Standardization Law) makes
an unconstitutional cut between two classes of employees in the BSP

Classifications necessarily mean making distinctions. And


grouping people or things always raises the issue as to whether
there is too much or there is too little in it

HELD: Yes. It is clear in the legislative deliberations that the exemption


of officers (SG 20 and above) from the SSL was intended to address the
BSPs lack of competitiveness in terms of attracting competent officers
and executives. It was not intended to discriminate against the rankand-file.

There should be no hesitation in using the equal protection


clause as a major cutting edge to eliminate every conceivable
irrational discrimination in our society
13 | P

LATON

HELD: No. The equal protection clause is not violated, because there is
a substantial basis for a different treatment of a member of a foreign
military armed forces allowed to enter our territory and all other
accused

Additional Cases
(C) Equal Protection
C1. Serrano v. Gallant Maritime Services, Inc.
582 SCRA 254 (2009)
th

Antonio Serrano, a Filipino seafarer, claims that the last clause in the 5
paragraph of Section 10, RA 8042 deprives them of equal protection
and denies them due process, for it treats OFWs differently from local
Filipino workers

Petitioner was hired by Gallant Maritime Services, Inc. and Marlow


Navigation Co., Ltd. under a POEA approved Contract of Employment.
He as constrained to accept a downgraded employment contract for
the position of Second Officer with a monthly salary of $1,000.00, upon
the assurance and representation of respondents that he would be
made Chief Officer by the end of April 1993. Respondents did not
deliver. Hence, petitioner refused to stay on as Second Officer and was
repatriated to the Philippines on May 26, 1998.
Petitioner filed with the Labor Arbiter a Complaint against respondents
for constructive dismissal and for payment of his money claims in the
total amount of $26,442.73. The dismissal was declared illegal; and
awarding him monetary benefits amounting to $8,770.00 based on the
salary period of three months only (in accordance with the 5th
paragraph of Section 10, RA 8042)
HELD: Yes. Section 18, Article II and Section 3, Article XIII accord all
members of the labor sector, without distinction as to place of
deployment, full protection of their rights and welfare xxx All monetary
benefits should be equally enjoyed by workers of similar category,
while all monetary obligations should be borne by them in equal
degree; none should be denied the protection of the laws which is
enjoyed by, or spared the burden imposed on, others in like
circumstances
The subject clause has a discriminatory intent against, and an invidious
impact on, OFWs
C2. Santos v. People
563 SCRA 341 (2008)
Criminal prosecution was recommended against MS. JUDY ANNE
LUMAGUI SANTOS for substantial underdeclaration of income, which
constitutes as prima facie evidence of false or fraudulent return
The information is void ab initio, being violative of due process and the
equal protection of the laws
HELD: No. Petitioner cannot aver that she has been denied equal
protection of the laws. Recognizing the existence of real differences
among men, the equal protection does not demand absolute equality.
It merely requires that all persons shall be treated alike, under like
circumstances and conditions, both as to the privileges conferred and
liabilities enforced
The only basis for petitioners claim of denial of equal protection of the
laws was the dismissal of the charges against Velasquez while those
against her were not
C3. Nicolas v. Romulo
578 SCRA 438 (2009)
Lance Corporal (L/CPL) Daniel Smith is a member of the United States
Armed Forces. He was charged with the crime of rape committed
against a Filipina, petitioner herein, sometime on November 1, 2005
Petitioners argue that to allow the transfer of custody of an accused to
a foreign power is to provide for a different rule of procedure for that
accused, which also violates the equal protection clause of the
Constitution

The rule in international law is that a foreign armed forces allowed to


enter ones territory is immune from local jurisdiction, except to the
extent agreed upon

Chapter 4
Searches and Seizures
The right of the people to be secure in their persons, houses,
papers, and effects against unreasonable searches and seizures
of whatever nature and for any purpose shall be inviolable, and
no search warrant or warrant of arrest shall issue except upon
probable cause to be determined personally by the judge after
examination under oath or affirmation of the complainant and
the witnesses he may produce, and particularly describing the
6
place to be searched and the persons or things to be seized.
This constitutional guarantee is a liberty safeguard that
prohibits and directs. While laying down the general rule
against unreasonable searches and seizures, it also provides the
guidelines for acceptable conduct on the part of government
authorities in regard to any intrusion into or invasion of the
peoples right to be secure in their persons, houses, papers,
and effects
It has also been held that the warrant is not meant to authorize
fishing expeditions
The finding of evidence cannot be the immediate reason for
issuing a search warrant. To use a search warrant for this
purpose would be unreasonable use of the remedy by search
warrant, which is prohibited by law
It assures ones entitlement to privacy and right to be left alone
and do whatever he wishes within bounds without having to be
subjected to the prying eyes of Big Brother
The embodiment of a spiritual concept: the belief that to value
the privacy of home and person and to afford its constitutional
protection against the long reach of government in no less than
to value human dignity, and that his privacy must not be
disturbed except in case of overriding social need, and then
only under stringent procedural safeguards
The privacy of the home has always been regarded by civilized
nations as one of the most sacred personal rights to which men
are entitled
A mans home is his castle, has become a maxim among the
civilized people of the earth
The right extends to all persons, including aliens. So are
corporations also included within its protection, though to a
lesser extent
Requirements for Issuance of Warrants
1. There should be a search warrant or warrant of arrest
2. Probable cause supported the issuance of such warrant

CONSTITUTION, Art. III, 2

14 | P

LATON

3. Such probable cause had been determined personally by a


judge
4. The judge personally examined the complaint and his
witnesses
5. The place to be searched and the persons or to be seized
have been particularly described
The Constitution requires that [the judge] carefully determine
the basis for the issuance of such warrant in order that the
protection declared be not set at naught
The test of sufficiency of an affidavit or testimony to justify the
issuance of a warrant is whether the affiant or the witness could
be held for perjury and made liable for damages in case his
statements are false
Nature of Search Warrants Proceedings
A search warrant proceeding is, in no sense, a criminal action or
the commencement of a prosecution. The proceeding is not
one against any person, but is solely for the discovery and to
get possession of personal property. It is a special and peculiar
remedy, drastic in nature, and made necessary because of
public necessity
Determination of Probable Cause
This responsibility of determining such cause has been vested
solely in judges, a duty which cannot be delegated to anybody
else
If the determination is done by one directly interested in law
enforcement the process would very likely be tainted and
clouded by his need to produce result in line with his job, and,
in the process, obscure some other considerations which would
otherwise be seen and appreciated by one not similarly
circumstanced
Roan v. Gonzales
145 SCRA 687 (1986)
FACTS: Petitioner challenged the admission of one Colt Magnum
revolver and 18 live bullets which were found after a search in his
house pursuant to a search warrant issued by the respondent judge,
which warrant he also assails. None of the articles listed in the warrant
was discovered during the search. However, the officers conducting the
search found in the premises the revolver and the bullets which they
confiscated. They are now the bases of the charge against the
petitioner.
HELD: To be valid, the search warrant must be supported by probable
cause to be determined by the judge or some other authorized office
after examining the complainant and the witnesses he may produce.
No less important, there must be a specific description of the place to
be searched and the things to be searched to prevent arbitrary and
indiscriminate use of the warrant. Probable cause, in the case of Burgos
V. Chief of Staff refers to such facts and circumstances which would
lead a reasonably discreet and prudent man to believe that an offense
has been committed and the objects sought in connection with the
offense are in the place sought to be searched. As held in a long line of
decision, the probable cause must refer to only one specific offense.
The petitioner has claimed that no deposition was taken by the
respondent judge , however, this is not entirely true because
depositions were taken of the complainants 2 witnesses in addition to
the affidavit executed by them. However, there is still the question of
the sufficiency of their deposition. A study of one of the depositions of
the witnesses, who claimed that their intelligence informers show that
they were suspicious of the petitioner because he was a follower of the

opposition candidate. This should have put the judge on guard as to the
motivations of the witnesses and alerted him of possible
misrepresentations from them. One may wonder why it did not occur
to the respondent judge to ask how the witnesses would be so certain
even to the caliber of the guns, or how far he was from the window or
whether it was on the first or second floor, or why his presence was not
noticed at all, or if the acts related were really done openly, in the full
view of the witnesses, considering that these acts were against the law.
These would have been judicious questions that were omitted and
instead, the declarations of the witnesses were readily accepted and
the search warrant sought was issued forthwith. The above discussed
defects have rendered the search warrant invalid. Nonetheless, the
Solicitor general argues that whatever defect there was, was waived
when the petitioner voluntarily submitted to the search and manifested
his conformity in writing. We do not agree because what we see here is
pressure exerted by the military authorities, who coerced the petitioner
to sign the supposed waiver was a guaranty against a possible challenge
later to the validity of the search they were conducting. Confronted
with the armed presence of the military and the presumptive authority
of the writ, the petitioner had no choice but to submit. The
respondents argued that the possession of colt magnum pistols and 18
live bullets are illegal per se, being malum prohibitum, hence it could be
taken even without a warrant. The SC held that it does not follow that
because an offense is malum prohibitum, the subject thereof is
necessarily illegal per se. Motive is immaterial in mala prohibita but the
subjects of this kind of offense may not be summarily seized because
they are prohibited. A search warrant is still necessary. WHEREFORE,
the search warrant issued by the respondent judge is declared null and
void.
Bache and Co (Phil.) v.Ruiz
37 SCRA 823 (1971)
FACTS: The Commissioner of Internal Revenue wrote the respondent
judge requesting the issuance of the search warrant against petitioners
for violating Sec. 46 (a) of the National Internal Revenue Code. The
following day, National Revenue examiner De Leon and his witness,
Logronio, went to the court together with documents i.e. application
for search warrant, affidavits, and other documents already filled out
but unsigned. At that time, Judge Ruiz was hearing a certain case and so
he simply instructed the deputy clerk of court to take their depositions.
After the session adjourned, the judge asked for the stenographer to
read to him the stenographic notes and without propounding any
questions, asked Logronio to take the oath and warned him that is his
deposition was found to be false, he would be charged with perjury.
The search warrant was then signed and BIR agents served the same on
the petitioners office. Petitioners lawyers protested on the ground
that no formal complaint or transcript of testimony was attached to the
warrant. Nevertheless, the agents proceeded with the search which
yielded 6 boxes of documents. Hence, this petition for certiorari,
prohibition and mandamus to declare the search warrant void.
HELD: The petition should be granted because the respondent judge
failed to personally examine the complainant and his witness. As
provided in the Constitution and the Rules of Court, the examination of
the complainant and witness he may produce shall be conducted by the
judge himself and not by others. The participation of the judge in the
proceedings which led the to the issuance of the search warrant was
thus limited to listening to the stenographers reading her notes, to a
few words of warning against the commission of perjury and to
administering the oath to the complainant and his witness. This cannot
be considered a personal examination. The reading of the stenographic
notes to respondent judge did not constitute sufficient compliance with
the constitutional mandate and the rule for by that manner respondent
judge did not have the opportunity to observe the demeanor of the
complainant and his witness, and to propound initial and follow up
questions which the judicial mind on account of its training, was in the
best position to conceive. These were important in arriving at a sound
inference on the all important question of whether or not there was
probable cause. Petition is granted, search warrant is declared null and
void.

15 | P

LATON

Particularity of Description and General Warrants


Notwithstanding the authority granted by a warrant, resulting
search or seizure may still be violative of the guarantee if such
instrument is in the nature of a general warrant, or one which
practically authorizes a blanket search or seizure, a roving
authority of officers serving it to look everywhere and get
anything that may be associated with what is being sought,
authorizing searches in any place for anything
The general warrants, whereby any officer or messenger may
be commenced to search suspected places without evidence of
a fact committed, or to seize any person or persons not named,
or whose offense is not particularly described and supported by
evidence, are grievous and oppressive, and ought not to be
granted
To obviate and prevent an avenue for abuse and arbitrary
exercise of power, the Constitution specifically requires
particularity of description of things or people to be searched
and seized
The warrant may be said to particularly describe the things to
be seized when the description therein is as specific as the
circumstances will ordinarily allow, or when the description
expresses conclusion of fact--not of law--by which the warrant
officer may be guided in making the search and seizure, or
when the things described are limited to those which bear
direct relation to the offense for which the warrant if being
issued
Stonehill v Diokno
20 SCRA 383 (1967)
FACTS; Upon application of the respondents-prosecutors, several
judges issued on several dates, a total of 42 search warrants against the
petitioners and/or corporations of which they are officers, to search the
persons above named and/or premises of their offices, warehouses
and/or residences, and to seize and take possession of certain personal
properties i.e. books of accounts, ledgers, financial records, vouchers ,
correspondences and other documents and/or papers showing all
business transactions as the subjects of the offense. Petitioners alleged
that said warrants are null and void because, inter alia, they do not
describe with particularity the documents, books and things to be
seized. Hence, this petition for certiorari, prohibition and mandamus.
HELD: The documents, papers and things under the alleged authority of
the warrants in question may be split into 2 major groups, namely a)
those found and seized in the offices of the aforementioned
corporations and b) those found seized in the residences of petitioners
herein. As regards the first group, we hold that the petitioners have no
cause of action to assail the legality of the contest warrants and of the
seizures made for the simple reason that said corporations have their
respective personalities, separate and distinct from the personality of
petitioners, It is well settled that the legality of the seizure can be
contested only by the party whose rights have been impaired and that
the objection to the unlawful search is purely personal and cannot be
availed of by third parties. With respect to the documents, papers and
things seized in the residences of the petitioners, the respondent
prosecutors are restrained from using them in evidence against the
petitioners. In connection with the documents, papers and things , 2
important questions need to be settled 1) whether the search warrants
in questions and the search and seizures made under the authority
thereof are valid or not and 2) if the answer to the preceding questions
is in the negative, whether said documents, papers and things may be
used in evidence against petitioners herein. Petitioners maintain that
the search warrants are in the nature of general warrants an that
accordingly the seizures effected are null and void. We agree. 2 points
must be stressed in connection with this constitutional mandate

namely 1) that no warrant shall issue but upon probable cause, to be


determined by the judge in the manner set forth in said provision and
2) that the warrant shall particularly describe the things to be seized.
None of these requirements has been complied with in the contested
warrants. Relying on the Moncado doctrine, the respondents
petitioners maintain that even if the search and seizures were
unconstitutional, the documents, papers and things thus seized are
admissible in evidence against petitioners. We are unanimously in the
opinion that this position be abandoned. Common law jurisdictions
have already given up in this approach and eventually adopt the
exclusionary rule realizing that this is the only practical means of
enforcing the constitutional injunction against unreasonable searches
and seizures.
Central Bank v. Morfe
20 SCRA 507 (1967)
FACTS: Respondent is a non stock corporation, the main purpose of
which is to encourage and implement savings and thrift among its
members and to extend financial assistance in the form of loans. CB
then published an announcement that such associations and others
similar in nature have never been authorized to engage in the banking
business. Moreover, CB directed the investigation and gathering of
evidences on the activities of such associations which are operating
contrary to law. Thereafter, a member of the intelligence division of CB
filed a verified application for a search warrant alleging that after close
observation and personal investigation, the organization is illegally
engaged in banking activities. A search warrant was issued. Petitioner
filed an action to annul the said search warrant on the grounds that it is
a roving commission, general in its terms and the use of the word and
others in the search warrant permits the unreasonable search and
seizure of documents which have no relation whatsoever to any specific
criminal act.
HELD: It is to noted that the action taken by CB is predicated on the
theory that the organization is illegally engaged in banking. The
respondent judge found the searches and seizure in question to be
unreasonable because the depositions given in support of the search
warrant states that the deponent personally knows threat the premises
of the organization were being used unlawfully for banking purposes.
He deduced that the deponent know specific baking transaction of the
petitioner with specific persons and then concluded that said deponent
could have if he really knew of actual violation of the law applied for a
search want to search and seize only books. To authorize and seize all
the records without referred to specific alleged victims of the
purported illegal banking transactions would be to harass the
petitioners.
The aforementioned order would seem to assume that an illegal
banking transaction of the kind contemplated in the contested action of
the offices of the bank, must always connote the existence of a victim
If this term is used to denote a party whose interests have been
actually injured, the assumption is not necessarily justified. The law
requiring compliance with certain requirements before anybody can
engage in banking obviously seeks to protect the public against actual,
as well as potential, injury.
It is true that such funds referred to are their savings and the
depositors thereof are members, but even a cursory examination of
said documents will readily show that anybody can be a depositors and
thus be a participating member. Hence, it is open to the public.
Moreover, the power to so dispose of the funds is placed exclusive
authority of the founding members and participating members are
expressly denied the right to vote or be voted for. This situation is
fraught, precisely with the very dangers or evils which RA 337 seeks to
forestall, by exacting compliance with the requirements of said act,
before the transaction in question could be undertaken. WHEREFORE,
the order is hereby annulled.

A search warrant may be partially void, such as when it


provided for the search and seizure of items which have not
been testified to during the application for such warrant
16 | P

LATON

159 SCRA 599 (1988)

The constitutional requirement of reasonable particularity of


the things to be seized is primarily meant to enable the law
enforcers serving the warrant to: (1) readily identify the
properties to e seized and this prevent them from seizing the
wrong items; and (2) leave said peace officers with no
discretion regarding the articles to be seized and thus prevent
unreasonable searches and seizures
The rule is that a description of a place to be searched is
sufficient if the officer with the warrant can, with reasonable
effort, ascertain and identify the place intended and distinguish
it from other places in the community
John Doe Warrants
The warrant should, as a rule, identify the person whose place
is to be searched, or the individual to be arrested. It may
happen, however, that the name could not be readily
ascertained, in which event some description sufficient to
identify the person may be resorted. If the description contains
enough details so that the officer serving the warrant would
have no difficulty identifying the person, then the warrant
would be considered valid
People v. Veloso
48 Phil. 169 (1925)
FACTS: Manila police, armed with a search warrant, raided the
Parliamentary Club, an establishment managed by Jose Ma. Veloso, a
member of the House of Representatives, acting on reliable
information that said club was actually a gambling house. They
apprehended nearly 50 people, including Veloso. Veloso asked what
they wanted and was shown the search warrant; he read it and told the
police that he was Representative Veloso and not John Doe (since the
name of Veloso did not appear in the search warrant but instead the
pseudonym John Doe was used), and that the police had no right to
search the establishment.
The police noticed that Velosos pockets were bulging as if containing
gambling utensils and as such they required him to show the evidence
they are searching for. After five minutes of conversation (with Veloso
constantly refusing to be searched all the while), the officers patience
reached its limit and one of them took hold of Veloso who promptly
retaliated, injuring the officer quite severely. It took two officers to
restrain Veloso, and with that, it was found that gambling utensils really
were in his pockets.
All of the persons apprehended Veloso included were arrested,
promptly searched and taken to the patrol wagons. Veloso again
refused to obey and shouted offensive epithets against the police
department, resisting so tenaciously that it took three policemen were
needed to place him in the patrol wagon. Because of his actions, Veloso
was charged and convicted of the crime of resistance to agents of the
authority (governed by Art. 252 of the Penal Code back then). On
appeal, Veloso held on to the proposition that his resistance was
justifiable on account of the illegality of the John Doe search warrant.
ISSUE: WON the search warrant and the arrest of Veloso were valid.
HELD: YES. Both the affidavit for the search warrant and the search
warrant itself described the address of the building where the
Parliamentary Club is. That sufficed as a sufficient designation of the
premises to be searched. As the search warrant stated that John Doe
had gambling utensils in his possession in the building occupied by him
(which, of course, is the building where the Parliamentary Club is) and
as that John Doe was Jose Ma. Veloso, the manager of the club, the
police could identify John Doe as Jose Mar. Veloso without difficulty.
Pangandaman v. Casar

FACTS: A shooting incident occurred in Lanao del Sur which left at least
five persons dead and two others wounded. In the course of events, a
warrant of arrest was issued against 64 persons: 14 (the petitioners in
this case) who were identified by three witnesses, and 50 John Does.
ISSUE: WON the warrant of arrest is valid; i.e. Can a court issue a
warrant of arrest against an unknown accused?
HELD: NO, it is not valid as far as the 50 John Does are concerned (as for
the 14 petitioners in this case, its an entirely different story because
their names and identities are already known; the warrant of arrest is
valid as regards to them). The warrant in question is of the nature of a
general warrant, one of a class of writs long prescribed as
unconstitutional and once anathematized as totally subversive of the
liberty of he subject. Clearly violative of the constitutional injunction
that warrants of arrest should particularly describe the person or
persons to be seized, the warrant as against unidentified subjects will
be considered as null and void.

Service Warrants - Time, Place and Manner


The time must not be one which is intrusive or violative of ones
privacy, like at the middle of the night
As for place, the warrant limits the area that may be searched
and the warrant could not be utilized as authority to search
everywhere until the item or articles sought for are found
The manner of service should not also be oppressive or
otherwise abusive
It could be served out at any time within its ten-day lifetime,
and if its object or purpose cannot be accomplished in one day,
the same may be continued the following day or days until
completed
Being armed with a warrant does not give leeway to a law enforcement
officer in doing whatever he wants without regard to the time, place,
and manner of execution of the warrant.
1.

2.

3.

The time must not also be one which is intrusive or violative


of ones privacy (e.g. in the middle of the night). Its also a
given that what is considered as reasonable time in one
locality may not be deemed the same way in another.
As for place, the warrant limits the area that may be
searched and the warrant could not be utilized as authority
to search everywhere until the items or articles sought for
are found.
Lastly, the manner of execution should not be oppressive or
otherwise abusive.

Q: May the implementation of search warrants be done on different


days?
A: Yes, it could be served at any time within its 10-day lifetime. If its
purpose cannot be accomplished in one day, it may be continued in the
next following day/s until completed.
Q: May the search warrant be used or a different purpose on each day?
A: No. A warrant can no longer be utilized as authority to make another
search once the items specified in it have already been seized.
People v. Court of Appeals
347 SCRA 453 (2000)
FACTS: Private respondent Ortiz was arrested carrying a pistol and
shabu. Soon after his arrest a search warrant was obtained from the
MTC of Paraaque commanding the PNP officers to make an
immediate search at any reasonable hour of day or night of the
house/s, closed receptacles and premises above-described and
17 | P

LATON

forthwith seize and take possession of the property subject of the


offense described (that is, unlicensed firearms a Baby armalite M-16,
a shotgun, a pistol cal. 45 and the like). Armed with the warrant, the
police accompanied by a representative of the MTC judge and a
barangay security officer went to Ortizs residence at about 7:30 PM
to search the premises, which resulted in the seizure of several
unlicensed firearms and ammunition. Ortiz was subsequently charged
with illegal possession of firearms; he moved to quash the search
warrant but the trial court denied it. The Court of Appeals (hereinafter
CA) reversed, holding as inadmissible in evidence the seized firearms
and ammunition.
ISSUE: WON the CA a quo erred in holding that the firearms and
ammunition seized from private respondents house are inadmissible as
evidence for being the fruits of an illegal search.
HELD: YES. In this case, there is no illegal search. The general rule is that
search warrants must be served in the daytime. However, when the
application asserts that the property is on the person or place ordered
to be searched, a search at any reasonable hour of the day and night
can be ordered. As to whether the time during which the search was
executed was unreasonable or not, the Supreme Court (hereinafter SC)
stated that the exact time of the execution of the warrant should be
left at the discretion of the law enforcement officers. Judicial notice
may be taken not just from the realities of law enforcement but also
the prevailing conditions in the place to be searched. As such, the SC
deemed that 7:30 PM was a reasonable time, taking notice that said
time in a suburban subdivision in Metro Manila in an hour at which the
residents are still up and about.

to abuse of the search process, and grant to officers executing a search


warrant that discretion which the Constitution has precisely removed
from them. The particularization of the description of the place to be
searched may properly be done only by the Judge, and only in the
warrant itself; it cannot be left to the discretion of the police officers
conducting the search.
Wilson v. Layne
526 US 603, 143 L Ed 2d 818, 119 S Ct 1692 (1999)
There is a warrant of arrest for the petitioners son in their home.
Respondents, deputy federal marshals and local sheriffs deputies,
invited the Washington Post (reporter and photographer) as part of
Marshals Service ride-along policy.
Warrant was silent about the media ride-along.
6:45 am petitioners were still in bed. Husband wearing pair of briefs ran
to the living room to investigate discovering 5 men wearing street
clothes with guns in their living room. The respondents, believing him
to be the person sought by the warrant, immediately subdued him on
the floor.
Wife follows wearing only nightgown saw her husband being restrained
by the armed officers.
Reporters observed and photographed the incident BUT WERE NOT
INVOLVED IN THE EXECUTION OF THE WARRANT. The newspaper
NEVER published the photographs.

People v. Court of Appeals


291 SCRA 400 (1998)

After a protective sweep of the house revealed that the petitioners son
is not in the house, the officers left.

FACTS: Quezon City police have procured a search warrant against Azfar
Hussain who had allegedly in his possession firearms and explosives at
Abigail Variety Store (hereinafter AVS) in Bulacan. However, the next
day, the search warrant against Hussain was served not at AVS but at
the adjacent apartment (hereinafter Apartment 1), resulting in the
arrest of four Pakistani nationals and the seizure of their personal
belongings, papers and effects, including cash (apparently quite a lot,
too) which was never mentioned in the warrant.

..Petitioners sued the officers in their personal capacities for money


damages.
..District court denied on basis of qualified immunity.
..Court of Appeals declined to decide whether it is a violation of the 4th
amendment but concluded that because no court had held at the time
of the search that media presence during a police entry into a residence
constituted such a violation, the right alleged was NOT CLEARLY
ESTABLISHED and thus the respondents are entitle to qualified
immunity
CHIEF JUSTICE REHNQUIST delivered the opinion of the court.
.. such a media ride-along does VIOLATE the 4th amendment. BUT
because the state of the law was not clearly established at that time
the search in this case took place, the officers are entitled to the
defense of qualified immunity.

Included allegedly among those taken were a piece of dynamite stick,


two pieces of C-4-type explosives and one fragmentation grenade,
along with some firearms and ingredients for explosives. On
arraignment, the four Pakistani nationals pleaded not guilty and
submitted a motion to quash the search warrant and declare the
evidence obtained by the police officers as inadmissible; the trial court
granted the motion. The People brought forth a petition for certiorari
but the CA dismissed it, basing on the proposition that the place
actually searched was different and distinct from the place described in
the search warrant.
ISSUE: WON a search warrant was validly issued as regards the
apartment in which private respondents were then actually residing (or,
put differently, WON the apartment had been specifically described in
the warrant).
HELD: NO. (The SC noted that the discrepancy concerning the premises
to be searched appeared to have resulted from the police officers own
faulty description of said premises in their application for the search
warrant, which was exactly what the Judge of the Quezon City Regional
Trial Court reproduced at AVS. The Judge also made the scope of the
search more particular and restrictive by his admonition in the warrant
that the search be limited only to the premises herein described.) The
place to be searched, as set out in the warrant, cannot be amplified or
modified by the officers own personal knowledge of the premises, or
the evidence they adduced in support of their application for the
warrant. Such a change is proscribed by the Constitution which requires
inter alia the search warrant to particularly describe the place to be
searched as well as the persons or things to be seized. It would concede
to police officers the power of choosing the place to be searched, even
if it not be that delineated in the warrant. It would open wide the door

Petitioners sued the federal officials under Bivens (Hanlon v. Bivens 525
US 981, [1988]) and the state officials under 1983. Both allows a
plaintiff to seek money damages from government officials who have
violated the 4th amendment. But government officials are shielded from
liability for civil damages insofar as their conduct does not violate
CLEARLY ESTABLISHED statutory or constitutional right of which a
reasonable person would have known. (Harlow v. Fitzgerald 457 US
800, [1982])
The court evaluating a claim for immunity Must first determine
whether the alleged right was clearly established at the time of
alleged violation. (Conn v. Gabbert 525 US 286 [1999])
In 1604 an English court made the now-famous observation that the
house of everyone is to him as his castle and fortress, as well for his
defense against injury and violence, as for his repose
The law of England has so particular and tender a regard to the
immunity of a mans house, that it stiles in its castle, and will never
suffer it to be violated with impunity.from this reason no doors
may be broken to execute any civil process; though, in criminal cases,
the public safety supersedes the private. William Blackstone
The 4th amendment: the right of the people to be secured of their
persons, houses, papers, and effects, against unreasonable search and
18 | P

LATON

seizures shall not be violated, and no warrants shall issue, but upon
probable cause, supported by Oath or affirmation, and particularly
describing the place to be searched, and the persons or things to be
seized.

This exception requires that there must be a lawful arrest which


precedes the search, the search must be contemporaneous with
the arrest, and the area searched is within the immediate
control of the person arrested

Respondents concede that the reporters did not assist them in their
task. But they are there to assert that the officers should be able to
exercise reasonable discretion, for good public relations, and also could
serve as to minimize abuse of the officers.
Own purpose of the Reporters, and the fact that it is not the officers
who kept the photographs, though in some cases, presences of 3rd
parties are justifiable, this is not.

Aguilar-Roque was one of the accused in the criminal case for rebellion
before Special Military Commission No.1, and also one of the accused in
a criminal case for subversion before the Military Commission no. 25.

The court held that it is a violation of the 4th amendment for police to
bring members of the media or other 3rd parties into a home during the
execution of a warrant when the presence of the third parties in the
house was not in aid of the execution of the warrant.

.August 6, 1984.. She and Nolasco was arrested at the intersection of


Mayon street and Piy Margal Street, Quezon City at 11:30 am. At noon
on the same day, elements of the Constabulary Security Group
searched the premises at 239-B mayon Street, Quezon City.

It is not unreasonable for the respondents at that time to believe that


bringing media along is lawful.

.The search resulted in the seizure of428 documents and written


materials, and additionally a portable typewriter, and 2 wooden boxes,
making 431 items in all, and the arrest of Tolentino, the person in
charge of the premises.

It is common practice to them. And there is no persuasive authority yet


on the case.
-judgment of CA is affirmed

Outright entry also is not justified
knock and announce rule (US)
Rule 126, section 7 of the rules of criminal procedure (Philippines)
Sec7. Right to break door or window to effect search the officer, if
refused admittance to the place of directed search after giving notice of
his purpose and authority, may break open any outer or inner door or
window of a house or any part of a house or anything therein to
execute the warrant or liberate himself or any person lawfully aiding
him when unlawfully detained therein.

Further, being armed with a warrant does not also justify


outright entry or barging into the place to be searched. In the
U.S., there is the so-called knock and announce rule whereby
an officer executing a warrant must knock and introduce
himself and announce his purpose and only in exceptional
circumstances may he dispense with the same, as when his
safety is at stake, or there is danger of the evidence being
destroyed
Warrantless Searches and Seizures
The recognized exceptions to the warrant requirement include:
1. Search incident to a lawful arrest
2. Evidence in plain view
3. Search of moving vehicles
4. Customs search
5. Stop and frisk
6. Exigent and emergency circumstances
7. Consented search
Search Incident to Lawful Arrest
This is primarily justified by the need to ensure the safety of the
arresting officers against any possible harm arising from the use
by the arrested individual of any weapon that he might have
concealed in his person or which is within immediate reach as
well as the need to preserve evidence that might otherwise be
destroyed

Nolasco v. Pao
139 SCRA 152 (1985)

Aguilar-Roque raised the issue of inadmissibility of evidence obtained


pursuant to the search warrant.
Judge Pao admitted to the amended Return and ruled that the
seized documents be subject to disposition of the tribunal trying the
case against the respondent.
Petitioners filed the instant petition for certiorari, prohibition and
mandamus to annul and set aside the
1. search warrant issued by the respondent Judge Pao
2. his Order admitting the Amended Return and granting
the Motion to Retain Seized Items
3. Order of the judge in the subversive documents case
denying petitioners Motion to Suppress.
..balancing of persons right and public interest.
Petitioners principally argue that Search warrant is void because it is a
general warrant since it did not sufficiently describe with particularity
the things subject of the search and seizure, and that probable cause
had not been properly established for lack of searching questions
propounded to the applicants witness. The SC held the warrants void,
but did not order the return of the seized items. Temporary restraining
order enjoining the respondents from introducing evidence obtained
pursuant to the search warrant in the subversive documents case is
made permanent, personalties seized may be retained by the CSG for
possible introduction as evidence in Criminal case no. SMC 1-1. Pending
before Special Military Commission No. 1, without prejudice to
petitioner Mila Aguilar-Roque objecting to their relevance and asking
said Commission to return to her any and all irrelevant documents and
articles.
Teehankee, J., concurring and dissenting:
Search warrant = void
Warrantless search = still void because she was arrested on board a
public vehicle. Warrantless search cannot be made in a place other
than the place of arrest.
Resolution on Motion for Reconsideration
147 SCRA 509 (1987)
The solicitor general, on behalf of public respondents, in deference to
the dissenting opinion of then SC Justice (now Chief Justice) Claudio
Teehankee, now offer no further objection to a declaration that the
subject search is illegal and to the return of the seized items to the
petitioners. Respondents state, however, that they cannot agree to
having the arrest of petitioners declared illegal
Search warrant =void
19 | P

LATON

Leangsiri was arrested at the arrival area of NAIA bringing 8225.31


grams of heroin hidden under the false bottom of a black suitcase.
Informed of the authorities that he is to deliver the contraband to 3
people at the Las Palmas Hotel in Manila.

FACTS: Detective Belland received information from a confidential


informer that unknown persons were smoking opium in a hotel. The
detective called for federal narcotic agents and went to the hotel with
such agents. The agents, who are experienced in narcotic work
recognized the smell of opium. The odor led them to Room 1. As
Belland went to the door and introduced himself, there was a slight
delay in the door before the defendant open the door. The defendant,
a woman, stepped back and admitted the officers. Belland then told her
about the opium smell, the defendant denied that there is such a smell.
Belland told her to consider herself under arrest because they were
going to search the room. The officers found opium and smoking
apparatus, the latter being warm, apparently from recent use. The
defendant objected to the admission of the evidence but the District
Court refused to suppress the same. She was convicted and the Court
of Appeals affirmed.

An entrapment was devised by NARCOM (narcotics command) and


agents of the bureau of customs then ensued.

ISSUE: Whether or not there was a violation of a valid search and


seizure (Specifically yung sa violation ng fourth amendment rights.)

The 3 were arrested.

Decision: Search and arrest not valid.

They learned that Amidu stays at room 413 of the same hotel. While
Omogbolahan and Bhola were billeted at Royal Palm Hotel.

Got there a piece of paper with the name Suchinda Leangsiri written on
it tucked within the pages of his telephone and address book. And
other possessions were confiscated.

Rationale: No reason for not obtaining a search warrant except


inconvenience to the officers and some slight delay necessary to
prepare papers and present the evidence to magistrate. There are no
convincing reasons to bypass the constitutional requirement. No
evidence or contraband was threatened with removal or destruction,
except perhaps the fumes which suppose in time will disappear. The
evidence of their existence before the search was adequate and the
testimony of the officers to that effect would not perish from the delay
of getting a warrant.

NARCOM went to Royal Palm Hotel. Coordinated with security officers


of the hotel who stood as witnesses when they entered and searched
the room. They yielded 2 black suitcases, each with false bottoms and
both smaller than that confiscated from Leangsiri. The appellants were
convicted of conspiring to transport heroin.

Since it was a search without warrant, it could be valid only if for a


crime committed in the presence of the arresting officer for a felony of
which he had reasonable cause to believe defendant guilty. The
arresting officer did not have probable cause to arrest petitioner until
he had entered her room and found her to be the sole occupant.

Whether or not the piece of paper found in the room of Amidu is


admissible as evidence.

An officer gaining access to private living quarters under color of his


office and of the law which he personifies must then have valid basis in
law for the intrusion. Valid basis is lacking.

Revised Rules of Court provides that a person lawfully arrested may be


searched for dangerous weapons or anything which may be used as
proof of the commission of an offense, without a search warrant.

Plain View Doctrine

Temporary restraining order enjoining the respondents from


introducing evidence obtained pursuant to the search warrant in the
subversive documents case is made permanent.
Personalities seized by virtue of the illegal search warrant are hereby
ordered returned to petitioners.
People v. Leangsiri
252 SCRA 213 (1996)

Accompanied by hotel owner and security officer, they searched


Amidus room.

Strict application of the exception that such warrantless search


obviously cannot be made at a place other than the place of arrest.
Immediate Control testwas enunciated in the American case of Chimel
vs. State of California. (Arrested at his house.. searched the entire
house and things found were allowed to be admitted as evidence. But
the SC reversed the conviction and held that the search went far
beyond his person and the area from which he might have obtained
either a weapon or something that could have been used as evidence
against him.)
The exception obtains when the plain view doctrine applies.
In the case a car, appellants were arrested in room 504 of the Las
Palmas Hotel. The piece of paper bearing the name of leangsiri
obtained in room 413 of the same hotel found through warrantless
search is illegal and the piece of paper is held to be inadmissible as
evidence against the appellants. The inadmissibility of the paper as
evidence will not however exculpate the appellants. Its exclusion does
not destroy the prosecutors case against the appellants. The remaining
evidence still established their guilt beyond reasonable doubt.
Johnson v. US
333 U.S. 10, 92 L Ed, 436, 68 S Ct 367 (1948)
Petitioner challenges her conviction on four counts charging violation of
federal narcotic laws on the ground that there were violations of her
Fourth Amendment rights.

Under this exception, contraband in plain view of officers who


have a right to be in that place where they are and see the
contraband have the right to seize it without having to secure a
warrant
Requisites:
1. Prior justification for the entry or intrusion
2. Inadvertent discovery of the evidence
3. Immediate apparent illegality of the item as evidence of a
crime, contraband or otherwise subject to seizure
4. Plain view justifies mere seizure of evidence without further
search
People v. Musa
217 SCRA 597 (1993)
In a buy-bust operation, a poseur buyer went to the house of Musa
(Present in the house of Musa: a boy, two women, one of which is his
wife) and asked for marijuana. After receiving the 20 Php marked
money, Musa gave the marijuana to the buyer, the buyer went to
the Narcotics Command and showed them marijuana. NARCOM team
then rushed to the buyers house. The marked money could not be
found. Musa said that he has given the money to his wife na wala na
dun nung panahon nay un. Apparently, two team members went to a
kitchen and noticed a cellophane colored white and stripe hanging at
the corner of the kitchen. The two asked Musa but Musa did not
answer. They opened it and found dried marijuana leaves. Musa was
convicted of selling marijuana.
20 | P

LATON

ISSUE: Whether or not the evidence was admissible.


Decision: No its not. (Under the Plain View Doctrine)
Rationale: In a buy bust operation, the authorities may seize the
marked money found on the person of the pusher immediately after
the arrest even without arrest and search warrants. Apparently, the
NARCOM agents searched the person of the appellant after arresting
him in his house but found nothing. Thats the time they searched the
whole house and found the plastic bag hanging in the corner which
contains the marijuana.
The warrantless search and seizure, as an incident to a suspects lawful
arrest, may extend beyond the person of the one arrested to include
the premises or surroundings under his immediate control. Objects in
the plain view of an officer who has the right to be in the position to
have that view are subject to seizure and may be presented as
evidence.
The plain view doctrine cannot justify the seizure of the object where
the incriminating nature of the object is not apparent from the plain
view of the object. THE PLASTIC BAGS WERE NOT WITHIN THEIR
PLAIN VIEW WHEN THEY ARRESTED THE APPELANT TO JUSTIFY ITS
SEIZURE. THE NARCOM AGENTS HAD TO MOVE FROM ONE PORTION
OF THE HOUSE TO ANOTHER BEFORE THEY SIGHTED THE PLASTIC
BAG. THE AGENTS HAD NO CLUE AS TO THE PLASTIC BAGS CONTENTS,
THEY HAVE TO OPEN IT FIRST TO KNOW WHETHER ITS CONTENTS IS A
CONTRABAND OR NOT. WHATS WITHIN THEIR PLAIN VIEW WAS THE
PLASTIC BAG, NOT THE MARIJUANA.
People v. Doria
301 SCRA 668 (1999)

12 small plastic bags containing and paper clip bags containing shabu
and two bricks of dried leaves which appeared to be marijuana. The
accused was charged with the unlawful possession of shabu and
marijuana. Accused was convicted. On appeal, he questions the validity
of the search warrant and the seizure of the bricks of marijuana.
ISSUE: whether or not the seizure of the latter drug (marijuana) was
justified on the ground that the drug was seized within the plain view of
the searching party.
Decision: Not justified. The decision of the trial court was reversed and
set aside. Accused is acquitted.
Rationale: For the plain view doctrine to apply, there must be: (a) prior
justification, (b) inadvertent discovery of the evidence, (c) immediate
apparent illegality of the evidence before the police.
The question is whether these requisites have been complied with.
(Hindi, lalo na yung a at b)
The only justification for an intrusion by the police is the conduct of a
search pursuant to accused appellants lawful arrest for possession of
shabu. The police failed to allege in this case the time when the
marijuana was found; whether or prior to, or contemporaneous with,
the shabu subject of the warrant, or whether it was recovered on
accused-appellants person or in an area within his immediate control.
Its recovery, therefore, presumably during the search conducted after
the shabu, had been recovered from the cabinet. Moreover, the
marijuana was in the form of two bricks wrapped in newsprint. Not
being in a transparent container, the contents wrapped in newsprint
could not have been readily discernible as marijuana. The recovery of
marijuana is INVALID. It is inadmissible in evidence against accused andappellant.

Search of Moving Vehicles


Buy-bust operation.
View of the Prosection: PO3 Manlangit acted as the buyer, gave 1600
pesos to Doria who thereafter told the former to wait for him as he
went to get the marijuana from his associate. After an hour, Doria gave
PO3 an object wrapped in plastic and gave it to PO3 who forthwith
arrested Doria. Marked money could not be found so they went to the
associates house. PO3 was standing by the door and saw a box which
has something wrapped in plastic which appear to be what just like
what Doria gave him. As PO3s suspicion aroused, he entered Neneths
house and took hold of the box. Other officers recovered the marked
money bills from Neneth. Defense denies these allegations.
ISSUE: Whether or not what happened is a reasonable seizure.(Plain
view doctrine again)
Decision: no.
Rationale: It is clear that an object is in plain view if the object itself is
plainly exposed to sight.
Standing by the door of appellant Gaddaos house, PO3 Manlangit had
a view of the interior of the said house. Two and a half meters away
was the dining table and underneath it was a carton box. The box was
partially open and revealed something wrapped in plastic. In a cross
examination, Manlangit admitted that he merely presumed the
contents to be marijuana because it had the same plastic wrapping as
the buy-bust marijuana. A close scrutiny of the records reveals that
the plastic wrapper was not colorless and transparent as to clearly
manifest its contents to a viewer. The marijuana was not in plain view
and its seizure without requisite search warrant was in violation of the
law and the constitution.
People v. Salanguit
356 SCRA 683
Police officers went to Salanguits house with a search warrant for
shabu. When they knocked on the door, no one opened the door. They
heard people panicking so they forced the door open and entered the
house. After showing the warrant, they searched the house, they found

If a warrant were first required before a car may be searched, it


may only be an exercise in futility as by the time the warrant is
issued the vehicle to be searched may have been driven to
some far away and unknown places
One has a lesser expectation of privacy in a motor vehicle
because its function is transportation, and it seldom serves as
ones residence or as the repository of personal effects. A car
has little capacity for escaping public scrutiny. It travels public
thoroughfares where both its occupants and its contents are in
plain view
Related to this kind of searches is the issue of checkpoints
where the rule is that only visual searches or inspection may be
had unless there is justifiable reason for conducting a more
extended search
The Court differentiated the checkpoint--which was primarily
intended to determine if the vehicles occupants were
committing a crime--from an information-seeking checkpoint
where police stop vehicles and ask the occupants for help in
providing information about a crime that has been committed
by others. This latter form was considered valid
People vs. Barros
231 SCRA 557 (1994)
FACTS: 2 police officers while riding a bus saw a man(Barros) carrying a
carton and placed it under his seat. When the officers reached their
destination, they informed another policeman to inspect the carton
carried by Barros. When the said policeman inspected the carton, he
found that it contained marijuana. When asked whether he owned the
carton of marijuana, he denied the same. But later on admitted the
ownership of such after the bus driver affirmed Barros ownership. He
was charged for the violation of the Dangerous Drug Act of 1972. The
21 | P

LATON

trial court convicted him. On appeal he claims that, among others, his
Constitutional right; against unreasonable search and seizures was
violated by the police authorities.

which has been declared no to be illegal per se, for as long as it is


warranted by the exigencies of public order and conducted in a way
least intrusive to the motorists.

ISSUE: whether or not the act of the policemen constitutes


unreasonable search and seizures?

Routine inspections are not regarded as violative of an individuals right


against unreasonable search. Limitations of routine inspections:
a.
Where the police officer merely draws aside the curtain of a
vacant vehicle which is parked on the public fair grounds.
b. Simply looks into a vehicle
c.
Flashes a light therein without opening the cars door
d. Where the occupants are not subjected to a physical or body
search
e. Where the inspection of vehicles is limited to a visual search
or inspection
f.
Routine check is conducted in a fixed area

HELD: there was an unreasonable search and seizures.


General rule, a search and seizure must be carried out through a
judicial warrant.
Exceptions (1): in case of moving vehicles, warrant less search can be
conducted because it is not practicable to secure a judicial warrant
before searching the same since such can be quickly moved out of the
locality or jurisdiction in which the warrant may be sought.
Limitations for the exception:
a.
it is limited only to routine checks, that is, the vehicle
are neither really searched not their occupants subject
to physical or body searches, the examination of the
vehicles being limited to visual inspection.
Exception (2): if there is probable cause.
In the case at bar, there was no probable cause for the policemen to
suspect that the carton he carried contained marijuana. Neither did
the carton emanate a distinct smell as that of marijuana. Nor did Barros
act suspiciously while boarding the bus. It did not indicate nor suggest
the presence of any such probable cause.
Waiver of unlawful search and seizure to constitute a waiver, it must
appear first that the right exists; second, that the person involved had
knowledge, actual or constructive, of the existence if such right; last,
the person had an actual intention to relinquish the right. The fact that
the accused failed to object to the entry to his house does not amount
to permission to make a search therein.
Ruling: Barros is acquitted.
Caballes v. Court of Appeals
373 SCRA 221 (2002)
FACTS: 2 policemen, while performing a routine patrol, spotted a
passenger jeep covered with kakawati leaves driven by the appellant.
The policemen stopped the jeep, and when asked what was loaded on
such, the appellant did not answer appearing pale and nervous. They
checked the cargo and discovered that it contained
aluminum/galvanized conductor wires exclusively owned by
NPC(national power corporation) He was convicted of theft. However
on appeal, he raised the validity of the warrantless search and the
admissibility of the evidence thus obtained.
ISSUE: whether or not police officers who were on routine patrol,
merely on suspicion that it might contain smuggled goods,
constitutes probable cause that will justify warrantless search and
seizure?
HELD: there is no probable cause and therefore illegal.
The mere mobility of vehicles, does not give the police officers
unlimited discretion to conduct indiscriminate searches without
warrant if made within the interior of the territory and the absence of
probable cause.
Probable cause signifies a reasonable ground of suspicion supported by
circumstances sufficiently strong, in themselves, to warrant a cautious
mans belief that the person accused is guilty of the offense with which
he is charged. The required probable cause will justify a warrantless
search and seizure is not determined by a fixed formula but is resolved
according to the facts of each case.
stop-and-search without warrant at military or police checkpoint

None of the following circumstances are present in the case at bar. The
police did not merely conducted a visual inspection of the vehicle, they
had to reach inside the vehicleit is not considered a simple routine
check.
The fact that the vehicle looked suspicious simply because it is not
common for such to be covered with kakawati leaves does not
constitute probable cause as would justify the conduct of a search
without warrant.
Rule: accused is acquitted.
Valmonte v. De Villa
178 SCRA 221 (1989)
FACTS: certain checkpoints in certain parts of Valenzuela and other
cities were set-up by the NCRDC which some of its residents complain
because they claim they are violation of their right against search and
seizures. Furthermore, they claim they are worried of being harassed
and of their safety being placed in arbitrary, capricious and whimsical
disposition of the military. The checkpoints by the military have been
issued to maintain, among others, peace and order.
ISSUE: whether the military checkpoints are constitutional and not an
infringement upon the right from warrantless search and seizures?
HELD: there was no issue to begin with because the petitioners are not
the real party in interest. It was ruled that petitioners who do not
allege that any of their rights were violated are not qualified to bring
action as real party in interest. In the case at hand, no proof to show
that, in the course of the routine checks, the military indeed committed
specific violations of petitioners right against unlawful search and
seizure or other rights.
Between the inherent right of the state to protect its existence and
promote public welfare and an individuals right against a warrantless
search, which is however reasonably conducted, the former should
prevail.
Rule: petition dismissed.
Anag, Jr. v. COMELEC
237 SCRA 424 (1994)
FACTS: COMELEC issued resolution 2323 otherwise known as the Gun
Ban. Pursuant to such resolution, the petitioner instructed his driver to
immediately send his gun that was given to him by the House of
Representatives. Following such order, the driver immediately went to
Manila. At about 20 meters from the Batasang Pambansa, police
authorities conducted a checkpoint. About 14 men search the car
driven by the driver, thus the gun that was to be delivered was seized
and he was charge for the violation of the gun ban.
ISSUE: whether or not the search and seizure of the gun was in violation
of an individuals right against warrantless search?
HELD: It was a violation of such right, since there were no justifying
22 | P

LATON

circumstances specifically pointing the culpability of the petitioner and


his driver, therefore, the search was invalid. The action of the
policemen unreasonably intruded to the privacy and the security of his
property. Consequently, the firearms obtained in the search cannot be
admitted as evidence for any purposes of proceeding.
The submissive actions of the driver with regards to the search, seems
like a waiver of the need for a warrant to search. However, considering
the circumstance wherein 14 armed men who were there to search the
vehicle, without his master, herein petitioner, thus he is in no way
capable of resisting such actions.

Aniag, Jr. Vs Comission on Elections


237 SCRA 424 (1994)
FACTS: In the preparation for the synchronized election for the national
and local elections in 1992. COMELEC issued resolution No. 2323 also
known as Gun Ban containing rules and regulations on bearing,
carrying and transporting of firearms or other deadly weapons.
COMELEC also issued resolution No. 2327 providing disqualification of
candidates engaged in gunrunning, using and transporting of firearms,
organizing special strike forces, and establishing spot checkpoints.
A congressman of 1st district of Bulacan was compelled to return 2
firearms issued to him by the House of Representatives. Petitioner
instructs his driver Arellano to pick the firearms and return it to the
congress. In the same day PNP set up a check point outside Batasan,
where Arellano was stop and searched finding the two firearms
properly packed. Arellano was detained and the firearms confiscated.
Later Arellano was released and the petitioner was invited to give light
to the situation. Petitioner explained that Arellano did not commit any
crime and that the driver actually complying with the law apprehended
by returning the firearms to the congress.
ISSUE: W/N such search and seizure is a violation to the constitutional
right to due process.
HELD: Extensive search without warrant could only be resorted to if
the officers conducting the search had reasonable or probable cause. In
the case at hand there was no evidence to show that the policeman
were impelled to do such, for there is no confidential report leading
them to reasonably believe that certain motorist is engage in
gunrunning. Therefore the search could not be valid.
People v. Usana
323 SCRA 754 (2000)
FACTS: During the COMELEC gun ban period, policemen set up a check
point at the corner of Senator Gil Puyat Ave. Those manning said
checkpoint, check merely those they found to be suspicious an
imposing merely a running stop on others. Past midnight they stopped
a KIA Pride driven car by Escano with two passengers with it namely
Usana and Lopez. One of the Policemen saw a long firearm on the lap
of Usana who was seated at the passenger seat. The three were
brought to the police station together with the car. The car was
searched and found within is hashish.
ISSUE: W/N such search and seizures is a violation of the constitutional
rights of the accused.
HELD: Court ruled that not all checkpoints are illegal. Those which are
warranted by exigencies of public order and are conducted in a way
least intrusive to motorists are allowed. As long as the inspection is
limited to a visual searched it cannot be regarded as violative of
individuals right against unreasonable searched. There are six
recognize exceptions to warrant requirement 1) searched incidental to
an arrest 2) searched moving vehicles 3) evidence in plain view 4)
custom searches 5) consented warrantless searched and 6) stop frisk
situations. Therefore checkpoint conducted was in pursuance of the
gun ban enforced by the COMELEC.

Almeida Sanchez v. United States


413 US 266, 37 L Ed 2d 596, 93 S Ct 2535 (1973)
FACTS: Mexican and holder of a valid work permit, was stopped by the
Border Patrol on a state highway 78 in California and hi car was
thoroughly searched. The point where the petitioner was stopped, the
road meanders north as well as east. But nowhere does the road reach
the Mexican border, and at all points it lies north of US 80 a major east
west highway entirely within the United States that connects the
Southwest with the west coast. It is obvious that the Border Patrol had
no search warrant, and that there was no probable cause of any kind
for the search. Marihuana was uncovered from the search. The only
asserted justification is 287a3 of the Immigration and Nationality Act,
66 Stat. 233 8 U.S.C. 1357a3, that provides for warrantless searches of
automobiles and other conveyances within reasonable distance from
any external boundary of United States.
ISSUE: W/N the Border Patrol search is constitutional
HELD: No claim is made nor could one be that the search of the
petitioners car was constitutional under any previous decision of this
court involving the search of an automobile. The search in this case was
conducted in the unfettered discretion of the members of the Border
Patrol without warrant, probable cause or consent. Search on a
California road that lies at all points at least 20 miles north of the
Mexican border was a wholly of different sort. In the absence of a
probable cause search violated the right to be free from unreasonable
search and seizures.
United States v, Brigoni
422 US 873 (1975)
Case involving the not the Border Patrol but to question the occupants
about their citizenship and immigration status. The officer may
question the driver and passengers about their citizenship and
immigration status.
United States v. Martinez Fuente
428 US 543 (1976)
Brief questioning is consistent with the fourth amendments, thus not
need ne authorized by warrant.

Customs Searches
Items which are imported and which are to be subjected to
payment of customs duties are not considered as properly
within the territory of the taxing authority if the appropriate
taxes have not yet been paid
Search and seizure without search warrant of vessels and air
crafts for violations of the customs laws have been the
traditional exception to the constitutional requirement of a
search warrant, because the vessel can be quickly moved out of
the locality or jurisdiction in which the search warrant must be
sought before such warrant could be secured; hence it is not
practicable to require a search warrant before search or seizure
can be constitutionally effected
Papa v. Mago
22 SCRA 857 (1968)
FACTS: Counter intelligence unit of the Manila Police Department
misdeclared that a certain shipment of personal effects would be
release from the customs zone pier in manila. When the trucks left the
vicinity other counter intelligence group seize the said trucks.
ISSUE: W/N such act by the counter intelligence of Manila Police
prejudiced the right to be free from unreasonable search and seizure.
HELD: 2d section of the Act of March 3, 1815 it was made lawful for
23 | P

LATON

customs offices not only to board and search vessels within their own
and adjoining examine any vehicle beast or person which is suspected
which are introduce contrary to the law. In the instant case the
petitioner could not question the search for their only complain they
were just intercepted without any search warrant. But even if there is a
search there is still authority to the effect that no search warrant would
be needed under the circumstances obtaining in the instant case. Thus
freedom from unreasonable searches and seizures is construed as to
recognize the difference between search in the dwelling and search of a
ship, motorboat, wagon or automobile where it is not practicable to
secure a search warrant. For the reason a vehicle could quickly move
out.

search is warranted for his and others' safety.


In delivering the opinion of the Court, Chief Justice Warren stated:
"Where a reasonably prudent officer is warranted in the circumstances
of a given case in believing that his safety or that of others is
endangered, he may make a reasonable search for weapons of the
person believed by him to be armed and dangerous regardless of
whether he has probable cause to arrest that individual for crime or the
absolute certainty that the individual is armed.
"(a) Though the police must, whenever practicable, secure a warrant to
make a search and seizure, that procedure cannot be followed where
swift action based upon on-the-spot observations of the officer on the
beat is required."

Stop and Frisk Exception - The Terry Patdown

The majority of the Court concluded the "stop and frisk" technique was
constitutional as long as the action could be rationally justified by
circumstances.

In certain situations, a police officer on the beat may observe


certain unusual and suspicious activity which his training and
experience would indicate that something wrong is afoot. He
need not wait for an explicit criminal conduct be manifested
before he may take action. In such an instance, he may
approach the suspicious person and undertake a limited check-a stop and frisk--patting down the outside clothing of the
person for possibly concealed weapons. This is justified by the
need to act expeditiously in a case where a minutes delay may
prove dangerous if not disastrous
A Terry stop must be justified at its inception and reasonably
related in scope to the circumstances which justified the initial
stop
The reasonableness of seizures that are less intrusive than a
traditional arrest depends on a balance between the public
interest and the individuals right to personal security from
arbitrary interference by law officers
Terry v. Ohio
392 U. S. 1, 20 L Ed 2d 889 S Ct 1868 (1968)
FACTS: A Cleveland police officer (Martin McFadden) observed two
unidentified men on a street corner in his beat. As the officer watched,
the men took turns walking a short distance down the sidewalk and
peering in a particular store window. After each circuit, the men would
appear to confer about something. This occurred a total of twenty-four
times (according to the Supreme Court opinion; McFadden's earlier
testimony indicates the men walked to the store and peered through
the window approximately 3-5 times each).
McFadden watched as the men were briefly joined by a third person
who left quickly. The suspicious activity caused McFadden to suspect
the men were planning to rob the store.
After the third individual left, the men began walking away in the
direction of the store. McFadden followed and observed from a
distance until they met up with the third party. McFadden then
approached the group, identified himself as police, and asked the men's
names.
The men mumbled something unintelligible, at which point McFadden
turned one of the suspects and patted down the outside of his clothing.
In the process, the officer found a gun in the pocket of the suspect's
jacket. He then directed all three men to enter a store with their hands
raised, and proceeded to pat down the other two suspects, recovering
a revolver from one. The three were taken to the local police station,
where two were charged with carrying a concealed weapon.
ISSUE: Whether it is always unreasonable for a policeman to seize a
person and subject him to a limited search for weapons unless there is
probable cause for an arrest.
HELD: The Court held that the stop, or seizure, and frisk, or search, was
valid when a "reasonably prudent officer" has cause to believe a limited

Adams v. Williams
407 US 143, 32 L Ed 2d 612, 92 S Ct 1921 (1972)
FACTS: While on duty, a police officer was approached by a reliable
informant who told the officer that a person sitting in a nearby car, i.e.,
the defendant, was carrying drugs and a gun. The officer went to
defendants car and grabbed a gun from exactly the same place where
the informant said the gun would be. Then the officer searched the car
and
found
additional
weapons
and
drugs.
ISSUE: Can the officer rely on information obtained from a reliable
informant for reasonable suspicion for a search?
HELD: Yes. The Court held that the officer had reasonable suspicion to
conduct the search. Here, the Court noted that the officer had
reasonable suspicion because the officer knew the informant to be
reliable. Thus, the officer had reason to believe that the suspect was
armed and dangerous and he could constitutionally frisk the suspect for
weapons.
Malacat v. Court of appeals
283 SCRA 159 (1997)
FACTS: On 27 August 1990, at about 6:30 p.m., allegedly in response to
bomb threats reported seven days earlier, Rodolfo Yu of the Western
Police District, Metropolitan Police Force of the Integrated National
Police, Police Station No. 3, Quiapo, Manila, was on foot patrol with
three other police officers (all of them in uniform) along Quezon
Boulevard, Quiapo, Manila, near the Mercury Drug store at Plaza
Miranda. They chanced upon two groups of Muslim-looking men, with
each group, comprised of three to four men, posted at opposite sides
of the corner of Quezon Boulevard near the Mercury Drug Store. These
men were acting suspiciously with their eyes moving very fast. Yu and
his companions positioned themselves at strategic points and observed
both groups for about 30 minutes. The police officers then approached
one group of men, who then fled in different directions. As the
policemen gave chase, Yu caught up with and apprehended Sammy
Malacat y Mandar. Upon searching Malacat, Yu found a fragmentation
grenade tucked inside the latters front waist line. Yus companion,
police officer Rogelio Malibiran, apprehended Abdul Casan from whom
a .38 caliber revolver was recovered. Malacat was charged with
violating Section 3 of Presidential Decree 1866. The trial court ruled
that the warrantless search and seizure of Malacat was akin to a stop
and frisk, where a warrant and seizure can be effected without
necessarily being preceded by an arrest and whose object is either to
maintain the status quo momentarily while the police officer seeks to
obtain more information; and that the seizure of the grenade from
Malacat was incidental to a lawful arrest. The trial court thus found
Malacat guilty of the crime of illegal possession of explosives under
Section 3 of PD 1866.
ISSUE: Whether the search made on Malacat is valid, pursuant to the
exception of stop and
frisk.
HELD: The general rule as regards arrests, searches and seizures is that
24 | P

LATON

a warrant is needed in
order to validly effect the same. The Constitutional prohibition against
unreasonable arrests, searches and seizures refers to those effected
without a validly issued warrant, subject to certain exceptions. As
regards valid warrantless arrests, these are found in Section 5, Rule 113
of theRules of Court. A warrantless arrest under the circumstances
contemplated under Section 5(a) has been denominated as one in
flagrante delicto, while that under Section 5(b) has been described as
a hot pursuit arrest. Turning to valid warrantless searches, they are
limited to the following: (1) customs searches; (2) search of moving
vehicles; (3) seizure of evidence in plain view; (4) consent searches; (5)
a search incidental to a lawful arrest; and (6) a stop and frisk. The
concepts of a stop-and-frisk and of a search incidental to a lawful
arrest must not be confused. These two types of warrantless searches
differ in terms of the requisite quantum of proof before they may be
validly effected and in their allowable scope. In a search incidental to a
lawful arrest, as the precedent arrest determines the validity of the
incidental search. Here, there could have been no valid in flagrante
delicto or hot pursuit arrest preceding the search in light of the lack of
personal knowledge on the part of Yu, the arresting officer, or an overt
physical act, on the part of Malacat, indicating that a crime had just
been committed, was being committed or was going to be committed.
Plainly, the search conducted on Malacat could not have been one
incidental to a lawful arrest. On the other hand, while probable cause is
not required to conduct a stop and frisk, it nevertheless holds that
mere suspicion or a hunch will not validate a stop and frisk. Here,
there are at least three (3) reasons why the stop-and-frisk was
invalid: First, there is grave doubts as to Yus claim that Malacat was a
member of the group which attempted to bomb Plaza Miranda 2 days
earlier. This claim is neither supported by any police report or record
nor corroborated by any other police officer who allegedly chased that
group. Second, there was nothing in Malacats behavior or conduct
which could have reasonably elicited even mere suspicion other than
that his eyes were moving very fast an observation which leaves us
incredulous since Yu and his teammates were nowhere near Malacat
and it was already 6:30 p.m., thus presumably dusk. Malacat and his
companions were merely standing at the corner and were not creating
any commotion or trouble. Third, there was at all no ground, probable
or otherwise, to believe that Malacat was armed with a deadly weapon.
None was visible to Yu, for as he admitted, the alleged grenade was
discovered inside the front waistline of Malacat, and from all
indications as to the distance between Yu and Malacat, any telltale
bulge, assuming that Malacat was indeed hiding a grenade, could not
have been visible to Yu. What is unequivocal then are blatant violations
of Malacats rights solemnly guaranteed in Sections 2 and 12(1) of
Article III of the Constitution.

Exigent and Emergency Circumstances


The Court justified the warrantless search by reason of the
urgency and exigency of the moment
People v. De Gracia
233 SCRA 716 (1994)
FACTS: The incidents involved in this case took place at the height of
the coup dtat staged in December, 1989. Accused-appellant Rolando
de Gracia was charged in two separate informations for illegal
possession of ammunition and explosives in furtherance of rebellion,
and for attempted homicide. Appellant was convicted for illegal
possession of firearms in furtherance of rebellion, but was acquitted of
attempted homicide. Surveillance was undertaken by the military along
EDSA because of intelligence reports & nbsp; &n bsp; about a coup.
Members of the team were engaged by rebels in gunfire killing one
member of the team. A searching team raided the Eurocar Sales Office.
They were able to find and confiscate six cartons of M-16 ammunition,
five bundles of C-4 dynamites, M-shells of different calibers, and
"molotov" bombs inside one of the rooms belonging to a certain Col.
Matillano. De Gracia was seen inside the office of Col. Matillano,
holding a C-4 and suspiciously peeping through a door. The team
arrested appellant. They were then made to sign an inventory, written
in Tagalog, of the explosives and ammunition confiscated by the raiding

team. No search warrant was secured by the raiding team. Accused was
found guilty of illegal possession of firearms.
ISSUE: Whether or not there was a valid search and seizure in this case.
Ruling: YES. It is admitted that the military operatives who raided the
Eurocar Sales Office were not armed with a search warrant at that time.
The raid was actually precipitated by intelligence reports that said
office was being used as headquarters by the RAM. Prior to the raid,
there was a surveillance conducted on the premises wherein the
surveillance team was fired at by a group of men coming from the
Eurocar building. When the military operatives raided the place, the
occupants thereof refused to open the door despite requests for them
to do so, thereby compelling the former to break into the office. The
Eurocar Sales Office is obviously not a gun store and it is definitely not
an armory or arsenal which are the usual depositories for explosives
and ammunition. It is primarily and solely engaged in the sale of
automobiles. The presence of an unusual quantity of high-powered
firearms and explosives could not be justifiably or even colorably
explained. In addition, there was general chaos and disorder at that
time because of simultaneous and intense firing within the vicinity of
the office and in the nearby Camp Aguinaldo which was under attack by
rebel forces. The courts in the surrounding areas were obviously closed
and, for that matter, the building and houses therein were deserted.
Under the foregoing circumstances, it is our considered opinion that
the instant case falls under one of the exceptions to the prohibition
against a warrantless search. In the first place, the military operatives,
taking into account the facts obtaining in this case, had reasonable
ground to believe that a crime was being committed. There was
consequently more than sufficient probable cause to warrant their
action. Furthermore, under the situation then prevailing, the raiding
team had no opportunity to apply for and secure a search warrant from
the courts. Under such urgency and exigency of the moment, a search
warrant could lawfully be dispensed with.

Consented Searches
Where the person to be searched acquiesces in the search of
his person or property, then obviously no warrant need be
procured. He in effect waives his right to otherwise have a
warrant justify the invasion of his liberty and privacy
It should not be presumed from a persons silence that he
waived the illegality of a search
Such a passive conformity given under coercive or intimidating
circumstances is considered no consent at all within the
purview of the constitutional guarantee
Consent must be made voluntarily, knowingly and intelligently
Requisites:
1. It must appear first that the right exists
2. The person involved had knowledge, actual or constructive,
of the existence of such right
3. Said person had an actual intention to relinquish the right
Consent to a search is not to be lightly inferred but must be
shown by clear and convincing evidence
Relevant to this determination are the following characteristics
of the person giving consent and the environment In which
consent is given:
1. The age of the defendant
2. Whether he was in public or secluded location
3. Whether he objected to the search of passively looked on
4. The education and intelligence of the defendant
5. The presence of coercive police procedures
25 | P

LATON

6. The defendants belief that no incriminating evidence will be


found
7. The nature of the police questioning
8. The environment in which the questioning took place
9. The possibly vulnerable subjective state of the person
consenting
It is the State which has the burden of proving, by clear and
positive testimony, that the necessary consent was obtained
and that it was freely and voluntarily given
United States v. Drayton
536 U.S. 194, 153 L Ed 2d 242, 122 S Ct 2105 (2002)
At a scheduled stop, police officers board the bus as part of a routine
drug and weapons interdiction effort. Lang approached the
respondents Drayton and Brown who were seated together, he
declared that the police were looking for drugs and weapons and asked
if the respondents had any bags. Lang searched the bag with Browns
consent, the bag revealed no contraband. The officer noticed that both
respondents were wearing heavy jackets and baggy pants despite the
warm weather. He asked Brown whether he minded if Lang checked his
person. Brown agreed, and a pat-down revealed hard objects similar to
drug packages in both thigh areas. Brown was arrested. Same thing
happened to Drayton. A further search revealed that respondents had
taped cocaine between their shorts.
Respondents moved to suppress the cocaine on the ground that the
pat-down searches was invalid. Lang did not inform the respondents
(passengers in that bus) of their right to refuse to cooperate.
Although Officer Lang did not inform respondents right to refuse the
search, he did request permission to search, and the totality of the
circumstances indicates that their consent was voluntary, so the
searches were reasonable. The Court has rejected in specific terms the
suggestion that police officers must always inform citizens of their right
to refuse when seeking permission to conduct a warrantless consent
search. While knowledge of the right to refuse consent is one factor to
be taken into account, the government need not establish such
knowledge as sine qua non of an effective consent.

Airport Searches
Correlated to the lessened expectation of privacy which a
passenger must necessarily have to recognize and accept, part
of the price for traveling in a mode of transportation that has
special concerns for safety and security

online tickets(Sec.9 RA 6235) that they are subject to search and, if any
prohibited materials or substances are found, such would be subject to
seizure. These announcements place passengers on notice that
ordinary constitutional protections against warrantless searches and
seizures do NOT apply to routine airport passengers.

The Terry search or the stop and frisk situation refers to a


case where a police officer approaches a person who is acting
suspiciously, for the purpose of investigating possibly criminal
behavior in line with the general interest of effective crime
prevention and detection. To assure himself that the person
with whim he is dealing is not armed with a weapon that could
unexpectedly and fatally be used against him, he could validly
conduct a carefully limited search of the outer clothing of such
person to discover weapons which might be used to assault him
Miscellaneous Searches and Seizures
Extends to non-tangible things and other forms of searches and
seizures involving alcohol and drug testing, for instance, or
transportation and communication, as well as in electronically
enhanced intrusions
Alih v. Castro
151 SCRA 279 (1987)
In 1984, two hundred (200) Philippine marines and elements of the
home defense forces conducted zona7. The initial reaction of the
people in the compound was to resist invasion with a burst of gunfire
no one was hurt; intended to warn the intruders. Unfortunately, the
situation aggravated. The soldiers returned fire and a bloody shooutout followed, resulting to a number of casualties. The besieged
compound surrendered the next morning, sixteen (16) male occupants
were arrested, finger-printed, paraffin-tested, and photographed over
their objection. The military also inventoried and confiscated nine M16
rifles, an M14 rifle, and nine rifle grenades, and several round of
ammunitions found in the premises.
The precarious state of Zamboanga City at the time in question
certainly did not excuse the non-observance of the constitutional
guaranty against unreasonable searches and seizures. The record does
not disclose that the petitioners were wanted criminals or fugitives
from justice. At the time of the zona they were merely suspected of the
mayors slaying and had not in fact even been investigated for it. Search
of petitioners premises is declared illegal and all articles seized as a
result thereof are inadmissible evidence against the petitioners in any
proceedings. However, said articles shall remain in custodia legis
pending the determination of legality of such.

People v. Johnson
348 SCRA 526 (2000)
Leila Johnson was about to fly back to USA. At NAIA departure area,
Olivia Ramirez, the lady in charge of frisking deporting passengers
frisked Johnson. Ramirez felt something hard on Johnsons abdominal
area. Upon inquiry, Johnson explained she needed to wear girdle as she
had undergone an operation. Not satisfied with the explanation and
with the consent of her superior, took Johnson to the ladys room for
inspection. Ramirez asked her to bring out the thing under her girdle.
Johnson brought out three plastic bags, which when examined turned
out to be methamphetamine hydrochloride (shabu). She was taken to
the to the Security office where her passport and ticket were taken.
She questions the legality of the warrantless search conducted at her
person.
Persons may lose the protection of the search and seizure clause by
exposure of their persons or property to the public in a manner
reflecting a lack of subjective expectation of privacy, which expectation
society is prepared to recognize as reasonable. Such recognition is
implicit in airport security procedures. Travellers are often notified
through airport public address systems, signs, and notices in their

Guazon v. De Villa
181 SCRA 623 (1990)
Petitioners, claiming to be bona fide residents of Metro Manila and
taxpayers and leaders in their respective communities, seek to prohibit
the military and police officers represented by public respondents from
conducting Areal Target Zonings or Saturation Drives in Metro
Manila. Saturation Drives were conducted in 1987in several critical
areas pinpointed by police and military as places where the subversives
were hiding. The petitioners claim that the saturation drives follow a
common pattern of human rights abuses.
The petition is REMANDED to the RTCs of Manila, Malabon, and Pasay.
The remedy is not an original action for prohibition brought through a
taxpayers suit. (1)No proper parties. Where one victim complains and
not one violator is properly charged, the problem is not initially for SC.
It is basically one for the executive departments and for trial courts. (2)
There is no proof. A method of pinpointing human rights abuses and
identifying violators is necessary.
Zona- military operation raid of a compound in search of loose firearms,
ammunition and other explosives.
7

26 | P

LATON

In the meantime and in the prima facie showing that some abuses were
probably committed and could be committed during future police
actions, banging on walls, kicking in of doors, violation of residences,
etc. are temporarily restrained.
Oliver v. United States
466 US 170 (1984)
Whether the open fields doctrine permits police officers to enter and
search marijuana fields without warrant where the fields are secluded
and contain no trespassing signs.
Reports that marijuana was being raised on the farm of oliver, two
narcotics agents went to the farm to investigate. They entered the farm
gate that has no trespassing sign and found a field of marijuana 1 mile
away from his house.
SC: as sated in Hester vs US, the governments intrusion upon open
fields is not one of those unreasonable searches proscribed in the
fourth amendment. The amendment does not protect the merely
subjective expectation of privacy but only those expectations that
society is prepare to recognize as reasonable. That an individual may
not legitimately demand privacy for activities conducted out of doors in
fields, except in the area immediately surrounding the home. An
individual has no legitimate expectation that open fields will be free
from warrantless intrusion by the government.
Oliver : circumstances may indicate reasonable expectations of privacy
were violated, it should be decided case to case basis.
Sc: no. it will make it difficult for the policeman to discern the scope of
his authority; it also creates a danger that constitutional rights will be
arbitrarily and inequitably enforced.
no trespassing sign and fences do not effectively bar the public from
viewing open fields and do not demonstrate that the expectation of
privacy was legitimate. The test is whether the intrusion of the
government infringes upon the personal and societal values protected
by the fourth amendment.sc finds no basis for concluding that a police
inspection of open fields accomplishes such infringement.
People v. Valdez
341 SCRA 25 (2000)
A tip was given about a plantation of marijuana allegedly owned by
valdez. The police were instructed to uproot the plants and arrest
cultivator. They found him in a nipa hut then looked around and saw
marihuana plant He admitted ownership but later on alleged that he
only admitted ownership out of fear.
General rule: search and seizure nust be carried on a judicial warrant.
Otherwise, it is unreasonable.
They first located the marijuana before appellant was arrested without
a warrant. There was no valid warrantless arrest which preceeded the
search of appellants premises. The seizure of evidence in plain view
applies only where the police officer is not searching for evidence
against the accused, but inadvertently comes across an incriminating
object. The marijuana plants were not in plain view because further
search was needed. There was illegal search and seizure thus the plant
cannot b admitted as evidence against him. The confession of
ownership without a counsel is also violative of the bill of rights.

In our view, requiring an employer to obtain warrant whenever


the employer wishes to enter an employees office, desk, or file
cabinets for a work-related purpose would seriously disrupt the
routine conduct of business and would be unduly burdensome
Camara v. Municipal Court Of The City And Country Of San Francisco
387 US 523 (1967)
Appellant was charged with violating san Francisco housing code for
refusing after 3 efforts by inspectors to secure his consent, to allow
warrantless inspection of the ground floor quarters which he leased
and whose residential used allegedly violated the apartments permit.
He argued that the sec503 of housing code is contrary to the 4th and
14th amendments in that it authorizes municipal officials to enter
private dwelling without a search warrant and without probable cause
to believe that a violation of the code exist.
Administrative searches are significant intrusions upon the interest
protected by the 4th amendment that such searches when authorized
and conducted without a warrant; lack the traditional safeguards which
the 4th amendment guarantees to the individual.
The warrant procedure is designed to guarantee that decision to search
private property is justified by a reasonable governmental interest. The
ultimate standard is the reasonableness.
Area inspection is reasonable search of private property within the
meaning of the 4th amendment. It is obvious that probable cause to
issue warrant to inspect must exist if reasonable legislative or
administrative standards are satisfied.
However, there was no emergency demanding immediate access; they
in fact made 3 trips to obtain consent. Yet no warrant was obtained and
thus the appellant was unable t verify either the need for or
appropriate limits of inspection... Appellant has constitutional right to
insist them to obtain a warrant to search.

Alcohol, Drugs and Related Tests


Requiring a person to submit urine or blood, or to undergo
breathalyzer testing for the purpose of determining whether he
is under the influence of alcohol or drugs are considered a
species of search that is governed by the constitutional
proscription against unreasonable searches and seizures. This
means, generally, that such a search may only be had pursuant
to a probable cause, or an individualized suspicion
Canine Sniff Test
While this may also implicate the privacy interest of the owner,
this is deemed as not approaching constitutional dimensions so
as to rule out the practice
A sniff by a dog that simply walks around a car is much less
intrusive than a typical search

Administrative and Other Searches

The Special Needs Exception

Searches and seizures by government employers or supervisors


of the private property of their employees are subject to the
restraints of the Fourth Amendment

Under this exception, the Court upheld an extended


warrantless search of a students purse after a report of
smoking in the school lavatory and following the students
denial of said report

Public employees expectations of privacy in their offices, desks,


and file cabinets, like similar expectations of employees in
private sector, may be reduced by virtue of actual office
practices and procedures, or by legitimate regulation

Our precedents establish that the proffered special need for


drug testing must be substantial--important enough to override
the individuals acknowledged privacy interest, sufficiently vital
27 | P

LATON

to suppress the Fourth Amendments normal requirement of


individualized suspicion
Warrantless Arrest
These arrests without warrant are justified by the fact that the
person to be arrested is caught in the act (in flagrante delicto),
is apprehended pursuant to a hot pursuit, or is an escaped
prisoner

Umil v. Ramos
187 SCRA 311 (1990)
Dural, a member of the NPA liquidation squad responsible for killing 2
soldiers on Jan. 31, 1988, was arrested on Feb. 1, while being treated
for a gunshot in a hospital. Dural was arrested for being a member of
the NPA, an outlawed subversive organization. Subversion as a
continuing offense, the arrest without warrant is justified.
The case filed involves subversion and illegal possession of firearm and
ammunition. On Aug. 17, 1988, the lower court found them guilty of
the charge.

In Flagrante Delicto
Under this exception, the arrest is justified by the very fact that
the crime is committed or is about to be committed in the very
presence of the person making the arrest
There is no more need for a warrant as the culprit is caught redhanded
Reliable information alone is not sufficient to justify a
warrantless arrest under Section 5(a), Rule 113. The rule
requires, in addition, that the accused perform such overt act
that would indicate that he has committed, is actually
committing, or is attempting to commit an offense
Elements:
1. The person to be arrested must execute an overt act
indicating he has just committed, is actually committing, or is
attempting to commit a crime
2. Such overt act is done in the presence or within the view of
the arresting officer
People v. Burgos
144 SCRA 1 (1986)
Cesar masalmok personally and voluntarily surrendered to the
authorities and gave intelligence information that he was forcibly
recruited by Burgos using a firearm. They found the accused plowing his
field, he denied possessing firearm. His wife pointed where the gun was
then the accused showed subversive documents that were allegedly
issued to him by a team leader of NPA. He was convicted of the crime
of illegal possession of firearm in furtherance of subversion.
In the constitutional provision against wanton and unreasonable
invasion of privacy and liberty of a citizen, his person, property papers
and effects, What is sought to be guarded is a mans prerogative to
choose who is allowed entry to his residence.
Under rule112, the officer arresting a person who has just committed,
is committing or is about to commit an offense must have a personal
knowledge of that fact. There is no personal knowledge in this case, it
came from masalmok and the location of gun was given by the wife. At
the time of his arrest, he was not in actual possession of the firearm
and the documents. Neither was he committing any act subversive.

Resolution on the Motion for Reconsideration


202 SCRA 292 (1991)
The courts decision on July 9, 1990 rules that the arrest of Dural
without warrant is justified because under Section 5 (a) Rule 113, Dural
was committing an offense, when arrested, because he was a member
of the NPA.
Sec 5 (b), Rule 113 of the Rules of Court requires 2 conditions for a valid
arrest without warrant:
1) That the person arrested had just committed an offense
2) That the arresting officer or the private person has personal
knowledge of facts (based on probable cause)
Accordingly, the motion for reconsideration of the decision dated July 9
is denied.
People v. Aminnudin
163 SCRA 40 (1988)
Aminnudin was arrested after disembarking from M/V Wilcon. Police
officers received a tip that he was carrying marijuana, and conducted a
warrantless search and arrest. The only justification was the tip
received 2 days before the arrest.
The accused-appellant was not caught inflagrante nor was a crime to be
committed or had just been committed to justify the warrantless arrest
under Rule 113 of the Rules of Court.
Without the evidence of marijuana allegedly seized from Aminnudin,
the case of prosecution shall fall. The evidence cannot be admitted for
the simple fact that the marijuana was seized illegally.

State of Rebellion
In quelling or suppressing the rebellion, the authorities may
only resort to warrantless arrests of persons suspected of
rebellion, as provided under Section 5, Rule 113 of the Rules of
Court, if the circumstances so warrant
Sanlakas v. Executive Secretary
421 SCRA 656 (2004)

In arrest without a warrant, it is not enough that there is a reasonable


ground to believe that the person to be arrested has committed a
crime. A crime must in fact or actually have been committed first. The
fact of commission must be undisputed. The test of reasonable ground
applies only to the identity of perpetrator.

On July 27, 2003, some 300 junior officers and enlisted men of the AFP,
armed with ammunitions and explosives stormed into the Oakwood
Apartments in Makati City. Bewailing the corruption in the AFP, the
soldiers demanded for the resignation of high officials, including the
President. The President then issued Proclamation No. 427 and General
Order No. 4 declaring a state of rebellion and calling out the AF to
suppress the rebellion. On Aug. 1, 2003, Proclamation No. 435 lifted the
declaration of the state of rebellion. Several petitions were filed
challenging the validity of the Proclamation, the Presidents authority
and the consequence of such declaration, specially in relation to the
arrest of those implicated in the rebellion.

No compelling reason for haste and not secure warrant of arrest. Arrest
of the accused while he was plowing is illegal. The arrest was unlawful,
the search and seizure is likewise not legal since these are mere
incidents of a valid arrest.

The mere declaration of the state of rebellion cannot diminish or


violate constitutionally protected rights. Simple declaration of the state
of rebellion does not suspend the operation of the constitution or
automatically suspend the privilege of writ of habeas corpus.

The right of a person to be secure against unreasonable seizure of his


body and any deprivation of liberty is most basic and fundamental. The
statute granting exceptions should be strictly construed.

28 | P

LATON

Hot Pursuit
When a crime has just been committed, the law enforcers
ordinarily would have to try to get the culprit as soon as
possible before he eludes them
They may be in possession of enough information or knowledge
by which to identify their quarry and make the arrest before
their job would become more difficult through the passage of
time
Go v. CA
206 SCRA 138 (1992)
FACTS: Rolito Go, petitioner, was driving in the opposite direction along
a one-way street in San Juan Metro Manila, when he nearly bumped
into Eldon Maguans car. Petitioner got out of his car, shot Maguan, and
left. A security guard nearby was able to take down the plate number of
Gos car then the police came at the crime scene. After obtaining
information from (1) the verification of LTO that the car was registered
under the petitioners wifes name, (2) the impression of the credit card
used by him in the bakeshop where he went before the incident
happened, and (3) the positive identification of the guard therein, the
police launched a manhunt. 6 days after the incident, Go, with 2
lawyers, presented himself to the San Juan police station for
verification. He was then detained and the police filed a complaint for
frustrated homicide in the Office of the Provincial Prosecutor of Rizal.
The Prosecutor filed before the RTC, an information for murder instead
of frustrated homicide, since Maguan died after a few days.
Nevertheless, petitioner was allowed to bail. The RTC judge (1) recalled
the bail, and gave petitioner 48 hrs from receipt of the Order to
surrender, (2) recalled and cancelled the Order which granted the leave
of the Prosecutor to conduct preliminary investigation, and (3) treated
as petition for bail the petitioners motion for immediate release and
preliminary investigation and set it for hearing.
Petitioner then filed for a petition for certiorari, prohibition and
mandamus before the SC, contending that the information was null and
void because no preliminary investigation has been previously
conducted. The SC remanded the petition to the CA wherein petitioner
was found not guilty since he refused to enter to a plea. The CA
dismissed the petitions and held that Gos warrantless arrest was valid
because the (1) offense was freshly committed, (2) his identity was
established through investigation, (3) when he showed up, there was
an existing manhunt and (4) there were witnesses.
ISSUE: WON a lawful warrantless arrest had been effected by the San
Juan police in respect of petitioner
HELD: The reliance of the petitioner and the Solicitor General in Umil v.
Ramos, is misplaced since in the said case, it was held that warrantless
arrests made from 1- 14 days after the actual commission of the crime
is legal in as much as such crime is a continuing crime. In the case at
bar, the crime committed was that of murder and cannot be considered
as a continuing crime since it was commenced and completed at once.
The warrantless arrest in this case does not fall under Sec. 5 of Rule 113
of the 1985 Rules on Criminal Procedure. Since the police arrested
petitioner 6 days after the shooting incident, it is apparent that the
arresting officers were not present during such incident and
therefore cannot be also regarded as one which had just been
committed.
Likewise, the said officers do not have personal knowledge of the
facts indicating that petitioner was the gunman. The information
derived from eyewitnesses did not constitute personal knowledge.
Thus, there was no lawful warrantless arrest.
Petitioner was not arrested at all. He walked in, with 2 lawyers, and
placed himself at the police disposal without stating the he is
surrendering. When the police filed complaint for frustrated homicide

with the Prosecutor, the latter should have immediately scheduled a


preliminary investigation to determine whether there was a probable
cause for charging petitioner. Since petitioner had not been arrested,
with or without warrant, he was entitled to be released immediately
and concerned only to his appearance in the preliminary investigation.
Posadas v. Ombudsman
341 SCRA 388 (2000)
FACTS: Dennis Venturina, a member of Sigma Rho at UP was killed in a
rumble between his fraternity & another on Dec. 8, 1994. Roger
Posadas, petitioner, and the chancellor of UP Diliman, asked the NBI for
the identification & apprehension of the suspects. Respondent Dizon,
the Chief of the Special operations group of the NBI and his men went
to UP and upon the basis of the identification by 2 eyewitnesses,
attempted to arrest Taparan and Narag who were members of the
other fraternity, as suspects but they just came to the UP police station
for a peace talk between their fraternities. Posadas objected because
the NBI agents do not have warrants of arrest and he & his lawyer
promised to take the 2 to the NBI office the next day. However, the
next day, they were not surrendered & have apparently escaped. Dizon
filed to the Office of the Prosecutor a complaint charging Petitioner for
violating PD 1829 which states that obstruction of the apprehension &
prosecution of criminal offenders is unlawful. Prosecutor
recommended for dismissal but was disapproved and was ordered by
the Ombudsman to proceed with the prosecution in Sandiganbayan.
Issues: (1) WON the attempted warrantless arrest of the student
suspects by the NBI could be validly made; (2) WON there was probable
cause for prosecuting petitioner for violation of PD 1829
HELD:
(1) Negative.
Art.3 Sec.2 of the Constitution:
No arrest may be made except in the case of a warrant issued by a
judge after examining the complainant and the witnesses he may
produce after finding probable cause to believe that the person to be
arrested has committed the crime. The case does not fall within the
exceptions provided in Rule 113 Sec. 5 of the Rules of Criminal
Procedure since neither the arresting officers witnessed the crime
being committed nor the students are fugitives from justice or
prisoners who had escaped from confinement.
The respondents invoked the ruling in People vs. Tonog,Jr wherein the
accused therein voluntarily went with the police after being invited and
the arresting officer found bloodstains on the pants of the accused
which made the former conclude that the latter is the suspect and the
arrest was also made on the day when the crime was committed. Thus,
Tonog case is not applicable in the case at bar since the NBI agents
tried to arrest Taparan & Narag 4 day after the commission of the
crime, they had no personal knowledge about the suspects to indicate
that they are guilty and they had obtained the information merely from
eyewitnesses, which is insufficient to justify a warrantless arrest.
Their attempt to arrest Taparan and Narag without a warrant was
illegal for their failure to comply with constitutional and procedural
requirements.
(2) As petitioners are also being prosecuted under PD 1829, it is a rule
that a criminal prosecution cannot be enjoined but it has been held that
respect for the citizen's right to be free from arbitrary arrest and
punishment and unwarranted and vexation prosecution is more impt.
than criminal procedure. As held in Venus vs. Desierto that the Court
does not interfere with the discretion of the Ombudsman in the
determination of the existence of a reasonable ground to believe that a
crime has been committed. The exceptions as stated in Brocka vs.
Enrile are as follows: (a)to afford protection to the constitutional rights
of the accused, (b) when necessary for the orderly administration of
justice or to avoid oppression or multiplicity of actions, (c) when there
is a prejudicial question which is sub judice, (d)where the acts of the
officer are without or in excess of authority, (e) where the prosecution
is under an invalid law, ordinance or regulation, (f)when double
jeopardy is clearly apparent, (g) where the court has no jurisdiction
over the offense, (h) where it is a case of persecution rather than
29 | P

LATON

prosecution, (i) where the charges are manifestly false and motivated
by the lust for vengeance, (j) where there is clearly no prima facie case
against the accused and a motion to quash on that ground has been
denied, and (k) preliminary injunction has been issued by the SC to
prevent the threatened unlawful arrest of the petitioners.
In the case at bar, PD 1829 was not violated since petitioner had a right
to prevent the arrest bec. it was illegal.
It was found out however by the Office of the Ombudsman that the
intervention by the petitioners allowed the escape of Taparan and
Narag. However, the student suspect, a certain Joel Carlo Denosta, was
not one of those who were attempted to be arrested by the NBI.
The NBI agents are at fault bec. they were unable to arrest Taparan and
Narag. If they believed the information given to them, they should have
applied first for a warrant before attempting to arrest.
Sanchez v. Demetriou
227 SCRA 627 (1993)
FACTS: Petitioner Sanchez, mayor of Calauan, Laguna, and other 6
people were accused of rape with homicide. Charges were filed against
them in connection with the rape-slay of Mary Eileen Sarmenta and the
killing of Allan Gomez. Preliminary investigation was conducted as
petitioner was represented by his counsel. The PNP sent petitioner an
invitation requesting him to appear for an investigation. When he was
taken to the camp, he was positively identified by 2 witnesses. He was
later placed on "arrest status" and taken to DOJ in Manila. An inquest
was conducted upon his arrival and a warrant of arrest was issued after
the hearing. He remained confined in Camp Crame while information
charges were filed with the others who were accused. A warrant arrest
was then issued and the SC ordered the transfer of such case to Pasig
City, M. Manila. Petitioner filed motion to quash the information since
his warrantless arrest was illegal and the court has no jurisdiction over
him. However, Judge Demetriou denied the petition.
Issues: (1) WON petitioners warrantless arrest was illegal; (2) WON the
court has no jurisdiction over him
HELD: (1) The warrantless arrest was illegal. The arresting officers were
not present during the commission of the alleged crime and they have
no personal knowledge that petitioner is responsible because their
basis was the statement by the witnesses. However, (2) the RTC has
jurisdiction over him because it issued a warrant arrest against him and
the others. It was delayed, but legal though.

Probable Cause for Issuance of Warrants of Arrest and Role of


Judges
What is required is that the judge must have sufficient
supporting documents upon which to make his independent
judgment, or at the very least, upon which to verify the findings
of the prosecutor as to the existence of probable cause
If the judge disagrees, or finds the evidence insufficient,
contrary to the conclusions of the prosecutor, the judge should
not dismiss the case but instead require the fiscal to present
additional evidence to show probable cause
The Court explained that the issuance of a warrant is not a
mere ministerial function. It calls for the exercise of judicial
discretion on the part of the issuing magistrate
While before, it was mandatory for the investigating Judge to
issue a warrant for the arrest of the accused if he found
probable cause, the rule now is that the investigating Judges
power to order the arrest of the accused is limited to instances
in which there is a necessity for placing him in custody in order
not to frustrate the ends of justice. The arrest of the accused

can be ordered only in the event that the prosecutor files the
case and the Judge of the Regional Trial Court finds probable
cause for the issuance of the warrant of arrest
Lim, Sr. v. Felix
194 SCRA 292 (1991)
FACTS: Vicente Lim, petitioner, was one of those who were charged
with multiple murder with frustrated murder in connection with the
ambush of Masbate Congressman Espinosa and his bodyguards (only
one survived) in the domestic airport of the said province Preliminary
investigation was conducted and the RTC judge was able to find a
probable cause for the issuance of the warrant of arrest. The Fiscal
ruled that the crime of the suspects must be murder for each of the 4
victims killed and physical injuries for the survivor. The Fiscal however
filed 4 separate Informations for murder against the 12 accused, with
no bail. As the petition for change of venue by Lim was granted by the
SC, the cases were transferred to Judge Felix of Makati. Petitioner filed
a motion and manifestation for the transmittal of initial records of
preliminary investigation for the best enlightenment of the court in its
determination of the existence of a probable cause based on the
Constitutional mandate that no warrant shall issue unless the issuing
magistrate have been personally convinced of such probable cause
but it was opposed by the prosecution & denied by the respondent
court. It later issued warrants of arrest against the petitioner and the
others who were accused.
ISSUE: WON a judge may issue a warrant of arrest without bail by
simply relying on the prosecution's certification & recommendation
that a probable cause exists
HELD: A judge may rely on the fiscals certification of the existence of
probable cause and issue a warrant of arrest. However, such
certification does not bind the judge to come out with the warrant of
arrest (Placer vs. Villanueva 126 SCRA 463 [1983]).
The judge must have a personal determination of the existence of a
probable cause for a warrant of arrest to be issued, but it does not
necessarily mean that he must personally examine the complaint
(Soliven vs. Makasiar 167 SCRA 393 [1988].
The determination of probable cause is a function of the judge.
Preliminary investigation is done by the prosecutor and does not bind
the judge. Also, there must be distinction between (1) the preliminary
inquiry which determines the probable cause for the issuance of the
warrant of arrest and (2) the preliminary investigation which ascertains
if the offender should be held for trial or be released (People v.
Honorable Enrique Inting GR No. 88919, July 25 1990).
RTC judges no longer have authority to conduct preliminary
investigations (Castillo v. Villaluz 171 SCRA 39 [1989]).
The judge may rely on the COMELEC's resolution to file for the
information in the same way that he may rely on the Prosecutor's
certification (People v Delgado GR Nos. 93419-32, Sept. 18, 1990.
*The constitutional mandate has not been satisfied and the judge
committed a grave abuse of discretion for relying solely on the
Prosecutor's certification where all the records of investigation are in
Masbate. He has not personally determined the probable cause but it
was the Provincial Prosecutor who had done such.
The extent of reliance depends on the circumstances of each case and
subject to the sound discretion of the judge. But when he issues a
warrant of arrest without evidence before him, he abuses such
discretion.
Webb v. De Leon
247 SCRA 652 (1995)
FACTS: The National Bureau of Investigation (NBI) filed with the
Department of Justice a letter-complaint charging petitioners Hubert
30 | P

LATON

Webb, Michael Gatchalian, Antonio Lejano and six (6) other persons,
with the crime of Rape with Homicide. Meanwhile, petitioner Webb
claimed during the preliminary investigation that he did not commit the
crime as he went to the United States on March 1, 1991 and returned
to the Philippines on October 27, 1992. Thereafter, the DOJ Panel
issued a 26-page Resolution finding probable cause to hold
respondents for trial and recommending that an Information for rape
with homicide be filed against petitioners and their co-respondents. It
then filed the corresponding Information against petitioners and their
co-accused with the Regional Trial Court. Respondent judge issued
warrants of arrest. Petitioner Webb voluntarily surrendered to police
authorities. Petitioners Gatchalian and Lejano likewise gave themselves
up to the authorities after filing their petitions before the Supreme
Court.
ISSUE: Whether or not the warrants of arrest issued by respondent
Judge Raul de Leon and later, respondent Judge Amelita Tolentino met
the constitutional requirement of probable cause.
HELD:
The Constitution, the Rules of Court, and our case law
repudiates the submission of petitioners that respondent
judges should have conducted searching examination of
witnesses before issuing warrants of arrest against them.
The Court also rejects the petitioners contention that a
judge must first issue an order of arrest before issuing a
warrant of arrest. There is no law or rule requiring the
issuance of an Order of Arrest prior to a warrant of arrest.
DOJ Panels 26-page report, testimonies of witnesses and
counter- affidavits of petitioners satisfied both respondent
judges that there is probable cause in issuing said warrants
of arrest.
Before issuing warrants of arrest, judges merely determine
personally the PROBABILITY, NOT THE CERTAINTY OF GUILT
of the accused. They just personally review the initial
determination of the prosecutor finding a probable cause to
see if it is supported by substantial evidence.
In search cases:
(1) Items sought are in fact seizable by virtue of their being
connected with criminal activity.
(2) The items will be found in the place to be searched.
In arrest cases:
(1) There must be probable cause that a crime has been
committed.
(2) The person to be arrested committed it.
*Upon filing of an information the Regional Trial Court may issue a
warrant for the arrest of the accused.
Talingdan v. Eduarte
366 SCRA 559 (2001)
FACTS: Petitioner, a private practitioner, charged respondent Judge
Eduarte, with improvidently issuing a warrant of arrest in a criminal
case for libel without the requisite preliminary investigation being first
conducted by the Office of the Public Prosecutor. He alleged that
sometime in April 2000, elements of PNP stormed into his residence to
arrest him and his client on the strength of a Warrant of Arrest issued
by respondent Judge. Complainant then filed a Very urgent Motion to
Quash and/or Set Aside Warrant of Arrest and Direct Prosecutors Office
to Conduct Preliminary Examination since they had not been previously
notified of the charge against them and no preliminary investigation
was ever conducted by the public prosecutors office yet. The
respondent granted the motion and recalled the warrant of arrest,
admitting that he issued the same under the mistaken belief that a
preliminary investigation had already been conducted and an
information filed in court. Thus, when he saw the Warrant of Arrest, he
signed the same honestly thinking that the Criminal Docket Clerk had
faithfully complied first with her duty of going over the records of the
case.

HELD: Respondents issuance of Warrant of Arrest was in violation of


the constitutional requirement of personal determination as to the
existence of probable cause.
REASON: No warrant of arrest shall issue except upon probable cause
to be determined personally by the judge after examination under oath
or affirmation of the complainant and the witnesses he may produce,
and particularly describing xxx the persons xxx to be seized. (Section 2,
Article III 1987 Constitution)
The phrase personal determination emphasizes the EXCLUSIVE
and PERSONAL RESPONSIBILTY of the issuing judge to satisfy himself as
to the existence of probable cause.
The Warrant of Arrest issues not on the strength of the certification
standing alone but because of the records that sustain it. In the case at
bench, there was not even a prosecutors certification to rely upon
since no information had even been filed yet in court.
Respondent cannot exculpate himself from administrative liability by
contending that the mistake was entirely attributable to the Criminal
Docket clerk who failed to faithfully comply with her duty of going
over the records of the criminal case and ensuring first that an
information had already been filed in court before preparing the
warrant of arrest.
* Options available to the judge upon personal determination of
probable cause:
(1) Personally evaluate the report and the supporting documents
submitted by the prosecutor regarding the existence of probable cause,
and on the basis thereof, issue a warrant of arrest.
(2) If on the basis thereof he finds no probable cause, disregard the
prosecutors report and require the submission of supporting affidavits
of witnesses to aid him in determining its existence.

Administrative Warrants
The Constitution is explicit that it is only a judge who can issue
warrants
The 1973 Charter allowed such other responsible officer as
may be authorized by law to determine probable cause
Qua Chee Gan v. Deportation Board
9 SCRA 27 (1963)
FACTS: Petitioners were charged before the Deportation Board of
having purchased US Dollars in the total amount of $130,000.00
without the necessary license from the Central Bank of the Philippines,
and of clandestinely remitting the same to Hong Kong. A warrant of
arrest of said aliens was issued by the presiding member of the
Deportation Board. Petitioners filed a motion to dismiss the charges
against them in the Deportation Board on the grounds of lack of
jurisdiction and that the charges do not constitute legal basis for
deportation. The lower court held that the Board has the power to
issue warrants of arrest and fix the amount of the bond for the
temporary release of the alien.
ISSUE:
(1)

(2)

Whether or not the President has the power to deport


aliens and consequently, the validity of delegation to the
Deportation Board of the ancillary power to investigate
Whether or not the Presidents power to conduct
investigation carries with it the power to order the arrest of
the alien complained of

* Pertinent Laws:
CA No. 613 (Immigration Act of 1940) Commissioner of Immigration
was empowered to effect the arrest and expulsion of an alien, after
previous determination by the Board of Commissioners of the existence
of grounds therefore.
Section 69 of Act No. 2711 (Revised Administrative Code) Lays down
the procedure to be observed should there be deportation
31 | P

LATON

proceedings.
HELD:
(1)
The charges against the herein petitioners constitute in effect an act
of profiteering, hoarding or blackmarketing of US dollars an economic
sabotage which is a ground for deportation.
There seems to be no doubt that the Presidents power of
investigation may be delegated. This is clear from a reading of Section
69 of the Revised Administrative Code which provides for a prior
investigation, conducted by said Executive (the President) or his
authorized agent.
(2)
Section 69 of the Revised Administrative Code, upon whose
authority the Presidents power to deport is predicated, does not
provide for the exercise of the power to arrest.
An implied grant of power, considering that no express authority
was granted by the law on the matter under discussion, that would
serve as a curtailment or limitation upon the fundamental right of a
person, such as his security to life and liberty, must be viewed with
caution. Then, a delegation of that implied power must be REJECTED as
inimical to the liberties of the people.
*The Executive Order insofar as it empowers the Deportation Board to
issue warrant of arrest upon the filing of formal charges against an alien
or aliens and to fix bond and prescribe the conditions for the temporary
release of said aliens, is declared ILLEGAL. The order of arrest issued by
the respondent Deportation Board is declared NULL AND VOID.
Harvey v. Defensor-Santiago
162 SCRA 840 (1988)
FACTS: Petitioners were apprehended from their respective residences
on February 27 1988 by agents of the Commission on Immigration and
Deportation (CID) by virtue of Mission Orders issued by Commissioner
Miriam Defensor-Santiago. Petitioners were among the twenty-two
(22) suspected alien pedophiles who were rounded up after three
months of close surveillance by CID agents. Seized during the
apprehension were photo negatives, pictures, posters and other
literature advertising the child prostitutes. After being denied bail,
petitioners availed a petition for a Writ of Habeas Corpus.
HELD: The petition is dismissed and the Writ of Habeas Corpus is
denied.
REASON:
*Probable Cause such facts and circumstances antecedent to the
issuance of the warrant that in themselves are sufficient to induce a
cautious man to rely on them and act in pursuance thereof.
In this case, the arrest of petitioners was based on probable cause
determined after close surveillance for three (3) months during which
period their activities were monitored. The existence of probable cause
justified the seizure of the photo negatives, photographs and posters
without warrant. Those articles were seized as an incident to a lawful
arrest and therefore, admissible in evidence.
The requirement of probable cause, to be determined by a Judge
does not extend to deportation proceedings.
What is essential is that there should be a specific charge against the
alien intended to be arrested and deported, that a fair hearing be
conducted with the assistance of counsel, if desired, and that the
charge should be substantiated by competent evidence.
In deportation proceedings, the right to bail is not a matter of right
but a matter of discretion on the part of the Commissioner of
Immigration and Deportation.
The power to deport aliens is an act of State, an act done by and
under the authority of the sovereign power.
Writ of habeas corpus will not be granted when the confinement is
or has become legal, although such confinement was illegal from the
beginning.
*Deportation proceedings are administrative in character. An order of
deportation is never construed as a punishment. It is preventive, not a
penal process.

* Order of Deportation-The return to his country of an alien who has


broken the conditions upon which he could continue to reside within
our borders.

Exclusionary Rule - The Fruit of the Poisonous Tree Doctrine


In the past it was held that sanctions against erring law
enforcers would be enough vindication of a violated right while
allowing the results of such an unreasonable search and seizure
to be admissible in evidence
A fruit of an illegal or unconstitutional act could not and should
not be given any form of legitimacy by its admission in evidence
Along with the discarding of the old rule came the demise of
the so-called Silver Platter Doctrine which allowed federal
judicial use of evidence seized in violation of the Constitution
by state agents
It is said that the exclusionary rule has three purposes:
First, the rule is calculated to prevent, not repair. Its purpose is
to deter--to compel respect for constitutional guaranty in the
only effective available way--by removing the incentive to
disregard it.
Second, the imperative of judicial integrity, i.e., that the
courts do not become accomplices in the willful disobedience
of a Constitution they are sworn to upholdby permitting
unhindered governmental use of the fruits of such invasionsA
ruling admitting evidence in a criminal trialhas the necessary
effect of legitimizing the conduct which produced the evidence,
while an application of the exclusionary rule withholds the
constitutional imprimatur
Third, that of assuring the people--all potential victim of
unlawful government conduct--that the government would not
profit from its lawless behavior, thus minimizing the risk of
seriously undermining popular trust in government
No man is to be convicted on unconstitutional evidence
Silverthorne Lumber Co., Inc. v. United States
251 US 385, 64 L Ed 319, 40 S Ct 182 (1920)
Facts: Indictment was filed against Frederick W. Silverthorne and his
father, who were arrested at their home. Representatives of
Department of Justice and the United States marshal without authority
go to the office of the company, made a clean sweep of all the books,
papers and documents found and directed all employees to the office
of the district attorney of US. Photographs and copies of material
papers were made, and a new indictment was framed based upon the
knowledge thus obtained. The District Court ordered the return of the
originals, but impounded the photographs and copies. Subpoenas to
produce the originals then were served, and on the refusal of the
plaintiffs in error to produce them, the Court made an order that the
subpoena should be compiled with. Contempt was filed against the
corporation and its owner
Issue: W/N there is an infringement of constitutional rights of the
parties under the Fourth Amendment which constitutes indictment?
Ratio: Taken from the dissenting opinion of CJ Holmes: The protection
of the Constitution covers physical possession, but not any advantages
that the government can gain over the object of its pursuit by doing a
forbidden act.
32 | P

LATON

If knowledge of them is gained from an independent source they may


be proved like any others, but the knowledge gained by the
Governments own wrong cannot be used by it in the way proposed

Additional Sanctions for Violations of the Guarantee


In addition to the exclusionary rule as a means of deterrence,
the erring officers may also be subjected to criminal and civil
liabilities for violating the constitutional proscription against
unreasonable searches and seizures
MHP Garments, Inc. v. Court of Appeals
236 SCRA 227 (1994)
Facts: MHP Garments, Inc was awarded to be the exclusive franchiser
to sell and distribute official Boy Scouts uniforms, supplies, badges and
insignias by the Boy Scouts of the Philippines. MHP has been given the
authority to undertake or cause to be taken the prosecution in court
of all illegal sources of scout uniforms and other scouting supplies. In
October 1983, petitioner received information that the private
respondents (Agnes Villa Cruz, Mirasol Lugatiman and Gertrudes
Gonzales) are selling unauthorized Boy Scout items and paraphernalia.
The petitioner, who was tasked to conduct surveillance and report,
together with two Philippine Constabulary officers (PC), went to the
respondents store and without warrant, seized boy and girl scouts
pants, dresses and suits which are on display. The respondents files a
criminal complaint for unfair competition, but was dismissed by the
Provincial Fiscal of Rizal, then later order returned the seized items to
the respondent. But seized items were not immediately returned, thus
private respondents personally went to the petitioners place of
business to recover the goods. Not all goods were returned and the
items that were returned were of that inferior quality.
Issue: W/N there is reasonable search and seizure even without
warrant
Ruling: Search and seizure is illegal
Ratio: The constitutional protection of our people against unreasonable
search and seizure is not merely a pleasing platitude. It vouchsafes our
right to privacy and dignity against undesirable intrusions committed by
any public officer or private individual. An infringement of this right
justifies an award for damages.
*Section 2, Article III of the 1987 Constitution protects not only those
who appear to be innocent but those who appear guilty, but are
nevertheless to be presumed innocent until the contrary is proved.
*There is a progression of time between the receipt of information and
the raid of the stores of private respondents. It shows sufficient time
for the petitioners and the PC raiding party to apply for a judicial
warrant.
Citing case of Lim vs. Ponce de Leon, recovery of damages for
violation of constitutional rights and liberties from public officer or
private individual as provided under Art. 32 of the Civil Code, in relation
to Article 2219 (6) of the same code. Recovery for Moral damages
*Art. 32 of the Civil Code make the persons who are directly, as well as
indirectly responsible for the transgression joint tortfeasors.
*Neither can it be said that only those shown to have participated
directly should be held liable. Art. 32 of the Civil Code encompasses
within the ambit of its provisions those directly, as well as indirectly,
responsible for its violations.
*Petitioners miserably failed to report the unlawful peddling of
scouting goods to the Boy Scouts of the Phil. for the proper application
of a warrant.
*Moral damages are not awarded to penalize the defendant but to
compensate the plaintiff for the injuries he may have suffered.
The wantonness of the wrongful seizure justifies the award of
exemplary damages. It will also serve as a stern reminder to all and
sundry that the constitutional protection against unreasonable search
and seizure is a virile reality and not a mere burst of rhetoric. The all

encompassing protection extends against intrusions directly done both


government and indirectly by private entities.
GROH v. Ramirez
540 U.S. 552, 257 L Ed 2d 1068, 124 S Ct 1284 (2004)
Facts: The petitioner, a Bureau of Alcohol, Tobacco and Firearms agent,
prepared and signed an application for a warrant to search the
respondents ranch for specified weapons, explosives and records
which is based on the information of a concerned citizen. Application
was supported by the petitioners affidavit that such items are there
together with a warrant form he has completed. The Magistrate (judge)
signed the warrant form even it did not identity any of the items that
the petitioner intended to seize. The description of the person or
property described respondents two story blue house rather than the
alleged stockpile of firearms. The petitioner led federal and local law
enforcement officers to the ranch the next day but found no illegal
weapons or explosives, then left the copy of the warrant but not the
application. Respondents sued petitioner in violation of the Fourth
Amendment.
Issue: (1) W/N the search violated the Fourth Amendment; (2) W/N is
entitled to qualified immunity, given the Magistrate Judge, relying on
an affidavit that particularly described the items in question; found
probable cause to conduct the search.

Ruling: Affirmed (decision of the Court of Appeals). Warrant is invalid


Ratio:
*Warrant was plainly invalid.
*Fourth Amendment states: no Warrants shall issue, but upon the
probable cause, supported by Oath or affirmation, and particularly
describing the place to be search and the persons or things to be seized
*The warrant complied with 1st three of the requirements:
-based on probable cause
-supported by sworn affidavit
-particularly described the place of search
*Warrant failed in particularity, because it did not provide description
of the type of evidence sought.
*Fourth Amendment by its terms requires particularity in the warrant,
not in supporting documents.
*The stated description of items to be seized in the warrant single
dwelling residenceblue in color did not describe the items to be
seized at all.
*The mere fact that the Magistrate issued a warrant does not
necessarily establish that he agreed that the scope of the search should
be as broad as the affiants request. Even though petitioner acted with
restraint in conducting the search, inescapable fact is that this
restraint was imposed by the agents themselves, not by a judicial
officer (Katz v. United States, 389 U.S. 347 [1967]).
*Purpose of the particularity requirement is not limited to the
prevention of general searches.
*Petitioner did not have in his possession a warrant particularly
describing the things he intended to seize; proceeding with the search
was clearly unreasonable under the Fourth Amendment.
*No reasonable officer could believe that a warrant that plainly did not
comply with that requirement was valid.
*If the law was clearly established, the immunity defense ordinarily
should fail, since a reasonably competent public official should know
the law governing his conduct (Harlow v. Fritzgerald, 457 U.S> 800,
818-819 [1982])
*Petitioner himself prepared the warrant and cannot reasonably relied
on the Magistrates assurance that the warrant contained an adequate
description of the things to be seize and was therefore valid. (Cf.
Sheppard, 468 U.S., at 989-990)
The uniformly applied rule is that a search conducted pursuant to a
warrant that fails to conform to the particularity requirement of the
Fourth Amendment is unconstitutional (Cf. Sheppard, 468 U.S., at 988,
n. 5)

Extra-territorial Reach of the Guarantee


33 | P

LATON

of those territories.

The Court held that the Fourth Amendment does not have
extraterritorial effect sp as to cover searches made in another
country involving non-American citizen. The social impact is
only between the government and those governed, including
aliens who have gone into the territory of the United States and
developed substantial connections with that country. Thus, if
the person affected is a citizen, it might be an entirely different
matter
United States v. Verdugo-Urquidez
494 U.S. 259, 108 Led 2d 222, 110 S Ct 1056 (1990)
Facts: Respondent is a citizen and resident of Mexico. He was believed
by the United States Drug Enforcement Agency (DEA) to be one of the
leaders of a large and violent organization in Mexico that smuggles
narcotics into the United States. He was apprehended by the Mexican
Police and transported him to United States Border Patrol station in
Calexico, California, then arrested by the United States Marshals and
moved him to a correctional center in San Diego, California, pending his
trial. DEA agents, working with Mexican officials, with Director General
of the Mexican Federal Judicial Police (MFJP), authorizing the searches,
searched his Mexican residences in Mexicali and San Felipe and seized
certain documents. The search of his residence uncovered a tally sheet,
which the Government believes reflects the quantities of marijuana
smuggled by the respondent into the United States. District Court
granted respondents motion to suppress the evidence, concluding that
the Fourth Amendment applied to the searches and DEA agents had
failed to justify searching of the premises without a warrant. Court of
Appeals for the Ninth Circuit Court, divided panel, held that American
citizens tried abroad by United States military officials were entitled to
Fifth and Sixth Amendment protections The court concluded that the
Constitution imposes substantive constraints on the Federal
Government, even it operates abroad. (citing Reid v. Covert, 354 U.S. 1,
[1957]). Majority assumed that illegal aliens in the United States have
Fourth Amendment rights. (relying on INS v. Lopez-Mendoza, 468 U.S.
1032 [1984]). Majority recognized that American search warrant would
be no legal validity in Mexico, but it is deemed sufficient that a warrant
would have substantial constitutional value in this country, because it
would reflect a magistrates determination that there existed probable
cause to search and would define the scope of the search.
Issue: W/N Fourth Amendment applies to the search and seizure by the
United States agents of property that is owned by a non-resident alien
and located in a foreign country.
Ruling: Reversed (Decision of the Court of Appeals)
Ratio:
*The Fourth Amendment operates in a different manner that the Fifth
Amendment, because the Fifth Amendment guaranteed the privilege
against self-incrimination, which is a fundamental trial right of criminal
defendants, which the constitutional violation will occur only at trial.
*As suggested by Madison, the driving force behind the adoption of
the Amendment was widespread hostility among the former Colonists
to the issuance of writs of assistance empowering revenue officers to
search suspected places for smuggled goods and general search
warrants permitting the search of private houses, often to uncover
papers that might be used to convict persons of libel. (Boyd v. United
States, 116 U.S. 616, 625 626, [1886])
*Purpose for the Amendment was to protect the people of the United
States against arbitrary action by their own Government; it was never
suggested to be intended to restrain the actions of the Federal
Government against aliens outside of the United States.
*Not every constitutional provision applies to governmental activity
even where the United States has sovereign power.
*Congress was not required to adopt a system of laws which shall
include the right of trial by jury and that the Constitution does not
without legislation and its own force, carry such right to territory so
situated.

Scientific and Technological Advancements and the Search and


Seizure Clause
As technology advances, the level of reasonably expected
privacy decreases. The measure of protection granted by the
reasonable expectation diminishes as relevant technology
becomes more widely accepted
In this area, again the courts would have to see how the
constitutional guarantee of privacy could be adjusted to meet
modern needs and demands, for as always there would be the
never-ending push and pull between the need of government
to maintain its role as protector against equally modernizing
criminal elements and the constant demand to safeguard
enduring liberty interests
Klyllo v. US
533 US 27, 150 L Ed 2d 94, 121 S Ct 2038 (2001)
Suspicious that marijuana was being grown in petitioner Kyllo`s home in
a triplex, agents used a thermal imaging device to scan the triplex to
determine if the amount of heat emanating from it was consistent with
the high intensity lamps typically used for indoor marijuana growth.
Kyllo was indicated on a federal drug charge of manufacturing
marijuana, he unsuccessfully moved to suppress the evidence seized
from his home and then entered a conditional guilty plea.
This case presents the question whether the use of a thermal imaging
device aimed at a private home from a public street detect relative
amounts of heat within the home constitutes a search within the
meaning of the fourth amendment.
The fourth amendment is to be construed in the light of what was
deemed an unreasonable search and seizure when it was adopted, and
in a manner which will conserve public interest as well as interests and
rights of individual citizens.
The government uses a devise that is not in general public use, to
explore details of the home that would previously have been
unknowable without physical intrusion, the surveillance is a search
and is presumptively unreasonable without warrant
The judgment of the court of appeals is reversed: the case is remanded
for further proceedings consistent with this opinion.

The Big Brother Spectre and the Right to Privacy


The limits of tolerable governmental intrusions and the extent
of privacy that society may reasonably recognize in various
situations would necessarily have to be considered by the
courts every now and then, as what might not be allowable
now would become a matter of necessity at some other time,
under different circumstances
It is necessary to stress that unless the creeping interference of the
government in essentially private matters is moderated, it is likely to
destroy that prized and peculiar virtue of the free society:
individualism. Every member of society, while paying proper deference
to the general welfare, must not be deprived of the right to be left
alone or, in the idiom of the day, to do his thing. As long as he does
not prejudice others, his freedom as an individual must not ne unduly
curtailed.
Ople v. Torres
293 SCRA 141 (1998)

Only fundamental constitutional rights are guaranteed to inhabitants


34 | P

LATON

The instant petition prays for the invalidation of Administrative order


no. 308 entitled Adoption of a National Computerized Identification
Reference System on constitutional grounds usurpation of the power
of congress to legislate and violation of the right to privacy. A.O. no.
308 was issued by then President Fidel V. Ramos on 12 December 1996,
was published in four newspaper of general circulation on 22 and 23
January 1997.
Yes. Assuming, arguendo, that A.O. No. 308 need not be the subject of
a law, still it cannot pass constitutional muster as an administrative
legislation because facially it violates the right to privacy. A.O. 308 is so
vague. The vagueness, the overbreadth of A.O. No. 308 which if
implemented will put our people's right to privacy in clear and present
danger. There are no vital safeguards the indefiniteness of A.O. No.
308 can give the government the roving authority to store and retrieve
information for a purpose other than the identification of the individual
through his PRN
The data may be gathered for gainful and useful government purposes;
but the existence of this vast reservoir of personal information
constitutes a covert invitation to misuse, a temptation that may be too
great for some of our authorities to resist.
Even that hospitable assumption will not save A.O. No. 308 from
constitutional infirmity for again said order does not tell us in clear and
categorical terms how these information gathered shall be handled. It
does not provide who shall control and access the data, under what
circumstances and for what purpose. These factors are essential to
safeguard the privacy and guaranty the integrity of the information.
It is plain and we hold that A.O. No. 308 falls short of assuring that
personal information which will be gathered about our people will only
be processed for unequivocally specified purposes.
They threaten the very abuses that the Bill of Rights seeks to prevent.
The petition is granted and A.O. no 308 declared null and void.

Additional Cases
(D) Searches and Seizures [11-12]
D1. NBI Microsoft Corp. v. Hwang
A former authorize distributor of Microsoft products was raided for
allegedly selling fake Microsoft products. During the raid, installer CDs
were found. What are installer CDs? They lump together in one CD
several programs so how could that be indicative of copyright
infringement. The DOJ however dismiss the case saying that there was
no probable cause because this person who was the subject of the
search warrant was an authorize distributor in the past.
HELD: the presence of installer CDs is indicative of a probable cause of
software infringement because Microsoft does not produce installer
CDs. Microsoft only comes up with CDs of particular programs but it
does not put them together in only one CD. If its windows its just
windows if its office its just office. But if you find them in only one CD,
thats installer CD, and then obviously those are fake or counterfeit. So
the SC said that is indicative of counterfeiting. Therefore there is
probable cause to continue with the case.

D2. AAA v. Carbonell


The names of victims in crimes involving violence against women and
their children would now have to be identified through this manner
AAA and so on in order to protect their privacy. In this case, there was a
charge for rape and the accused asked the judge for judicial
determination of probable cause. Instead of the judge simply issuing
the warrant as a result of the filing of the case, he was asked to
determine if there is really a probable cause for the purpose of issuing a
warrant. Acting on this motion, the judge required the complainant to
appear together with her witnesses to determine if there was probable

cause indeed, to issue a warrant of arrest. The complainant and the


witnesses did not show up and because of that, the trial court judge
dismiss the case. This was elevated to the SC.
HELD: it is not mandatory for the judge to conduct a hearing for the
purpose of determining probable cause if there has already been a case
filed before him and the records are sufficient to justify the finding of a
probable cause for the issuance of a warrant to arrest. In this case, the
SC itself determined that the records are already enough to justify the
issuance of a warrant of arrest, such that the act of the judge in
requiring the complainant to show up together with her witnesses and
eventually dismissing the case for their failure to show up was
considered as a grave abuse of discretion.
When it comes to the kind of information that would justify a probable
cause theres of course the requirement that those appearing before
the judge must have personal knowledge and not simply hearsay.
Because if its hearsay then the judge could obviously not carry on a
follow up manner of questioning. The witness would simply say that is
what was told me so how could the judge now prove further and
deeper. But if the person appearing before him has personal knowledge
then he could satisfy his curiosity of the judge whether he is really
telling the truth or not.
D3. United States v. Grubbs
547 U.S. 90 (2006)
FACTS: Jeffrey Grubbs purchased a videotape containing child
pornography from a Website operated by an undercover postal
inspector. Officers from the Postal Inspection Service arranged a
controlled delivery of a package containing the videotape to Grubbs
residence. A postal inspector submitted an "anticipatory" search
warrant application to a Magistrate Judge for the Eastern District of
California, accompanied by an affidavit describing the proposed
operation in detail, explaining that the warrant would be executed only
upon the receipt of the parcel by a person(s) and has been physically
taken into the residence (triggering condition). The warrant was issued.
Two days later, an undercover postal inspector delivered the package.
Grubbs wife signed for it and took the unopened package inside. The
inspectors detained Grubbs as he left his home a few minutes later,
then entered the house and commenced the search. Roughly 30
minutes into the search, Grubbs was provided with a copy of the
warrant, which included both attachments but not to supporting
affidavit that explained when the warrant would be executed. Grubbs
consented to interrogation by the postal inspectors and admitted
ordering the videotape. He was placed under arrest, and various items
were seized, including the videotape.
A grand jury for the Eastern District of California indicted Grubbs on
one count of receiving a visual depiction of a minor engaged in sexually
explicit conduct. Grubbs moved to suppress the evidence seized during
the search of his residence, arguing as relevant here that the warrant
was invalid because it failed to list the triggering condition. The district
court denied the motion. The court of appeals for the ninth circuit
reversed; it held that the particularity requirement of the Fourth
Amendment applies with full force to the conditions precedent to an
anticipatory search warrant, because the postal inspectors failed to
present the affidavit the warrant was inoperative, and the search was
illegal.
ISSUE: Whether anticipatory search warrants are categorically
unconstitutional
DECISION: No. The judgment of the Court of Appeals is reversed, and
the case is remanded for further proceedings consistent with this
opinion
REASON: The Fourth Amendment does not set forth some general
particularity requirement, it specifies only two matters that must be
particularly described in the warrant: (1) the place to be search and (2)
the persons or things to be seized. The court defined an anticipatory
search warrant as a warrant based upon an affidavit showing probable
cause that at some future time (but not presently) certain evidence of
35 | P

LATON

crime will be located at a specified place. The court further held that
the probable-cause requirement looks to whether evidence will be
found when the search is conducted, all warrants are in a sense,
anticipatory. Anticipatory warrants are, therefore, no different in
principal from ordinary warrants. They require a magistrate to
determine (1) that it is now probable that (2) contraband, evidence of a
crime, or a fugitive will be on the described premises (3) when the
warrant is executed. It should be noted, however, that where the
anticipatory warrant places a condition (other than the mere passage of
time) upon its execution, the first of these determinations goes not
merely to what will probably be found if the condition is met. Rather,
the probability determination for a conditioned anticipatory warrant
looks also to the likelihood that the condition will occur, and thus that a
proper object of seizure will be on the described premises. Two prerequisites of probability must be satisfied (1) It must be true that if the
triggering condition occurs there is a fair probability that contraband
or evidence of a crime will be found in a particular place (2) there is
probable cause to believe that the triggering condition will occur.
D4. Los Angeles County v. Rettele
550 U.S. 90 (2006)
FACTS: Los Angeles County Sheriffs Department Deputy Dennis
Watters investigated a fraud and identity-theft crime ring. There were
four suspects of the investigation. The four suspects were known to be
African-Americans. Watters obtained a search warrant for two houses
in Lancaster, California, where he believed he could find the suspects.
The warrant authorized him to search the homes and three of the
suspects for documents and computer files. Watters briefed six other
deputies in preparation for the search of the houses. Watters informed
them they would be searching for three African-American suspects.
However, Watters did not know that one of the houses (the first to be
searched) had been sold to Max Rettele. He had purchased the home
and moved into it three months earlier with his girlfriend Judy Sadler
and Sadlers 17-year-old son Chase Hall. All three, respondents here,
are Caucasians.
The deputies announcement awoke Rettele and Sadler.The deputies
entered their bedroom with guns drawn and ordered them to get out
of their bed and to show their hands. They protested that they were
not wearing clothes. Rettele and Sadler were held at gunpoint for one
to two minutes before Rettele was allowed to retrieve a robe for
Sadler. He was then permitted to dress. By that time the deputies
realized they had made a mistake, they apologized to Rettele and
Sadler.
Rettele and Sadler, individually and as guardians ad litem for Hall, filed
this suit against Los Angeles County, the Los Angeles County Sheriffs
Department, Deputy Watters, and other members of the sheriffs
department. Respondents alleged petitioners violated their Fourth
Amendment rights by obtaining a warrant in reckless fashion and
conducting an unreasonable search and detention. The District Court
held that the warrant was obtained by proper procedures and the
search was reasonable. It concluded in the alternative that any Fourth
Amendment rights the deputies violated were not clearly established
and that, as a result, the deputies were entitled to qualified immunity.
On appeal respondents did not challenge the validity of the warrant;
they did argue that the deputies had conducted the search in an
unreasonable manner. The Court of Appeals concluded that the search
and detention were unnecessarily painful, degrading, or prolonged,
and involved an undue invasion of privacy. Turning to whether
respondents Fourth Amendment rights were clearly established, the
majority held that a reasonable deputy should have known the search
and detention were unlawful.
ISSUE: whether the act of the deputies constitute an unreasonable
manner of conducting a search.
DECISION: No. The court held that the search was reasonable under the
circumstances. The judgment of the Court of Appeals is reversed, and
the case is remanded for further proceedings consistent with this
opinion.

REASON: When the deputies ordered respondents from their bed, they
had no way of knowing whether the African-American suspects were
elsewhere in the house. The presence of some Caucasians in the
residence did not eliminate the possibility that the suspects lived there
as well. The deputies, who were searching a house where they believed
a suspect, might be armed, possessed authority to secure the premises
before deciding whether to continue with the search. In executing a
search warrant officers may take reasonable action to secure the
premises and to ensure their own safety and the efficacy of the search.
Unreasonable actions include the use of excessive force or restraints
that cause unnecessary pain or are imposed for a prolonged and
unnecessary period of time. The orders by the police to the occupants,
in the context of this lawful search, were permissible, and perhaps
necessary, to protect the safety of the deputies. Blankets and bedding
can conceal a weapon, and one of the suspects was known to own a
firearm, factors which underscore this point. The Constitution does not
require an officer to ignore the possibility that an armed suspect may
sleep with a weapon within reach. The deputies needed a moment to
secure the room and ensure that other persons were not close by or
did not present a danger. The Fourth Amendment allows warrants to
issue on probable cause, a standard well short of absolute certainty.
Valid warrants will issue to search the innocent, and people like Rettele
and Sadler unfortunately bear the cost. When officers execute a valid
warrant and act in a reasonable manner to protect themselves from
harm, however, the Fourth Amendment is not violated.
D5. Valeroso v. Ca
598 SCRA 41 (2009)
FACTS: For resolution is the Letter-Appeal of Senior Inspector Jerry
Valeroso praying that the Feb. 22, 2008 decision and June 30, 2008
resolution be set aside and a new one be entered acquitting him of the
crime of illegal possession of firearm and ammunition.
During trial there were two versions as to where Valeroso was arrested.
Prosecution claims that Valeroso was arrested near the INP central
Police Station in Culiat, Quezon City, while he was about to board a
tricycle; after placing him under arrest, the arresting officers bodily
searched him, and they found the subject firearms and ammunition. On
the other hand, the defense insists that he was sleeping inside a room
in the boarding house of his children in Quezon City and was awakened
by four heavily armed men in civilian attire who pointed their guns at
him and pulled him out of the room, tied his hands and placed him near
the faucet outside the room then went back inside, searched and
ransacked the room and forcibly opened a locked cabinet where they
discovered the subject firearm.
The RTC, branch 97, QC, convicted Valeroso as charged. On appeal, the
CA affirmed the RTC decision with modification to the penalty. On
petition for review, SC affirmed in full the CA decision. He then filed a
motion for reconsideration which was denied with finality on June 30,
2008. The present letter-appeal focused on his breached constitutional
rights against unreasonable search and seizure.
OSG filed a manifestation in lieu of comment recommending Valerosos
acquittal, considering the testimonies of the witnesses for the defense
more credible. The OSG agrees with Valeroso that the subject firearms
was obtained by the police officers in violation of his constitutional
right against illegal search and seizure, and should thus be excluded
from the evidence for the prosecution.
ISSUE: whether the warrantless search and seizure of the firearm and
ammunition valid.
DECISION: No. The Feb. 22, 2008 decision and June 30, 2008 resolution
are reconsidered and set aside. Sr. Insp. Jerry Valeroso is acquitted of
illegal possession of firearm and ammunition.
REASON: Must give more credence to the version of the defense.
Sec. 2 of Art. III of the Constitution , as a general rule, the procurement
of a warrant is required before a law enforcer can validly search or
36 | P

LATON

seize the person, house, papers, or effects of any individual.


Furthermore, Art. III, Sec. 3(2) states that any evidence obtained in
violation of this or the preceding section shall be inadmissible in
evidence for any purpose in any proceeding. However, this is not
absolute, there are exceptions or instances where searches and
seizures are allowed even without a valid warrant among these are: (1)
warrantless search incidental to a lawful arrest and (2) seizure of
evidence in plain view.

cocaine possession. On the other hand, the Arizona Supreme Court


concluded that the search of Gants car was unreasonable within the
meaning of the Fourth Amendment.

The Court explained that when an arrest is made, it is reasonable for


the arresting officer to search the person arrested in order to remove
any weapon that the latter might use in order to resist arrest or effect
his escape, or for the protection of the officer, as well as to prevent the
concealment or destruction of evidence on the suspects person. A valid
arrest allows the seizure of evidence or dangerous weapon either on
the person of the one arrested or within the area of his immediate
control, within which he might gain possession of weapon or
destructible evidence.
In the present case, the arresting officers served the warrant of arrest
without any resistance from Valeroso, his hands were tied and he was
placed outside the room, and also the cabinet, which was locked, could
no longer be considered as an area within his immediate control
because there was no way for him to take any weapon or to destroy
any evidence that could be used against him. The purpose of the
exception is to protect the arresting officer from being harmed by the
person arrested, based on the said circumstances, the search exceeded
the bounds of what may be considered as an incident to a lawful arrest.
The warrantless search in this case cannot also be justified under the
plain view doctrine because it may not be used to launch unbridled
searches and indiscriminate seizure or to extend a general exploratory
search made solely to find evidence of defendants guilt. The doctrine
usually applied where a police officer is not searching for evidence
against the accused, but nonetheless unintentionally comes across an
incriminating object. However, in this case, the police officers did not
just accidentally discover the subject firearm and ammunition; they
actually searched for evidence against Valeroso.
Clearly, the search made was illegal, a violation of Valerosos right
against unreasonable search and seizure. Consequently, the evidence
obtained in violation of said right is inadmissible in evidence against
him. Without the illegally seized firearm, Valerosos conviction cannot
stand.

DECISION: Yes. The judgment of the State Supreme Court is affirmed.

D6. Arizona v. Gant


556 U.S. ___ (2009)
FACTS: On August 25, 1999, acting on an anonymous tip that the
residence at 2524 North Walnut Avenue was being used to sell drugs,
Tucson police officers Griffith and Reed knocked on the front door and
asked to speak to the owner. Gant answered the door and, after
identifying himself, stated that he expected the owner to return later.
The officers left the residence and conducted a records check, which
revealed that Gants drivers license had been suspended and there was
an outstanding warrant for his arrest for driving with a suspended
license. When the officers returned to the house that evening, the
officers recognized his car as it entered the driveway Gant parked at
the end of the driveway, got out of his car, and shut the door. Griffith
immediately arrested Gant and handcuffed him. They locked Gant in
the back-seat of their patrol car, after which two officers searched his
car: One of them found a gun, and the other discovered a bag of
cocaine in the pocket of a jacket on the backseat. Gant was charged
with two offensespossession of a narcotic drug for sale and
possession of drug paraphernalia.
He moved to suppress the evidence seized from his car on the ground
that the warrantless search violated the Fourth Amendment's
prohibition of unreasonable searches and seizures. Among other things,
Gant argued that it did not authorize the search of his vehicle because
he posed no threat to the officers after he was handcuffed in the patrol
car and because he was arrested for a traffic offense for which no
evidence could be found in his vehicle. The trial court declined Gant's
request, stating that the search was a direct result of Gant's lawful
arrest and therefore an exception to the general Fourth Amendment
warrant requirement. The court convicted Gant on two counts of

ISSUE: whether the search conducted by police officers after


handcuffing the defendant and securing the scene a violation of the
Fourth Amendment's protection against unreasonable searches and
seizures.

REASON: Police may search a vehicle incident to a recent occupants


arrest only if the arrestee is within reaching distance of the passenger
compartment at the time of the search or it is reasonable to believe the
vehicle contains evidence of the offense of arrest. When these
justifications are absent, a search of an arrestees vehicle will be
unreasonable unless police obtain a warrant or show that another
exception to the warrant requirement applies. The Arizona Supreme
Court correctly held that this case involved an unreasonable search.
The court observed that the search-incident-to-arrest exception to the
warrant requirement is justified by interests in officer safety and
evidence preservation. In this case, the justifications no longer exist
because the scene is secure and the arrestee is handcuffed, secured in
the back of a patrol car, and under the supervision of an officer, the
court concluded, a warrantless search of the arrestees car cannot be
justified as necessary to protect the officers at the scene or prevent the
destruction of evidence. Accordingly, the court held that the search of
Gants car was unreasonable.
D7. Brigham city v. Stuart
547 U. S. 398 (2006)
Facts: Four police officers responded to a call regarding a loud party at
a residence. Upon arriving at the house, they heard shouting from
inside, and proceeded down the driveway to investigate. There, they
observed two juveniles drinking beer in the backyard. They entered the
backyard, and sawthrough a screen door and windowsan
altercation taking place in the kitchen of the home. Four adults were
attempting, with some difficulty, to restrain a juvenile. At this point, an
officer opened the screen door and announced the officers presence.
The officers subsequently arrested respondents and charged them with
contributing to the delinquency of a minor, disorderly conduct, and
intoxication. In the trial court, respondents filed a motion to suppress
all evidence obtained after the officers entered the home, arguing that
the warrantless entry violated the Fourth Amendment.
Issue: Whether police may enter a home without a warrant when they
have an objectively reasonable basis for believing that an occupant is
seriously injured or imminently threatened with such injury?
Held: It is a basic principle of Fourth Amendment law that searches
and seizures inside a home without a warrant are presumptively
unreasonable. Groh v. Ramirez, 540 U. S. 551, 559 (2004) (quoting
Payton v. New York, 445 U. S. 573, 586 (1980) (some internal quotation
marks omitted)). Nevertheless, because the ultimate touchstone of the
Fourth Amendment is reasonableness, the warrant requirement is
subject to certain exceptions. Flippo v. West Virginia, 528 U. S. 11, 13
(1999) (per curiam); Katz v. United States, 389 U. S. 347, 357 (1967).
The officers had an objectively reasonable basis for believing both that
the injured adult might need help and that the violence in the kitchen
was just beginning. Nothing in the Fourth Amendment required them
to wait until another blow rendered someone unconscious or semiconscious or worse before entering. The role of a peace officer
includes preventing violence and restoring order, not simply rendering
first aid to casualties; an officer is not like a boxing (or hockey) referee,
poised to stop a bout only if it becomes too one-sided.
The officers announcement of his presence was at least equivalent to a
knock on the screen door. Indeed, it was probably the only option that
had even a chance of rising above the din. Under these circumstances,
there was no violation of the Fourth Amendments knock-and37 | P

LATON

announce rule. Furthermore, once the announcement was made, the


officers were free to enter; it would serve no purpose to require them
to stand dumbly at the door awaiting a response while those within
brawled on, oblivious to their presence.

D8. Georgia v. Randolph


547 U.S. 103 (2006)
Facts: Respondent Scott Randolph and his wife, Janet, separated in late
May 2001, when she left the marital residence in Americus, Georgia,
and went to stay with her parents in Canada, taking their son and some
belongings. In July, she returned to the Americus house with the child,
though the record does not reveal whether her object was
reconciliation or retrieval of remaining possessions. On the morning of
July 6, she complained to the police that after a domestic dispute her
husband took their son away, and when officers reached the house she
told them that her husband was a cocaine user whose habit had caused
financial troubles. She mentioned the marital problems and said that
she and their son had only recently returned after a stay of several
weeks with her parents. Shortly after the police arrived, Scott Randolph
returned and explained that he had removed the child to a neighbor's
house out of concern that his wife might take the boy out of the
country again; he denied cocaine use, and countered that it was in fact
his wife who abused drugs and alcohol. One of the officers, Sergeant
Murray, went with Janet Randolph to reclaim the child, and when they
returned she not only renewed her complaints about her husband's
drug use, but also volunteered that there were items of drug
evidence in the house. Sergeant Murray asked Scott Randolph for
permission to search the house, which he unequivocally refused. The
sergeant turned to Janet Randolph for consent to search, which she
readily gave. She led the officer upstairs to a bedroom that she
identified as Scott's, where the sergeant noticed a section of a drinking
straw with a powdery residue he suspected was cocaine. He then left
the house to get an evidence bag from his car and to call the district
attorney's office, which instructed him to stop the search and apply for
a warrant. When Sergeant Murray returned to the house, Janet
Randolph withdrew her consent. The police took the straw to the police
station, along with the Randolphs. After getting a search warrant, they
returned to the house and seized further evidence of drug use, on the
basis of which Scott Randolph was indicted for possession of cocaine
.
Procedural Posture: Trial court denied motion to suppress ruling that
Janet Randolph had common authority to consent to the search. Court
of Appeals reversed.

individuals claim to security against the governments intrusion into his


dwelling place. However, society can have the benefit of these interests
without relying on a theory of consent that ignores inhabitants refusal
to allow a warrantless search. The cotenant acting on his own initiative
may be able to deliver evidence to the police, and can tell the police
what he knows, for use before a magistrate in getting a warrant. Lastly,
the court drew a fine line between other similar cases Matlock and
Rodriguez by stating if a potential defendant with self-interest in
objecting is in fact at the door and objects, the co-tenants permission
does not suffice for a reasonable search, whereas the potential
objector, nearby but not invited to take part in the threshold colloquy,
loses out.
Concurring: Justice Breyer who write a concurring opinion, stated that if
Fourth Amendment law forced us to choose between two bright-line
rules; (1) a rule that always found one tenants consent sufficient to
justify a search without a warrant and (2) a rule that never did, I believe
we should choose the first. A rule permitting such searches can serve
important law enforcement needs (for example, in domestic abuse
cases) and the consenting partys joint tenancy diminishes the objecting
partys reasonable expectation of privacy. Furthermore, he stressed
that the court should take a totality of the circumstances approach
because if the circumstances change significantly, so should the result.

D9. Dizon v. Lambino,


498 SCRA 233 (2006)
Facts: The killing during a rumble on December 8, 1994 of University of
the Philippines graduating student Dennis Venturina, the chairperson of
the UP College of Public Administration Student Council, drew the then
Chancellor of UP Diliman Roger Posadas to seek the assistance of the
National Bureau of Investigation (NBI).
As two student-suspects in the killing, Francis Carlo Taparan and
Raymundo Narag, were at the time in the office of Col. Bentai, Atty.
Marichu Lambino, Legal counsel of UP Diliman, who repaired to the
Office of Col. Bentain, advised against Atty. Dizons move, however, he
not being armed with a warrant for their arrest.
Chancellor Posadas and Vice Chancellor for students Rosarion TorresYu, who also repaired to the office of the colonel, joined Atty. Lambino
in opposing the turn-over of the suspects to Atty. Dizon, despite the
latters claim that under its Charter the NBI was authorized to make
warrantless arrests.

Issue(s): Whether such an evidentiary seizure is likewise lawful with the


permission of one occupant when the other, who later seeks to
suppress the evidence, is present at the scene and expressly refuses to
consent.

Issues: (1) Whether the attempted arrest of the student suspects by the
NBI could be validly made without a warrant; and (2) Whether there
was probable cause for prosecuting petitioner for violation of P.D. No.
1829.

Judgment/Disposition: Affirmed

Held: Respecting the complaint against Atty. Dizon, this court, also in
Posadas v. Ombudsman, held that [f]or the failure of the NBI agents to
comply with the constitutional and procedural requirements, their
attempt to arrest [the two student-suspects] without a warrant was
illegal.

Holding: A physically present co-occupants stated refusal to permit


entry prevails.
Reasoning: Justice Souter, who authored the majority opinion, began
by providing an analogy similar to this case by stating To begin with, it
is fair to say that a caller standing at the door of shared premises would
have no confidence that one occupants invitation was a sufficiently
good reason to enter when a fellow tenant stood there saying stay
out. Without some very good reason, no sensible person would go
inside under those conditions. Fear for the safety of the occupant
issuing the invitation, or of someone else inside, would be thought to
justify entry, but the justification then would be the personal risk, the
threats to life or limb, nit the disputed invitation. He further states
that there is no common understanding that one co-tenant generally
has a right or authority to prevail over the express wishes of another,
whether the issue is the color of the curtains or invitations to outsiders.
The court also applied the balancing test by stating that in the balancing
of competing individual and governmental interest entailed by the bar
to unreasonable searches, the cooperative occupants invitation adds
nothing to the governments side to counter the force of an objecting

The NBI Charter clearly qualifies the power to make arrests to be in


accordance with existing laws and rules.
Members of the investigation staff of the Bureau of Investigation shall
be peace officers, and as such have the following powers:
(a) To make arrests, searches and seizures in accordance
with existing laws and rules. xxxx(Emphasis supplied)
D10. Social Justice Society (SJS) v. Dangerous Drugs Board
570 SCRA 410 (2008)
Facts: In these kindred petitions, the constitutionality of Section 36 of
Republic Act No. (RA) 9165, otherwise known as the Comprehensive
Dangerous Drugs Act of 2002, insofar as it requires mandatory drug
testing of candidates for public office, students of secondary and
tertiary schools, officers and employees of public and private offices,
38 | P

LATON

and persons charged before the prosecutors office with certain


offenses, among other personalities, is put in issue.
The Commission on Elections issued Resolution No. 6486, prescribing
the rules and regulations on the mandatory drug testing of candidates
for public office in connection with the May 10, 2004 synchronized
national and local elections.
Petitioner Aquilino Q. Pimentel, Jr., a senator of the Republic and a
candidate for re-election in the May 10, 2004 elections. He seeks (1) to
nullify Sec. 36(g) of RA 9165 and COMELEC Resolution No. 6486 dated
December 23, 2003 for being unconstitutional in that they impose a
qualification for candidates for senators in addition to those already
provided for in the 1987 Constitution; and (2) to enjoin the COMELEC
from implementing Resolution No. 6486. Pimentel Invokes Sec. 3,
Article VI of the Constitution.
Petitioner Social Justice Society (SJS), a registered political party, seeks
to prohibit the Dangerous Drugs Board (DDB) and the Philippine Drug
Enforcement Agency (PDEA) from enforcing paragraphs (c),(d),(f), and
(g) of Sec. 36 of RA 9165 on the ground that they are constitutionally
infirm. For one, the provisions constitute undue delegation of
legislative power when they give unbridled discretion to schools and
employers to determine the manner of drug testing.
Petitioner Atty. Manuel J. Laserna, Jr., seeks in his petition that Sec. 36
(c),(d),(f), and (g) of RA 9165 be struck down as unconstitutional for
infringing on the constitutional right to privacy, the right against
unreasonable search and seizure, and the right against selfincrimination, and for being contrary to the due process and equal
protection guarantees.
Issues: (1) Do Sec. 36(g) of RA 9165 and COMELEC Resolution No. 6486
impose an additional qualification for candidates for senator?
Corollarily, can Congress enact a law prescribing qualifications for
candidates for senator in addition to those laid down by the
Constitution? And (2) Are paragraphs (c),(d),(f), and (g) of Sec. 36, RA
9165 unconstitutional? Specifically, do these paragraphs violate the
right to privacy, the right against unreasonable searches and seizure,
and the equal protection clause? Or do they constitute undue
delegation of legislative power?
Held: The congress cannot validly amend or otherwise modify these
qualification standards, as it cannot disregard, evade, or weaken the
force of a constitutional mandate, or alter or enlarge the Constitution.
Accordingly, Sec. 36(g) of RA 9165 should be declared unconstitutional.
The Constitution is the basic law to which all laws must conform; no act
shall be valid if it conflicts with the Constitution.
The provisions of RA 9165 requiring mandatory, random, and
suspicionless drug testing of students are constitutional. The right to
enroll is not absolute; it is subject to fair, reasonable, and equitable
requirements. The Court can take judicial notice of the proliferation of
prohibited drugs in the country that threatens the will-being of the
people, particularly the youth and school children who usually end-up
as victims. Accordingly, and until a more effective method is
conceptualized and put in motion, random drug testing of students in
secondary and tertiary schools is not only acceptable but may even be
necessary if the safety and interest of the student population,
doubtless a legitimate concern of the government, are to be promoted
and protected.
Just as in the case of secondary and tertiary level students, the
mandatory but random drug test prescribed by Sec. 36 of RA 9165 for
officers and employees of public and private offices is justifiable, albeit
not exactly for the same reason. Reasonableness is the touchstone of
the validity of a government search or intrusion. And whether a search
at issue hews to the reasonableness standard is judged by the balancing
of the government-mandated intrusion on the individuals privacy
interest against the promotion of some compelling interest. The
employees privacy interest in an office is to a large extent
circumscribed by the companys work policies, the collective bargaining
agreement, if any, entered into by management and the bargaining

unit, and the inherent right of the employer to maintain discipline and
efficiency in the workplace. Their privacy expectation in a regulated
office environment is, in fine, reduced; and a degree of impingement
upon such privacy has been upheld.
The reduced expectation of privacy on the part of the employees, the
compelling state concern likely to be met by the search, and the welldefined limits set forth in the law to properly guide authorities in the
conduct of the random testing, we hold that the challenged drug test
requirement is, under the limited context of the case, reasonable and,
ergo, constitutional.
The situation is entirely different in the case of persons charged before
the public prosecutors office with criminal offenses punishable with six
years and one day imprisonment. The operative concepts in the
mandatory drug testing are randomness and suspicionless. In the
case of persons charged with a crime before the prosecutors office, a
mandatory drug testing can never be random or suspicionless. To
impose mandatory drug testing on the accused is a blatant attempt to
harness a medical test as a tool for criminal prosecution, contrary to
the stated objectives of RA 9165. Drug testing in this case would violate
a persons right to privacy guaranteed under Sec. w, Art. III of the
Constitution. Worse still, the accused persons are veritably forced to
incriminate themselves.
Sec. 36(g) of RA 9165 and COMELEC Resolution No. 6486
UNCONSTITUTIONAL
Sec. 36(c) and (d) of RA 9165 - CONSTITUTIONAL but declaring its Sec.
36(f) UNCONSTITUTIONAL

Chapter 5
Privacy of Communications and Correspondence
The privacy of communication and correspondence shall be
inviolable except upon lawful order of the court, or when public
safety or order requires otherwise, as prescribed by law.
Any evidence obtained in violation of this or the preceding
section shall be inadmissible for any purpose in any
8
proceeding.
Part of the right to be let alone is necessarily the corresponding
freedom to communicate in confidence with some other
persons of ones choosing without the contents of that
communication being disclosed to others, especially the
government
Privacy of Communications and Searches
To ensure respect and observance of the guarantee, the
Constitution requires that there be a court order, or some
weighty, justifiable and substantial state interest, such as public
safety or order, before interference with the privacy of
communications and correspondence could be allowed
The guidelines are supposed to be set out in law. Finally, as
further deterrence, it mandates that any evidence obtained in
violation of its proscriptions shall be useless--inadmissible for
any proceeding
Katz v. US
389 US 347, 19 L Ed 2d 576, 88 S Ct 507 (1967)
8

CONSTITUTION, Art. III, 3(1) and (2)

39 | P

LATON

Petitioner was convicted under an indictment charging him with


transmitting wagering information by telephone across state lines-from LA to Miami and Boston--in violation of a federal statute. Evidence
of petitioners end of the conversations, overheard by FBI agents who
had attached an electronic listening and recording device to the
outside of the telephone booth from which the calls were made, was
introduced and admitted at the trial
Held: No. The Governments activities in electronically listening and
recording the petitioners words violated the privacy upon which he
justifiably relied while using the telephone booth, and thus constituted
a search and seizure within the meaning of the Fourth Amendment
Wherever a man may be, he is entitled to know that he will remain free
from unreasonable searches and seizures
The Fourth Amendment protects people, not places
There is a twofold requirement:
1. That a person have exhibited an actual )subjective) expectation of
privacy
2. That the expectation be one that society is prepared to recognize as
reasonable
the point is not that the booth is accessible to the public at other
times, but that it is a temporarily private place whose momentary
occupants expectations of freedom from intrusion are recognized as
reasonable

The aforestated provision clearly and unequivocally makes it illegal for


any person, not authorized by all the parties to any private
communication to secretly record such communication by means of a
tape recorder. The law makes no distinction as to whether the party
sought to be penalized by the statute ought to be a party other than or
different from those involved in the private communication

The word communicate comes from the latin word


communicare, meaning to share or to impart. In its ordinary
signification, communication connotes the act of sharing or
imparting, as in a conversation, or signifies the process by
which meanings or thoughts are shared between individuals
through a common system of symbols (as language signs or
gestures) these definitions are broad enough to include verbal
or non-verbal, written or expressive communications of
meanings or thoughts which are likely to include the
emotionally-charged exchange
What the law prohibits is the overhearing, intercepting, or
recording of private communications. Thus, a tape recording of
an altercation or verbal exchange between a policeman and a
radio reporter at a police station is admissible even if said
recording was done without the knowledge of the participants.
And, as noted in Ramirez, the use of a telephone extension for
the purpose of overhearing does not violate RA 4200

Zulueta v. CA
253 SCRA 699 (1996)

Salcedo-Ortaez v. CA
235 SCRA 111 (1994)
Private respondent Rafael S. Ortaez filed with the RTC of QC a
complaint for annulment of marriage with damages against petitioner
Teresita Salcedo-Ortaez on grounds of lack of marriage license and/or
psychological incapacity of the petitioner. Among the exhibits offered
by private respondent were three (3) cassette tapes of alleged
telephone conversations between petitioner and unidentified persons
The trial court issued the assailed order admitting all of the evidence
offered by private respondent, including tape recordings of telephone
conversations of petitioner with unidentified persons. these tape
recordings were made and obtained when private respondent allowed
his friends from the military to wire tap his home telephone
Held: No. RA 4200 entitled An Act to Prohibit and Penalize WireTapping and Other Related Violations of the Privacy of Communication,
and for other purposes expressly makes such tape recordings
inadmissible in evidence
Clearly, respondents trial court and Court of Appeals failed to consider
the afore-quoted provisions of the law in admitting in evidence the
cassette tapes in question. Absent a clear showing that both parties to
the telephone conversations allowed the recording of the same, the
inadmissibility of the subject tapes is mandatory under RA 4200
The subject cassette tapes are declared inadmissible in evidence
Ramirez v. CA
248 SCRA 590 (1995)
A civil case for damages was filed by petitioner Socorro D. Ramirez in
the RTC of QC alleging that the private respondent, Ester S. Garcia, in a
confrontation in the latters office, allegedly vexed, insulted and
humiliated her in a hostile and furious mood and in a manner
offensive to petitioners dignity and personality, contrary to morals,
good customs and public policy--Garcia essentially belittled Ramirezs
intelligence, and attributing her employment to Garcias help. In
support of her claim, petitioner produced a verbatim transcript of the
event and sought moral damages, attorneys fees and other expenses
of litigation. the transcript on which the civil case was based was culled
from a tape recording of the confrontation made by petitioner

Petitioner Cecilia Zulueta, wife of private respondent Alfredo Martin,


entered the clinic of her doctor husband, a doctor of medicine, and in
the presence of her mother, a driver and private respondents
secretary, forcibly opened the drawers and cabinet in her husbands
clinic and took 157 documents consisting of private correspondence
between Dr. Martin and his alleged paramours, i.e. greeting cards,
cancelled checks, diaries, Dr. Martins passport, and photographs. The
documents and papers were seized for use in evidence in a case for
legal separation and for disqualification from the practice of medicine
which petitioner had filed against her husband
Held: No. The only exception to the prohibition in the Constitution is if
there is a lawful order [from a] court or when public safety or order
requires otherwise, as prescribed by law. Any violation of this
provision renders the evidence obtained inadmissible for any purpose
in any proceeding

Warrant Requirement
Consistent with Section 2 of Article III, searches and seizures,
whether of tangible or intangible things, must be authorized by
a warrant supported by probable cause and with a particularity
of description of what is sought to be searched or seized
Accordingly, what might only be possible is a reasonable
description of the persons whose communication is sought to
be intercepted, identification of the crime that might be
committed by means of such communication, as well as a
delimitation of the period of the allowable search and seizure
Additional Cases
(E) Privacy of Communications and Correspondence
E1. City of Ontario, California, et al. v. Quon et al
560 U. S. ____ (2010)
Petitioner acquired alphanumeric pagers able to send and receive text
messages. Its contract with its service provider, Arch Wireless, provided
40 | P

LATON

for a monthly limit on the number of characters each pager could send
or receive, and specified that usage exceeding that number would
result in an additional fee. The City issued the pagers to respondent
Quon and other officers in its police department (OPD). When Quon
and others exceeded their monthly character limits for several months
running, petitioner Scharf, OPDs chief, sought to determine whether
the existing limit was too low, i.e., whether the officers had to pay fees
for sending work-related messages or, conversely, whether the
overages were for personal messages. After Arch Wireless provided
transcripts of Quons and another employees August and September
2002 text messages, it was discovered that many of Quons messages
were not work related, and some were sexually explicit. McMahons
report noted that Quon sent or received 456 messages during work
hours in the month of August 2002, of which no more than 57 were
work related; he sent as many as 80 messages during a single day at
work; and on an average workday, Quon sent or received 28 messages,
of which only 3 were related to police business. The report concluded
that Quon had violated OPD rules. Quon was allegedly disciplined.
The employee contends that the privacy of the messages is protected
by the ban on unreasonable searches and seizures found in the
Fourth Amendment to the United States Constitution
HELD: No. [O]ffices of government employees...are [generally] covered
by Fourth Amendment protections, but government searches to
retrieve work-related materials or to investigate violations of workplace
rulessearches of the sort that are regarded as reasonable and normal
in the private-employer contextdo not violate the...Amendment

[T]he correct analysis has two steps: First, because some


[government] offices may be so open...that no expectation of
privacy is reasonable, a court must consider [t]he operational
realities of the workplace to determine if an employees
constitutional rights are implicated. Second, where an
employee has a legitimate privacy expectation, an employers
intrusion on that expectation for non investigatory, workrelated purposes, as well as for investigations of work-related
misconduct, should be judged by the standard of
reasonableness under all the circumstances.
E1. Ontario vs Quon
560 U.S. ___ (2010)
Petitioner Ontario (hereinafter City) acquired alphanumeric pagers able
to send and receive text messages. Its contract with its service
provider, Arch Wireless, provided for a monthly limit on the number of
characters each pager could send or receive, and specified that usage
exceeding that number would result in an additional fee. The City
issued the pagers to respondent Quon and other officers in its police
department (OPD), also a petitioner here. When Quon and others
exceeded their monthly character limits for several months running,
petitioner Scharf, OPDs chief, sought to determine whether the
existing limit was too low, i.e., whether the officers had to pay fees for
sending work-related messages or, conversely, whether the overages
were for personal messages. After Arch Wireless provided transcripts
of Quons and another employees August and September 2002 text
messages, it was discovered that many of Quons messages were not
work related, and some were sexually explicit. Scharf referred the
matter to OPDs internal affairs division. The investigating officer used
Quons work schedule to redact from his transcript any messages he
sent while off duty, but the transcript showed that few of his on-duty
messages related to police business. Quon was disciplined for violating
OPD rules.
The employee contends that the privacy of the messages is protected
by the ban on unreasonable searches and seizures found in the
Fourth Amendment to the United States Constitution, made applicable
to the States by the Due Process Clause of the Fourteenth
Amendment. He and the other respondents each of whom had
exchanged text messages with Quon during August and September
filed this suit, alleging, inter alia, that petitioners violated their Fourth
Amendment rights and the federal Stored Communications Act (SCA) by
obtaining and reviewing the transcript of Quons pager messages, and

that Arch Wireless violated the SCA by giving the City the transcript.
Whether the audit was nonetheless reasonable, the court concluded,
turned on. Whether Scharf used it for the improper purpose of
determining if Quon was using his pager to waste time, or for the
legitimate purpose of determining the efficacy of existing character
limits to ensure that officers were not paying hidden work-related
costs. After the jury concluded that Scharfs intent was legitimate, the
court granted petitioners summary judgment on the ground they did
not violate the Fourth Amendment. The Ninth Circuit reversed.
Although it agreed that Quon had a reasonable expectation of privacy
in his text messages, the appeals court concluded that the search was
not reasonable even though it was conducted on a legitimate, workrelated rationale. The opinion pointed to a host of means less intrusive
than the audit that Scharf could have used. The court further
concluded that Arch Wireless had violated the SCA by giving the City
the transcript.

Chapter 6
Freedom of Expression and Assembly
No law shall be passed abridging the freedom of speech, of
expression, or of the press, or the right of the people peaceably
to assemble and petition the government for redress of
9
grievances.
Among the most cherished liberties in a free society, where
freedom of thought and conscience is a bedrock principle, one
that occupies a preferred and predominant status, is the right
to freely speak ones mind
Freedom of expression is the matrix, the indispensable
condition of nearly every freedom. The guarantee of the
freedom of speech has been defined as the instrument and
guarantee and the bright and consummate flower of all liberty
[I]t was made part of the First Amendment to the American
Constitution
Under this guarantee, the people are to determine their own
direction and chart their own destiny through the free
exchange of ideas and not through dictation from or coercion
of the government or anybody elses
The theory of freedom of expression involves more than a
technique for arriving at better social judgments through
democratic procedures. It comprehends a vision of society, a
faith and a whole way of life
Scope of Guarantee -- Prior Restraint, Subsequent Punishment
and Damages
The Freedom of Expression Clause is basically directed against
prior restraint or censorship and subsequent punishment
[I]t means that the people are kept free from any undue
interference from the government in their thoughts and words
[T]he people should be allowed to see and discuss for
themselves what is best for them
The First Amendments guarantee of the freedom of speech,
or of the press prohibits a wide assortment of government
restraints upon expression, but the core abuse against which it
was directed was the scheme of licensing laws implemented by

CONSTITUTION, Art. III, 4

41 | P

LATON

the monarch and Parliament to contain the evils of the


th
th
printing press in the 16 - and 17 -century England
It punished the publication of any book or pamphlet without a
license and required that all works be submitted for approval to
a government official, who wielded broad authority to suppress
works that he found to be heretical seditious, schismatical, or
offensive
The freedom has also expanded its coverage through the years.
While it may have been primarily meant to assure the right to
speak ones mind on matters affecting government affairs and
politics, it has gradually encompass expressions which are of
private and commercial concerns
The guarantee has also come to ensure that claims for damages
arising from the utilization of the freedom be not so
unreasonable or exorbitant as to practically still or chill its
exercise
[T]he right of free speech is not absolute at all times and under
all circumstances
These include the lewd and obscene, the profane, the libelous,
and the insulting or fighting words--those which, by their very
utterance, inflict injury or tend to incite an immediate breach of
the peace
The First Amendment protects the press from governmental
interference; it confers no analogous protection on the
Government
The Chilling Effect Principle -- The Danger of Self-Censorship
If the freedom to speak is not adequately shielded from
overbearing and misdirected zeal to check or restrict it, people
who might otherwise be minded to say something would rather
keep quiet than risk the danger of being prosecuted or
otherwise subjected to disagreeable and unwelcome
consequences
This form of censorship is likewise something that the Free
Speech Clause seeks to prevent, for in having people keep quiet
out of their own self-induced fears society suffers just as much
as when their mouths are kept shut by the authorities
The chilling effect need not emanate only from threats coming
from the government itself. It may also be effected by means of
the use of libel laws that may directly enforce silence through
the threat of financial ruin brought about by claims for
damages or prosecution under criminal laws for defamation
Facial Challenges and the Overbreadth Doctrine
The overbreadth doctrine permits a party to challenge the
validity of a statute even though as applied to him it is not
unconstitutional but it might be if applied to others not before
the Court whose activities are constitutionally protected
Accordingly, considering the preferred and paramount position
of the freedom of speech and of the press, such extraordinary
safeguards as the facial challenge and the use of the
overbreadth doctrine are allowed to be engaged in whenever
speech is threatened

General Considerations
To properly understand the value of the freedom of speech, of
the press and of expression, it would be best to consider the
background, the history and the circumstances which called
forth such guarantee
Near v. Minnesota
283 US 697, 75 L Ed 2d 1357, 51 S Ct 625 (1931)
A Minnesota statute declared that one who engages "in the business of
regularly and customarily producing, publishing," etc., "a malicious,
scandalous and defamatory newspaper, magazine or other periodical,"
is guilty of a nuisance, and authorizes suits, in the name of the State, in
which such periodicals may be abated and their publishers enjoined
from future violations. In such a suit, malice may be inferred from the
fact of publication. The defendant is permitted to prove, as a defense,
that his publications were true and published "with good motives and
for justifiable ends." Disobedience of an injunction is punishable as a
contempt.
Under said statute, the County Attorney of Hennepin County brought
action to enjoin the publication of what was describe as a "malicious,
scandalous and defamatory newspaper, magazine and periodical"
known as "The Saturday Press," published by the defendants in the city
of Minneapolis.
Held: Yes. The statute is not directed at threatened libel, but at an
existing business which, generally speaking, involves more than libel"
The object of the statute is not punishment, in ordinary sense, but
suppression of the offending newspaper or periodical
The statute not only operates to suppress the offending newspaper or
periodical, but to put the publisher under an effective censorship (in a
way that, unless the owner or publisher is able and disposed to bring
competent evidence to satisfy the judge that the charges are true and
are published with good motives and for justifiable ends, his
newspaper or periodical is suppressed and further publication is made
punishable as a contempt)
For these reasons we hold the statute, so far as it authorized the
proceedings in this action under clause (b) of section one, to be an
infringement of the liberty of the press guaranteed by the Fourteenth
Amendment
New York Times Co v. US
403 US 713, 29 L Ed 2d 822, 91 S Ct 2140 (1971)
The United States brought these actions to enjoin publication in the
New York Times and in the Washington Post of the contents of a
classified study entitled "History of the US Decision-Making Process on
Viet Nam Policy"
Held: Any system of prior restraints of expression comes to this Court
bearing a heavy presumption against its constitutional validity.
J. Black & J. Douglas: The amendments were offered to curtail and
restrict the general powers granted to the Executive, Legislative and
the Judicial Branches two years before in the original Constitution.
In the case at bar, we are asked to hold that, despite the First
Amendment's emphatic command, the Executive Brach, the Congress,
and the Judiciary can make laws enjoining publication of current news
and abridging freedom of the press in the name of "national security"
The dominant purpose of the First Amendment was to prohibit the
widespread practice of governmental suppression of embarrassing
information.
Babst v. National Intelligence Board
132 SCRA 316 (1984)
42 | P

LATON

The petitioners are journalists and columnists. On different dates in July


1980, they were summoned by military authorities for interrogation
regarding their work, feelings, sentiments, beliefs, associations and
even private lives. In addition, one of them was charged with libel by a
General who sought to recover P10 million in damages. They brought
an action for prohibition to stop the NIB from questioning them and
from filing libel suits on matters that had been the subject of inquiry by
the NIB.
The petition has become moot and academic. Be that as it may, it is not
idle to note that, while ordinarily, an invitation to attend a hearing and
answer some questions is not illegal or constitutionally objectionable,
under certain circumstances, however, such an invitation can easily
assume a different appearance as when it comes from a powerful
group composed predominantly of ranking military officers and the
designate interrogation site is a military camp.

Forms and Variations of the Freedom and Relativity of State


Regulation
Depending on the form and medium in which speech is being
exercised, to that extent may it also affect the extent of
governmental power expended
It is also true that speech has its own hierarchy, that is, some
specie of speech are given more weight and importance, and
thus a greater extent of protection, compared to others. One
could not equate, for instance, the right to speak on matters
affecting public matters with the right to comment on some
private concerns
Eastern Broadcasting Corporation v. Dans, Jr.
137 SCRA 628 (1985)
The SC held that radio broadcast also enjoys the protection of the
freedom of expression. If close down, the owner enjoys the rights to
due process according to the standards set in Ang Tibay v. CIR.
But radio deserves greater regulation than newspapers because it could
invade the privacy of everyone for no fee, and it is such that one is
likely to listen to what is being said.
The petitioner filed this action to compel respondent government
officials to allow the reopening of Radio Station DYRE after it had been
closed for allegedly having been used to incite the people to sedition.
The petitioner contended that it was denied due process because no
hearing was held and no proof was submitted to establish a factual
basis for the closure. However, before the Court could promulgate its
decision the petitioner filed a motion to withdraw its action on the
ground that it had sold the radio station to Manuel Pastrana and that
the National telecommunications Commission had expressed its
willingness to grant the requisite license.
The case has been moot and academic. However, for the guidance of
the inferior courts and administrative bodies, the following guidelines
must be observed: The cardinal primary requirements in administrative
proceedings as laid down in Ang Tibay v. CIR should be followed before
a broadcast station may be closed; All forms of communication are
entitled to the broad protection of the freedom of expression clause.
Necessarily, the freedom of television and radio broadcasting is
somewhat lesser in scope than the freedom accorded to newspapers
and print media. This limitation derives from the fact the broadcast
media have a uniquely pervasive presence in the lives of all Filipinos;
The government has a right to be protected against broadcasts which
incite listeners to violently overthrow it; and Broadcast stations deserve
the special protection given to all forms of media by the due process
and freedom of expression clauses of the Constitution.

Standards for Regulations and Restrictions

Just like any other liberty, the freedom of expression is not


absolute
There are three basic standards--the dangerous tendency
rule, the clear and present danger test and the balancingof-interest test. Of the three, he clear and present danger test
is the most liberal and latitudinarian
Under the dangerous tendency rule, if the words uttered
create a dangerous tendency which the state has a right to
prevent, ten such words are punishable. It is not necessary that
some definite or immediate acts of force, violence, or
unlawfulness be advocated. It is sufficient that such acts be
advocated in general terms. Nor it is necessary that the
language used be reasonably calculated to incite persons to
acts of force, violence, or unlawfulness. It is sufficient if the
natural tendency and probable effect of the utterance be to
bring about the substantive evil which the legislative body
seeks to prevent
The clear and present danger test posits that the evil
consequence of the comment or utterance must be extremely
serious and the degree of imminence extremely high before
the utterance can be punished. The danger to be guarded
against is the substantive evil sought to be prevented
The question in every case is whether the words used are used
in such circumstances and are of such a nature as o create a
clear and present danger that they will bring about the
substantive evils that Congress has a right to prevent. It is a
question of proximity and degree
This [clear and present danger] test then as a limitation on
freedom of expression is justified by the danger or evil of a
substantive character that the state has a right to prevent.
Unlike the dangerous tendency doctrine, the danger must not
only be clear but also present. The term clear seems to point to
a causal connection with the danger of the substantive evil
arising from the utterance questioned. Present refers to the
time element. It used to be identified with imminent and
immediate danger. The danger must not only be probable but
very likely inevitable
The essential difference between the two doctrines related to
the degree of proximity of the apprehended danger which
justified the restriction upon speech
Dangerous Tendency
Doctrine
Permits the restrictions once
a
rational
connection
between
the
speech
restrained and the danger
apprehended--the
tendency of one to create
the other--was shown

Clear and Present Danger Rule


Requires the Government to
defer application of restrictions
until the apprehended danger
was much more visible, until its
realization was imminent and
nigh at hand

The latter rule was thus considerably more permissive of


speech than the former, in context for the testing of which they
were originally designed
The third test, the balancing-of-interests test, the crucial
question is: how much deference should be given to the
legislative judgment?
43 | P

LATON

Factors in ascertaining the point or line of equilibrium:


1. The social value and importance of the specific aspect of the
particular freedom restricted by the legislation
2. The specific thrust of the restriction, i.e., whether the
restriction is direct or indirect, whether or not the persons
affected are few
3. The value and importance of the public interest sought to be
secured by the legislation--the reference here is to the nature
and gravity of the evil which the Congress seeks to prevent
4. Whether the specific restrictions decreed by the Congress is
reasonably appropriate and necessary for the protection of
such public interest
5. Whether the necessary safeguarding of the public interest
involved may be achieved by some other measures less
restrictive of the protected freedom
Relevant also to any discussion of the balancing test would be
the so-called OBrien test, as well as the time, place and
manner restrictions
Under the OBrien test, a government regulation is sufficiently
justified if:
1. It is within the constitutional power of the Government
2. It furthers an important or substantial government interest
3. The governmental interest is unrelated to the suppression of
free expression
4. The incident restriction on expression is no greater than is
essential to the furtherance of the interest
Under the reasonable time, place and manner rule, an
expression, whether oral or written or symbolized by conduct,
is subject to reasonable time, place or manner restrictions.
Restrictions of this kind are valid, provided that they are:
1. Justified without reference o the content of the regulated
speech
2. They are narrowly tailored to serve a significant
governmental interest
3. They leave open ample alternative channels for
communication of the information
And then again, one must take note of the differences between
a law or regulation, on one hand, and court injunction, on the
other
An injunction, by its nature, applies only to a particular group
(or individuals) and regulates the activities, and perhaps the
speech, of that group. It does so, however, because of the
groups past actions in the context of a specific dispute
between real parties
Ordinances represent a legislative choice regarding the
promotion of particular societal interests. Injunctions, by
contrast, are remedies imposed for violations (or threatened
violations) of a legislative or judicial decree
Injunctions also carry greater risks of censorship and
discriminatory application than do general ordinances

The defendants were indicted in three counts. The first charges a


conspiracy to violet the Espionage Act by causing and attempting to
cause insubordination, in the military and naval forces of the United
States, and to obstruct the recruiting and enlistment service of the
United States. The second count alleges a conspiracy to commit an
offence against the United States. The third count charges an unlawful
use of the mails for the transmission of the same matter. The
defendants were found guilty on all the counts.
But the character of every act depends upon the circumstances in
which it is done.
The question in every case is whether the words used are used in such
circumstances and are of such a nature as to create a clear and present
danger that they will bring about the substantive evils that Congress
has a right to prevent. It is a question of proximity and degree.
If the act (speaking, or circulating a paper), its tendency, and the intent
with which it is done are the same, we perceive no ground for saying
that success alone warrants making the act a crime. (Goldman v. United
States, 245 U.S. 474, 477). Indeed, that case might be said to dispose of
the present contention if the precedent covers all media concludendi.

Content-Based and Content-Neutral Regulations


Content-based are those which either approve or disapprove
based on contents of expression, such as favoring or disfavoring
some topics
Content-based prohibitions, enforced by severe criminal
penalties, have the constant potential to be a repressive force
in the lives and thoughts of a free people
Consistent with the idea that the government is not supposed to be
dictating to the people what to think or believe in, what to read or see
or hear, regulations which are content based-those which either
approve or disapprove based on the contents of the expression, are
subjected to strict scrutiny. To justify them by the presence of a
compelling state interest and a showing of an absence of any other
means which the state objective could be attained.
Content based-prohibitions, have the constant potential to be a
repressive force in the lives and thoughts of the free people. The
government through its public libraries, public television and public
funding for the arts, has broad discretion to make content-based
judgments in deciding what private speech to make available to the
public.
R.A.V. v. City of St. Paul
In the predawn hours one day in June 1990, petitioner R.A.V. and
several other teenagers allegedly assembled a crudely made cross by
taping together broken chair legs which they then burned inside the
fenced yard of a black family that lived across the street from the house
where petitioner was staying. Respondent city chose to charge
petitioner was the St. Paul Bias- Motivated Crime Ordinance, which
prohibits the display of a symbol which one knows or has reason to
know arouses anger, alarm or resentment in others on the basis of
race, color, creed, religion or gender. The trial court dismissed this
charge on the ground that the ordinance was substantially overbroad
and impermissibly content-based, but the State Supreme Court
reversed.

Injunctions, of course, have some advantages over generally


applicable statutes in that they can be tailored by a trial judge
to afford more precise relief than a statute where a violation of
the law has already occurred

The court also concluded that the ordinance was not impermissibly
content-based, because it was narrowly tailored to serve a compelling
governmental interest in protecting the community against the biasmotivated threats to public safety and order.

Schenck v. United States


39 S Ct 247 (1919)

Let there be no mistake about our belief that burning across in


someones front yard is reprehensible. But St. Paul has sufficient means
44 | P

LATON

at its disposal to prevent such behavior without adding the First


Amendment to the fire.

apply to public figures who sued in libel on the basis of alleged


defamatory falsehoods.

The judgment of the Minnesota Supreme Court is reversed, and the


case is remanded for proceedings not inconsistent with this opinion.

If a matter is a subject of public or general interest, it cannot suddenly


become less so merely because a private individual is involved or
because in some sense, the individual did not voluntarily choose to
become involved. The publics primary interest is in the event, the
public focus is on the conduct of the participant and the content, effect
and significance of the conduct, not the participants prior anonymity or
notoriety. The present case illustrates the point.

Libel and the Deliberate Falsehood


As noted earlier, one of the exceptions to the freedom of
expression would be libel or defamation. Freedom to speak
does not include the knowing lie or falsehood
At the same time, he freedom also means that one may not be
accurate, pr may even be wrong, in the presentation of facts, or
might be guilty of a lot of exaggerations and overstatements in
the narration of events and other circumstances, but these
would not be enough to take them out of the mantle of the
protection accorded by the guarantee
New York Times Co. v. Sullivan
376 US 254, 11 L Ed 2d 686, 84 S Ct 710 (1964)
ACTUAL MALICE TEST
Particularly applicable to public officials and public figures. Under this
test, it is not enough that what might have been published is false.
There must be an element of malice in it. Namely, that the one who
publish it knew very well that it was false or he did it with other
disregard of whether it was true or false.
With regard to people who are not public officials but considered as
public figures, they are also open to criticisms and comments
Rosenbloom v. Metromedia
403 US 29, L ED 2d 296, 91 S Ct 1811 (1971)
FACTS: Petitioner was a distributor of nudist magazines in the
Philadelphia metropolitan area. During the fall of that year, in response
of citizen complaints, the Philadelphia Police arrested several
newsstand operators, including the petitioner, in charge of selling
obscene material. Three days later, the police obtained a warrant to
search petitioners home and the rented barn he used as a warehouse
and seized the inventory of magazines and books found there. Upon
learning of the seizure, the petitioner who had been released on bail
after his first arrest, surrendered to the police and was arrested for a
second time. Following the second arrest, the police informed the
respondents radio station WIP and another local radio station, a wire
service and a local newspaper of the raid on the petitioners home and
of his arrest. WIP reported news stories of the petitioners arrest for
possession of obscene literature and of the police seizure of the
obscene books. It did not mention petitioners name but used the
terms smut literature racket and girlie book peddlers. Following
petitioners acquittal of criminal obscenity charges, he filed action in
District Court seeking damages under Pennsylvanias libel law. The jury
found for petitioner but CA reversed the decision holding that the fact
that plaintiff was not a public figure cannot be accorded decisive
significance. The issue here is whether, because he is not a public
official or a public figure, but a private individual, those limits
required that he prove that the falsehoods resulted from a failure of
the respondent to exercise reasonable care, or required that he prove
that the falsehoods were broadcast with knowledge of their falsity or
with reckless disregard of whether they were false or not.
HELD: In the series of cases beginning with New York Times Co v.
Sullivan, 376 US 254 (964), the Court has considered the limitations
upon state libel laws imposed by the constitutional guarantees of
freedom of speech and of the press. It held that in a civil libel action by
a pubic official against a newspaper those guarantees required clear
and convincing proof that a defamatory falsehood alleged as libel was
uttered with knowledge that it was false or with reckless disregard of
whether it was false or not. The same requirement was later held to

Petitioners argument that the Constitution should be held to require


that the private individual prove only that the publisher failed to
exercise reasonable care in publishing defamatory falsehoods
proceeds along two lines. First, he argues that the private individual,
unlike the public figure, does not have access to the media to counter
the defamatory material and that the private individual, unlike the
public figure, has not assumed the risk of defamation by thrusting
himself in to the public arena. Second, petitioner focuses on the
important values served by the law of defamation in preventing and
redressing attacks upon reputation. We have recognized the force of
petitioners arguments.
Borjal v. CA
301 SCRA 1 (1999)
FACTS: During the congressional hearings on the transport crises in
1988, the attendees agreed to organize the First National Conference
on Land Transportation (FNCLT) to be participated by the private and
government sector in order to find ways to solve the transportation
crises. The estimated cost of conference estimated at P 1.8M would be
funded thru solicitations. During its first organizational meeting, private
respondent Francisco Wenceslao was elected as Executive Director and
he then undertook to solicit support for the conference from the
business sector. Between May and July 1989, a series of articles written
by petitioner Borjal was published in his column in the Philippine Star.
The articles dealt with the alleged anomalous activities of an organizer
of a conference without naming private respondent nor referring to
FNCLT. Private respondent reacted to the articles and refuted the
matters adverted to by Borjal. Thereafter a complaint was lodged by
private respondent with the National Press Club for unethical conduct.
This was followed by a criminal case for libel which was, however,
dismissed for insufficiency of evidence. A civil action for damages based
on libel was then filed against petitioners. Borjal and Soliven, publisher
and chairman of the editorial board of Philippines today, owner of
Philippine Star. The trial court decided in favor of private respondent,
which decision was affirmed by CA, although the amount of damages
was reduced. Hence, this petition.
HELD: The petition is impressed with merit. In order to maintain a libel
suit, it is essential that the victim be identifiable although it is not
necessary that he be named. It is also not sufficient that the offended
party recognized himself as the person attacked or defamed but it
must be shown that at least, a third person could identify him as the
object of the libelous publication. Regrettably, these requirements
have not been complied with in the case at bar. Identification is grossly
inadequate when even the alleged offended party is himself unsure
that he was the object of the verbal attack. It is well to note that the
revelation of the identity of the person alluded to came not from
petitioner Borjal but from the private respondent himself when he
supplied the information through his June 4 letter to the editor. Had
private respondent not revealed that he was the organizer of the FNCLT
referred to in the Borjal articles, the public would have remained in
blissful ignorance of his identity. It is therefore clear that on the
element of identifiability alone the case fails.
Indisputably, Borjals articles are neither private communications nor
fair and true report without any comments or remarks. However, this
does not necessarily mean that they are not privileged. To be sure, the
enumeration on Art 354 of the Revised penal code (on the requirement
for publicity of defamatory materials) is an exclusive list of qualifiedly
privileged communications since fair commentaries on matters of
public interest are likewise privileged. The rule on privileged
45 | P

LATON

communication had its genesis not in the nations penal code but in the
Bill of Rights of the Constitution guaranteeing freedom of speech and of
the press.
Borjal did not act with malice, we find the petitioner Borjal to have
acted in good faith. Moved by the civic duty and prodded by his
responsibility as a newspaperman, he proceeded to expose and
denounced what he perceived to be a public deception. Every citizen
has the right to enjoy a good name and reputation, but we do not
consider that petitioner Borjal has violated that right (of the
respondents) nor abused his press freedom. We must however take
this opportunity to likewise remind media practitioners of the high
ethical standards attached to and demanded by their noble profession.
Filipinas Broadcasting Network Inc (FBNI) v. Ago Medicaland
Educational Center-Bicol Christian College of Medicine (AMEC-BCCM)
448 SCRA 413 (2005)
FACTS: Mel Rima and Jun Alegre, host of radio program Expose aired
every morning over DZRC-AM owned by FBNI heard over Legaspi City.
Rima and Alegre exposed various alleged complaints from students,
teachers and parents against AMEC-BCCM. Alegre said among other
things that 1) AMEC students in Physical Theraphy complained that the
course is not recognized by DECS 2)students are required to take and
pay for he subject even if he subject does not have an instructor,
commenting such greed for money on the part of AMECs
administration 3) the administrators of AMEC BCCM, AMEC Science HS
and the AMEC Institute of Mass Communication in their effort to
minimize expenses in terms of salary are absorbing or continues to
reject making reference to many teachers in AMEC who were former
teachers of Aquinas University but were removed because of
Immorality. In offering to prove that AMEC is a dumping ground,
garbage, not merely of moral and physical misfits, mention was made
of the case of the Dean of Student Affairs of AMEC, Justita Lola, which
according to Alegre, as the family name implies she is too old to work,
being an old woman. On the other hand, Rima echoed Alegres
allegations about Dean Lola. AMEC filed a complaint for damages
claiming that the broadcasts were defamatory, against FBNI, Rima and
Alegre. In reply, they claimed that they were plainly impelled by a sense
of public duty to report the going on in AMEC, which is an institution
imbued with public interest. The trial court rendered a decision finding
FBNI and Alegre liable for libel and held that the broadcasts are libelous
per se. In absolving Rima from the charge, the trial court ruled that
Rimas only participation was when he agreed with Alegres expose.
The parties appealed to the CA, which affirmed the trial courts
judgment with modification, but made Rima solidarily liable with FBNI
and Alegre. Hence, this petition.
HELD: We deny the petition. There is no question that the broadcasts
were made public and imputed to AMEC defects or circumstances
tending to cause it dishonor, discredit and contempt. We do not agree
with FBNI contention that Rima and Alegre did not act with malicious
intent. Every defamatory imputation is presumed malicious. Rima and
Alegre failed to show adequately their good intention and justifiable
motive in airing the supposed gripes of the students.
Some US courts apply the privilege of neutral reportage in libel cases
involving matter so of public interest or public figures. Under this
privilege, a republisher who accurately and disinterestedly reports
certain defamatory statements made against public figures is shielded
from liability regardless of the republishers subjective awareness of the
truth or falsity of the accusation. Rima and Alegre cannot invoke the
privilege of neutral reportage because unfounded comments abound in
the broadcasts. Moreover, there is no existing controversy involving
AMEC when the broadcasts were made. The privilege of neutral
reportage applies where the defamed person is a public figure who is
involved in an existing controversy and a party to that controversy
makes the defamatory statement.
Had the comment been an expression of opinion based on established
facts, it is immaterial that the opinion happened to be mistaken, as long
as it might reasonably be inferred from the facts. However, the
comments of Rima and Alegre were not backed up by facts. Therefore,

the broadcasts are not privileged and remain libelous per se. FBNI is
solidarily liable to pay for damages arising from the libelous broadcasts.
An employer and employee are solidarily liable for defamatory
statement by the employee within the scope and course of his
employment, at least when the employer authorizes or ratifies the
defamation. Moreover, FBNI, as shown by circumstances (e.g. no clear
and convincing evidence shows that they underwent FBNIs regimented
process of application and their deficiencies in their KBP accreditation)
lacked the diligence in selecting and supervising Rima and Alegre,
Hence, FBNI is solidarily liable to pay damages together with Rima and
Alegre.
Flor v. People
454 SCRA 440 (2005)
FACTS: Petitioner Flor and Nick Ramos, managing editor and news
correspondent respectively of the Bicol forum, a local weekly
newspaper circulated in the Bicol Region were charged with libel, for
having published an allegedly defamatory news article regarding
financial irregularities involving then minister of the Presidential
Commission on Govt. Reorganization and concurrently Camarines Sur
Gov. Luis Villafuerte. The news article reported, inter alia that the
officials denial that he did not spend government money for his recent
trips to Japan and Israel failed to convince the people and that the
people knew that the trips were purely junket. It also stated that about
P700,000 was collected by way of cash advances by ranking provincial
officials, at the instance of the Governor and without resolution
approving its release, was allegedly used for the two trips. It also
reported that Villafuerte claimed that he spent his own money for the
trips. The petitioner admitted that he wrote the questioned news items
on the basis of a note given to him by a source whom he refused to
identify. Said source was allegedly connected with the Provincial
Treasurers office. He said that prior to writing the article, he went to
his source to ask some clarificatory questions and was given
authenticated records of the cash advances. Villafuerte claimed that no
one from Bicol Forum made any attempt to get his side of the story nor
confirm the veracity of the contents of the article from any source at
the provincial capitol. To him, the Bicol Forum seemed to be making a
mockery of his previous explanations regarding the cash advances and
his trips abroad and such a sweeping statement subjected him to public
ridicule and humiliation. The trial court rendered a decision convicting
both accused which CA affirmed.
HELD: The sole issue here is if the questioned news is libelous. We
reverse. Clearly when confronted with libel cases involving publication
which deal with public official and the discharge of their official
functions, this Court is not confined within the wordings of the libel
statute; rather, the case should likewise be examined under the
constitutional precept of the freedom of the press. A public official is
barred from recovering damages in cases involving defamations. His
entitlement, however, is limited to instances when the defamatory
statement was made with actual malice-that is with the knowledge
that it was false or with reckless disregard of whether it was false or
not.
Vasquez v. CA
314 SCRA 460 ( 1999)
FACTS: Petitioner Vasquez, a resident of Tondo Foreshore area,
together with other families went to see then NHA General manager
Lito Atienza regarding their complaint against their Bgy. Chairman,
Jaime Olmedo. After their meeting with Atienza and other NHA
officials, petitioner and his companions were interviewed by newspaper
reporters concerning their complaint. The following day, a news article
appeared in the newspaper Ang Tinig ng Masa saying that the families
of the Tondo Foreshore Area complained about their Bgy. Chairman
who, in connivance with some of the project managers of NA, managed
to get for themselves some 14 lots in the said area. It was also reported
that Olmedo was involved in illegal gambling and theft of fighting cocks.
Based on the article, Olmedo filed a complaint for libel against
petitioner alleging that the latters statements cast aspersions in him
and his damaged reputation. The trail court found petitioner guilty of
libel. The CA affirmed.
46 | P

LATON

HELD:The question for determination in this case is the liability for libel
of a citizen who denounces a bgy official for misconduct in office. SC
held that the decision of CA must be reversed. Petitioner contends that
what he said was true and was made with good motives and for
justifiable ends which the SC found merit.
To find a person guilty of libel under Art 353 of the Revised Penal Code,
the following elements must be proved: a) the allegation of a
discreditable act or condition concerning another, b) publication of
the charge, and c) identity of the person defamed, and d) existence of
malice. In this case, the first 3 elements are present. The question is
whether from the fact that the statements were defamatory, malice
can be presumed so that it was incumbent upon petitioner to
overcome such presumption. Under Art. 361 of the Revised Penal code,
if the defamatory statement is made against a public official with
respect to the discharge of his official duties and functions and the
truth of the allegation is shown, the accused will be entitled to an
acquittal even though he does not prove that the imputation was
published with good motives and for justifiable ends. In this case,
contrary to the findings of the lower court, petitioner was able to prove
the truth of his charges against the bgy official.
It was error for the lower court to hold that petitioner only tried to
prove that the complainant (bgy official) is guilty of the crimes alluded
to; Accused, however, has not proven that the complainant committed
the crimes. For that is not what petitioner said as reported in the Ang
Tinig ng Masa. The fact that charges had been filed against the bgy.
official, not the truth of such charges, was the issue.

Privacy, Expression and Damages


The question here is in determining to what extent such an
action for vindication of private rights or sense of self-esteem
and worth may be held consistent with the guarantees of the
liberty to speak ones mind and publicize what he may have
discovered or learned
Baas, Jr. v. CA
325 SCRA 259 (2000)
FACTS: In 1976 petitioner sold to Ayala Investment Corp. 28,265 sq.
mtrs. Located in Muntinlupa, for P 2.3M. The Deed of Sale provided
that upon signing of the contract, Ayala shall; pay 400K with the
balance of 1.8M++ was to be paid in 4 equal consecutive annual
installments, with 12% interest pa on the outstanding balance. The
periodic payment of 400K shall be payable starting on Feb 20 1977 and
every year thereafter or until Feb 20 1980. On the same day, however,
petitioner discounted the promissory note with AYALA, for its face
value of P 1.8M evidenced by a Deed of Assignment signed by the
petitioner and AYALA. The latter then issued 9 checks to petitioner, all
dated Feb. 20 1976, drawn against BPI with the uniform amount of
200K++. In his 1976 Income tax Return, petitioner reported the 400K++
initial payment as income from disposition of capital asset. In the
succeeding years, until 1979, petitioner reported a uniform income of
200K++ as gain from sale of capital asset. In is 1980 income tax amnesty
return, petitioner also reported the same amount of 200K++ as the
realized gain on disposition of capital asset for the year. In 1978, BIR
examined the books of petitioner for 1976. They discovered that he had
no outstanding receivable from the 1976 land sale, concluding that the
sale was cash and the entire profit should have been taxable in said
year since the income was wholly derived then instead of being spread
over 4 years. They assessed a tax deficiency of P 2.4M. After reviewing
the examiners report, Larin as Regional Director of Manila Region IVA
of BIR directed the revision of the report in order to consider the land
as capital asset, which resulted to the reduced tax deficiency of 900K++.
On Sept. 1980, petitioner acknowledged receipt of the BIR assessment
but insisted that the sale of his land to AYALA was on installment. In
1981, BIR charged the petitioner with tax evasion. It appeared in the
Evening News Express, Evening Post and Bulletin Today. All news items
mentioned petitioners false income tax return concerning the sale of
land to AYALA. Meanwhile on July 2 1981, petitioner availed of tax

amnesty under PD 1740 and 1840. In both, petitioner did not recognize
that his sale of land to AYALA was on cash basis. Reacting to the news
article, petitioner filed with the RTC an action for damages against BIR
for extortion and malicious publication of the BIRs tax audit report,
claiming that the filing of criminal complaints against him for violation
of tax laws were improper because he had already availed of 2 tax
amnesty decrees, The trial court decided in favor of the respondents
and the CA affirmed. Before the SC, petitioner questions the propriety
of awarding damages to Larin.

HELD: Moral damages may be recovered in cases involving acts referred


to in Art 21 of the Civil code. As a rule, a public official may not recover
damages for charges of falsehood related to his official conduct unless
he proves that the statement was made with actual malice.
We appreciate petitioners claim that he filed his 1972 return in good
faith and he had honestly believed that the law allowed him to declare
the sale of the land in installment. We can further grant that the
pertinent tax laws needed construction. The petitioner was offended by
the headlines alluding him as a tax evader is fully understandable,
however, all these, do not justify what amounted to a baseless
prosecution of respondent Larin. Petitioner presented no evidence to
prove Larin extorted money from him. He even admitted that he never
met nor talked to the respondent. On Larins instruction, the tax
assessment was reduced. Petitioner went on to file the extortion cases
against Larin in different fora and this is where actual malice could
attached on petitioners part. We are constrained to agree that there is
sufficient basis for the award of moral and exemplary damages in favor
of the respondent. He suffered anxiety and humiliation because of the
unfounded charges against him.
The award is in favor of a government official in connection with his
official function, it is with caution that we affirm granting moral
damages, for it might open the floodgates for government officials
counter claiming damages in suits filed against them in connection with
their functions. Moreover, we must be careful lest the amounts
awarded make citizens hesitate to expose corruption in the
government, for fear of lawsuits from vindictive government officials.
Hence, we reduce the moral and exemplary damages in this case.
Lopez vs. Court of Appeals
34 SCRA 116 (1970)
Sanitary inspector assigned to babuyan islands, Fidel Cruz , sent a
distress signal that there are series of killings in the island, however
when Philippine defense team got there, they found out that he only
wanted transportation to manila. He was called hoax of the year. But
Together with the article, The Manila Chronicle mistakenly published
the photograph of Fidel G. Cruz,former mayor of bulacan. The
publishing was corrected by the MC. Cruz filed for damages, He was
awarded P11,000 by CA.
SC: So long as it is done in good faith, newspapers have the legal right
to have and express opinions on legal questions. To deny them of that
right would infringe upon the freedom of the press. The newspapers
should be given leeway and tolerance to enable them to courageously
and effectively perform their important role in our democracy. They
should not be punished for honest mistakes and imperfect choice of
words if done in good faith. However in this case, there was no
pressure of a deadline to meet, no occasion to haste and they should
have imposed reasonable care. The correction promptly made would
reduce the damages awarded. The practice being, more likely to
reduce damages for libel than to increase them
BULLETIN PUBLISHING CORPORATION V. NOEL
167 SCRA 255 (1988)
Bulletin publishing corp published an article claiming Lanao politics
being dominated by big royal families and the only time one who was
not of any royal house became a leader was during the American era
when the late Amir Mindalano held some sway. Relatives of mindalano
filed a complaint for damages against petitioners for libel because
47 | P

LATON

mandalanos belong to royal houses and he did not live with an


American family.
SC: PETITION GRANTED. Libel has not been committed. Personal hurt or
embarrassment is not automatically equivalent to defamation. The law
against defamation protects ones interest in acquiring, retaining, and
enjoying a reputation as good as ones character and conduct
warrant in the community and it is to community standards- not
personal or family standards-that a court must refer in evaluating a
publication claimed to be defamatory. A newspaper should be free to
report on events and developments in which the Public has legitimate
interest, wherever they may take place within the nation and as well as
in the outside world, with minimum fear of being hauled to court so
long as newspapers keep within the standards of morality and civility
prevailing within the general community. Any other rule on defamation
in a national community like ours with many, diverse cultural, social
religious and other groupings is likely to produce an unwholesome
chilling effect upon the constitutional protected operations of the
press.
Ayer Productions PTY. LTD. VS Capulong
160 SCRA 861 (1988)
Petitioner proposed a motion picture to re enact the events that made
possible the EDSA revolution. Juan Ponce Enrile did not approve his and
his familys inclusion in the docu-drama. He was deleted from the
script. Thereafter, he still filed to enjoin production of the film without
his consent and that it constitutes violation of his right to privacy.
Petitioners claim they are exercising their freedom of speech and
expression.
The right of privacy like the right of free expression is not absolute.
Limited intrusion is permissible where the person is a public
figure(anyone who has arrived at a position where public attention is
focused upon him as a person) and the information is matters of public
character. The interest protected by the right of privacy is the right to
be free from unwarranted publicity, wrongful publicizing of private
affairs and activities outside the realm of legitimate public concern. 1.
Clear and present danger rule 2. Balancing of interest test are
limitations of freedom of speech and of the press.
Such public figures have lost to some extent their right to privacy. The
press had a privilege under the constitution to inform the public about
those who have became legitimate matters of public interest. The
privilege of enlightening the public was not limited to the dissemination
of news in the sense of current events. It extended to information or
education or even entertainment and amusement, by books, articles,
pics, films and broadcasts concerning interesting phases of human
activity in general, as well as the reproduction of the public scene in
newsreels and travelogues.
The film itself limits portraying only the participation of Enrile that are
directly related to the public facts of Edsa. Such can be carried out
without license from him. However, there must be no reckless
disregard of truth in depicting of Enrile, no representation of private
life.

Arts, Letters and Obscenity


The problem is in discerning to what extent it may be
considered art and allowed expression and display and where it
crosses over to obscenity, an area that falls beyond the
protection of freedom of expression

Collective bargaining negotiations between chief negotiator(bartnicki)


and president (kane) of union representing teachers were unlawfully
intercepted and recorded. Vopper, a radio commentator played it in his
show. Yocum, who delivered the tape to vopper, said that he found it in
his mailbox. The information was acquired by the respondents lawfully.
According to district court, under statutory language, an individual
violates the federal act by intentionally disclosing the contents of an
electronic communication when he knows or has reason to know that
the information was obtained by illegal interception even if the
individual was not involved in the interception. CA stated the statute
invalid because they dettered more speech than necessary to protect
the private interest at stake.
Whether the punished publisher of information has obtained the
information in question in a manner lawful in itself but from a source
who has obtained it unlawfully, may the government punish the
ensuing publication of that info based on the defect of the chain?
Privacy of communication is an important interest. Privacy concerns
give way when balanced against the interest in publishing matters of
public importance.
SC . AFFIRMED. Strangers illegal conduct does not suffice to remove
the first amendment shield from speech about a matter of public
concern.
Miller vs California
413 US 15, 37 ED 2d 419, 93 S Ct 2607 (1973)
Appellant conducted mass mailing campaign to advertise adult
materials. An unsolicited mail containing pictures and drawings
depicting men and women engaging in various sexual activities was
received by a restaurant manager and his mother. He was convicted of
misdemeanor.
The first amendment protects works which , taken as a whole, have
serious literary, artistic, political or scientific value regardless of
whether the government or a majority of the people approve of the
ideas these works represent. The protection given speech and press
was fashioned to assure unfettered interchange of ideas for the
bringing about of political and social changes desired by the people. But
the portrayal of hardcore sexual conduct for its own sake and for
commercial gain is a different matter. Roth holding that Obscene
material is not protected by the first amendment is affirmed. Obscenity
is determined by applying contemporary community standards not
national standards
Stanley vs. Georgia
394 US 557 , 22 L Ed 2d 524 (1969)
An investigation of appelants bookmaking activities led to a search
warrant. While searching, obscene films were found and seized.
Won a statute imposing criminal sanctions upon mere possession of
obscene matter is constitutional.
No. First and 14th amendments prohibit making mere possession of
obscene material as a crime. The states retain broad power to regulate
obscenity but that power does not extend to mere possession by the
individual in the privacy of his home. If the first amendment means
anything it means that the state has no business telling a man what
books he may read or what films he should watch. The case cannot be
decided simply by citing roth. Publication and mere possessing does not
impose same dangers.

The constant challenge to the courts then is how to provide


guidelines by which people may determine what is allowed
form what is proscribed

New York v. Ferber


458 U.S. 747, 73 L Ed 2d 1113, 102 S Ct 3348 (1982)

Bartnicki vs. Vopper


532 US 514, 149 Led 787, 121 S Ct 1753 (2001)

Facts: A New York statute prohibits person from knowingly promoting a


sexual performance by a child under age of 16 by distributing material
which depicts such performance. Respondent sold to an undercover
police officer two films devoted almost exclusively to depicting young
48 | P

LATON

boys masturbating.
Issue: Whether or not New York criminal statute which prohibits
persons from knowingly promoting sexual performances b children
under the age of 16 by distributing material which depicts such
performances is constitutional or overbroad.
Held: No, a trier of fact need not find that the material appeals to the
prurient interest of the average person, it is also not required that
sexual conduct portrayed be done so in a offensive manner and the
material at issue need not be considered as a whole. The law should
not be invalidated for overbreadth unless it reaches a substantial
number of permissible application is hardly novel. Therefore it is
considered as a paradigmatic case of a state statute whose legitimate
reach dwarf its arguably impermissible applications.
Ashcroft v. Free Speech Coalition
535 U.S. 234, 152 L Ed 2d 403, 122 S Ct 1389 (2002)
Facts: Child Pornography Act of 1996 (CCPA) expanded the prohibition
on child pornography to include not only pornographic images made
using actual children but also any visual depiction like photograph, film,
video, picture or computer or computergenerated image or picture
that depict a minor engage in sexual explicit conduct, also known as
virtual child pornography. Free speech coalition an adult entertainment
trade association filed a suit alleging that the appears to be and
conveys the impression provisions are overboard and vague, chilling
production of works protected by First amendment.
Issue: Whether or not the mentioned provision abridges the freedom of
speech.
Held: CPPA prohibits speech despite its serious literary, artistic, political
or scientific value. The statute proscribes the visual depiction of an idea
teenager engaging in sexual activity that is a fact of modern society and
has been a theme in art and literature throughout the ages. Virtual
child pornography is not intrinsically related to the sexual abuse of
children. The harm does not necessarily follow from the speech, but
depends upon some unquantified potential for subsequent criminal
acts. Therefore provision is considered overbroad and
unconstitutional.
Pita v. Court of Appeals
178 SCRA 362 (1989)
Facts: Pursuant to an Anti Smut Campaign, Western Police district,
INP of Metropolitan Police Force of Manila, seized and confiscated from
dealers, distributors, newsstand owners and peddlers along Manila
sidewalks magazines, publications and other reading materials believed
to be obscene, pornographic and indecent and later burned the seized
materials. Among seized publication seized was Pinoy Playboy
magazine. The company filed a case for injunction seeking to restrain
respondents confiscating petitioners magazine claiming that the
magazines are decent, artistic and educational magazine, which
protected by the constitution which guarantees of freedom of speech
and of the press.
Issue: Whether or not such confiscation is unconstitutional.
Held: The court rejected the argument that there is neither
constitutional nor legal provision which would free the accused from all
criminal responsibility because of the absence of the warrant. The
reasons are first there is no accused to speak of and second would be
the Mayor could have directly ordered the raid without search warrant
for the reason that the violation of the penal law was already
committed.

Films, Review, Classifications and Censorship


An issue that also presents itself to the courts is the matter of
censorship or prior review of motion pictures or pre-recorded
broadcasts before exhibition

As to what might be permissible regulations or restrictions


beyond that is something that the courts may continue to
grapple with as cases are presented before the, questioning, for
instance, the classification that might have been accorded a
particular work, and so on
Times Film Corporation v. City of Chicago
365 U.S. 43, 5 L Ed 2d 403, 81 S Ct 391 (1961)
Facts: Municipal Code of Chicago requires submission of all motion
pictures for examination or censorship prior to their public exhibition
and forbids their exhibition unless they meet certain standards. The
film known as Don Juan applied for a permit but refused to submit
the film for examination. Appropriate city officials refused to issue
permit for the reason of the petitioners denial to submit the film. The
petitioner on the other hand brought a suit seeking injunctive relief
ordering the issuance of the permit. For the reason that the provision
of the ordinance requiring submission of the film constitutes, on its face
a prior restraint within the prohibition of the first and Fourteen
amendments.
Issue: the precise question at issue here never having been specifically
decided.
Held: Motion picture censorship cases which reached the court
involved question of standards. The challenge here is the censors basic
authority; it does not go to any statutory standards employed by the
censor or procedural requirements as to the submission of the film.
Movie and Television Review and Classification Board (MTRCB) v. ABS
CBN Broadcasting Corporation 448 SCRA 575 (2005)
Facts: The Inside Story a public affairs show of the ABS CBN
network, had aired Prostituition episode which the main topic is
about students who enters into prostitution in order to earn money, to
be use for their enrolment. It was alleged that the school the most of
these students came from is from Philippine Womens University
(PWU). The reason was in the said episode PWU was the background.
The parents and teacher association of PWU filed a complaint under
the MTRCB, alleging that the episode is a besmirched the name of
PWU. That the petitioner did not submit the episode to the petitioner
for review and that they exhibit it without permission violating
presidential decree No. 1986.
Issue: whether or not the MTRCB has authority to review the above
mentioned show.
Held: Yes, there are only two exceptions where MTRCB cannot review a
certain show; 1) that the television program imprinted or exhibited by
the Philippine Government, its department or agencies 2) newsreels.
Newsreels are straight news reporting and therefore such program is
not considered to be one for it is more of a public affairs program which
is in no contest is subjected to MTRCB review.
Freedman v. Maryland
380 US 51, 13 L Ed 2d 649, 85 S Ct 734 (1965)
Facts: Appellant sought to challenge the constitutionality of the
Maryland motion picture censorship statute Md. Ann. Code, 1957, Art.
66A, and exhibited the film Revenge at Daybreak at his Baltimore
theatre without first submitting the picture to the State Board of
Censors as required.
Issue: Whether or not non issuance of license prior to the failure to
submit the picture is unconstitutional.
Held: Maryland does not satisfy the following criteria 1) once the
censor disapproves the film, the exhibitor must assume the burden of
instituting judicial proceedings and persuading the courts that the film
is protected expression 2) once the board has acted against a film,
exhibition is prohibited pending judicial review 3) Maryland statute
49 | P

LATON

provide no assurance of prompt judicial determination. Maryland


scheme fails to provide safeguards against inhibition of protected
expression, and this renders the requirement of prior submission of
films to the Board an invalid previous restraint.
Gonzalez v. Kalaw Katigbak
137 SCRA 717 (1985)
Facts: Gonzalez president of the Malaya Films produced the movie
Kapit sa patalim. In a resolution of a sub committee of respondent
Board of Review for Motion Pictures and Television (BRMT) a permit
was exhibit but the film was under the classification For adults only. A
motion for reconsideration was filed by the petitioners stating that the
classification of the film was without legal basis.
Issue: Whether or not there was been a grave abuse of discretion by
the board in the light of the difficulty and travail undergone by
petitioners before Kapit sa Patalim was classified as for Adults Only.
Held: Obscene material is material which deals with sex in a manner
appealing to prurient interest. The perception of what constitutes
obscenity appears to be unduly restrictive. Light of the facts in this case
should be construed in such fashion top avoid any taint of
unconstitutionality. Court concludes then that there was an abuse in
discretion nonetheless there is no enough votes to maintain that such
abuse can be considered grave. All that remains to be said is that the
ruling is to be limited to the concept of obscenity applicable to motion
pictures.

Modern Amenities and State Regulations


There would necessarily have to be adaptations and
adjustments as some features of these advances may not
readily fit into the traditional molds in which the rights and
regulations coexisted in the past
Reno v. American Civil Liberties Union
521 U.S. 844, 138 L Ed 2d 874, 117 S Ct 2329 (1997)
Two provisions of Communications Decency Act of 1996 seek to protect
minors from harmful material on the internet. 223 (a)(1)(b)(ii)
criminalizes the knowing transmission of obscene and indecent
message to any recipient under 18 years of age. Section 223(d)
prohibits the knowin[g]. Sending or displaying to person under 18 of
any message that, in context, depicts, describes, in terms patently
offensive as measured by contemporary community standards, sexual
or excretory activities or organs. A number of plaintiff files suit
challenging the constitutionality of 223(a)(1) and 223(d). Three judge
of District court convened and entered a preliminary injunction against
enforcement of both challenged provisions. The courts judgment
enjoins the government from enforcing 223(a)(1)(B)s prohibitions
insofar as they relate to indecent communications, but reserves the
right of the Government to investigate and prosecute the obscenity or
child pornography activities prohibited therein. The injunction against
223(3) is unqualified because that section contains no separate
reference to obscenity and child pornography. Government appealed
arguing the DC erred in its ruling.
Issue: Whether or not the two provisions enacted to protect minors
from indecent and patently offensive communications on the internet
constitutional
Decision: Yes there is a violation. The Court upheld the decision of the
District Court.
Rationale: The stature abridges the freedom of speech protected by
the First Amendment.

to potentially harmful speech, the CDA effectively suppresses a large


amount of speech that adults have a constitutional right to receive and
to address to one another.
Where obscenity is involved, we have consistently held that the fact
that protected speech may be offensive to some does not justify its
suppression. It is true that we have repeatedly recognized the
governmental interest in protecting children from harmful materials
but that interest does not justify an unnecessarily broad suppression of
speech addressed to adults ( example: since ang gusto ng government
para macheck kung minor ka is magswipe pa ng credit card, etc. E since
yung ibang adult walang credit card, Boom!)
Moreover, the general, undefined terms indecent and patently
offensive cover large amounts of nonpornographic material with
serious educational or other value.
United States v. Playboy Entertainment Group, Inc.
529 U.S. 803, 146 L Ed. 2d 865, 120 S Ct 1878 (2000)
Playboy Entertainment Group owns and prepares programs for adult
television networks. Playboy transmits its programming to cable
television operators, who retransmit it to their subscribers. Playboy
challenges 505 of the Television Communication Act of 1996, which
requires cable television operators who provide channels primarily
dedicated to sexually-oriented programming either to fully scramble
or otherwise fully block those channels or to limit their transmission to
hours when children are unlikely to be viewing, set by administrative
regulation as the time between 10 pm to 6am. Playboy contends that
the stature is unnecessarily restrictive content-based legislation
violative of the First Amendment. Disctrict court ruled content based
restriction on speech violates the First Amendment because the
Government might further its interests in less restrictive ways.
Issue: Whether or not DC erred in its ruling that the legislation violated
the First Amendment
Decision: Laws designed or intended to suppress or restrict the
expression of specific speakers contradict the First Amendment
principles. Section 505 limited Playboys market as a penalty for its
programming choice, though other channels capable of transmitting
like material are altogether exempt.
505 is a content based speech restriction, it can only stand of it satisfies
strict scrutiny. If a statute regulates speech based on its content, it
must be narrowly tailored to promote compelling Government interest.
If a less restrictive alternative would serve the Governments purpose,
the legislature must use that alternative.
As we consider a content-based regulation, the answer should be clear:
The standard is strict scrutiny. This case involves speech alone, and
even where speech is indecent and enters the home, the objective of
shielding children does not suffice to support a blanket ban if the
protection can be accomplished by a less restrictive alternative.
Cable systems have the capacity to block unwanted channels on a
household-by-household basis. Targeted blocking is less restrictive than
banning, and the Government cannot ban speech if targeted blocking is
a feasible and effective means of furthering its interests.
In problems like these, the Government has the burden to formulate a
solution to address such problems. It must do so in way consistent
with First Amendment Principles. The Government has failed to show
that 505 is the least restrictive means for addressing a real problem.
Fighting Words Doctrine
-One of the exceptions to the guarantee of freedom of expression.
-Those which, by their very utterance, inflict injury or tend to incite an
immediate breach of the peace.

The CDA is a content based regulation on speech.


The CDA lacks the precision that the First Amendment requires when a
stature regulates the content of speech. In order to deny minors access

Hecklers veto
-The opposition of a rowdy or obstreperous crowd might as well drown
out the voice of the one seeking to exercise the right to speak. In legal
50 | P

LATON

context, that might come from in the form of regulations tending to


prevent the expression of an idea in view of the reaction that might be
engendered among those opposed to it.
-This may be in the guise of a permit requirement in the holding of
rallies, parades or demonstrations conditioned on the payment of a fee
computed on the basis of the cost needed to keep order in view of the
expected opposition by persons holding contrary views.

The Fighting Words Doctrine and the Hecklers Veto


As noted earlier, one of the exceptions to the guarantee of
freedom of expression is the species of speech referred to as
fighting words--those which, by their very utterance, inflict
injury or tend to incite an immediate breach of the peace--since
they are no essential part if any exposition of ideas, and are
such slight social value as a step to truth that any benefit that
may be derived from them is clearly outweighed by the social
interest in order and morality
Hecklers veto comes in the form of regulations tending to
prevent the expression of an idea in view of the reaction that
might be engendered among those opposed to it. This may be
in the guise of a permit requirement in the holding of rallies,
parades or demonstrations conditioned n the payment of a fee
computed on the basis of the cost needed to keep order in view
of the expected opposition by persons holding contrary views
Governance, Elections and Speech
At the core of the freedom of expression is speech directed at
the political order

is a valid limitation under the clear and present danger doctrine.


Once such situation is found to exist, there is no limit to the allowable
limitations on such constitutional rights. Evils substantial in character
taint the purity of the electoral process.
It is no exaggeration to state that violence and even death did
frequently occur because of the heat engendered by such political
activities. The opportunity for dishonesty and corruption, with the right
to suffrage being bartered, was further magnified. The legislative body
must have felt impelled to impose the foregoing restrictions. Without
such restrictions, the laudable purpose of R.A. 5880 would be
frustrated and nullified.
We give due recognition to the legislative concern to cleanse, and if
possible, render spotless, the electoral process.
Sanidad v. Commission on Elections
181 SCRA 529 (1990)
In relation to the holding of a plebiscite for the ratification of the
Organic Act for Cordillera Autonomous Region, Resolution no. 2167
prohibits columnists, commentators or announcers during the
plebiscite campaign period, on the day before and on plebiscite day,
from using their columns or radio or television times to campaign for or
against the plebiscite issues. Petitioner, who claims to be a newspaper
columnist of overview fir the Baguio Midland Carrier, alleges that
such provision is void. He maintains that unlike regular news reporter,
his column obviously and necessarily contains his opinions, views, and
beliefs on any issue. Comelec counters that the provision is a valid
implementation of their power to supervise and regulate media during
election or plebiscite period under Article IX-C, section 4 of the
Constitution Moreover; he may still express his views for or against the
act through the Comelec space and airtime.
Issue: Whether or not Resolution 2167 is unconstitutional.

The assertion of ones freedom to speak or to express ones


thoughts would have to be balanced against the other interests
of the state, be it in keeping peace and order, clean, honest and
credible elections, making equitable access to media for
publicity or in preventing visual blights, all of which may come
as an inevitable consequence of campaign activities, spending
and posting election materials
Gonzales v. Commission on Elections
27 SCRA 835 (1969)
Petitioners, a private individual and a councillor and a candidate for
vice-mayor of Manila, challenged the validity of two sections in the
Revised Election Code, under R.A. 4880, which prohibited the too early
nomination of candidates and limiting the period of election campaign
or partisan political activity. The law provides however that simple
expression of opinion and thoughts concerning the election shall not be
considered as part of an election campaign, and further provision that
nothing stated in the Act shall be understood to prevent any person
from expressing his views on current political problems or issues, or
from mentioning the candidates for public office whom he supports.
Issue: Whether or not the enforcement of R.A. 4880 prejudice the basic
rights such as freedom of speech and assembly.
Decision: Does not prejudice. R.A. is constitutional.
Rationale: Freedom of speech or of the press involves the liberty to
discuss publicly and truthfully any matter of public interest without
censorship or punishment. It means something more than the right to
approve existing political beliefs or economic arrangements, to lend
support to official measures, to take refuge in the existing climate of
opinion on any matter of public consequence.
Freedom of speech could only be limited if there through the clear and
present danger rule and the dangerous tendency rule. The said R.A.

Decision: Unconstitutional.
Rationale: It is clear from Art. IX-C that the evil sought to be avoided is
the possibility that a franchise holder may favour or give any undue
advantage to a candidate in terms of advertising space or radio or
television time. This is also the reason why a columnist, commentator,
announcer or personality, who is a candidate for any elective office is
required to take a leave of absence from his work during the campaign
period. It shall not be construed to mean that the Comelec has also
been granted the right to supervise and regulate the exercise by media
practitioners themselves of their right to expression during plebiscite
periods. Media practitioners exercising their freedom of expression are
neither the franchise holder nor the candidates; in fact there are no
candidates involved in a plebiscite.
Osmena v. Commission on Elections
288 SCRA 447 (1998)
In National Press Club v. Comelec, the court upheld the constitutionality
of Section 11(b) of R.A. no. 6646 which prohibits mass media from
selling or giving free of charge print space or air time for campaign or
other political purposes, except the COMELEC. Petitioners, candidates
for public office, seek a re-examination of the validity of aforecited
provision, contending that events after the ruling in National Press Club
have called into question the validity of the very premises if that
decision.
There is no case or controversy to decide, only an academic discussion
to hold.
Decision: Petition is dismissed.
Petitioners claim that the experience in the last five years since the
decision has shown undesirable effects of the law. However, petitioners
do not complain of any harm suffered as a result of the operation of the
law. What petitioners seek is not the adjudication of a case but simply
51 | P

LATON

the holding of an academic exercise.


The laws concern is not with the message or content of the ad but with
ensuring media equality between candidates with deep pockets and
those with less resources. There is not total ban on political ads, much
less restriction on the content of the speech. Given money could be of a
disadvantage of a poor candidate, there is a substantial or legitimate
governmental interest justifying exercise of the regulatory power of the
COMELEC.
Puno, J. Concurring:
The guaranty of freedom of speech should not be used to frustrate
legislative attempts to level the playing field in politics. R.A. 6646 does
not curtail speech as it no more than prevents the abusive wealth by
the rich to frustrate the poor candidates access to media. If we allow
money to monopolize the media, the political framework will cease to
be a market of ideas but a market for influence of the rich.
Blo Umpar Adiong v. COMELEC
207 SCRA 712 (1992)
Comelec promulgated Resolution no. 2347, regulating election
propaganda. The said resolution allows pamphlets, decals, stickers, etc.
To be posted only in any of the authorized posting areas provided in the
same resolution and declares it unlawful to draw, paint, inscribe, post,
display, or publicly exhibit, any election propaganda in any place,
whether public or private, mobile or stationary, except in Comelec
common posting area. Petitioner assails the resolution as regards the
prohibition of posting of decals and stickers in mobile places like cars
and other moving vehicles. The posting of decals and stickers, according
to him, shall be his last medium to inform the electorate that he is a
senatorial candidate (neophyte sya) since there is already a ban on
radio, television, and print political advertisements.
Issue: Whether or not the COMELEC may prohibit the posting of decals
and stickers on mobile places, public or private and limit their
location or publication to the authorized posting areas that it fixes.
Decision: Null and void.
Rationale: First: The prohibition unduly infringes on the citizens right of
free speech. Considering the period of legitimate campaign activity is
limited, it becomes obvious that unduly restrictive regulations may
prove unfair to affected parties and the electorate as well.
The posting of decals and stickers in mobile places like cars and other
moving vehicles does not endanger and substantial government
interest. There is no clear public interest threatened by such activity so
as to justify the curtailment of such right. (Di siya pasok sa clear and
present danger rule)
Second: The restriction as to where the decals and stickers should be
posted is so broad that it encompasses even citizens private property,
which in this case a privately-owned vehicle. (Violation ng property
without due process of law na to.)
There are many candidates whose names alone evoke qualifications,
platforms, programs and ideologies which voter may accept or reject.
When a person attaches a sticker with such a candidates name on his
car and bumper, he is pressing more than the name, he is espousing
ideas. Our view of the validity of the challenged regulation includes its
effects in todays particular circumstances. We are constrained to rule
against the Comelec prohibition.

conflict with official COMELEC count as well as the unofficial quick


count of NAMFREL, and ABS-CBN did not have any authorization nor
deputized by the Commission. Thus, the petition for certiorari,
petitioner arguing that holding of exit polls and the nationwide
reporting of their results are valid exercises of freedoms of speech and
of the press.
Holding of exit polls and the dissemination of their results through mass
media constitute an essential part of the freedoms of speech and of the
press. The COMELEC cannot ban them totally in the guise of promoting
clean, honest, orderly and credible elections.
Social Weather Stations, Inc. v. Commission on Elections
357 SCRA 496 (2001)
Social Weather Stations, Inc. (SWS) is a non-stock social research
institution which conducts surveys in various fields and thereafter
processes, analyzes, and publicly reports the results thereof, while
petitioner Kamahalan Publishing Corporation publishes the Manila
Standard, a newspaper of general circulation. Petitioners bring action
for prohibition from implementing Sec. 5.4 of R.A. No. 9006, the Fair
Election Act, which the former claim unconstitutional. Sec. 5.4 provides:
Surveys affecting national candidates shall not be published fifteen
(15) days before an election and surveys affecting local candidates shall
not be published seven (7) days before an election. SWS, which wishes
to conduct survey throughout the period of elections and to release to
the media the results as well as publish them directly, and Kamahalan
Publishing, which intends to publish election survey results to the last
day of election survey results to the last day of elections, assail the
aforecited provision as equivalent to prior restraint without any
justification.
OBrien test used to determine the constitutional validity. A
government regulation is sufficiently justified: [1] if it is within the
constitutional power of the government; [2] if it furthers an important
or substantial government interest; [3] if the governmental interest is
unrelated to the suppression of free expression.
Sec. 5.4 of RA 9006 constitutes an unconstitutional abridgment of
freedom of speech, expression, and the press. It is invalid because: [1] it
imposes a prior restraint on the freedom of expression; [2] it is a direct
and total suppression of a category of expression even though such
suppression is only for a limited period, and [3] the governmental
interest sought to be promoted can be achieved by other means other
than the suppression of freedom of expression.

Courts, Criticism and Contempt


The courts may also express sensitivity at times to their delicate
and important ole in society such that they feel the need to
preserve the kind of respect and dignity that they should
deserve in order that they could effectively continue to
discharge their critical function of dispensing justice
United States v. Bustos
37 Phil. 731 (1918)

ABS-CBN Broadcasting Corporation v. Commission on Elections


323 SCRA 811 (2000)

Numerous citizens of Pampanga assembled, prepared and signed a


petition to the Executive Secretary charging Roman Punsalan, justice of
the peace and Macabebe and Masantol, Pampanga, with malfeasance
in office and asked for his removal. It was contended that said justice of
peace exacted money and property from a complainant, asked for
money in exchange of victory in litigation and paid a complaint in
another to justify the shelving of his case. Charges had been instituted
and Punsalan was acquitted.

During the 1998 National Elections, ABS-CBN prepared a project to


conduct radio-TV coverage of the elections and to make an exit survey
of the vote during the elections for national officials particularly for the
President and the Vice President. COMELEC issued a Resolution which
approved the restraining order to stop ABS-CBN or any other groups
from conducting such exit survey, believing that such project might

Criminal action for libel against those who petitioned for Punsalans
removal was then instituted. Defendants contend that their petition for
removal of the justice of the peace falls within the protection of the
freedom of speech and right to assembly and to petition for the redress
of their grievances. Moreover, they contend that the content of their
petition is to be considered privileged communication and thus, cannot
52 | P

LATON

be the basis for a libel case.


The guaranties of a free speech and a press include the right to criticize
judicial conduct. The administration of law is a matter of public
concern. The right to assemble10 and petition11 is the necessary
consequence of republican institutions and the complement of the
right of free speech. A communication made bona fide upon any
subject-matter in which the party communicating has an interest, or in
reference to which he has a duty, is privileged. Qualified privilege is a
complaint made in good faith and without malice in regard to the
character or conduct of a public official when addressed to an officer or
board having some interest on the duty or in the matter.
In the case, it is not a case of direct and vicious accusations published in
the press, but of charges predicated on affidavits made to proper
official and thus qualifiedly privileged. Although charges are probably
not true as to the justice of peace but believed to be true by the
petitioners. Good faith surrounded their action. Probable cause for
them to think that malfeasance or misfeasance in office existed is
apparent. The manner in commenting on the conduct of justice of the
peace was proper. And finally, charges and the petition were submitted
through reputable attorneys to proper functionary, the Executive
Secretary.
Nestle Philippines, Inc. v. Sanchez
154 SCRA 542 (1987)
Union of Filiro Employees and Kimberly Independent Labor Union for
Solidarity, activism and Nationalism-Olalia intensified the intermittent
pickets they have been conducting in front of Padre Faura gate of the
Supreme Court building. They set up pickets quarters on the pavement
in front of the SC building, constructed provisional shelters along the
sidewalks, set up a kitchen and littered the place. They waved their red
streamers and placards with slogans, and took turns haranguing the
court all day using loudspeakers. These acts were done even after their
leaders had been received by the SC Justices who were chairpersons of
the Division s where their cases are pending. Thus, thereafter, the SC en
banc issued a resolution giving the said unions the opportunity to
withdraw graciously and requiring the union leaders and their counsels
to show cause why they should not be held in contempt of court. The
counsel for one of the unions apologized to the Court for the acts,
together with an assurance that they will not be repeated.
The apologies offered by the respondents were accepted by the Court.
The individuals cited who are non-lawyers are not aware that even as
the rights of free speech and of assembly are protected by the
Constitution, any attempt to pressure or influence courts of justice
through the exercise of either right amounts t an abuse thereof, is no
longer within the ambit of constitutional protection, nor did they
realize any such efforts to influence the course of justice constitutes
contempt of Court. The duty and responsibility of advising them,
therefore, rest primarily and heavily upon the shoulders of their
counsel of record. Atty. Jose Espinas, when his attention was called by
the Court, did his best to demonstrate to the pickets the untenability of
their acts. It is their duty as officers of the court to properly apprise
their clients on matters of decorum and proper attitude toward courts
of justice. No demonstrations or pickets intended to pressure or
influence courts of justice into acting one way or the other on pending
cases shall be allowed in the vicinity and/or within the premises of any
and all courts.

In Re Petition to Annual En Banc Resolution A.M. No. 98-7-02-SC


296 SCRA xi (1998)
Petitioner Valmonte applied for a Mayors permit to hold a rally and
camp out in front of the Justice Hall of Las Pias. Office of the Mayor
refused to issue the permit on the ground that the holding of a rally in
Assembly a right on the part of the citizens to meet peaceably for
consultation in respect to public affairs.
11 Petition any person or group of persons can apply, without fear of penalty,
to appropriate branch or office of the government for redress of grievances.
10

front of the Justice Hall of Las Pias was prohibited under the Supreme
Courts En Banc Resolution dated 7 July 1998, entitled, Re: Guidelines
on the Conduct of Demonstrations, Pickets, Rallies and Other Similar
Gatherings in the Vicinity of the Supreme Court and All Other Courts.
They submit that the Supreme Court gravely abused its discretion
and/or acted without or in excess of jurisdiction in promulgating those
guidelines. Freedom of speech and expression despite its
indispensability has its limitations. It has never been understood as the
absolute right to speak whenever, however, and wherever one pleases,
for the manner, place, and time of public discussion can be
constitutionally controlled. As well put by our Justice Isagani Cruz, the
better policy is not liberty untamed but liberty regulated by law where
every freedom is exercised in accordance with law and with due regard
for the rights of others. Court reiterates that judicial independence and
the fair and orderly administration of justice constitutes paramount
governmental interests that can justify the regulation of publics right
of free speech and peaceful assembly in the vicinity of the courthouses.
Even in the United States, a prohibition against picketing and
demonstrating in or near courthouses has been ruled as valid
constitutional.
Petitioners also claim that this Court committed an act of judicial
legislation in promulgating the assailed resolution. They charged that
this Court amended the provisions of Batas Pambansa (B.P.) Blg. 880,
otherwise known as The Public Assembly Act, by converting the
sidewalks and streets within a radius of two hundred (200) meters from
every courthouse from a public forum place into a no rally zone.
Contrary therefore to petitioners impression, B.P. Blg. 880 did not
establish streets and sidewalks, among other places, as public fora. A
close look at the law will reveal that it in fact prescribes reasonable
time, place, and manner regulations. It requires a written permit for the
holding of public assemblies in public places subject, even, to the right.
Existence of B.P. Blg. 880, however, does not preclude this Court from
promulgating rules regulating conduct of demonstrations in the vicinity
of courts to assure our people of an impartial and orderly
administration of justice as mandated by the Constitution.
SC is especially vested by the Constitution with the power to adopt
measures essential to an orderly administration of justice. These rules
are designed to ensure the orderly and expeditious conduct of court
business as well as to secure the rights of parties. These court-made
rules have the force and effect of law.
Social Weather Stations, Inc. v. Asuncion
228 SCRA xi (1993)
Manila Standard published an item entitled Judiciary worse than PNP,
which reported that, according to the opinion polls conducted by the
Social Weather Station (SWS), the Judiciary had an even lower
satisfaction rating than the PNP. The report prompted Judge
Maximiano Asuncion to initiate, motu proprio, proceedings entitled In
the Matter of Findings of Social Weather Research Group Derogatory to
the Judiciary. He then ordered the President of SWS, Prof. Mahar
Mangahas, to explain why he should not be held in contempt for
distributing to the general public without prior permission from any
court findings which tend to directly or indirectly degrade the
administration of justice. An explanation was submitted to the court
and the contempt charge against SWS President was dismissed after
finding the explanation submitted satisfactory. After some time, Prof.
Mangahas addressed a letter to the Chief Justice intended as formal
complaint against Judge Asuncion for grave abuse of authority and
gross ignorance of the law, in connection with the contempt charge
that was initiated by respondent judge.
A publication which tends to impede, obstruct, embarrass or influence
the courts in administering justice in a pending suit or proceeding,
constitutes criminal contempt which is summarily punishable by courts.
A publication which tends to degrade the courts and to destroy public
confidence in them or that which tends to bring them in any way into
disrepute, constitutes likewise criminal contempt, and is equally
punishable by courts. In the first there is no contempt where there is no
decision which might in any way be influenced by newspaper
publication. In the second, the contempt exists, with or without a
53 | P

LATON

pending case, as what is sought to be protected is the court itself and


its dignity would lose their utility if public confidence is destroyed.
The contention that Professor Mangahas was improperly cited for
contempt for acts or utterances not related to a pending action must,
therefore, be rejected. What was clearly implicit in the newspaper
report about the results of the SWS poll in the words of Judge
Asuncion, that the people have more confidence with the police that
with the judges in light of the fact, of which judicial notice is taken,
that said report came out at a time when there already was widespread
publicity adverse to the judiciary, there can be no doubt of its clear
tendency to degrade the administration of justice. Judge Asuncion can
hardly be faulted for what, at a minimum, he must have felt dutybound to do in the circumstances. No question of prior restraint or
violation of the guarantee of free speech arises here, what he did being,
in essence, merely to initiate an inquiry into the source and basis of the
derogatory news report. And he forthwith abated the proceedings
upon receiving an explanation he deemed satisfactory.
In Re Emil P. Jurado
243 SCRA 299 (1995)
Facts: Jurado is a journalist who writes a column entitled Opinion in a
newspaper of general circulation, the Manila Standard. He describes
himself as a columnist, who incidentally happens to be a lawyer. He
wrote about alleged improprieties and irregularities in the judiciary
over several months. In light of these abnormal developments, the
Chief Justice took an extraordinary step. He issued Administrative Order
No. 11-93 dated 25 January 1993, Creating an Ad Hoc Committee to
Investigate Reports of Corruption in the Judiciary, tasked to ascertain
the truth respecting said reports and statements, and to interview at
closed-door sessions or otherwise, such persons as may appear to it to
have some knowledge of the matter and who may be appealed to share
that knowledge with the court, and otherwise gather such evidence as
may be available.
Material to the present inquiry are Jurados published statements from
late 1992 to the middle of February, 1993 where he wrote of graft and
corruption in the judiciary. There were calls for impeachment of the
justices, for resignation of judges. There were insistent and more
widespread reiterations of denunciations of incompetence and
corruption in the judiciary. For shortly afterwards, on 10 February 1993,
Mr. Vicente R. Samson, First Vice-President of the PLDT addressed a
letter to the chief Justice and requesting that the Court take such
action as may be appropriate.
Jurado moved for the termination of the proceeding on the following
posited premises: 1) the court has no administrative supervision over
him as a member of the press or over his work as a journalist; 2) the
present administrative matter is not a citation for (a) direct contempt
as there is no pending case or proceeding out of which a direct
contempt charge against him may arise, (b) indirect contempt as no
formal charge for the same has been laid before the court in
accordance with Section 3 (Rule 71) of the Rules of Court; and,, 3) his
comments would be more relevant and helpful to the Court if taken
together with the other evidence and reports of other journalists
gathered before the Ad Hoc Committee. He perceives no reason why
his comments should be singled out and taken up in a separate
administrative proceeding.
Issue: Whether or not Jurados acts are protected by the Constitutional
provision of Freedom of Expression?
Held: There is no constitutional value in false statements of fact, and
the erroneous statement of fact is not worthy of constitutional
protection (although) ** nevertheless inevitable in free debate.
Neither the intentional lie nor the careless error, it said, materially
advances societys interest in unhibited, robust, and wide-open
debate on public issues.
Jurado [failed] to undertake even the most cursory verification of their
objective truth; the abdication of the journalists duty to report and
interpret the news with scrupulous fairness; and the breach of the laws
injunction that a person act with justice, give everyone his due and
observe honesty and good faith both in the exercise of his rights and in

the performance of his duties.


[His] actuations, in the context in which they were done, demonstrate
gross irresponsibility, and indifference to factual accuracy and the
injury that he might cause to the name and reputation of those whom
he wrote. They constitute contempt of court, directly tending as they
do to degrade or abase the administration of justice and the judges
engaged in that function.
WHEREFORE, the Court declares Atty. Emil (Emiliano) P. Jurado guilty
of contempt of court.

Speech, Professions and Callings


While the right to freely speak ones mind may be available to
the people, it does not necessarily mean that it is allowed to
the same extent for everyone. Depending on ones occupation
or calling, the extent to which he may exercise that right may
be affected, and his behavior, including the manner by which
he may express himself, would have to be accordingly adjusted
Zaldivar v. Sandiganbayan
166 SCRA 316 (1988)
Facts: Petitioner Zaldivar is one of the several defendants in criminal
cases pending before the Sandiganbayan for violation of the Anti-Graft
and Corrupt Practices Act. Petitioner asserts that respondent Gonzales,
as Tanodbayan and under the provisions of the 1987 Constitution, was
no longer vested with power and authority independently to
investigate and to institute criminal cases for graft and corruption
against public officials and employees. Hence, the criminal informations
filed were all null and void.
The Court issued a Temporary Restraining Orders directing the
Tanodbayan from acting upon cases already filed and from filing
additional criminal informations. Thereafter, petitioner Zaldivar filed
with the Court a Motion to Cite in Contempt directed against Gonzales
alleging that Gonzales issued contemptuous statements to the media.
He either released his Motion for Reconsideration with facsimiles of
said notes to the press or repeated the same to the press. And, for the
next several days, the metropolitan papers carried long reports on
those statements and variations and embellishments thereof. The
Court issued a resolution ordering Gonzales to explain why he should
not be punished for contempt and/or subjected to administrative
sanctions.
Issue: Whether or not respondent Gonzales statements are covered by
Constitutional provision of freedom of speech?
Held: His [Gonzales] defense is not that he did not make the statements
ascribed to him but that those statements give rise to no liability on his
part, having been made in the exercise of his freedom of speech.
*** A free press is not to be preferred to an independent judiciary, nor
an independent judiciary to a free press. Neither has primacy over the
other; both are indispensable to a free society.
The freedom of the press in itself presupposes an independent judiciary
through which that freedom may, if necessary, be vindicated. And one
of the potent means for assuring judges their independence is a free
press.
This constitutional right must be protected in its fullest extent. The
Court has heretofore given evidence of its tolerant regard for charges
under the Libel Law which come dangerously close to its violation. But
license or abuse of liberty of the press and of the citizens should not be
confused with liberty in its true sense. As important as is the
maintenance of unmuzzled press and the free exercise of the rights of
the citizens is the maintenance of the independence of the Judiciary.
Some courts have held, persuasively it appears to us, that a lawyers
right of free expression may have to be more limited than that of a
54 | P

LATON

layman.

Bates v. State Bar of Arizona


433 US 350, 53 L Ed 2d 810,97 S Ct 2691 (1977)

Respondent Gonzalez claims to be and he is, of course, entitled to


criticize the rulings of this court. But it is the cardinal condition of all
such criticism that it shall be bona fide, and shall not spill over the walls
of decency and propriety. Intemperate and unfair criticism is a gross
violation of the duty of respect to courts. It is such a misconduct that
subjects a lawyer to disciplinary action. The lawyers duty to render
respectful subordination to the courts is essential to the orderly
administration of justice
The Court concludes that respondent Gonzales is of guilty both of
contempt court in facie curiae and of gross misconduct as an officer of
the court and member of the Bar.
Resolution on Motion for Reconsideration
170 SCRA 1 (1989)
Facts: In his point D, respondents [Gonzales] counsel urges that it is
error for this Court to apply the visible tendency rule rather than the
clear and present danger rule in disciplinary and contempt charges.
Held: The clear and present danger doctrine has been an accepted
method for marking out the appropriate limits of freedom of speech
and of assembly in certain contexts. It is not, however, the only test
which has been recognized and applied by courts.
From the language of the specific constitutional provision, it would
appear that the right is not susceptible of any limitation. No law may be
passed abridging the freedom of speech and of the press. The realities
of life in a complex society preclude however, a literal interpretation.
Freedom of expression is not an absolute. It would be too much to
insist that all times and under all circumstances it should remain
unfettered and unrestrained. There are other societal values that press
for recognition.

Facts: The appellants, licensed attorneys and members of the Arizona


State Bar, placed a newspaper advertisement for their legal clinic,
listing their fees for certain routine services, namely, uncontested
divorces, uncontested adoptions, simple personal bankruptcies, and
changes of name. Because of this they were charged in a complaint
filed by the State Bars president with violating the State Supreme
Courts disciplinary rule, which prohibits attorneys from advertising in
newspapers or other media. The Arizona Supreme Court upheld the
conclusion of a bar committee that appellants had violated the rule,
rejecting, among others, appellants claim that the rule infringed
appellants First Amendment rights.
Issue: Whether or not the State may prevent the publication in a
newspaper of appellants truthful advertisement concerning the
availability and terms of routine legal services?
HELD: We rule simply that the flow of such information may not be
restrained, and we therefore hold the present application of the
disciplinary rule against appellants to be violative of the First
Amendment.

Symbolic Speech, Expressive Conduct and the Public Forum


Doctrine
Speech is not confined to words. It can also be expressed
through a persons conduct or other symbolic manner of
delivering a message, either alone or in collaboration with
other. The Constitution itself guarantees as part of the freedom
of expression the right of the people peaceably to assemble
and petition the Government for redress of grievances

The prevailing doctrine is that the clear and present danger rule is such
a limitation. Another criterion for permissible limitation on freedom of
speech and of the press is the balancing of interests test. The principle
requires a court to take conscious and detailed consideration of the
interplay of interests observable in a given situation or type of
situation.

Public Forum Doctrine posits that public areas such as streets


and parks are traditionally considered as proper venues for the
free articulation of ideas and views and that, therefore, the
exercise of such freedom should not be unnecessarily restricted
or impaired

ACCORDINGLY, the Court Resolved to DENY the Motion for


Reconsideration for lack of merit. The denial is FINAL.

The public forum doctrine holds that restrictions on speech


should be subject to higher scrutiny when, all other things
being equal, that speech occurs in areas playing a vital role in
communication--such as in those places historically associated
with first amendment activities, such as streets, sidewalks, and
parks--especially because of how indispensable communication
in these places is to people who lack access to more elaborate
(and more costly) channels

Commercial Speech
As noted earlier, at the core of the freedom of speech and of
the press is political expression, or those in connection with the
peoples relationship with their government and politics. It was
only subsequently that the freedom was also considered to
encompass commercial speech
For commercial speech to come within that provision, it at least
must concern lawful activity and not be misleading. Next, we
ask whether the asserted governmental interest is substantial.
If both inquiries yield positive answers, we must determine
whether the regulation directly advances the governmental
interest asserted, and whether it is not more extensive than is
necessary to serve that interest
Intrinsically related to commercial speech is the advertising
industry, whose power to influence could either be for good or
bad. Accordingly, the Constitution provides for its regulation by
holding that The advertising industry is impressed with public
interest, and shall be regulated by law for the protection of
consumers and the promotion of the general welfare

United States v. OBrien


391 U.S. 367, 20 L Ed 2d 672, 88 S Ct 1673 (1968)
Facts: OBrien and three companions burned their Selective Service
certificates on the steps of the South Boston Courthouse. A sizeable
crowd, including several agents of the Federal Bureau of Investigation,
witnessed the event and immediately after the burning members of the
crowd began attacking OBrien and his companions. OBrien stated to
FBI agents that he had burned his registration certificate because of his
beliefs, knowing that he was violating federal law. He stated in
argument to the jury that he burned the certificate publicly to influence
others to adopt his antiwar beliefs. He also argued that the 1965
Amendment prohibiting the knowing destruction or mutilation of
certificates was unconstitutional because it was enacted to abridge free
speech, and because it serve no legitimate legislative purpose, which
arguments the District Court rejected.
Issue: Whether or not the 1965 Amendment prohibiting the knowing
destruction or mutilation of certificates was unconstitutional because it
was enacted to abridge free speech?
55 | P

LATON

Held: This court has held that, when speech and non-speech
elements are combined in the same course of conduct, a sufficiently
important governmental interest in regulating the non-speech element
can justify incidental limitations on First Amendment freedoms.
Whatever impression inheres in these terms, we think it clear that a
government regulation is sufficiently justified if it is within the
constitutional power of the Government; if it furthers an important or
substantial governmental interest; if the governmental interest is
unrelated to the suppression of free expression; and if the incidental
restriction on alleged First Amendment freedoms is no greater than is
essential to the furtherance of that interest.
The governmental interest and the scope of the 1965 Amendment are
limited to preventing harm to the smooth and efficient functioning of
the Selective Service System. When OBrien deliberately rendered
unavailable his registration certificate, he willfully frustrated this
governmental interest. For this non-communicative impact of his
conduct, and for nothing else, he was convicted.
Since the 1965 Amendment to 12(b)(3) of the Universal Military
Training and Service Act is constitutional as enacted and as applied.
Accordingly, we vacate the judgment of the Court of Appeals, and
reinstate the judgment and sentence of the District Court. This
disposition makes unnecessary consideration of OBriens claim that the
Court of Appeals erred in affirming his conviction on the basis of the
non-possession regulation.
Clark v. Community for Creative Nonviolence
468 U.S. 288, 82 L Ed 2d 221, 104 S Ct 3065 (1984)
Facts: In 1982, the National Park Service issued a renewable permit to
respondent Community for Creative Non-Violence (CCNV) to conduct a
wintertime demonstration in Lafayette Park and Mall, which are
National Parks in the heart of Washington, D.C., for the purpose of
demonstrating the plight of the homeless. The permit authorized the
erection of two symbolic tent cities. However, the Park Service, relying
on its regulations particularly on that permits camping (defined as
the use of park land for living accommodation purposes such as
sleeping activities, or making preparations to sleep) only in designated
campgrounds, no campgrounds having ever been designated in
Lafayette Park or the Mall denied CCNVs request that demonstrators
be permitted to sleep in the symbolic tents. CCNV and the individual
respondents then filed an action in the District Court, alleging, inter
alia, that application of the regulations to prevent sleeping in the tents
violated the First Amendment. The District Court granted summary
judgment for the Park Service, but the Court of Appeals reversed.
Issue: Whether or not a National Park Service regulation prohibiting
camping in certain parks violates the First Amendment when applied to
prohibit demonstrators from sleeping in Lafayette Park and the Mall in
connection with a demonstration intended to call attention to the
plight of the homeless?
HELD: Expression, whether oral or written or symbolized by conduct, is
subject to reasonable time, place, or manner restrictions. Symbolic
expression of this kind may be forbidden or regulated if the conduct
itself may constitutionally be regulated, if the regulation is narrowly
drawn to further a substantial governmental interest, and if the interest
is unrelated to the suppression of free speech.
In relation to regulation of activities or conduct on
government property, a distinction would have to be made on whether
such property is traditionally considered as public forums streets
and parks or not.
The Governments ownership of property does not
automatically open that property to the public. It is a long-settled
principle that governmental actions are subject to a lower level of First
Amendment scrutiny when the government function operating [is]
not the power to regulate or license, as lawmaker but, rather, as
proprietor, to manage [its] internal operation[s]
Regulation of speech activity on governmental property that
has been traditionally open to the public for expressive activity, such as

public streets and parks, is examined under strict scrutiny. Regulation


of speech on property that the Government has expressly dedicated to
speech activity is also examined under strict scrutiny. But regulation of
speech activity where the Government has not dedicated its property
to First Amendment activity is examined only for reasonableness.

Regulation of speech activity n governmental property that has


been traditionally open to the public for expressive activity,
such as public streets and parks, is examined under strict
scrutiny
Regulation of speech on property that the Government has
expressly dedicated to speech activity is also examined under
strict scrutiny
But regulation of speech activity where the Government has
not dedicated its property to first Amendment activity is
examined only for reasonableness
Anonymous Speech
The speaker need not always have to identify himself in order
that he may avail of the protection of the guarantee of freedom
of speech. Anonymity need not result in loss of ones right to
speak freely
The anonymity of an author is not ordinarily a sufficient reason
to exclude her work product from the protections of the First
Amendment
The freedom to publish anonymously extends beyond the
literary realm
If the State can show a compelling interest to be subserved by a
regulation which requires identification of the speaker,
however, then the same may be upheld, but again it would
have to be subjected to an exacting scrutiny which is the usual
standard in relation to speech-related restrictions
Freedom of Assembly and Right to Petition
The expression of ones opinion or plaints need not only be
limited to pure words. An individual with a grievance that needs
to be articulated may find a more effective manner of having
his message get across by associating and joining with others
and expressing themselves in a collaborative manner and
through a collective voice
As stated in Reyes v. Bagatsing, it was not by accident or
coincidence that the rights to freedom of speech and of the
press were coupled in a single guarantee with the rights of the
people peaceably to assemble and to petition the government
for redress of grievances. All these rights, while not identical,
are inseparable
Reyes v. Bagatsing
125 SCRA 553 (1983)
Facts: Petitioner, retired Justice J.B.L. Reyes, on behalf of the Anti-Bases
Coalition, sought a permit from the City of Manila to hold a peaceful
march and rally on 26 October 1983, starting from Luneta, a public
park, to the gates of the United States Embassy, hardly two blocks
away. It turned out that on 19 October such permit was denied.
Petitioner was unaware of such a fact as the denial was sent by
ordinary mail. The reason for refusing a permit was due to persistent
intelligence reports affirm[ing] the plans of subversive/criminal
elements to infiltrate and/or disrupt and assembly or congregations
56 | P

LATON

where a large number of people is expected to attend. On the same


day the Court heard the parties on oral argument and in the afternoon,
a minute resolution was issued by the Court granting the mandatory
injunction prayed for on the ground that there was no showing of the
existence of a clear and present danger of a substantive evil that could
justify the denial of a permit.

individual or a group. There can be no legal objection, absent the


existence of a clear and present danger of a substantive evil, on the
choice of Luneta as the place where the peace rally would start. Time
immemorial Luneta has been used for purposes of assembly,
communicating thoughts between citizens, and discussing public
questions.

Issue: Whether or not the mayor of the city of Manila acted in lack or
excess of jurisdiction in denying the rally permit to petitioner and if
such decision abridge their right of expression and assembly?

Such use of the public places has from ancient times, been a part of the
privileges, immunities, rights, and liberties of citizens.
With regard to the ordinance, there was no showing that there was
violation and even if it could be shown that such a condition is satisfied
it does not follow that respondent could legally act the way he did. The
validity of his denial of the permit sought could still be challenged.

Held: It is true that the licensing official, here respondent Mayor, is not
devoid of discretion in determining whether or not a permit would be
granted. It is not, however, unfettered discretion. While prudence
requires that there be a realistic appraisal not of what may possibly
occur but of what may probably occur, given all the relevant
circumstances, still the assumption especially so where the assembly
is scheduled for a specific public place is that the permit must be for
the assembly being held there.
While the general rule is that a permit should recognize the right of the
applicants to hold their assembly at a public place of their choice,
another place may be designated by the licensing authority if it be
shown that there is a clear and present danger of a substantive evil if
no such change were made. Hence the discretion reached by the Court.
The mere assertion that subversives may infiltrate the ranks of the
demonstrators does not suffice.
Ordinarily, the remedy in cases of this character is to set aside the
denial or the modification of the permit sought and order the
respondent official to grant it. Nonetheless, as there was urgency in this
case, the proposed march and rally being scheduled for the next day
after the hearing, this Court, in the exercise of its conceded authority,
granted the mandatory injunction in the resolution of October 25,
1983.

A summary of the application for permit for rally: The applicants for a
permit to hold an assembly should inform the licensing authority of the
date, the public place where and the time when it will take place. If it
were a private place, only the consent of the owner or the one entitled
to its legal possession is required. Such application should be filed well
ahead in time to enable the public official concerned to appraise
whether there may be valid objections to the grant of the permit or to
its grant but at another public place. It is an indispensable condition to
such refusal or modification that the clear and present danger tests be
the standard for the decision reached. Notice is given to applicants for
the denial.

Academic Freedom
The Constitution guarantees that academic freedom shall be
enjoyed in all institutions of higher learning. This freedom
essentially involves the right of such institutions of learning to
determine what to teach, how to teach them, who may teach
them, and who to admit to study therein
The classroom is peculiarly the marketplace of ideas

Reyes v. Bagatsing
125 SCRA 553 (1983)
Facts: Petitioner sought a permit from the City of Manila to hold a
peaceful march and rally on October 26, 1983 from 2:00 to 5:00 in the
afternoon, starting from the Luneta to the gates of the United States
Embassy. Once there, and in an open space of public property, a short
program would be held. The march would be attended by the local and
foreign participants of such conference. That would be followed by the
handing over of a petition based on the resolution adopted at the
closing session of the Anti-Bases Coalition. There was likewise an
assurance in the petition that in the exercise of the constitutional rights
to free speech and assembly, all the necessary steps would be taken by
it "to ensure a peaceful march and rally. However the request was
denied. Reference was made to persistent intelligence reports affirming
the plans of subversive/criminal elements to infiltrate or disrupt any
assembly or congregations where a large number of people are
expected to attend. Respondent suggested that a permit may be issued
if it is to be held at the Rizal Coliseum or any other enclosed area where
the safety of the participants themselves and the general public may be
ensured. An oral argument was heard and the mandatory injunction
was granted on the ground that there was no showing of the existence
of a clear and present danger of a substantive evil that could justify the
denial of a permit. However Justice Aquino dissented that the rally is
violative of Ordinance No. 7295 of the City of Manila prohibiting the
holding of rallies within a radius of five hundred (500) feet from any
foreign mission or chancery and for other purposes. Hence the Court
resolves.
Issue: Whether or not the freedom of expression and the right to
peaceably
assemble
violated.
Ruling: Yes. The invocation of the right to freedom of peaceable
assembly carries with it the implication that the right to free speech has
likewise been disregarded. It is settled law that as to public places,
especially so as to parks and streets, there is freedom of access. Nor is
their use dependent on who is the applicant for the permit, whether an

Academic freedom, the term as it evolved to describe the


emerging rights related to intellectual liberty, has traditionally
been associated with freedom of thought, speech, expression,
and the press; in other words, the right of individuals in
university communities, such as professors, researchers and
administrators, to investigate, pursue, discuss and, in the
immortal words of Socrates, to follow the argument wherever
it may lead, free from internal and external interference or
pressure
The component of this aspect of academic freedom have been
categorized under the areas of: (1) who may teach and (2) how
to teach
As corporate entities, educational institutions of higher learning
are inherently endowed with the right to establish their
policies, academic and otherwise, unhampered by external
controls or pressure. In the Frankfurter formulation, this is
articulated in the areas of: (1) what shall be taught, e.g., the
curriculum and (2) who may be admitted to study
Admission to an institution of higher learning is discretionary
upon a school, the same being a privilege on the part of the
student rather than a right
The Constitution guarantees that academic freedom shall be enjoyed
in all institutions of higher learning. This freedom essentially involves
the right of such institutions of learning to determine what to teach,
how to teach them, who may teach them, and who to admit to study
therein. It has also been held that if, pursuant to its academic freedom,
such institutions of higher learning can decide who can and who cannot
study in it, it certainly can also determine on whom it can confer the
honor and distinction of being its graduates, including the revocation or
57 | P

LATON

withdrawal of the honor or distinction it may have conferred where it is


subsequently shown that the same was obtained through fraud.
Garcia v. Faculty Admission Committee, Loyola School of Theology
68 SCRA 277 (1975)
Facts: This is a mandamus case compelling the respondent to allow the
petitioner to continue studying there. The petitioner contends that she
was denied re admission for the reason given by the respondent
school, namely, that her frequent questions and difficulties were not
always pertinent and had the effect of slowing down the progress of
the class, is not a valid ground for her expulsion. While the respondent
contends that the school has discretion to admit or continue admitting
in said school any particular student, considering not only academic or
intellectual standards but also other students, space limitations,
facilities, professors and optimum classroom size.
Issue: Whether or not the petitioner is deemed possessed of such a
right that has to be respected.
Ruling: The petition is deemed not only on general principle, but also in
view of the character of the particular educational institution involved.
It is a seminary. Also, there is the autonomy recognized by the
Constitution in this explicit language: All institutions of higher learning
shall enjoy academic freedom. In connection to this, the four essential
freedoms of a university are to determine for itself on academic
grounds who may teach, what may be taught, how it shall be taught,
and who may be admitted to study.
Justice Makasiar dissenting: What is involved here is not merely
academic freedom of the higher institutions of learning as guaranteed
by Section 8 (2) of Art. XV of the 1973 Constitutions. The issue here
strikes at the broader freedom of expression of the individual the very
core of human liberty.
Miriam College Foundation, Inc. v. CA
348 SCRA 265 (2000)
Facts: The petitioner took disciplinary action against some of its
students who were members of the editorial boards of the colleges
school paper (Chi Rho) and magazine (Ang Magazing Pampanitikan Ng
Chi Rho) for contents therein that were allegedly described by some
members of the Miriam College community as obscene, vulgar,
indecent, gross, sexually explicit, injurious to young readers,
and devoid of all moral values. The students then filed a petition for
prohibition and certiorari with preliminary injunction/restraining order
before the RTC of Quezon City questioning the jurisdiction of the
Discipline Board of Miriam College over them. The RTC eventually
granted the writ of preliminary injunction but subsequently dismissed
the petition on the ground raised by both parties that it is the DECS
which has jurisdiction. Eight of the eleven students elevated to the SC,
but the court referred to the C.A. in due time the appellate court
granted the petition of the students and declared the RTC order, as well
as the students supervision and dismissed, void.

the special characteristics of the school environment.


The power of the school to investigate is an adjunct of its power to
suspend or expel. It is a necessary corollary to the environment
conducive to learning: that power, like the power to suspend or expel,
is an inherent part of the academic freedom of institutions of higher
learning guaranteed by the Constitution. Therefore, the Miriam College
has the authority to her and decide.

Among the reasons for the indispensability of the peoples


freedom of speech and of assembly to democracy are that
freedom of expression is essential as a means of assuring
individual fulfillment and that it is an essential process for
advancing knowledge and discovering
Additional Cases
(F) Freedom of Expression, Right of Assembly
and Academic Freedom [3-8]
Chavez vs. Gonzales
545 SCRA 441 (2008)
The case originates from events that occurred a year after the 2004
elections. On June 5, 2005 Press Secretary Ignacio Bunye told reporters
that the opposition is planning to destabilize the administration by
releasing an audiotape of a mobile phone conversation allegedly
between Pres. GMA
and a high-ranking official of the Comelec
(Garcillano) through wire-tapping. He produced two kinds of tape a
complete version and a doctored version. That which instructed the
comelec official to manipulate the election results in the presidents
favor. There were also conversations of President, First gentleman,
Garci and the late senator Barbers.
On the June, 2005 respondent DOJ secretary Gonzales warned
reporters that who had copies of the CD or those who will publish the
contents will be liable under the Anti-wiretapping Act because of the
continuing the offense that had the personal knowledge of the crime
that included Sec. Bunye and Atty. Paguia.
Because of the incident the NTC gives fair warning to radio and
television owners/operators to observe anti-wire-tapping law and
pertinent circulars on program standards. It has been subsequently
established that the said tapes are false and/or fraudulent after a
prosecution or appropriate investigation, the concerned radio and
television companies are hereby warned that their broadcast/airing of
such false information and/or willful misrepresentation shall be just
cause for the suspension, revocation and/or cancellation of the licenses
or authorizations issued to the said companies.
On June 14, 2005, NTC held a dialogue with the Board of Directors of
the Kapisanan ng mga Brodkaster sa Pilipinas (KBP). NTC allegedly
assured the KBP that the press release did not violate the constitutional
freedom of speech, of expression, and of the press, and the right to
information

Issue: Who has the jurisdiction over the case?


Ruling: Sec. 5(2), Article XIV of the Constitution guarantees all
institutions of higher learning academic freedom. The essential
freedoms subsumed in the term academic freedom encompasses the
freedom to determine for itself an academic grounds:
(1) Who may teach,
(2) What may be taught,
(3) How it shall be taught, and
(4) Who may be admitted to study.
The right of the school to discipline its students is at once apparent in
the third freedom, i.e., how it shall be taught. Accordingly, the right
to discipline the student likewise finds basis in the freedom what to
teach.
The right of the students to free speech in school premises, however, is
not absolute. The right to free speech must always be applied in light of

Petitioner Chavez filed a petition under Rule 65 of the Rules of Court


against respondents Secretary Gonzales and the NTC, praying for the
issuance of the writs of certiorari and prohibition, as extraordinary legal
remedies, to annul void proceedings, and to prevent the unlawful,
unconstitutional and oppressive exercise of authority by the
respondents. Alleging that the acts of respondents are violations of
the freedom on expression and of the press, and the right of the people
to information on matters of public concern.
We rule that not every violation of a law will justify straitjacketing the
exercise of freedom of speech and of the press. Our laws are of
different kindsand doubtless, some of them provide norms of conduct
which even if violated have only an adverse effect on a persons private
comfort but do not endanger national security. There are laws of great
significance but their violation, by itself and without more, cannot
support suppression of free speech and free press. In fine, violation of
law is just a factor, a vital one to be sure, which should be
58 | P

LATON

weighed in adjudging whether to restrain freedom of speech and of the


press. The totality of the injurious effects of the violation to private and
public interest must be calibrated in light of the preferred status
accorded by the Constitution and by related international covenants
protecting freedom of speech and of the press. In calling for a careful
and calibrated measurement of the circumference of all these factors
to determine compliance with the clear and present danger test, the
Court should not be misinterpreted as devaluing violations of law. By all
means, violations of law should be vigorously prosecuted by the
State for they breed their own evil consequence. But to repeat, the
need to prevent their violation cannot per se trump the exercise of
free speech and free press, a preferred right whose breach can
lead to greater evils. For this failure of the respondents alone to offer
proof to satisfy the clear and present danger test, the Court has no
option but to uphold the exercise of free speech and free press. There
is no showing that the feared violation of the anti-wiretapping law
clearly endangers the national security of the State.
This is not all the faultline in the stance of the respondents. We slide to
the issue of whether the mere press statements of the Secretary of
Justice and of the NTC in question constitute a form of content-based
prior restraint that has transgressed the Constitution. In
resolving this issue, we hold that it is not decisive that the press
statements made by respondents were not reduced in or followed up
with formal orders or circulars. It is sufficient that the press statements
were made by respondents while in the exercise of their official
functions. Undoubtedly, respondent Gonzales made his statements as
Secretary of Justice, while the NTC issued its statement as the
regulatory body of media. Any act done, such as a speech uttered, for
and on behalf of the government in an official capacity is covered by
the rule on prior restraint. The concept of an act does not limit itself
to acts already converted to a formal order or official
circular. Otherwise, the non formalization of an act into an official
order or circular will result in the easy circumvention of the prohibition
on prior restraint. The press statements at bar are acts that should be
struck down as they constitute impermissible forms of prior restraints
on the right to free speech and press.
There is enough evidence of chilling effect of the complained acts on
record. The warnings given to media came from no less the NTC, a
regulatory agency that can cancel the Certificate of Authority of the
radio and broadcast media. They also came from the Secretary of
Justice, the alter ego of the Executive, who wields the awesome power
to prosecute those perceived to be violating the laws of the land. After
the warnings, the KBP inexplicably joined the NTC in issuing an
ambivalent Joint Press Statement. After the warnings, petitioner Chavez
was left alone to fight this battle for freedom of speech and of the
press. This silence on the sidelines on the part of some media
practitioners is too deafening to be the subject of misinterpretation.
In VIEW WHEREOF, the petition is GRANTED. The writs of certiorari and
prohibition are hereby issued, nullifying the official statements made by
respondents on June 8, and 11, 2005 warning the media on airing the
alleged wiretapped conversation between the President and other
personalities, for constituting unconstitutional prior restraint on the
exercise of freedom of speech and of the press
Soriano v. Laguardia
(2009)
This is a petition regarding Eliseo Soriano who seeks to nullify and set
aside an order and decision of the MTRCB in connection with certain
utterances he made in the television show, Ang Dating Daan.
On August 10, 2004, at 10pm, the petitioner, as host of the television
show Ang Dating Daan made following remarks:
Lehitimong anak ng demonyo, sinungaling;
Gago ka talaga Michael, masahol ka pa sa putang babae o dib ba.
Yung putang babae ang gumagana lang doon ung ibaba, ditto kay
Michael ang gumagana ang itaas. O dib a! o masahol pa sa putang
babae yan. Sabi ng lola ko masahol pa sa putang babae yan sora ang
kasinungalingan ng mga demonyong ito.
Because of that the MTRCB ordered the suspension of the program for
20 days in accordance with Section 3(d) of Presidential Decree No. (PD)

1986, creating the MTRCB, in relation to Sec. 3, Chapter XIII of the 2004
Implementing Rules and Regulations (IRR) of PD 1986 and Sec. 7, Rule
VII of the MTRCB Rules of Procedure.5The same order also set the case
for preliminary investigation. The following day after the suspension
the petitioner sought for reconsideration of the preventive suspension
and Laguardia ( MTRCB chair person) recuse from hearing the case.
After the case was heard in this court it was given 3 months suspension
of the program Ang Dating Daan.
After which they again filed for petition for certiorari and prohibition
with prayer for injunctive relief with the following issues: (A) BY
REASON THAT THE [IRR] IS INVALID INSOFAR AS IT PROVIDES FOR THE
ISSUANCE OF PREVENTIVE SUSPENSION ORDERS; (B) BY REASON OF
LACK OF DUE HEARING IN THE CASE AT BENCH; (C) FOR BEING
VIOLATIVE OF EQUAL PROTECTION UNDER THE LAW; (D) FOR BEING
VIOLATIVE OF FREEDOM OF RELIGION; AND (E) FOR BEING VIOLATIVE
OF FREEDOM OF SPEECH AND EXPRESSION.
In ending, what petitioner obviously advocates is an unrestricted
speech paradigm in which absolute permissiveness is the norm.
Petitioners flawed belief that he may simply utter gutter profanity on
television without adverse consequences, under the guise of free
speech, does not lend itself to acceptance in this jurisdiction. We
repeat: freedoms of speech and expression are not absolute freedoms.
To say "any act that restrains speech should be greeted with furrowed
brows" is not to say that any act that restrains or regulates speech or
expression is per se invalid. This only recognizes the importance of
freedoms of speech and expression, and indicates the necessity to
carefully scrutinize acts that may restrain or regulate speech.
WHEREFORE, the decision of the MTRCB in Adm. Case No. 01-04 dated
September 27, 2004 is hereby AFFIRMED with the MODIFICATION of
limiting the suspension to the program Ang Dating Daan. As thus
modified, the fallo of the MTRCB shall read as follows:
WHEREFORE, in view of all the foregoing, a Decision is hereby
rendered, imposing a penalty of THREE (3) MONTHS SUSPENSION on
the television program, Ang Dating Daan, subject of the instant
petition.
Co-respondents Joselito Mallari, Luzviminda Cruz, and UNTV Channel
37 and its owner, PBC, are hereby exonerated for lack of evidence.Costs
against petitioner.
Soriano v. Laguardia
(2009)
Ang Dating Daan host Eliseo S. Soriano uttered the following
statements in his TV program against Michael Sandoval (Iglesia ni
Cristos minister and regular host of the TV program Ang Tamang
Daan):
Lehitimong
anak
ng
demonyo[!]
Sinungaling
[!]
Gago ka talaga[,] Michael[!] [M]asahol ka pa sa putang babae[,] o di
ba[?] []Yung putang babae[,] ang gumagana lang doon[,] []yung
ibaba, dito kay Michael[,] ang gumagana ang itaas, o di ba? O, masahol
pa sa putang babae []yan. Sobra ang kasinungalingan ng mga
demonyong ito.
As a result, The MTRCB initially slapped Sorianos Ang Dating Daan,
which was earlier given a G rating for general viewership, with a 20day preventive suspension after a preliminary conference. Later, in a
decision, it found him liable for his utterances, and was imposed a
three-month suspension from his TV program Ang Dating Daan. Soriano
challenged
the
order
of
the
MTRCB.
HELD: The SC ruled that Sorianos statement can be treated as
obscene, at least with respect to the average child, and thus his
utterances cannot be considered as protected speech. Citing decisions
from the US Supreme Court, the High Court said that the analysis
should be context based and found the utterances to be obscene
after considering the use of television broadcasting as a medium, the
time of the show, and the G rating of the show, which are all factors
that made the utterances susceptible to children viewers. The Court
emphasized on how the uttered words could be easily understood by a
child literally rather than in the context that they were used.
The SC also said that the suspension is not a prior restraint, but rather
a form of permissible administrative sanction or subsequent
punishment. In affirming the power of the MTRCB to issue an order of
suspension, the majority said that it is a sanction that the MTRCB may
59 | P

LATON

validly impose under its charter without running afoul of the free
speech clause. visit fellester.blogspot.com The Court said that the
suspension is not a prior restraint on the right of petitioner to
continue with the broadcast of Ang Dating Daan as a permit was
already issued to him by MTRCB, rather, it was a sanction for the
indecent contents of his utterances in a G rated TV program.
(Soriano v. Laguardia; GR No. 165636, April 29, 2009)

and criticism. His irresponsible and baseless statements, his


unrepentant stance and smug insistence of his malicious and
unfounded accusation against Justice Tinga have sullied the dignity and
authority of this Court. Beyond question, therefore, De La Serna's
culpability for indirect contempt warrants the penalty of a fine not
exceeding P30,000.00 or imprisonment not exceeding six (6) months or
both under the Rules.

Dissenting Opinion: Chief Justice Reynato S. Puno, in a separate


dissenting opinion, said that a single government action could be both a
penalty and a prior restraint. The Chief Magistrate pointed out that the
three month suspension takes such form because it also acts as a
restraint to petitioners future speech and thus deserves a higher
scrutiny than the context based approach that the majority applied.
In voting to grant Sorianos petition, the Chief Justice said that in the
absence of proof and reason, he [Soriano] should not be penalized with
a three-month suspension that works as a prior restraint on his
speech.

The power to declare a person in contempt of court and in dealing with


him accordingly is a means to protect and preserve the dignity of the
court, the solemnity of the proceedings therein and the administration
of justice from callous misbehavior and offensive personalities. Respect
for the courts guarantees the stability of the judicial institution.
Without such guarantee, the institution would be resting on a very
shaky foundation. The Court will not hesitate to wield this inherent
power to preserve its honor and dignity and safeguard the morals and
ethics of the legal profession.

F9. In Re: Amado P. Macasaet


561 SCRA 395 (2008)
The case stemmed from certain articles that appeared in the "Business
Circuit" column of Amado P. Macasaet in the Malaya, a newspaper of
general circulation of which he is the publisher. The articles, containing
statements and innuendoes about an alleged bribery incident in the
Supreme Court, came out in four (4) issues of the newspaper on
September 18, 19, 20 and 21, 2007.
Upon evaluation of the columns "Business Circuit" of Amado P.
Macasaet in the September 18, 19, 20, and 21, 2007 issues of the
Malaya, it appears that certain statements and innuendoes therein
tend, directly or indirectly, to impede, obstruct, or degrade the
administration of justice, within the purview of Section 3(d), Rule 71 of
the 1997 Rules of Civil Procedure. Amado P. Macasaet is ordered to
explain why no sanction should be imposed on him for indirect
contempt of court
The High Court created an investigating committee composed of retired
Supreme Court justices. The Investigating Committee held hearings and
gathered affidavits and testimonies from the parties concerned
The Court declares respondent Amado P. Macasaet guilty of indirect
contempt of court. Unfortunately, the published articles of respondent
Macasaet are not of this genre. On the contrary, he has crossed the
line, as his are baseless scurrilous attacks which demonstrate nothing
but an abuse of press freedom. They leave no redeeming value in
furtherance of freedom of the press. They do nothing but damage the
integrity of the High Court, undermine the faith and confidence of the
people in the judiciary, and threaten the doctrine of judicial
independence
He published his highly speculative articles that bribery occurred in the
High Court, based on specious information, without any regard for the
injury such would cause to the reputation of the judiciary and the
effective administration of justice. Nor did he give any thought to the
undue, irreparable damage such false accusations and thinly veiled
allusions would have on a member of the Court.
F10. Fudot v. Cattleya Land, Inc.
570 SCRA 86 (2008)
Mr. Chan had already bought the interest of Cattleya Land, Inc.
(Cattleya) over a property adjacent to the property subject of the case
and that he was interested in putting up a resort/hotel in the property.
He wanted to purchase Carmelita Fudot's interest in the property as
well to put an end to the litigation. On 9 November 2007, the Court
received from De La Serna a request for the inhibition of Associate
Justice Dante O. Tinga, claiming that Justice Tinga received P10 Million
from Mr. Johnny Chan (Mr. Chan) in exchange for a favorable decision
in the instant case.
After hearing the explanation of both parties, the court holds that Atty.
De La Serna has transcended the permissible bounds of fair comment

F11. Pleasant Grove City v. Summum


555 U.S. __ (2009)
Summum, a religious organization, sent a letter to the mayor of
Pleasant Grove, Utah asking to place a monument in one of the city's
parks. Although the park already housed a monument to the Ten
Commandments, the mayor denied Summum's request because the
monument did not "directly relate to the history of Pleasant Grove."
Summum filed suit against the city in federal court citing, among other
things, a violation of its First Amendment free speech rights. The U.S.
District Court for the District of Utah denied Summum's request for a
preliminary injunction.
The U.S. Court of Appeals for the Tenth Circuit reversed the district
court and granted Summum's injunction request. The Tenth Circuit held
that the park was in fact a "public" forum, not a non-public forum as
the district court had held. Furthermore, Summum demonstrated that
it would suffer irreparable harm if the injunction were to be denied,
and the interests of the city did not outweigh this potential harm. The
injunction, according to the court, was also not against the public
interest.
Does a city's refusal to place a religious organization's monument in a
public park violate that organization's First Amendment free speech
rights when the park already contains a monument from a different
religious group?
No. The Supreme Court reversed the Tenth Circuit holding that the
placement of a monument in a public park is a form of government
speech and therefore not subject to scrutiny under the Free Speech
Clause of the First Amendment. The Court reasoned that since Pleasant
Grove City had retained final authority over which monuments were
displayed, the monuments represented an expression of the city's
viewpoints and thus government speech. Although a park is traditional
public forum for speeches and other transitory expressive acts, the
display of a permanent monument in a public park is not a form of
expression to which forum analysis applies.
F12. PHCAP v Duque III
535 SCRA 265 (2007)
Facts: Before the Court is a petition for certiorari under Rule 65 of the
Rules of Court, seeking to nullify A.O No. 2006-0012 entitled, Revised
Implementing Rules and Regulations of Executive Order No 51,
Otherwise known as the "Milk Code,, Relevant International
Agreements, Penalizing Violations Thereof and for Other Purposes
(RIRR). Petitioner posits that the RIRR is not valid as it contains
provisions that are not constitutional and go beyond the law it is
supposed to implement.
Issue: WON RIRR is valid and constitutional
Held: Only Sections 4(f), 11 and 46 of A.O. No. 2006-0012 are declared
as beyond the authority of the DOH and are therefore null and void.
The rest of the Code is in consonance with the objective, purpose and
60 | P

LATON

intent of the Milk Code since it constitutes reasonable regulation of an


industry which affects public health and welfare. Thus, the rest of RIRR
do not constitute illegal restraint of trade nor are they violative of the
due process clause of the Constitution.
Concurring Opinion of Chief Justice Puno:
Another reason why the absolute ban on advertising and promotion of
breast milk substitutes found under Sections 4(f) and 11 of A.O. No.
2006-0012 should be struck down:

same day, she issued G.O. No. 5 setting the standards which the AFP
and the PNP should follow in the suppression and prevention of acts of
lawless violence. Prof. Randolf David and others were then arrested
without warrant while they were exercising their right to peaceful
assembly. The authorities also raided the office of the newspaper
Tribune, threatened the media, imposed censorship and threatened
take over public utilities. Petitioners were charged with the crime of
inciting to sedition and violation of BP 880, The Public Assembly Act of
1985.

The advertising and promotion of breast milk substitutes is considered


a commercial speech which is a kind of speech that proposes an
economic transaction. It is a separate category of speech which is not
accorded the same level of protection as that given to other
constitutionally guaranteed forms of expression but is nonetheless
entitled to protection. Commercial speech must concern lawful activity
and not be misleading if it is to be protected under the First
Amendment. The asserted governmental interest must be substantial.
If both of these requirements are met, it must next be determined
whether the state regulation directly advances the government interest
asserted, and whether it is not more extensive than is necessary to
serve the interest.

Issue: WON PP 1017 and G.O. No. 5 are constitutional

The absolute ban on advertising prescribed under Sections 4 (f) and 11


of the RIRR is unduly restrictive and is more than necessary to further
the avowed governmental interest of promoting the health of infants
and young children. It must be self-evident, for instance that the
advertisement of such products which are strictly informative cuts too
deep on free speech. The laudable concern of the respondent for the
promotion of the health of infants and young children cannot justify the
absolute, overarching ban.

However, the warrantless arrest of the petitioners, the dispersal of


rallies and warrantless arrest of others and the imposition of standards
on media or any prior restraint on the press, and the warrantless search
of the Tribune offices and the seizures of some articles for publication
are neither authorized by the Constitution nor by the provisions of PP
1017 and G.O. No. 5.

F13. Bayan v. Ermita


488 SCRA 226 (2006)
Facts: Petitioners come in three groups and they assail BP 880 The
Public Assembly Act of 1985 , some of them in toto and others only
Sections 4, 5, 6, 12, 13(a), and 14(a), as well as the policy of Calibrated
Preemptive Response (CPR). They also seek to stop violent dispersals of
rallies under the "no permit, no rally" policy and the CPR policy recently
announced.
Issues: (1) WON the Calibrated Preemptive Response policy is valid
(2) WON BP 880 is valid and constitutional
Held: The right to peaceably assemble and petition for redress of
grievances is, together with freedom of speech, of expression, and of
the press, a right that enjoys primacy in the realm of the constitutional
protection. Since these rights constitute the very basis of a functional
democratic polity, without which all the other rights would be
meaningless and unprotected. However, such right is not absolute.
Thus, BP 880 provides restriction that simply regulates the time, place
and manner of the assemblies. It does not impose an absolute ban on
public assemblies.
(1) The Calibrated Preemptive Response (CPR) policy has no place in
our legal firmament and must be struck down. It serves no valid
purpose if it is also means maximum tolerance under BP 880 and is
illegal if it means something else.
(2) BP 880 is valid and constitutional since it does not curtail or unduly
restrict freedom. It merely regulates the use of public places as to the
time, place and manner of assemblies. Moreover, maximum
tolerance" is for the benefit of rallyists, not the government. The
delegation to the mayors of the power to issue rally "permits" is
likewise valid because it is subject to the constitutionally-sound "clear
and present danger" standard.
F14. David v. Macapagal-Arroyo
486 SCRA 160 (2006)
Facts: As part of the commemoration of EDSA I, Pres. GMA
promulgated PP 1017 declaring the state of national emergency. On the

Held: PP 1017 is constitutional on the ground that it constitutes a call


by the President for the AFP to prevent pr suppress lawless violence.
But its provisions regarding the Presidents power to issue decrees,
direct the AFP to enforce obedience to all laws even those which are
unrelated to lawless violence and to impose standards on media or any
for, or prior restraint on the press are unconstitutional.
G.O. No. 5 is valid since it is as order issued by the President acting as
the Commander-in Chief addressed to the AFP to carry out the
provision of PP 1017.

*Assembly is a right of the citizens to meet peaceably for consultation


regarding public affairs. It is a necessary consequence of our republican
institution and complements the right of speech. In connection with
the freedom of expression, such right is not limited or denied except on
a showing of a clear and present danger of a substantive evil that
Congress has a right to prevent. The right to assemble, like other rights
of freedom of expression, is not subject to previous restraint or
censorship. It may not also be conditioned upon the prior issuance of a
permit or authorization from the government authorities except if the
assembly is intended to be held in a public place, a permit for the use of
the place and not the assembly itself may be validly required.
In the case at bar, the petitioners were not notified and heard on the
revocation of their permits. The first time they learned of it was at the
time of the dispersal. Thus, such absence of notice is a fatal defect.
*The search conducted in the office of Tribune, even though it has
anti-government sentiments, is considered as illegal as it violated
petitioners freedom of the press. It must be noted that freedom to
comment on public affairs is essential to the vitality of a representative
democracy.
*(1) The overbreadth doctrine is an analytical tool developed for testing
on their faces statutes in free speech cases known under the
American laws First Amendment Cases. In connection to that, a plain
reading of PP 1017 shows that it is not primarily directed to speech or
even a speech-related conduct. Also, it is not intended for testing the
validity of a law that reflects legitimate interest in maintaining
comprehensive control over harmful constitutionally unprotected
conduct. The claims of facial overbreadth are entertained in cases
involving statutes which, by their terms, seek to regulate only spoken
words and again that overbreathed claims, if entertained at all , have
been curtailed when invoked against ordinary criminal laws that are
sought to be applied to protected conduct. Therefore, PP 1017
pertains to a spectrum of conduct, not free speech, which is manifestly
subject to state regulation.
(2) Facial invalidation of laws is considered as manifestly strong
medicine to be used sparingly and only as a last resort and is
generally disfavored for the reason that a person to whom a law may
be applied will not be heard to challenge a law on the ground that it
may be conceivably be applied unconstitutionally to others, for
example, in situations not before the Court. Thus, a facial challenge
61 | P

LATON

using the overbreadth doctrine will require the Court to examine pp


1017 and pinpoint its flaws and defects not on the basis of its actual
operation to petitioners but on the assumption that its very existence
may cause others not before the Court to refrain from constitutionally
protected speech or expression.
(3) A facial challenge on the ground of overbreadth is the most difficult
challenge to mount successfully because the challenger must establish
that there can be no instance when the assailed law may be valid. In
the case at bar, petitioners did not even attempt to show whether this
situation exists.

Chapter 7
Freedom of Religion
No law shall be made respecting an establishment of religion,
or prohibiting the free exercise thereof. The free exercise and
enjoyment of religious profession and worship, without
discrimination or preference, shall forever be allowed. No
religious test shall be required for the exercise of civil or
12
political rights.
A mans faith and belief are his alone and the State has no
business interfering with that
The Constitution also provides in emphatic terms The
separation of Church and State shall be inviolable
The assurance of religious freedom under the Constitution
principally consists of two guarantees, embodied in the socalled Establishment Clause and Free Exercise Clause
Aglipay v. Ruiz
64 Phil. 201 (1937)
Facts: Petitioner seeks the issuance of a writ of prohibition against
respondent Director of Posts from issuing and selling postage stamps
commemorative of the 33rd International Eucharistic Congress.
Petitioner contends that such act is a violation of the Constitutional
provision stating that no public funds shall be appropriated or used in
the benefit of any church, system of religion, etc. This provision is a
result of the principle of the separation of church and state, for the
purpose of avoiding the occasion wherein the state will use the church,
or vice versa, as a weapon to further their ends and aims. Respondent
contends that such issuance is in accordance to Act No. 4052, providing
for the appropriation funds to respondent for the production and
issuance of postage stamps as would be advantageous to the
government.

Although such issuance and sale may be inseparably linked with


theRoman Catholic Church, any benefit and propaganda incidentally
resulting from it was no the aim or purpose of the Government.

Defining and Divining Religion


In having to decide issues that implicate freedom of religion,
courts must necessarily have to define what religion is
Defining religion is a difficult task for even theologians,
philosophers and moralists cannot agree on a comprehensive
definition
Religion is derived from the Middle English religioun, from Old
French religion, from Latin religio, vaguely referring to a bond
between man and the gods
The term religion has reference to ones views of his relations
to his Creator, and to the obligations they impose of reverence
for his being and character, and of obedience to his will
The test of belief in a relation to s Supreme Being is whether
a given belief that is sincere and meaningful occupies a place in
the life of its possessor parallel to that filled by the orthodox
belief in God of one who clearly qualifies for the exemption
Four criteria to qualify as religion under the First Amendment:
1. There must be belief in God or some parallel belief that
occupies a central place in the believers life
2. The religion must involve a moral code transcending
individual belief, i.e., it cannot be purely subjective
3. A demonstrable sincerity in belief is necessary, but the court
must not inquire into the truth or reasonableness of the belief
4. There must be some associational ties, although there is also
a view that religious beliefs held by a single person rather than
being part of the teachings of any kind of group or sect are
entitled to the protection of the Free Exercise Clause
In Aglipay v. Ruiz, religion has been described as that
profession of faith to an active power that binds and elevates
man to his Creator
American Bible Society v. City of Manila also spoke of religion as
having reference to ones view of his relations to His Creator
and to the obligations they impose of reverence to His being
and character, and obedience to His Will

Issue: Whether or not there was a violation of the freedom to religion.

The Establishment Clause


Ruling: What is guaranteed by our Constitution is religious freedom and
not mere religious toleration. It is however not an inhibition of
profound reverence for religion and is not a denial of its influence in
human affairs. Religion as a profession of faith to an active power that
binds and elevates man to his Creator is recognized. And in so far as it
instills into the minds the purest principles of morality, its influence is
deeply felt and highly appreciated. The phrase in Act No. 4052
advantageous to the government does not authorize violation of the
Constitution. The issuance of the stamps was not inspired by any
feeling to favor a particular church or religious denomination. They
were not sold for the benefit of the Roman Catholic Church. The
postage stamps, instead of showing a Catholic chalice as originally
planned, contains a map of the Philippines and the location of Manila,
with the words Seat XXXIII International Eucharistic Congress. The
focus of the stamps was not the Eucharistic Congress but the city of
Manila, being the seat of that congress. This was to to advertise the
Philippines and attract more tourists, the officials merely took
advantage of an event considered of international importance.
12

This clause principally prohibits the state from sponsoring any


religion, or favouring any religion as against other religions. It
mandates a strict neutrality in affairs among religious groups
The state may not favor religion to the extent of enforcing
people to believe in a god or any other supernatural being.
With the freedom to believe also comes the liberty not to
believe in any other being
In the Philippine jurisdiction, there us substantial agreement on
the values sought to be protected by the Establishment Clause,
namely voluntarism and insulation of the political process from
interfaith dissension
Religion requires voluntarism because compulsory faith lacks
religious efficacy

CONSTITUTION, Art. III, 5

62 | P

LATON

Such voluntarism cannot be achieved unless the political


process is insulated from religion and unless religion is
insulated from politics
School District of Abington Township v. Schempp
374 U.S. 203, 10 L Ed 2d 844, 83 S Ct 1560 (1963)
Facts:
Applicable Amendments:
First Amendment, Establishment Clause: Congress shall make no law
respecting an establishment of religion, or prohibiting the free exercise
thereof.
-No. 142 The Commonwealth of Pennsylvania requires that at least
ten verses from the Holy Bible shall be read without comment, at the
opening of each Public school on each school day. Any child shall be
excused from such Bible reading, or attending such Bible reading, upon
the request of his parent or guardian. The exercises are broadcast into
each room in the building through an intercom system. This is followed
by the recitation of the Lords Prayer. Participation in the open
exercises was considered voluntary. The student reading the bible must
select the passages and read any form or version he chases. (King James
version, Douay or the Revised Standard versions as well as the Jewish
Holy Scriptures)
-The constitutionality of the said statute was assailed by Edward
Schempp, a member of the Unitarian faith who, along with his wife and
children, questioned the validity of the statute, contending that his
rights have been violated, under the 14th of table and to the
Constitution of the United States.
-The children study in Abington Senior High School
-Schempp testified that he at first refused to exercise his prerogative of
excusing his children from the morning exercises upon fear that his
children would be labeled as odd balls. Their classmates would be liable
for lumping religious differences and objections as atheism with
immoral and un-patriotic overtones.
-Doctor Solomon Grayzel (witness for the appellees): The reading of
such
verses without explanation may be psychologically harmful to the
children and may cause a divisive force in the social media of the
school.
-Doctor Luther A. Weigle (witness for the defense): The Bible is a nonsectarian piece of literature within among the Christian faiths. The
exclusion of the New Testament would be in itself a sectarian practice.
-The trial court struck down the practices and the statute requiring
them after making the specific findings of fact that attendance to
Abington and undergoing the practices were compulsory. The court
further found that the reading of the verses without comment would
constitute in effect a religious observance.
-The court rejected the defences argument that the children were
allowed to excuse themselves via their parents request, saying that it
did not mitigate the obligatory nature of the ceremony. This was still in
violation of the establishment clause in that it threatens religious
liberty by putting a premium upon belief as opposed to non-belief,
rendering sinister, alien, and suspect the beliefs, ideals, and even
morality of the petitioners.
Issue: WON rule 142 of the Commonwealth of Pennsylvania is
unconstitutional under the violation of the Establishment Clause under
the Fourteenth Amendment.
Ruling: Yes. It is true that religion has been closely identified with
American history and government. This background is evidence today
in our public life through the continuance in our oaths of office from
the Presidency to the Alderman of the final supplication, So help me
God. Indeed, only last year, an official survey of the country indicated
that 64% of our people have church membership while 3% profess no

faith at all. This is not to say, however that religion has been so
identified with our history and government that religious freedom is
not likewise as strongly imbedded in our public and private life. Nothing
but the most telling of personal experiences in religious persecution
could have implanted such belief.
Marsh v. Chambers
463 U.S. 783, 77 L Ed 2d 1019, 103 S Ct 3330 (1983)
Facts: The Nebraska Legislature begins each of its sessions with a prayer
by a chaplain paid by the State with the legislature's approval.
Respondent member of the Nebraska Legislature brought an action in
Federal District Court, claiming that the legislature's chaplaincy practice
violates the Establishment Clause of the First Amendment, and seeking
injunctive relief. The District Court held that the Establishment Clause
was not breached by the prayer but was violated by paying the chaplain
from public funds, and accordingly enjoined the use of such funds to
pay the chaplain. The Court of Appeals held that the whole chaplaincy
practice violated the Establishment Clause, and accordingly prohibited
the State from engaging in any aspect of the practice.
Issue: Whether or not the legislature's chaplaincy practice violates the
Establishment Clause of the First Amendment.
Ruling: By a 6-3 vote the Supreme Court permitted the practice of
beginning a legislative session with a prayer delivered by a publicly
funded chaplain, with Chief Justice Warren Burger writing the majority
opinion.
The Court relied almost entirely on historical practice and tradition.
Congress had paid a chaplain and opened sessions with prayers for
almost 200 years. Indeed, the fact that Congress had continued the
practice after considering constitutional objections in the Court's view
strengthened rather than weakened the historical argument.
The opening of sessions of legislative and other deliberative public
bodies with prayer is deeply embedded in the history and tradition of
this country. From colonial times through the founding of the Republic
and ever since, the practice of legislative prayer has coexisted with the
principles of disestablishment and religious freedom. In the very
courtrooms in which the United States District Judge and later three
Circuit Judges heard and decided this case, the proceedings opened
with an announcement that concluded, "God save the United States
and this Honorable Court." The same invocation occurs at all sessions of
this Court.
In light of the unambiguous and unbroken history of more than 200
years, there can be no doubt that the practice of opening legislative
sessions with prayer has become part of the fabric of our society. To
invoke Divine guidance on a public body entrusted with making laws is
not, in these circumstances, an "establishment" or a step toward
establishment; it is simply a tolerable acknowledgement of beliefs
widely held among the people of this country.
Basically, the decision argued that both the Supreme Court and
Congress have traditionally begun their sessions with prayers. Since
individual states do not have to abide by more stringent First
Amendment limits than the federal government, then they, too, are
permitted to use prayers. The "Establishment Clause does not always
bar a state from regulating conduct simply because it harmonizes with
religious concerns."
Marsh vs. chambers
463 us 783 (1983)
Facts: Nebraska Legislature begins its sessions with a prayer offered by
a chaplain who is chosen biennially by the Executive Board of
Legislative Council and paid out of public funds. Respondent, claims
that the Legislatures chaplaincy practice violates the Establishment
Clause of the First Amendment.
Issue: whether or not the prayer offered upon the start of every session
of the Nebraska Legislature and the payment of public funds thereof
constitutes a violation of the Establishment Clause of the First
63 | P

LATON

Ammendment?
Held: the prayer offered by the chaplain and the funds paid thereon are
not a violation of the Establishment Clause.
The offering of prayer in the opening of sessions is deeply rooted in the
history and tradition of this country. The practice has coexisted with
the principles of disestablishment and religious freedom. It has
continued without interruption ever since that early session of
Congress. However, standing alone, historical patterns cannot justify
contemporary violations of constitutional guarantees. In this context,
historical evidence shed light not only on what the draftsmen intended
the Establishment Clause mean, but also on how they thought that
Clause applied to the practice authorized by the First Congress- their
actions reveal their intent.
BOARD OF EDUCATION VS. ALLEN
392 US 236 (1968)
Facts: Section 701 of New Yorks Education law requires local public
school authorities to lend textbooks free of charge to all students in
grade including those in private and parochial schools. The appellant
school board sought a declaration that the statutory requirement was
invalid as violative of the State and Federal Constitutions, an order
barring the appellee Commissioner of Education from removing
appellants members from office for failing to comply with it, and an
order preventing the use of state funds for the purchase of textbooks
to be lent to parochial students.
Held: the express purpose of the law is the furtherance of the
educational opportunities available to the young. There is nothing that
shows about the necessary effects of the statute that is contrary to its
stated purpose. The law merely makes available to all children the
benefits of a general program to lend school books free of charge.
WALZ VS. TAX COMMISSION OF THE CITY OF NEW YORK
397 US 664 (1970)
facts: appellant imsiccessfully sought an injunction in the New York
courts to prevent the NY Tax Commission fom granting property tax
exemptions to religious organizations for properties used solely for
religious worship, as authorized by the state constitution and the
implementing statue. The appellant contends that the grant of tax
exemptions to church property indirectly requires the appellant to
make a contribution to religious bodies, and thereby violates the
Establishment and Free Exercise Clause.
Held: the legislative purpose of the property tax exemptions is neither
the advancement nor the inhibition of religion; it is neither sponsorship
nor hostility. NY has determined thath certain entities that exist in
harmonious relationship to the community at large, and that foster its
moral and mental improvement should not be inhibited in their
activities by property taxation or the hazard of loss of its property for
nonpayment of taxes.
Nothing in this national attitude towards
religious tolerance and two centuries of uninterrupted freedom from
taxation has given the remotest sign of leading to an established church
or religion, it has operated affirmatively to guarantee the free exercise
of all forms of religious beliefs.
Lemon v. Kurtzman
403 US 602 (1971)
Facts: a Pennsylvania and Rhode Island statutes providing state aid to
church-related elementary and secondary schools. Both statutes are
challenged as violative of the Establishment and Free Exercise Clause.
The Pennsylvanian statutes reimburse the cost of teachers salaries,
textbooks, and instructional materials in specified secular subjects.
Rhode Island pays directly to teachers in nonpublic elementary shools a
supplement of 15% of their salaries. A federal court upheld the
Pennsylvania law while a District Court ruled that the Rhode Island law
fostered 'excessive entanglement'.

Held: the assistance was unconstitutional. There are three criteria that
should be used to assess legislation: "First, the statute must have a
secular legislative purpose; second, its principal or primary effect must
be one that neither advances or inhibits religion; finally, the statute
must not foster and excessive government Entanglement with religion."
The two statutes in question violate the third of these criteria. The
teachers whose salaries are being partially paid by the State are
religious agents who work under the control of religious officials. There
is an inherent conflict in this situation of which the state should remain
clear. To ensure that teachers play a non-ideological role would require
the state to become entangled with the church. Allowing this
relationship could lead to political problems in areas in which a large
number of students attend religious schools.
Tilton v Richardson
403 us 672 (1971)
facts: the higher education facilities act was passed in response to a
strong nationwide demand for the expansion of college and university
facilities to meet the sharply rising number of young people demanding
higher education. Act provides federal construction grants and loan for
college and university facilities, excluding any facility used or to be
used for sectarian instruction or as a place of religious worship, or
primarily in connection with any part of the program of a school or
department of divinity. The act stipulated that after twenty years, the
school could use the facilities for whatever purpose they chose.
Held:
the Court decided that the grants for non-religious school facilities did
not violate the Establishment Clause. it decided that the provision
limiting the statutes interest to twenty years was unconstitutional.
The primary effect of the Higher Education Facility Act was not to aid
religious institutes. The objective was to encourage education among
the countrys youth. In an earlier case (Bradfield v. Rob) the Court
decided that not all of financial aid to church-sponsored activities
violates the religious clauses of the constitution. The beneficiaries of
the act are secondary schools in which children are not as susceptible
to religious coercion and in which religious instruction is not as central
to the curriculum. Because the States interest in the structure remains
after twenty years, the provision giving the schools the ability to use
the facility for religious purposes is unconstitutional. This finding does
not require the invalidating of the entire act because it was not
essential to the whole law. The HEFA(higher education financial aid) did
not lead to excessive entanglement because the aid was aimed at
religiously neutral facilities. Also, the aid was non-ideological and was a
one-time, single-purpose program. The taxpayers rights were not
violated by the act because there was no coercion directed at the
practice
or
exercise
of
their
religious
beliefs.

In making this decision the Court did not discuss whether the assistance
to the religious schools for non-religious purposes would enhance their
ability to further their religious instruction. Taxpayers, whose money
was given to religious institutions, were not harmed provided their own
religious practices were not affected.
Agostini v. Felton
521 US 203 (1997)
Facts: A New York parochial school board challenged the District Court's
upholding of a twelve year-old decision in Aguilar v. Felton which
prohibited public school teachers from teaching in parochial schools.
The current proposal offered help to needy students in private schools
by sending public school teachers to tutor them after school. New York
was forced to offer remedial help to students through 'local educational
agencies'. Students did not need to attend public schools in order to be
eligible for the assistance. Those who were to receive tutoring were
students who a) reside in low income areas or b) failed or were at risk
of failing the state's student performance standards.

64 | P

LATON

Held: the Court allowed public school teachers to tutor private school
students in their private schools. Because of the Supreme Court's ruling
in Aguilar, the State had been forced to tutor students in either public
schools or mobile units outside of the parochial schools. As a result, the
school board was forced to deduct $7.9 million dollars from their
budget for transportation and establishing the mobile units. An earlier
ruling in Zobrest disavowed a ban of placing all public employees in
sectarian schools (a sign language interpreter had been provided for a
deaf student). Additionally, not all government aid that directly assists
in the educational function of religious schools is invalid. The location of
the classroom (either in public or religious schools) should not matter.
Furthermore, there is little difference between providing a sign
language interpreter, which the Court already allowed, and a tutor. The
interaction that would result between the state and church is allowable
because a relationship between the two is inevitable. This decision
reinforces the belief that the state can conduct public programs in
religious schools without becoming excessively entangled with the
religion. This is contrary to the earlier attitude that there must be an
absolute wall between public and religious schools.

The Free Exercise Clause


Under this part of the religious freedom guarantee, the state is
prohibited from unduly interfering with the outward
manifestations of ones beliefs and faith
With this free exercise guarantee, the individual is assured that
his religion would not be confined to thought only but also may
find expression through action. The Free Exercise Clause
accords absolute protection to individual religious convictions
and beliefs and proscribes government from questioning a
persons beliefs or imposing penalties or disabilities based
solely on those beliefs. The Clause extends protection to both
beliefs and unbelief
Further, while freedom to believe may be absolute, the liberty
to act on said belief is not so. The same may be subject to valid
state regulations designed to promote the general welfare and
the interests of society
The government must be able to show a compelling state
interest in order to justify any regulation that may impair
religious exercise
The Free Exercise Clause principally guarantees voluntarism,
although the Establishment Clause also assures voluntarism by
placing the burden of advancement of religious groups on their
intrinsic merits and not on the support of the state
West Virginia State Board of Education v. Barnette
319 US 624 (1943)
Facts: s part of instituting a required curriculum teaching American
values, the state of West Virginia forced students and teachers to
participate in saluting the flag. Failure to comply with this resulted in
expulsion and the student was considered illegally absent until
readmitted. A group of Jehovah's Witnesses refused to salute the flag
because it represented a graven image that was not to be recognized.
Held: the Court ruled that the school district violated the rights of
students by forcing them to salute the American flag. The refusal of the
students to say the pledge did not infringe on the rights of other
students. The flag salute required students to declare a belief that was
contrary to their faiths. The state did not claim that a clear and present
danger would be created if the students remained passive during the
pledge. Unlike the decision in Gobitis, this Court does not believe that
allowing an individual's rights to be supported over government
authority is a sign of a weak government. "Conscientious scruples have
not, in the course of the long struggle for religious toleration, relieved

the individual from obedience to a general law not aimed at the


promotion or restriction of religious beliefs. The mere possession of
religious convictions which contradict the relevant concerns of a
political society does not relieve the citizen from the discharge of
political responsibilities." Finally, compulsion is not a legitimate means
for creating national unity.
Gerona v. Secretary of Education
106 Phil. 2 (1959)
Facts: In accordance to Sec. 2 of R.A. 1265 of June 11, 1995, authorizing
and directing the Sec. of Education to issue or cause to be issued rules
and regulations for the proper conduct of the flag ceremony, he issued
Department Order No. 8 series of 1995 making it compulsory for public
and private schools to have daily flag ceremonies. Petitioners,
members of the religious group known as Jehovahs Witnesses, have
children attending a school in Masbate and said children refused to
salute the flag, sing the national anthem and recite the patriotic pledge.
As a result, they were expelled from school. They wrote to the
Secretary of Education petitioning that their children be allowed to
remain silent, stand at attention and that they be exempted from
saluting the flag, singing the national anthem and reciting the patriotic
pledge. Their ground is their belief as Jehovahs witnesses that they
must not bow down nor serve any image (Ex 20: 4-5), and the flag, for
them, is considered as an image of the State. However, the Secretary
denied such petition. Petitioners filed a complaint against them and
prayed for the declaration of Department Order No. 8 as invalid and
contrary to the Bill of Rights, but the trial court dismissed such
complaint.
Issue: WON Department Order No. 8, which made it compulsory for
public and private schools to have daily flag ceremonies, is valid and
constitutional
Held: The Filipino flag is not an image which requires religious
veneration but it is a symbol of the Republic of the Philippines. Also, the
flag salute is not a religious ceremony but an act of profession of love
and allegiance and pledge of loyalty to the fatherland which the flag
stands for. By the authority of the legislature, the Secretary of
Education was duly authorized to promulgate Department Order No. 8
which does not violate the Constitutional provision about freedom
and religion and exercise of religion. And since compliance with the
non-discriminatory and reasonable rules and regulations and school
discipline, which includes the observance of the flag ceremony, is a prerequisite to attendance in public schools, the failure and refusal of the
petitioners to participate in the flag ceremony led to their proper
exclusion and dismissal.
Ebralinag v. Division of Superintendent of schools of Cebu
219 SCRA 256 (1993)
Facts: The petitioners, assisted by their parents, are high school and
grade school students who belong to the religious group of Jehovahs
Witnesses. They were expelled from school by the public authorities in
Cebu because they refuse to salute the flag, sing the national anthem,
and recite the patriotic pledge as required by R.A. No. 1265 of July 11,
1995 and by Department Order No. 8 of July 21, 1995 of DECS which
made the flag ceremony compulsory in all educational institutions.
Jehovahs Witnesses admittedly teach their children not to salute the
flag, sing the national anthem and recite the patristic pledge for they
consider such acts as acts of worship which should be only given to
God. They feel bound by the Bibles command to guard themselves
against idols (1 John 5:21) as they also consider the flag as an image or
idol representing the State.
Issue: WON the school children who are members of a religious sect
known as Jehovahs Witnesses may be expelled from school for
refusing, on account of their beliefs, to take part in the flag ceremony,
which includes playing by a band or singing the Philippine national
anthem, saluting the Philippine flag, and reciting the patriotic pledge
Held: Although the petitioners do not take part in the compulsory flag
ceremony, they do not, however, engage in external acts or behavior
65 | P

LATON

that would offend their countrymen who express their love of country
through the flag ceremony. They quietly stand at attention during the
flag ceremony to show respect for the right of those who participate in
the solemn proceedings. Since they do not engage in disruptive
behavior, there is no warrant for their expulsion.
The sole justification for a prior restraint or limitation on the exercise of
religious freedom is the existence of a grave and present danger of a
character both grave and imminent, of a serious evil to public safety,
public morals, public health, or any other legitimate public interest,
that the State has a right and duty to prevent. Without such threat to
public safety, the expulsion of the petitioners from school is not
justified.
Also, the expulsion of the members Jehovahs Witnesses from schools
where they are enrolled will violate their right as citizens of the
Philippines, under the 1987 constitution, to receive free education,
because it is the duty of the State to protect and promote the right of
all citizens to quality education... and to make such education
accessible to all Sec. 1, Art. XIV.
Resolution on Motion for Reconsideration
251 SCRA 569 (1995)
Facts: The State moves for a reconsideration of the decision on March
1, 1993 which granted private respondents petition for certiorari and
prohibition and annulled the expulsion orders issued by said
respondents on the ground that the decision created and exemption in
favor of the Jehovah Witnesses religious group members , in violation
of the Establishment Clause of the Constitution. Although they refuse
to salute the flag, they are willing to stand quietly and peacefully at
attention in order not to disrupt the ceremony or to disturb those who
believe differently.
Issue: WON the compulsory flag salute is valid
Held: The refusal of the Jehovahs Witnesses in saluting the flag is
based on their religious belief which is shared by their entire
community. As the petitioners were expelled because of their religious
belief, such action, as stated by the Court, was against religious
practice. It is likewise apparent that the said orders and memoranda
would gravely endanger the free exercise of the religious beliefs of the
members of their sects. The refusal to salute the flag is not the same as
the refusal to pay taxes and to submit to compulsory vaccination since
the former has no threat to the life or health of the State. Thus, there is
no reason for compulsory or coercive flag salute. Although the
Constitution provides for a national flag, it does not give the State the
power to compel a salute to the flag.
Cantwell v. Connecticut
310 U.S. 296, 84 L Ed 1213, 60 S Ct 900 (1940)
Facts: Newton Cantwell and his 2 sons, Jesse and Russell, are members
of the religious group Jehovahs Witnesses. They were arrested because
they were engaged in selling books and soliciting in a neighborhood
street which is thickly populated by 90% Roman Catholics. Jesse
Cantwell, asked two men to listen to a phonograph record which
attacked their religion as they were Catholics. They were tempted to
strike Cantwell unless he went away. Thus, Cantwell went away and
there was no evidence that he was personally offensive or had entered
into any argument with them. However, the Cantwells were arrested in
violation of the General Statutes of Connecticut which prohibited
solicitation of money, services, subscriptions, or any valuable thing for
any alleged religious, charitable or philanthropic cause unless such
cause is approved by the secretary of public welfare council. They were
also convicted of the common law offense of inciting breach of peace.
Issue: WON the method adoption by Connecticut to that end
transgresses the liberty safeguarded by the Constitution
Held: The statute deprives the appellants of their liberty without due
process of law in contravention of the 14th Amendment for which they
were arrested. They were also right in their insistence that the Act is

not a regulation. If a certificate is procured, solicitation is permitted


without restraint, but in the absence of which, solicitation is altogether
prohibited. However, they argue that requiring them to procure a
certificate would be a restraint on the exercise of their religion. The
conviction of Jesse must be reversed since there was no evidence that
his deportment was noisy, truculent, overbearing or offensive. He
actually had not invaded a right or interest of the public or of the men
accosted. It likewise did not amount to a breach of the peace. The
petitioners communication raised no such clear and present danger
to public peace and order.
Iglesia Ni Cristo v. CA
259 SCRA 529 (1996)
Facts: Petitioners were members of the Iglesia ni Cristo religious
group. As they have TV programs aired during Saturdays and Sundays,
about the propagation of their religion, they submitted to the Board of
Review for Moving Pictures and Television the VTR series of their
program. However, the Board classified such series as X or not for
public viewing since they offend and constitute an attack against other
religions which is expressly prohibited by law. Petitioner filed two
courses of action against the Board and appealed to the Office of the
President which reversed the decision of the respondent Board and
allowed the series for public viewing. Petitioner also filed an case
against the respondent Board with the RTC of Quezon City on the
ground that the Board acted without jurisdiction or with grave abuse of
discretion in requiring them to submit their VTR tapes and x-rating
them. The RTC ordered the Board to grant the petitioner the necessary
permit for public viewing but it directed the petitioner to refrain from
attacking or offending other religions. Upon the petitioners motion for
reconsideration, the directive to refrain from attacking other religions
was deleted and the Board was prohibited from requiring the petitioner
to submit a review of their VTR of their program. The CA however
reversed the RTC decision and held that the Board has the jurisdiction
and power to review the petitioners TV program and that it did not act
with grave abuse of discretion. The CA likewise found the series
indecent, contrary to law and contrary to good customs.
Issues: (1) WON respondent Board has the power to review petitioners
TV program
(2) assuming it has power, WON it gravely abused its discretion when it
prohibited the airing of petitioners religious program series on the
ground that they constitute an attack against other religions and that
they are indecent, contrary to law and good customs.
Held: (1) PD No. 1986 gives the Board the power to screen, review,
examine all television programs, and to approve, delete, and/or
prohibit the exhibition and/or television broadcast of TV programs. It
also directs the Board to apply contemporary Filipino cultural values as
standard to determine those which are objectionable for being
immoral, indecent, contrary to law and/or good customs, injurious to
the prestige of the Republic of the Philippines and its people, or with a
dangerous tendency to encourage the commission of violence or of a
wrong or a crime. Thus, it has jurisdiction to review petitioners TV
program entitled Ang Iglesia ni Cristo.
*The right to religious profession and worship has a 2-fold aspect:
freedom to believe and freedom to act on ones belief. The first is
absolute as long as the belief is confined within the realm of thought
while the second is subject to regulation where the belief is translated
into external acts that affect the public welfare.
(2) However, the Court reversed the ruling of the CA since the claim of
the Board that such TV programs attack the Catholic faith, has no basis.
The CA has not even viewed the tapes as they were not presented as
evidence but it considered them as indecent, contrary to law and good
customs. Such ruling suppresses petitioners freedom of speech and
interferes with its right to free exercise of religion. The respondents
cannot also rely on the ground attacks another religion in x-rating
the religious program of the petitioner since such claim was merely
added by the Board to its Rules. They have also failed o apply the clear
and present danger rule since there is no showing of the type of harm
that the tapes would bring about especially the gravity ad imminence of
the threatened harm.
66 | P

LATON

US v. Ballard
322 U.S. 78, 88 L Ed 1148, 64 S Ct 882 (1944)
Facts: Respondents belong to the I Am movement which promotes
the religious belief and doctrines that a certain Guy W. Ballard, now
deceased, alias Saint Germain, had been selected and designated by
alleged ascertained masters as a divine messenger. They also
believed that the messages of such ascertained masters and of the
alleged divine entity, Saint Germain, would be transmitted to mankind
through Guy W. Ballard. Edna W. Ballard and Donald Ballard were also
allegedly selected as messengers just like Guy. They also believed that
they have supernatural powers which enable them to heal persons with
ailments. They were indicted and convicted for using and conspiring to
use mails to defraud in the organization and promotion of their
movement. They were also charged since they well knew that what
they believed in was false.
Issues: WON in respondents good faith, i.e. whether they did not
believe those things, that Jesus or Saint Germain came down and
dictated, or those things that they wrote, and preached, they used the
mail for the purpose of getting money
Held: On appeal, the Circuit CA reversed, holding that the decision of
the District Court in restricting the jury to the issue of respondents
good faith was error and granted a new trial. The SC granted certiorari
because of the importance of the question presented. The Circuit CA
held that the question of the truth of the representations concerning
respondents religious beliefs or doctrines should have been submitted
to the jury and thus, it remanded the case for a new trial.
We do not agree that the truth or verity of respondents religious
doctrines or beliefs should have been submitted to the jury. The
District court ruled properly when it withheld from the jury all
questions concerning the truth or falsity of the religious beliefs or
doctrines of respondents. The law knows no heresy, and is
committed to the support of no dogma, the establishment of no sect
(Watson v Jones, 13 Wall. 679, 728). The First Amendment does not
only forestall compulsion by law of the acceptance of any creed or
the practice of any form of worship but it also safeguards the free
exercise of the chosen form of religion (Cantwell v Connecticut, 310
U.S. 296, 303). It also does not select any one group or any type of
religion for preferred treatment.
American Bible Society v. City of Manila
101 Phil. 386 (1957)
FACTS: The American Bible Society is a foreign, non-stock, non-profit,
religious, missionary corporation doing business in the Philippines
through its Philippine agency established in manila in 1898. In the
course of its ministry, the Society has been distributing and selling
bibles and/or gospel portions thereof. Sometime in 1953, the acting
City treasurer of manila required the Society to secure the
corresponding Mayors permit and license fees. The Society paid the
accumulated permit and license fees (P5,891.45) under protest and
then filed a complaint in court to question the constitutionality and
legality of the ordinances under which the said fees were being
collected.
ISSUE: Whether or not Ordinance 2529 (requiring every entity engaged
in business to pay a license fee based on gross sales) and ordinance
3000 (requiring that a municipal permit to be first obtained before
engaging in business) could be made applicable to the American Bible
Society.
THE COURTS RULING: Defendant was sentenced to return to the
plaintiff the sum of P5,891.45 unduly collected from it.
REASON: The Constitutional guarantee of the free exercise and
enjoyment of religious profession and worship carries with it the right
to disseminate religious information. Any restraint of such right can
only be justified like other restraints of freedom of expression on the
grounds that there is a CLEAR AND PRESENT DANGER OF ANY
SUBSTANTIVE EVIL which the State has the right to prevent.

It is a flat license tax levied and collected as a condition to the pursuit of


activities whose enjoyment is guaranteed by the constitutional liberties
of press and religion and inevitably tends to suppress their exercise.
Ordinance No. 3000 does not impose any charge upon the enjoyment
of a right granted by the Constitution, nor tax the exercise of religious
practice.
In Re Summers
325 US 561, 89 L Ed 1795, 65 S Ct 1307 (1945)
FACTS: Petitioners prayer for admission to the practice of law in Illinois
was denied by the States Supreme Court allegedly on the sole ground
that he is a conscientious objector to war, or, because of his
conscientious scruples against participation in war. From the record, it
appears that Clyde Wilson Summers has complied with all the
prerequisites for admission to the bar of Illinois except that he has not
obtained the certificate of the Committee on Character and fitness,
which committee declined to sign a favourable certificate on the
ground that petitioner was a conscientious objector to war.
THE COURTS RULING: The decision of the Supreme Court of Illinois
denying the petitioners admission to the practice of law in Illinois was
AFFIRMED.
REASON: A conscientious belief in nonviolence to the extent that the
believer will not use force to prevent wrong, no matter how
aggravated, and so cannot swear in good faith to support the Illinois
Constitution, must disqualify such a believer for admission.
The responsibility for choice as to the personnel of its bar rests with
Illinois. Only a decision which violated a federal right secured by the
Fourteenth Amendment would authorize our intervention...We cannot
say that any such purpose to discriminate motivated the action of the
Illinois Supreme Court.
It is impossible for us to conclude that the insistence of Illinois that an
officer who is charged with the administration of justice must take an
oath to support the Constitution of Illinois and Illinois interpretation of
that oath to require a willingness to perform military service violates
the principles of religious freedom.
*However, Mr. Justice Black filed a DISSENT stating among other things
that, Under our (US) Constitution, men are punished for what they do
or fail to do, and not for what they think and believe.
Wisconsin v. Yoder
406 US 205, 32 L Ed 2d 15, 92 S Ct 1526 (1972)
FACTS: Respondents were members of the Amish religion. They and
their families are residents of Green County, Wisconsin. Wisconsins
compulsory school attendance law required them to cause their
children to attend public or private school until reaching age 16, but the
respondents declined to send their children. They believed that, by
sending their children to high school, they would not only expose
themselves to the danger of censure of the church community, but also
endanger their own salvation and that of their children. The Wisconsin
Supreme Court sustained respondents claim under the Free Exercise
Clause of the First Amendment, and reversed the convictions.
THE COURTS RULING: The First and Fourteenth Amendments prevent
the State from compelling respondents to cause their children to attend
formal high school to age 16.
REASON: Old Order Amish communities today are characterized by a
fundamental belief that salvation requires life in a church community
separate and apart from the world and worldly influence. In the Amish
belief, higher learning tends to develop values they reject as influences
that alienate man from God.
As the record strongly shows, the values and programs of the modern
secondary school are in sharp conflict with the fundamental mode of
67 | P

LATON

life mandated by the Amish religion.


The enforcement of the States requirement of compulsory formal
education after the eighth grade would gravely endanger, if not
destroy, the free exercise of respondents religious beliefs.
The Amish alternative to formal secondary school education has
enabled them to function effectively in their day-to-day life under selfimposed limitations on relations with the world, and to survive and
prosper in contemporary society as a separate, sharply identifiable and
highly self-sufficient community for more than 200 years in this
country.

*To have the protection of the religion Clauses, the claims must be
rooted in religious belief.
Victoriano v. Elizalde Rope Workers Union
59 SCRA 54 (1974)
FACTS: Appellee, a member of the religious sect known as the Iglesia
ni Cristo had been in the employ of the Elizalde Rope Factory, Inc.
since 1958. As such employee, he was a member of the Elizalde Rope
Workers Union. A closed-shop provision was contained in the
collective bargaining agreement which required membership in the
Union as a condition of employment for all permanent employees.
However, RA 3350 was enacted, providing that, such agreement shall
not cover members of any religious sects which prohibit affiliation of
their members in any such labor organization. Thereupon, the Union
wrote a formal letter to the Company asking the latter to separate
appellee from the service in view of the fact that he was resigning from
the Union as a member. The management of the company informed
appellee that unless he could achieve a satisfactory arrangement with
the Union, the company would be constrained to dismiss him from the
service.
THE COURTS RULING: The decision of the Court of First Instance of
Manila enjoining the Company from dismissing the appellee and
sentencing the Union to pay Appellee P500 for attorneys fees and
costs of the action is AFFIRMED.

familiar inscribed yellow T-shirts, they started to march down said


street with raised clenched fists and shouts of government invectives.
However, they were barred from proceeding any further, on the ground
that St. Jude Chapel was located within the Malacaang security area.
However, because of the alleged warning given them by respondent
Major Lariosa that any similar attempt by petitioners to enter the
church in the future would likewise be prevented, petitioners took this
present recourse.
THE COURTS RULING: The instant petition is hereby dismissed.
REASON: Undoubtedly, the yellow T-shirts worn by some of the
marchers, their raised clenched fists, and chants of anti-government
slogans strongly tend to substantiate the respondents allegation that
the petitioners intend to conduct an anti-government demonstration.
While it is beyond debate that every citizen has the undeniable and
inviolable right to religious freedom, the exercise thereof, and of all
fundamental rights for that matter, must be done in good faith.
The restriction imposed is necessary to maintain the smooth
functioning of the executive branch of the government, which
petitioners mass action would certainly disrupt.
DOCTRINE LAID DOWN IN THIS CASE: If the exercise of said religious
belief clashes with the established institutions of society and with the
law, then the former must yield and give way to the latter.
However, several justices filed separate opinions:
FERNANDO, C.J. concurs in the result but dissents insofar as the
opinion fails to declare that the freedom of exercise of religious
profession and worship could only be limited by the existence of a clear
and present danger of a substantive evil.
ABAD SANTOS, J. (dissenting) It is highly presumptuous for both the
respondents and this Court to attribute unstated and unadmitted
motives to the petitioners. I fail to perceive the presence of any clear
danger to the security of Malacaang due to the action of petitioners.

REASON: It is clear that the right to join a union includes the right to
abstain from joining any union.

MELENCIO-HERRERA, J. (dissenting) The location of the St. Jude


Chapel within the perimeter of the Malacaang security area is not, to
my mind, sufficient reason for a prior restraint on petitioners right to
freedom of religious worship.

RA 3350 merely excludes ipso jure from the application and coverage of
the closed shop agreement the employees belonging to any religious
sects which prohibit affiliation of their members with any labor
organization. In spite of any closed shop agreement, members of said
religious sects cannot be refused employment or dismissed from their
jobs on the sole ground that they are not members of the collective
bargaining union.

RELOVA, J. (Separate vote and statement) The fact that petitioners


chose a Tuesday to hear mass and/or pray for their special intention
negates the suspicion that they were out to stage a demonstration.
Respondents should have allowed petitioners to hear mass and/or pray
and, thereafter, see what they would do.

The free exercise of religious profession or belief is superior to contract


rights. It is only where unavoidably necessary to prevent an IMMEDIATE
AND GRAVE DANGER to the security and welfare of the community that
infringement of religious freedom may be justified, and only to the
smallest extent necessary to avoid the danger.
The exemption from the effects of the closed shop agreement does not
directly advance, or diminish, the interests of any particular religion.
The benefit upon the religious sects is merely incidental and indirect.
*A statute in order to withstand the strictures of constitutional
prohibition, must have a secular legislative purpose and a primary
effect that neither advances nor inhibits religion.
German v. Barangan
135 SCRA 514 (1985)
FACTS: Petitioners composed of about 50 businessman, students and
office employeesconverged at J.P. laurel Street, Manila, for the
ostensible purpose of hearing mass at the St. Jude Chapel which adjoins
the Malacaang grounds located in the same street. Wearing the

Estrada v. Escritor
408 SCRA 1 (2003)
FACTS: Compalinant Alejandro Estrada wrote to the presiding judge of
Branch 253, Regional Trial Court of Las Pias City, requesting for an
investigation of rumors that respondent Soleded Escritor, court
interpreter in said court, is living with a man not her husband. They
allegedly have a child of eighteen to twenty years old. She admitted
that she has been living with Luciano Quilapio Jr., without the benefit of
marriage for twenty years and that they have a son. Quilapio was
likewise married at that time, but had been separated in fact from his
wife. But as a member of the religious sect known as the Jehovahs
Witnesses and the Watch Tower and Bible Tract Society, their conjugal
arrangement is in conformity with their religious beliefs. In fact, the
respondent has executed a Declaration of Pledging Faithfulness that is
recognized as giving the parties the right to marital relationship even if
not recognized by civil authorities.
PRINCIPAL ISSUE: Whether or not respondent should be found guilty
of the administrative charge of gross and immoral conduct.
SUB-ISSUE: Whether or not respondents right to religious freedom
68 | P

LATON

should carve out an exception from the prevailing jurisprudence on


illicit relations for which government employees are held
administratively liable.
THE COURTS RULING: The case is REMANDED to the Office of the
Court Administrator.
The Solicitor General is ordered to intervene in the case where it will be
given the opportunity: (a) to examine the sincerity and centrality of
respondents claimed religious belief and practice; (b) to present
evidence on the states compelling interest to respondents religious
belief and practice and (c) that the means the state adopts in pursuing
its interest is the least restrictive to respondents religious freedom.
REASON: The jurisdiction of the Court extends only to public and
secular morality.
Should the Court declare respondents conduct as immoral and held
her administratively liable, the Court will be holding that in the realm of
public morality, her conduct is reprehensible or there are state
interests overriding her religious freedom. But in so ruling, the Court
does not and cannot say that her conduct should be made
reprehensible in the realm of her church where it is presently
sanctioned and that she is answerable for her immorality to Jehovah
God nor that other religions prohibiting her conduct are correct.
In any event, even if the court deems sufficient respondents evidence
on the sincerity of her religious belief and centrality to her faith, the
case at bar cannot still be decided using the compelling state interest
test.
The government should be given the opportunity to demonstrate the
compelling state interest it seeks to uphold in opposing the
respondents stance that her conjugal arrangement is not immoral and
punishable as it comes within the scope of the free exercise protection.
*The benevolent neutrality approach is used in interpreting the
religious clauses in the Philippine Constitution.
*Benevolent Neutrality Approach looks further than the secular
purposes of government action and examines the effect of these
actions on religious exercise.
*Tests must be applied to draw the line between permissible and
forbidden religious exercise:
CLEAR AND PRESENT DANGER TEST; IMMEDIATE AND GRAVE DANGER
TEST involves religious speech as this test is often used in cases of
freedom of expression.
COMPELING STATE INTEREST TEST is proper where conduct is
involved.

The Tension Between the Establishment Clause and the Free


Exercise Clause
The two subclauses of the provision on religion are meant to
ensure the guarantee that they were meant to vivify

in carefully balancing the interests and values implicated in


each case that may be brought before them
Religious Affairs, Intramural Disputes and Secular Jurisdiction
While generally it should be no business of the government
trying to meddle in problems among members of the same
sectarian group, the same would hold true only on matters of
purely religious dogma and discipline. It cannot, withhold its
judicial or administrative machineries when the dispute spills
over to the secular concerns of society
When the problem is about more mundane matters, however,
such as when they involve conflicting claims to the same
property, or when they are concerned with civil aspects of
membership in society, such as employment, then the courts
and secular authorities would have to interpose their power
In matters purely ecclesiastical the decisions of the proper
church tribunals are conclusive upon the civil tribunals
Austria v. NLRC
312 SCRA 410 (1999)
Petitioner was a pastor of private respondent Central Philippine Union
Mission Corporation of the Seventh Day Adventist, a religious
corporation duly organized and existing under Philippine Law.
Petitioner worked with the SDA for 28 years. Petitioner one day
received a letter of dismissal citing misappropriation of funds, willful
breach of trust, serious misconduct, gross and habitual neglect of duties
and the like as grounds for the termination of his services. Petitioner
then filed a complaint against SDA and its officers before the Labor
Arbiter for illegal dismissal and ruled in his favor. On appeal, the NLRC
vacated the arbiters decision and dismissed the case for want of merit.
The private respondents filed a motion for reconsideration and ruled
that it has no jurisdiction over the matter because it involves
ecclesiastical affair to which state cannot interfere.
HELD: The separation of church and state finds no application in this
case. What is involved is the relationship of the church as an employer
and the minister as an employee. The grounds invoked for petitioners
dismissal are all based on Article 282 of the Labor code which
enumerates just causes for termination of employment. By this alone, it
is palpable that the reason for petitioners dismissal from the service is
not religious in nature. And it is clear that when the SDA dismissed the
petitioner, it was merely exercising its management prerogative to fire
an employee which it believes to be unfit for the job.
In a vain attempt to support their claim of breach of trust, private
respondents pin on the petitioner the alleged non-remittance of the
tithes collected by his wife. As proven by convincing and substantial
evidence it was the wife of the petitioner who collected the tithes and
failed to remit the same and not the petitioner. There being no
conspiracy or collusion, the petitioner cannot be held accountable for
the alleged infraction of his wife.

The First Amendment forbids both the abridgment of the free


exercise of religion and the enactment of laws respecting an
establishment of religion

Wherefore, the petition for certiorari is granted. The challenged


resolution of the public respondent NLRC, is nullified and set aside. The
decision of the Labor Arbiter is reinstated and hereby affirmed.

Even as they might have common purpose, the two clauses


may somehow come into conflict if they were allowed to be
extended to their logical end

Islamic Dawah Council Of The Phils., Inc v. Office of the Executive


Secretary
405 SCRA 497 (2003)

There is a natural antagonism between a command not to


establish religion and a command not to inhibit its practice; this
tension between the religion clauses often leaves the courts
with a choice between competing values in religion cases.
Resolving the tension, of course, would again involve the courts

Petitioner is a non-governmental organization that extends voluntary


services to the Filipino, especially to Muslim communities. It claims to
be a federation of national Islamic organization and an active member
of international organizations such as RISEAP which had accredited the
petitioner to issue halal certifications in the Philippines. However, EO
46 was issued creating the Philippine Halal Certification Scheme and
69 | P

LATON

designing the Office of the Muslim Affairs to oversee its


implementation. OMA sent letters to food manufacturers asking them
to secure the halal certification only from it lest they violate EO 46 and
RA 4109. As a result, petitioner lost revenues after food manufacturers
stopped securing certification from it. Petitioner filed instant petition
for prohibition, contending that EO 46 violates the constitutional
provision on separation of the church and state since the halal
certification scheme is a function only of religious organization.
HELD: The petition is granted. OMA deals with the societal, legal,
political and economic concerns of the Muslim community as a
national cultural community and not as a religious group. Without
doubt, classifying food product as halal is a religious function because
the standards used are drawn from the Quran and Islamic beliefs. By
giving OMA the exclusive power to classify food products as halal, EO
46 encroached on the religious freedom of Muslim organizations to
interpret for Filipino Muslims what food product are fit for Muslim
consumption. The halal certifications issued by petitioner and similar
organizations come forward as the religious approval of a food product
fit for Muslim consumption.
Wherefore, the petition is granted. EO 46 is hereby declared null and
void. Consequently, respondents are prohibited from enforcing the
same.

Conscientious Objectors
The right of conscientious objectors is based more on statutory
provisions than the constitutional freedom of religion. It is a
governmental accommodation extended to those, who out of
religious scruples, may find themselves opposed to war and the
concomitant taking of lives
In order to qualify for classification as a conscientious objector,
a registrant must satisfy three basic tests:
1. He must show that he is conscientiously oppose to war in any
form
2. He must show that his opposition is based upon religious
training and belief, as the term has been construed in our
decisions
3. He must show that this objection is sincere
Religious Test and Exercise of Political and Civil Rights
The last sentence of the religion clause declares that no
religious test shall be required for the exercise of civil and
political rights. This should follow if the State is really to be free
from meddling into religious affairs
This prohibition against religious test has reference to the
historically and constitutionally discredited policy of probing
religious beliefs by test oaths or limiting public offices to
persons who have, or perhaps more properly profess to have, a
belief in some particular kind of religious concept
McDaniel v. Paty
435 US 618, 55 L Ed 2d 593, 98 S Ct 1322 (1978)
Paty, a candidate for delegate to the Tennessee constitutional
convention, sued for a declaratory judgment that McDaniel, an
opponent who was Baptist minister, was disqualified as serving as
delegate. That court held that the statutory provision violated the First
and Fourteenth Amendments. After the election, the Tennessee SC
reversed, holding that the clergy disqualification imposed no burden on
religious belief, and restricted religious action . . . [only] in the law
making process of the government where religious action is
absolutely prohibited by the establishment clause.
WON, Minister or priest is barred from serving as delegates to the

states limited constitutional convention deprived appellant McDaniel,


of the right to the free exercise of religion guaranteed by First and
Fourteenth Amendment.
Tennessee has failed to demonstrate that its views of the dangers of
clergy participation in the political process have not lost whatever
validity they may once have enjoyed. The essence of the rationale
restriction on ministers is that, if elected to public office, they will
necessary exercise their powers and influence and promote the
interests of one sect or thwart the interest of another, thus pitting one
against the others, contrary to the anti-establishment principle with its
command of neutrality. However, the American experience provides no
persuasive support for the fear that clergymen in the public office will
be less careful of anti-establishments interests or less faithful to their
oaths of civil office than their unordained counterparts.
We hold that S4 of ch. 848 violates McDaniels First Amendment right
to free exercise of his religion made applicable to the states by the
Fourteenth Amendment. Accordingly, the judgment of Tennessee SC is
reversed, and the case is remanded to that court for further
proceedings.

Religious Leaders and Participation in Political Affairs


While on one side may be the claim that as members of society,
as citizens themselves, such religious leaders and their flock are
still entitled to participate in political affairs and help chart the
course of the country, on the other side is the need to keep
those same sectarian principals from intruding into purely
secular concerns thereby limiting them to their spiritual
province
Additional Cases
(G) Freedom of Religion [1-2]
Chapter 8
Liberty of Abode and Freedom of Movement
The liberty of abode and of changing the same within the
limits prescribed by law shall not be impaired except upon
lawful order of the court. Neither shall the right to travel be
impaired except in the interest of national security, public
13
safety, or public health, as may be provided by law.
The assurance of the right to live and stay where [one] wants
within legal bounds. Thus, unless there be important societal
considerations and interests that are implicated by a persons
decision to stay anywhere, he should be given complete
freedom of choice as to where he may want to dwell and set up
his abode. This right, pursuant to the Constitution, may only be
impaired by lawful order of the court
Corollary to this liberty to establish ones abode is the freedom
of locomotion, of moving about, travelling from place to place
as an individual may so deem appropriate or desirable
If there are no countervailing national security, public safety or
public health interests that are implicated, a person should be
free to go where he pleases
Also, another limitation on freedom of locomotion involves
those applicable to people who might have pending criminal
cases, in which event their movement or choice of abode may
be restricted by the courts as part of the condition of their
being out on bail

13

CONSTITUTION, Art. III, 6

70 | P

LATON

Rubi v. Provincial Board of Mindoro


39 Phil. 660 (1919)
Resolution No. 25 of the Provincial Board of Mindoro creates for a
permanent settlement of the Manguianes. Pursuant to Resolution No.
25 and Section 2145 of the Administrative Code of 1917, the Governor
directed the Mangyanes in the province to take up their habitation on
Sitio of Tigbao on Lake Naujan. Any mangyan who refuses to comply
with the order shall, upon conviction, be sentenced to imprisonment.
Petitioners challenge the constitutionality of Section 2145 of the AC on
among others, due process and equal protection grounds.
The reasons for the action of the Resolution of the Provincial board are
as follows: (1) the failure of former attempts for the advancement of
non-Christian people of the province; (2) the only successful method for
educating the Mangyans was to oblige them to live in a permanent
settlement; (3) the protection of the Mangyans; (4) the protection of
the public forests in which they roam; and (5) the necessity of
introducing civilized customs among the Mangyans. What the
government wished to do by bringing them into a reservation was to
gather the children for educational purposes, and to improve the health
and ,orals was in fine, to begin the process of civilization.
The Mangyans, in order to fulfill this governmental policy, must be
confined for a time, for their own good and good of the country.
The action pursuant to section 2145 of the AC does not deprive a
person of hid liberty without due process of law and does not deny to
him the equal protection of laws, and that confinement in reservation
in accordance with said section does not constitute slavery and
involuntary servitude. Further, section 2145 of the AC is a legitimate
exertion of the police power. Section 2145 of the AC of 1917 is
constitutional.
Petitioners are not lawfully imprisoned or restrained of their liberty.
Habeas corpus can, therefore, not issue.
Edwards v. California
62 S Ct 164 (1941)
FACTS: Edwards, a citizen of the United States and a resident of
California, bought to California from Texas his indigent brother in law,
likewise a citizen of the US and a resident of Texas. He was
subsequently charged with and convicted of violating a California law
which penalizes one who knowingly brings into the State an indigent
person who is not a resident of the State, knowing him to be such an
indigent person.
ISSUE: WON the prohibition embodied in said California law against the
bringing or transportation of indigent persons into California is within
the police power of that State.
RULING: NO. It is not a valid exercise of the police power of California, it
imposes an unconstitutional burden upon interstate commerce, and
thus the conviction under it cannot be sustained. Article 1, 8 of the
Constitution delegates to the Congress the authority to regulate
interstate commerce. And it is settled beyond question that the
transportation of persons is commerce within the meaning of that
provision. But this does not mean that there are no boundaries to the
permissible area of State legislative authority. There are. And none is
more certain than the prohibition against attempts on the part of any
single state to isolate itself from difficulties common to all of them by
restraining the transportation of persons and property across its
borders. The California law in question is squarely in conflict with this as
its express purpose and inevitable effect is to prohibit the
transportation of indigent persons across the California border.
CONCURRING OPINION: The right to move freely from State to State is
an incident of national citizenship protected by the privileges and
immunities clause of the Fourteenth Amendment against state
interference.
Zemel v. Rusk
85 S Ct 1271 (1965)

FACTS: After the US had broken diplomatic relations with Cuba and the
Department of State had eliminated Cuba from the area for which
passports were not required, Louis Zemel applied to have his passport
validated for travel to Cuba to satisfy [his] curiosity . . . and to make
[him] a better informed citizen. His request was denied, and he filed
suit seeking a judgment declaring that:
1. He was entitled under the Constitution and the laws of the
US to travel to Cuba and to have his passport validated for
that purpose;
2. The Secretary of States travel restrictions were invalid; and
that
3. The Passport Act of 1926 and Section 215 of the Immigration
and Nationality Act of 1952 were unconstitutional.
ISSUE: WON the Secretary of State is statutorily authorized to refuse to
validate the passports of US citizens for travel to Cuba, and, if he is,
WON the exercise of that authority is constitutionally permissible.
RULING: YES. The court thinks that the Passport Act of 1926 embodies a
grant of authority to the Executive to refuse to validate the passports of
US citizens for travel to Cuba. That Act provides, in pertinent part: The
Secretary of State may grant and issue passports . . . under such rules as
the President shall designate and prescribe for and on behalf of the
United States. . . . The right to travel within the United States is, of
course, also constitutionally protected. But that freedom does not
mean that areas ravaged by flood, fire or pestilence cannot be
quarantined when it can be demonstrated that unlimited travel to the
area would directly and materially interfere with the safety and welfare
of the area or the nation as a whole. So it is with international travel (as
is the case here).
Marcos v. Manglapuz
177 SCRA 668(1989)
FACTS: In February 1986, Ferdinand E. Marcos was deposed from the
presidency via the non-violent people power revolution and forced
into exile in Hawaii. Corazon Aquino was declared President of the
Republic. Soon, the Marcos family sought to be allowed to return to the
country but President Aquino barred their return. The Marcos family
filed the instant petition for mandamus and prohibition asking the
Court to order the respondents to issue travel documents to Mr.
Marcos and the immediate members of his family and to enjoin the
implementation of the Presidents decision to bar their return from the
Philippines.
ISSUE: WON, in the exercise of the powers granted by the Constitution,
the President may prohibit the Marcoses from returning to the
Philippines.
RULING: YES. The President did not act arbitrarily or with grave abuse
of discretion in determining that the return of former President Marcos
and his family at the present time and under present circumstances
poses a serious threat to national interest and welfare and in
prohibiting their return to the Philippines. That the President has such
power under the Constitution has been recognized by members of the
Legislature. The request or demand of the Marcoses to be allowed to
return to the Philippines cannot be considered in the light solely of the
constitutional provisions guaranteeing liberty of abode and the right to
travel, subject to certain exceptions, or of case law which clearly never
contemplated situations even remotely similar to the present one. It
must be treated as a matter that is appropriately addressed to those
residual unstated powers of the President which are implicit in and
correlative to the paramount duty residing in that office to safeguard
and protect general welfare. In that context, such request or demand
should submit to the exercise of a broader discretion on the part of the
President to determine whether it must be granted or denied. The
petition was dismissed.
DISSENTING OPINION: It is of the dissenters belief that Marcos, as a
citizen of the Philippines, is entitled to return to and live and die in
his own country. The government failed dismally to show that the
return of Marcos dead or alive would pose a threat to the national
71 | P

LATON

security as it had alleged.


Resolution on the Motion for Reconsideration
178 SCRA 760 (1989)
FACTS: The SC, in its decision on Marcos v. Manglapuz, dismissed said
petition (close fight, voting 8 to 7). On September 28, 1989, Marcos
died in Honolulu, Hawaii. President Aquino then declared that his
remains will not be allowed to be brought to the Philippines until such
time as the government shall otherwise decide, and thus the
petitioners filed their motion for reconsideration.
RULING: The death of Marcos has not changed the factual scenario
under which the Courts decision was rendered. The threats to the
government have not been shown to have ceased and on the contrary,
instead of erasing fears as to the destabilization that will be caused by
the return of the Marcoses, Mrs. Marcos reinforced the basis for the
decision to bar their return when she called President Aquino illegal,
claiming that Mr. Marcos is the legal President of the Philippines, and
declared that the matter should be brought to all the courts of the
world. President Aquinos decision to bar the return of the Marcoses
and subsequently, Mr. Marcos remains at the present time and under
the present circumstances is in compliance with her bounden duty to
protect and promote the interest of the people. In the absence of a
clear showing that she had acted with arbitrariness or with grave abuse
of discretion in arriving at this decision, the Court will not enjoin its
implementation. The motion was thus denied for lack of merit.

Additional Cases
(H) Liberty of Abode and Freedom of Movement [1]

Chapter 9
Right to Information
The right of the people to information on matters of public
concern shall be recognized. Access to official records, and to
documents and papers pertaining to official acts, transactions,
or decisions, as well as to government research data used as
basis for policy development, shall be afforded the citizen,
14
subject to such limitations as may be provided by law.
If the people are really and effectively to participate in charting
their own destiny and that of their government, then they must
be given sufficient data and information upon which to base
any intelligent and meaningful decisions
The incorporation of the right to information in the
Constitution is a recognition of the fundamental role of free
exchange of information in a democracy--there can be no
realistic perception by the public of the nations problems, nor
a meaningful democratic decision-making if they are denied
access to information of general interest
A person could not be presumed to know what was not open to
view, much more so if deliberately kept from sight
Publication and Effectivity
The Civil Code proclaim: Ignorance of the law excuses no one
from compliance therewith. But it would hardly be fair for the
law to presume knowledge if the government does not
disseminate the laws and rules that govern the norms of
conduct that it expects from its citizens

14

CONSTITUTION, Art. III, 7

Publication should be a prerequisite to effectivity


In so far as publication is concerned, it would be preferable that
the same be done through the means which is more readily
accessible to the people, i.e., newspapers of general circulation,
in addition to the traditional manner of doing through the
Official Gazette
Taada v. Tuvera
136 SCRA 27 (1985)
FACTS: Invoking the peoples right to be informed on matters of public
concern (Sec. 6, Art. IV of the 1973 Constitution), petitioners seek a writ
of mandamus to compel respondent public officials to publish or cause
the publication in the Official Gazette of various presidential decrees,
letters of instruction, letters of implementation, GOs, EOs,
proclamations and administrative orders. The respondents seek the
outright dismissal of the petition on the ground that petitioners have
no legal personality or standing to bring said petition, as there is no
showing that petitioners are personally and directly affected by the
alleged non-publication of the presidential issuances concerned.
ISSUE: WON publication in the Official Gazette is a sine qua non
requirement for the effectivity of laws where the laws themselves
provide for their own effectivity dates, and WON petitioners have legal
standing to bring the petition in the first place.
RULING: YES. The SC held that Article 2 of the Civil Code does not
preclude the requirement of publication in the Official Gazette, even if
the law itself provides for the date of its effectivity. Publication of all
presidential issuances of a public nature or of general applicability
is mandated by law. Presidential issuances of general application which
have not been published shall have no force and effect. This is covered
by Section 1 of Commonwealth Act 638 which object is to give the
general public adequate notice of the various laws which are to
regulate their actions and conduct as citizens. Without such notice and
publication, there would be no basis for the application of the maxim
ignorantia legis non excusat. As for the second issue, when the
question is one of public right and the object of the petition is to
procure the enforcement of a public duty, the people are regarded as
the real party in interest and need not show that he has any legal or
special interest in the result, it being sufficient to show that he is a
citizen and as such interested in the execution of the laws.
Resolution on the Motion for Reconsideration
146 SCRA 446 (1986)
FACTS: Petitioners move for clarification in regard to the following:
a.
Meaning of law of public nature or general applicability;
b. Whether distinction must be made between laws of general
applicability and laws which are not;
c.
Meaning of publication;
d. Where the publication is to be made; and
e. When the publication is to be made.
RULING: The SC held that clause unless it is otherwise provided refers
to the date of effectivity and not to the requirement of publication
itself, which cannot in any event be omitted. Petition is indispensable in
any case, but the legislature may in its discretion provide that the usual
15-day period be shortened or extended. The term laws should refer
to all laws and not only to those of general application, for strictly
speaking all laws relate to the people in general albeit there are some
that do not apply to them directly. As such, all statutes, including ones
of local application and private laws, shall be published as condition for
their effectivity. Interpretative regulations and those merely internal in
nature need not be published, and so are letters of instruction issued
by administrative superiors concerning rules and guidelines to be
followed by their subordinates in the performance of duties.
Publication should be done in the Official Gazette (Art. 2 of the Civil
Code was not yet amended to its present form at the time of this
decision). Publication must be made forthwith, or at least as soon as
possible, to give effect to the law pursuant to said Art. 2.
72 | P

LATON

Bengzon v. Drilon
208 SCRA 133 (1992)
FACTS: The petitioners are retired Justices of the Supreme Court and
Court of Appeals who are currently receiving monthly pensions under
R.A. 910 as amended by R.A. 1797.
R.A. 910 provides for the retirement pension of Justices of the Supreme
Court and Court of Appeals, it was amended in 1957 by R.A. 1797
providing in Section 3-A thereof for automatic adjustments of pensions
if salaries of justices were increased or decreased. Identical retirement
benefits were also given, by President Marcos, to the members of the
constitutional commissions under R.A. 3595 and to the members of the
Armed Forces under P.D. 578. Two months later, he issued P.D. 644
repealing Section 3-A of the previous R.As and P.Ds. Subsequently, the
president decreed the restoration of the automatic readjustment of the
retirement pension of officers and enlisted men. A later decree also
issued providing for the automatic readjustment of the pensions of
members of the Armed Forces. On the other hand, the same was not
restored for the retired Justices of the Supreme Court and Court of
Appeals.
In 1990, Congress, realizing the unfairness of the discrimination,
approved a bill for the reenactment of the repealed provisions of R.A.
1797 and R.A. 3595. However, the president vetoed the bill, stating that
it would erode the very foundation of the governments collective
effort to adhere faithfully to and enforce strictly the policy on
standardization of compensation as articulated in the Compensation
and Position Classification Act of 1989. The following year, retired
Justices of the Court of Appeals filed a letter/petition asking the court
for a readjustment of their monthly pensions in accordance with R.A.
1797 claiming that P.D. 644 repealing R.A. 1797 did not become law as
there was no valid publication, it only appeared for the first time in the
supplemental issue of the Official Gazette. Since, P.D. 644 has no
binding force and effect of law, it therefore did not repeal R.A. 1797.
The court, in its resolution, acted favorably on the request and
pursuant to the resolution, Congress included in the General
Appropriations Bill for the Fiscal Year 1992 certain appropriations for
the judiciary intended for the payment of the adjusted pension rates
due to them. The President vetoed provisions of the bill related to
aforesaid appropriations, reiterating the earlier reasons for vetoing the
former house bill.
ISSUE: (1) Whether or not the veto by the President (cory) of certain
provisions in the general appropriations act for the fiscal year 1992
relating to the payment of the adjusted pensions of retired justices of
the aforementioned courts constitutional.
(2) Whether or not P.D. 644 became law.
DECISION: Petition is GRANTED. (1) The court declared the questioned
veto invalid, set aside as illegal and unconstitutional. (2) The court
agreed that P.D. 644 never became a law.
REASON: (1) It is invalid for being in contravention of the constitutional
guidelines in the exercise of the same, i.e., that it vetoed provisions and
not items. And also what were really vetoed are the R.A. 1797 of 1957
and the Resolution of the Supreme Court in 1991, thus no President
may veto the provisions of a law enacted 35 yrs. before his or her term
of office and neither may the President set aside or reverse a final and
executory judgment of the Supreme Court through the exercise of the
veto power.
(2) P.D. 644 which purportedly repealed R.A. 1797 never achieved that
purpose because it was not properly published, it never became a law
and consequently, it did not have the effect of repealing R.A. 1797. As
justified in the case of Taada v. Tuvera all laws shall immediately
upon their approval or as soon thereafter as possible, be published in
full in the Official Gazette, to become effective only after 15days from
their publication, or on another date specified by the legislature, in
accordance with Article 2 of the Civil Code. P.D. 644 was promulgated
by President Marcos in 1975, but was not immediately or soon
thereafter published. It took more than 8 years to publish the decree
after its promulgation.

Relative to the case of Bengzon, timely publication of laws


would also be a manner of giving flesh to the principle of
accountability of public officers
Umali v. Estanislao
209 SCRA 446 (1992)
FACTS: In 1991, Congress enacted R.A. 7167 providing for upward
adjustments in basic personal and additional exemptions allowable to
individuals for income tax purposes. Its effectivity clause provided:
This Act shall take effect upon its approval. The Act was signed and
approved by the President on Dec. 19, 1991 and was published on Jan.
14, 1992 in a newspaper of general circulation. On Dec. 1992
respondents promulgated Revenue Regulations No. 1-92, providing that
said regulations shall take effect on compensation income from Jan. 1,
1992. Two petitions were then filed by several taxpayers to compel the
respondents to implement R.A. 7167 with respect to taxable income
earned or received in 1991.
ISSUE: (1) Whether or not R.A. 7167 took effect upon its approval by
the President on Dec. 1991, or on Jan. 30, 1992, after 15days following
its publication on Jan. 14, 1992 in a newspaper of general circulation.
(2) Whether or not, assuming it took effect on Jan. 30, 1992, the said
law nonetheless covers or applies to compensation income earned or
received during calendar year 1991.
DECISION: (1) The court rules that R.A. 7167 took effect on Jan. 30,
1992, which is after 15 days following its publication on Jan. 14. (2) Sec.
1, 3, and 5 of Revenue Regulations No. 1-92 which provide that the
regulations shall take effect on compensation income earned or
received from Jan. 1 1992 are hereby SET ASIDE. They should take
effect on compensation income earned or received from Jan. 1, 1991.
REASON: (1) The clause unless it is otherwise provided, in Article 2 of
the Civil Code, refers to the date of effectivity and not to the
requirement of publication itself, which cannot in any event be
omitted. Publication is indispensable in every case, but the legislature
may in its discretion provide that the usual 15 day period shall be
shortened or extended.
(2) The personal exemptions as increased by R.A. 7167 cannot be
regarded as available only in respect of the compensation income
received during 1992. Revenue Regulations No. 1-92 would in effect
postpone the availability of the increased exemptions to Jan 1 April
15, 1993, and thus literally defer the effectivity of R.A. 7167 to Jan. 1,
1993. Thus, the implementing regulations collide frontally with Sec. 3 of
the said Act which states that the statute shall take effect upon its
approval.

An effectivity clause which provides that the law shall take


effect immediately upon its approval is defective which, while
not invalidating the whole laws, means that, pursuant to Article
2 of the Civil Code, the enactment takes effect fifteen days after
its publication
People v. Veridiano II
132 SCRA 523 (1984)
FACTS: Benito Go Bio, Jr. was charged with violation of BP Blg. 22 for a
check he issued in the second week of May 1979. Bio filed a motion to
quash the information on the ground that it failed to charge an offense,
pointing out that on the date of the alleged commission of the offense
BP. Blg. 22 had not yet taken effect. Although BP. Blg. 22 was published
in April 9, 1979 issue of the Official Gazette, the said issue was only
released on June 14, 1979 and, considering that the questioned check
was issued about the second week of May 1979, then he could not
have violated the said statute because it was not yet released for
circulation at that time.
Petitioner contends that B.P. Blg. 22 was published in April 9, 1979
issue of the Official Gazette and 15 days therefrom would be April 24,
1979, or several days before respondent Go Bio, Jr. issued the
73 | P

LATON

questioned check around the second week of May 1979 and that
respondent judge should not have taken into account the date of
release of the Gazette for circulation because Sec. 11 of the Revised
Administrative Code provides that for the purpose of ascertaining the
date of effectivity of a law that needed publication, the Gazette is
conclusively presumed to be published on the day indicated therein as
the date of issue.
ISSUE: whether or not Go Bio, Jr. violated B.P. Blg. 22 when he issued a
check around the second week of May 1979.
DECISION: No. The accused cannot be held liable for bouncing checks
prior to the effectivity of B.P. Blg. 22 although the check may have
matured after the effectivity of the said law.
REASON: The Solicitor General admitted the certification issued by Ms.
Charito A. Mangubat stating This is to certify that Vol. 75, No. 15, of
the April 9, 1979 issue of the Official Gazette was officially released for
circulation on June 14, 1979. It is therefore, certain that the penal
statute in question was made public only on June 14, 1979 and not on
the printed date April 9, 1979. Therefore, June 14, 1979 was the date of
publication of B.P. Blg. 22, before the public may be bound by its
contents especially its penal provisions, the law must be published and
the people officially informed of its contents and/or its penalties. For, if
a statue had not been published before its violation, then in the eyes of
the law there was no such law to be violated and, consequently, the
accused could not have committed the alleged crime. When Go Bio, Jr.
committed the act complained of in May 1979 there was then no law
penalizing such act.
De Roy v. Court of Appeals
157 SCRA 757 (1988)
FACTS: On Aug. 17, 1987, the Court of Appeals promulgated a decision
affirming a lower court judgment adverse to the petitioners. Said
decision of the appellate court was received by petitioners on Aug. 25,
1987. On Sept. 9, 1987, the last day of the 15-day period to file an
appeal, petitioners filed a motion for extension of the time to file a
motion for reconsideration, which was eventually denied by the
appellate court. The petitioners herein filed this instant petitione for
certiorari. The appellate court applied the rule laid down in Habaluyas
Enterprises, Inc. v. Japzon, that the 15-day period for appealing or for
filing a motion for reconsideration cannot be extended. This ruling has
been reiterated in several cases relating to such. Petitioners contend
that the rule enunciated in the Habaluyas case should not be made to
apply to the case at bar owing to the non-publication of the Habaluyas
decision in the Official Gazette as of the time the subject decision of the
Court of Appeals was promulgated.
ISSUE: whether or not the Court of Appeals committed grave abuse of
discretion when it denied petitioners motion for extension of time to
file a motion for reconsideration, directed entry of judgment and
denied their motion for reconsideration.
DECISION: This court finds that the Court of Appeals did not commit a
grave abuse of discretion.
REASON: The CA correctly applied the rule laid down in Habulayas case.
Also, contrary to petitioners view, there is no law requiring the
publication of Supreme Court decisions in the Official Gazette before
they can be binding and as a condition to their becoming effective. It is
the bounded duty of counsel as lawyer in active law practice to keep
abreast of decisions of the Supreme Court particularly where issues
have been clarified, consistently reiterated, and published in the
advance reports of Supreme Court decisions and in such publications as
the Supreme Court Reports Annotated and law journals.
STATE PROSECUTORS v. MURO
236 SCRA 505 (1994)

violation of Central Bank Foreign Exchange Restrictions. These cases


had been filed by the complainant prosecutors against Mrs. Imelda
Marcos. The prosecutors alleged that respondent judge issued his
Order dated Aug. 13, 1992 solely on the basis of newspaper reports
concerning the announcement on Aug. 10, 1992 by the President of the
lifting by the government of all foreign exchange restrictions. The
judge, taking judicial notice of what allegedly were of public knowledge,
dismissed the cases even before the Central Bank Circular or Monetary
Board Resolution had been officially issued.
Respondent judge contended that there was no need to await
publication of the Central Bank Circular repealing the existing law on
foreign exchange controls for the simple reason that the public
announcement made by the President in several newspapers of general
circulation lifting foreign exchange controls was total,, absolute,
without qualification, and was immediately effective, and that having
acted only on the basis of such announcement, he cannot be blamed
for relying on the erroneous statement of the President that the new
foreign exchange rules rendered moot and academic the cases filed
against Mrs. Marcos, which erroneous statement was corrected only on
Aug. 17, 1992 but published in the newspapers on Aug. 18, 1992, and
only after he had issued his order of dismissal dated Aug. 13, 1992.
ISSUE: whether or not respondent judge committed gross ignorance of
the law in dismissing the eleven cases solely on the basis of newspaper
reports concerning the announcement by the President of the lifting of
all foreign exchange restrictions.
DECISION: Yes. The court finds respondent judge guilty of gross
ignorance of the law and is dismissed from the service.
REASON: The doctrine of judicial notice rests on the wisdom and
discretion of the courts. Matters of judicial notice have three material
requisites: (1) the matter must be one of common and general
knowledge; (2) it must be well and authoritatively settled and not
doubtful or uncertain; and (3) it must be known to be within the limits
of the jurisdiction of the court. Things of common knowledge are
matters coming to the knowledge of men generally in the course of the
ordinary experience of life, or mattes which are generally accepted by
mankind as true and are capable of ready and unquestioned
demonstration. Respondent judge, in the guise of exercising discretion
and on the basis of a mere newspaper account which is sometimes
even referred to as hearsay evidence twice removed, took judicial
notice of the supposed lifting of foreign exchange controls, a matter
which was not and cannot be considered of common knowledge or of
general notoriety. Worse, he took cognizance of an administrative
regulation which was not yet in force when the order of dismissal was
issued. Jurisprudence dictates that judicial notice cannot be taken of a
statute before it becomes effective.
The assertion of respondent judge that there was no need to await
publication of the circular for the reason that the public announcement
made by the President in several newspapers of general circulation is
beyond comprehension. As a judge of the RTC of Manila he is supposed
to be well-versed in the elementary legal mandates on the publication
of laws before they take effect. The very act of respondent judge in
altogether dismissing the eleven criminal cases without even a motion
to quash having been filed by the accused, and without at least giving
the prosecution the basic opportunity or an oral argument, is not only a
blatant denial or elementary due process to the Government but is
palpably indicative of bad faith and partiality. In order that bias may not
be imputed to a judge, he should have the patience and circumspection
to give the opposing party a chance to present his evidence even if he
thinks that the oppositors proof might not be adequate to overthrow
the case for the other party. At the very least, respondent judge acted
injudiciously and with unjustified haste in the outright dismissal of the
eleven cases, and thereby rendered his actuation highly dubious.

Nature of the Guarantee and Matters of Public Concern

FACTS: Respondent Judge Manuel Muro was charged with ignorance of


the law, grave misconduct and violation of Rules 2.01, 3.01 and 3.02 of
the Code of Judicial Conduct, for having dismissed eleven cases for
74 | P

LATON

The guarantee is a self-executory provision under both the


1973 and 1987 Constitution. It can be invoked by any citizen
before the courts
The information and the transactions referred to in the
subject provision of the Constitution have as yet no defined
scope and extent
Recognized restrictions:
1. National security matters and intelligence information
2. Trade secrets and banking transactions
3. Criminal matters
4. Other confidential information

Confidential Matters
While people have the general right to know most everything, it
does not mean, however, that they have an unfettered access
to everything in the possession of the government. In the very
nature of things, some matters have to be kept confidential if
the government is to be effective. One such instance is in
diplomatic negotiations
The nature of diplomacy requires centralization of authority
and expedition of decision which are inherent in executive
action. Another essential characteristic of diplomacy is its
confidential nature
Indeed, while the Constitution guarantees a right to
information on matters of public concern, for which purpose
access to official records, documents, and papers pertaining to
official acts, transactions, or decisions, as well as to government
research data used as basis for policy development, shall be
afforded the citizens, it states at the same time that the same
shall be subject to such limitations as may be provided by law
What is sought to be had then is the grant of enough
information to enable the citizens to exercise their political
rights as the ultimate source of power without sacrificing the
need for the government to keep sacrosanct and confidential
those that need to be kept secret. Where the demarcation line
might be between what should be publicly accessible and what
is to be kept sub rosa is a question that the courts may have to
visit every now and then
Privacy Interests
The people also have the right to keep matters to themselves,
and having certain data and information about them which had
been gathered and stored by the Government does not
authorize others, pursuant to their right to information, to have
access to the same either for their personal consumption or for
publication
Additional Cases
(I) Right to Information [1]

I2. Bantay vs. COMELEC


G.R. No. 177271 May 4, 2007
The Comelec issued Resolution No. 7804 prescribing rules and
regulations to govern the filing of manifestation of intent to participate

and submission of names of nominees under the party-list system of


representation in connection with the May 14, 2007 elections. Pursuant
thereto, a number of organized groups filed the necessary
manifestations. Some were ostensibly subsequently accredited by the
Comelec to participate in the 2007 elections
Two consolidated petitions for certiorari and mandamus to nullify and
set aside certain issuances of the Commission on Elections (Comelec)
respecting party-list groups which have manifested their intention to
participate in the party-list elections on May 14, 2007.
In the first petition, petitioners Bantay Republic Act (BA-RA 7941, for
short) and the Urban Poor for Legal Reforms (UP-LR, for short) assail
the various Comelec resolutions accrediting private respondents
Biyaheng Pinoy et al., to participate in the forthcoming party-list
elections on May 14, 2007 without simultaneously determining
whether or not their respective nominees possess the requisite
qualifications defined in Republic Act (R.A.) No. 7941, or the "Party-List
System Act" and belong to the marginalized and underrepresented
sector each seeks to represent.
In the second, petitioners Loreta Ann P. Rosales, Kilosbayan Foundation
and Bantay Katarungan Foundation impugn Comelec Resolution 070724 dated April 3, 2007 effectively denying their request for the
release or disclosure of the names of the nominees of the fourteen (14)
accredited participating party-list groups mentioned in petitioner
Rosales previous letter-request.
While both petitions commonly seek to compel the Comelec to disclose
or publish the names of the nominees of the various party-list groups
named in the petitions, the first have the following additional prayers:
1) that the 33 private respondents named therein be "declare[d] as
unqualified to participate in the party-list elections as sectoral
organizations, parties or coalition for failure to comply with the
guidelines prescribed by the [Court] in [Ang Bagong Bayani v. Comelec]"
and, 2) correspondingly, that the Comelec be enjoined from allowing
respondent groups from participating in the May 2007 elections.
Issue
1. Whether respondent Comelec, by refusing to reveal the names of the
nominees of the various party-list groups, has violated the right to
information and free access to documents as guaranteed by the
Constitution; and
2. Whether respondent Comelec is mandated by the Constitution to
disclose to the public the names of said nominees.
Comelec based its refusal to disclose the names of the nominees of
subject party-list groups on Section 7 of R.A. 7941.
SEC. 7. Certified List of Registered Parties.- The COMELEC shall, not
later than sixty (60) days before election, prepare a certified list of
national, regional, or sectoral parties, organizations or coalitions which
have applied or who have manifested their desire to participate under
the party-list system and distribute copies thereof to all precincts for
posting in the polling places on election day. The names of the party-list
nominees shall not be shown on the certified list.
COMELEC Chairman Benjamin S. Abalos, Sr. said he and [the other
five COMELEC] Commissioners --- believe that the party list elections
must not be personality oriented.Abalos said under [R.A.] 7941 ,
the people are to vote for sectoral parties, organizations, or coalitions,
not for their nominees.He said there is nothing in R.A. 7941 that
requires the Comelec to disclose the names of nominees. xxx (Words in
brackets and emphasis added)Insofar as the disclosure issue is
concerned, the petitions are impressed with merit.
Assayed against the non-disclosure stance of the Comelec and the given
rationale therefor is the right to information enshrined in the selfexecutory Section 7, Article III of the Constitution, viz:
Sec.7. The right of the people to information on matters of public
concern shall be recognized. Access to official records, and to
documents, and papers pertaining to official acts, transactions, or
decisions, as well to government research data used as basis for policy
75 | P

LATON

development, shall be afforded the citizen, subject to such limitations


as may be provided by law.
Complementing and going hand in hand with the right to information is
another constitutional provision enunciating the policy of full disclosure
and transparency in Government. We refer to Section 28, Article II of
the Constitution reading:
Sec. 28. Subject to reasonable conditions prescribed by law, the State
adopts and implements a policy of full public disclosure of all its
transactions involving public interest.
..The right to information is a public right and for every right of the
people recognized as fundamental lies a corresponding duty on the part
of those who govern to respect and protect that right. However, the
right to information and its companion right of access to official records
are not absolute and is limited to "matters of public concern" and is
further subject to such limitation as may be provided by law.
The terms "public concerns" and "public interest" have eluded precise
definition. But both terms embrace, to borrow from Legaspi, a broad
spectrum of subjects which the public may want to know, either
because these directly affect their lives, or simply because such matters
naturally whet the interest of an ordinary citizen. At the end of the day,
it is for the courts to determine, on a case to case basis, whether or not
at issue is of interest or importance to the public.

advanced in their petition. The exercise would require the Court to


make a factual determination, a matter which is outside the office of
judicial review by way of special civil action for certiorari.
In certiorari proceedings, the Court is not called upon to decide factual
issues and the case must be decided on the undisputed facts on record.
The sole function of a writ of certiorari is to address issues of want of
jurisdiction or grave abuse of discretion and does not include a review
of the tribunals evaluation of the evidence. (note that nowhere in R.A.
No. 7941 is there a requirement that the qualification of a party-list
nominee be determined simultaneously with the accreditation of an
organization. )
I3. Senate of the Philippines v. Ermita
488 SCRA 1 (2006)
The people as well as their representatives are entitled to know what
are happening around them especially those of public concern involving
the use of public funds, as well as other deals entered into by those
who are in temporary ascendancy in the government. Also, there are
things that should be kept confidential for the proper and efficient
functioning of the Government.
a transparent government is one of the hallmarks of a truly republican
state however, even in the early history of republican thought, it has
been recognized that the head may keep certain information
confidential in pursuit of the public interest.

Here.. no national security or like concerns is involved in the disclosure


of the names of the nominees of the party-list groups in question.
Doubtless, the Comelec committed grave abuse of discretion in refusing
the legitimate demands of the petitioners for a list of the nominees of
the party-list groups subject of their respective petitions. Mandamus,
therefore, lies.

EO 464 (does it ring a bell? Buset.. minumulto ako ng poli bat gnun?)

The last sentence of Section 7 of R.A. 7941 is certainly not a justifying


card for the Comelec to deny the requested disclosure.

EO 64 practically sealed the lips of everyone in the executive


department. Not only department heads but also minor functionaries
are prevented as the executive privilege was also bestowed on themmeaning no appearance without clearance.

The prohibition imposed on the Comelec under said Section 7 is limited


in scope and duration, meaning, that it extends only to the certified list
which the same provision requires to be posted in the polling places on
election day. To stretch the coverage of the prohibition to the absolute
is to read into the law something that is not intended.
The Comelec obviously misread the limited non-disclosure aspect of the
provision as an absolute bar to public disclosure before the May 2007
elections. The interpretation thus given by the Comelec virtually tacks
an unconstitutional dimension on the last sentence of Section 7 of R.A.
No. 7941.
The Comelecs reasoning that a party-list election is not an election of
personalities is valid to a point. It cannot be taken, however, to justify
its assailed non-disclosure stance which comes, as it were, with a
weighty presumption of invalidity, impinging, as it does, on a
fundamental right to information, While the vote cast in a party-list
elections is a vote for a party, such vote, in the end, would be a vote for
its nominees, who, in appropriate cases, would eventually sit in the
House of Representatives.
DENIED insofar as it seeks to nullify the accreditation of the
respondents named therein. However, insofar as it seeks to compel the
Comelec to disclose or publish the names of the nominees of party-list
groups, sectors or organizations accredited to participate in the May
14, 2007 elections, the same petition and the petition in G.R. No.
177314 are GRANTED.
Comelec is hereby ORDERED to immediately disclose and release the
names of the nominees of the party-list groups, sectors or organizations
accredited to participate in the May 14, 2007 party-list elections. The
Comelec is further DIRECTED to submit to the Court its compliance
herewith within five (5) days from notice hereof.
The Court is unable to grant the desired plea of petitioners BA-RA 7941
and UP-LR for cancellation of accreditation on the grounds thus

.. for the purpose of ensuring the observance of the separation of


powers
Executive privilege and power of legislative inquiries in aid of
legislation.

Inquiry in aid of legislation..


If the info is necessary for wise legislation, Congress has the right to
compel disclosure thereof.
Executive privilege- (Schwartz) the power of the government to
withhold information from the public the courts and the congress.
(Rozell) the right of the president and high level officials to withhold
information from Congress, the courts, and ultimately the public
(tribe) 3 distinct kinds
1.

2.

3.

State secrets privilege- (on the ground that


the information is of such nature that
disclosure would subvert crucial military or
diplomatic objectives)
Informers privilege- (not to disclose the
identity of persons who furnish info of
violations of law to officers charged with
the enforcement of that law)
Generic privilege (for internal deliberations
attaching to intragovernmental documents
reflecting
advisory
opinions,
recommendations
and
deliberations
comprising part of a process by which
governmental decisions and policies are
formulated.

A president and those who assist him must be free to explore


alternatives in the process of shaping policies and making decisions and
to do so in a way many would be unwilling to express except privately.
The privilege is fundamental to the operation of government and
inextricably rooted in the separation of powers under the Constitution.
76 | P

LATON

Recognized only in certain types of information of a sensitive character.


A claim therefore maybe valid or invalid. Noticeably absent is the
recognition that executive officials are exempt from the duty to disclose
information by the mere fact of being executive officials. The
extraordinary character of the exemptions indicates that the
presumption inclines heavily against executive secrecy and in favor of
disclosure.
Sec 2(a) of EO 464, the reference to persons being covered by the
executive privilege may be read as an abbreviated way of saying that
the person is in possession of info which is in the judgment of the head
office concerned, privileged. Executive privilege refers to information
and not to persons. The claim therefore under section 3 of EO 464 in
relation to section 2(b)is thus invalid per se because, it is not asserted
but merely implied instead of providing precise and certain reasons for
the claim, it merely invokes eo464 coupled with the announcement
that the President has not given consent. It severely frustrates the
power of inquiry of Congress. In Fine, Section 3 and 2 (b) must but
invalidated.
The president may not authorize her subordinates to exercise such
power.
Right of people to information v the right of Congress to information.
The demand of a citizen has no same obligatory force as that of a
subpoena duces tecum.
Also, it doesnt give an individual to exact a testimony from government
officials.
It does not follow in every sense that when the congress exercises the
power of inquiry, that the people are exercising their right to info.
To the extent that investigations in aid of legislation are generally
conducted in public, however, any executive issuance tending to unduly
limit disclosures of information in such investigations necessarily
deprives the people of information which being presumed to be in aid
of legislation, is presumed to be a matter of public concern. The citizens
are thereby denied access to information which they can use
formulating their own opinions on the matter before congressopinions which they can then communicate to their representatives
and other government officials through the various legal means
allowed by their freedom of expression.
Moreover, even as EO 464 applies only to officials of the executive
branch, it does not follow that the same is exempt from publication.
Since this concerns public interest, the challenged order must be
covered by the publication requirement.
I4. Sabio v. Gordon
504 SCRA 704 (2006)
FACTS: Then-President Corazon Aquino issued E.O. No. 1, creating the
PCGG, a body tasked with recovering the ill-gotten wealth obtained by
deposed President Ferdinand Marcos and his close associates. Section 4
(b) of said E.O. provides that [n]o member or staff of the Commission
shall be required to testify or produce evidence in any judicial,
legislative or administrative proceeding concerning matters within its
official cognizance. This was to ensure the PCGG's unhampered
performance of its duties. 20 years later, the Senate invited PCGG
Chairman Camilo Sabio to be one of the resource persons in a Senate
investigation, who declined the invitation invoking Section 4 (b) of E.O.
No. 1.
ISSUE: May Section 4 (b) of E.O. No. 1 be invoked by Chairman Sabio to
justify non-appearance on legislative investigations?
HELD: No. Section 4 (b) of E.O. No. 1 has been repealed by the 1987
Constitution, particularly by Sec. 21, Art. VI (power of legislative
inquiry).
(The Supreme Court said, "[T]he conduct of inquirires in aid of

legislation is not only intended to benefit Congress but also the


citizenry. The people are equally concerned with this proceeding and
have the right to participate therein in order to protect their interests.
The extent of their participation will largely depend on the information
gathered and made known to them. In other words, the right to
information really goes hand-in-hand with the constitutional policies of
full public disclosure and honesty in the public service. It is meant to
enhance the widening role of the citizenry in governmental decisionmaking as well as in checking abuse in the government." They also
mentioned that some of our standing jurisprudence, such as the case of
Taada v. Tuvera, "have recognized a citizen's interest and personality
to enforce a public duty and to bring an action to compel public officials
and employees to perform that duty.)
I5. Air Philippines Corporation v. Pennswell, Inc.
540 SCRA 215 (2007)
FACTS: Air Philippines Corporation (APC) is a domestic corporation
engaged in air transportation services. APC purchased chemical
lubricants from Pennswell, a company that manufactures and sells
industrial chemicals. In the course of events, APC alleged that
Pennswell defrauded it: APC insisted that Pennswell committed to
deliver lubricants of the type belonging to a new line instead of what it
had previously delivered to them, which were apparently lubricants of
second-grade quality not fit for APC's specific use. The second batch of
lubricants demanded for, though, turned out to be of the same type as
that of the first delivery. APC demanded return of its money, but
Pennswell ignored the demand and billed APC instead for its
outstanding debts. APC now seeks to convince the Supreme Court that
it has a right to obtain the chemical composition and ingredients of
respondents products to conduct a comparative analysis of its
products.
ISSUE: Can chemical components of Pennswell products be accessed
pursuant to right to information under Sec. 7, Art III?
HELD: No. Jurisprudence has consistently acknowledged the private
character of trade secrets, as there is a privilege not to disclose them.
The Supreme Court has declared that trade secrets and banking
transactions are among the recognized restrictions to the right of the
people to information as embodied in the Constitution.
I6. Antolin vs. Domondon15
G.R. No. 165036; 5 July 2010
Facts: Hazel Ma. C. Antolin (Petitioner) failed the Certified Public
Accountant (CPA) Licensure Exam she took in October 1997. Convinced
she deserved to pass the Exam, she wrote to the Board of Accountancy
(Board), requesting that her answer sheets be re-corrected. She was
shown her answer sheets but since these showed only shaded marks,
she was unable to determine why she failed the Exam. Consequently,
she asked the Board for copies of the questionnaire, her answer sheets,
the answer keys and an explanation of the grading system (collectively,
the Examination Papers). Her request was denied on two grounds: (1)
Section 36, Article III of the Rules and Regulations Governing the
Regulation and Practice of Professionals, as amended by Professional
Regulation Commission (PRC) Resolution No. 332, series of 1994, only
allowed access to her answer sheets, and reconsideration of the result
of her examination can be made only on grounds of mechanical error in
the grading of the answer sheets, or malfeasance; and (2) the Board
was precluded from releasing the Examination Papers (other than the
answer sheets) by Section 20, Article IV of PRC Resolution No. 338,
series of 1994. The Board later informed her that her exam was
investigated and no mechanical error was
found in the grading.
Issues: Whether or not petitioner has the constitutional right to have
access to the Examination papers.
Held: Like all the constitutional guarantees, the right to information is
not absolute; it is limited to matters of public concern and is further
15

Source: Lexforiphilippines.com

77 | P

LATON

subject to such limitations as may be provided by law (Section 7,


Article III, 1987 Constitution). Similarly, the States policy of full
disclosure is limited to transactions involving public interest, and is
subject to reasonable conditions prescribed by law (Sec. 28, Art. II,
1987 Constitution). National board examinations such as the CPA Board
Exams are matters of public concern. The populace in general, and the
examinees in particular, would understandably be interested in the fair
and competent administration of these exams in order to ensure that
only those The Court, nonetheless, realizes that there may be valid
reasons to limit access to the Examination Papers in order to properly
administer the exam. That there exist inherent difficulties in the
preparation, generation, encoding, administration, and checking of
these multiple choice exams that require that the questions and
answers remain confidential for a limited duration. The PRC had not
been given an opportunity to explain the reasons behind their
regulations for keeping the Examination Papers confidential. The Court
deemed it best to remand the case to the RTC for further proceedings.

Chapter 10
Right of Association
The right of the people, including those employed in the public
and private sectors, to form unions, associations, or societies
16
for purposes not contrary to law shall not be abridged.
Man being gregarious by nature would find it natural to
associate with others, whether for intimate companionship or
for social political and other reasons
An individual may join or organize whatever associations and
societies he and kindred minds and hearts like him might want
to form
Just like any other rights, however, the right to associate is still
subject to limitation that it must not be for purposes contrary
to law
Unlike the cases of other guarantees, which are mostly
American in origin, this particular freedom has an indigenous
cast. It can trace its origin to the Malolos Constitution
General Considerations
The right to association may involve such intimate and personal
relations as friendship or marriage to the more impersonal
groupings as those of unions and societies where objectives
might range from purely social to economic and political. The
first one may be considered as the freedom of intimate
association and the latter the freedom of expressive association
The right to form associations also include the liberty not to
join at all. A person can not be compelled to join a group that
he does not want to associate with. The constitutionally
guaranteed freedom of association includes the freedom not to
associate. The right to choose with whom one will associate
oneself is the very foundation and essence of that partnership.
It should be noted that the provision guarantees the right to
form an association. It does not include the right to compel
others to form or join one
National Association for the Advancement of Colored People v.
Alabama ex rel. Patterson
357 US 449, 2 L Ed 2d 1488, 78 S Ct 1163 (1958)
Petitioner is a nonprofit membership corporation organized under the
laws of New York for the purpose of advancing the welfare of Negroes.
16

CONSTITUTION, Art. III, 8

It operates through chartered affiliates which are independent


unincorporated associations with membership therein equivalent to
petitioner.
Had local affiliates in Alabama and operated there without complying
with an Alabama statute that requires a foreign corporation to qualify
before doing business there by filing its corporate charter with the
secretary of the state and designating a place of business and an agent
to receive service of process.
The association has never complied with the statute from which it
considered itself exempt.
Alleging the petitioners activities- such as opening a regional office,
organizing organized various activities in Alabama, recruiting members
and soliciting contributions within the state, giving financial support
and furnishing of legal assistance to Negro boycott of the bus lines in
Montgomery to compel the seating of passengers without regard to
race-were causing irreparable injury to the citizens of the state for
which criminal prosecution and civil actions at law afforded no
adequate relief.
The Attorney general of Alabama brought an equity suit to enjoin the
petitioner from conducting further activities in and taking steps to
further activities in the state.
Court issued a restraining order.
Petitioner demurred to the allegations of the bill and moved to dissolve
the restraining order. It contended that what the state sought to
accomplish by its suit would violate rights to freedom of speech and
assembly guaranteed by the 14th amendment to the constitution of the
US.
The state moved for the production of a large number of records and
papers, including bank statements, lease, deeds, and records containing
the names of all Alabama members and agents of the association.
Petitioner produced all except its membership list and was adjudged in
contempt for that and was fined $100000.
Whether Alabama, consistently with the due process clause of the 14th
amendment, can compel petitioner to reveal the names and addresses
of the members and agents without regard to their positions and
functions in the association.
Petitioners claim is that the order, in the circumstances shown by this
record violated rights assured to petitioner and its members under the
constitution.
Can the association validly assert the right of its members?
because it and its members are in the practical sense identical.

Yes

The rights are personal to the members, who are not immediately
before the court, may be asserted by the association on their behalf for
the right could not be effectively vindicated except through an
appropriate party before the court.
Petitioner argues that the effect of compelled disclosure of
membership list will be to abridge the rights of their rank-and-file
members to engage in lawful association in support of their common
beliefs.
The fact that Alabama has taken no direct action to restrict the right of
the petitioners members to associate freely, does not end inquiry into
the effect of the production order.
Inviolability of privacy in group association may in many circumstances
be indispensable to preservation of freedom of association, particularly
when the group espouses dissident beliefs.
It is not sufficient to answer that whatever repressive effect
compulsory disclosure of names of petitioners members, may have
78 | P

LATON

upon participation by Alabama citizens in petitioners activities follows


not from private community pressures. The crucial factor here is the
interplay of governmental and private action, for it is only after the
initial exertion of state power represented by the production order that
private action takes hold.

2.

Whether Alabama has demonstrated an interest in obtaining the


disclosures it seeks from petitioner which is sufficient to justify the
deterrent effect which we have concluded these disclosures may well
have on the free exercise by petitioners members of their
constitutionally protected right of association.

3.

4.

The exclusive purpose was to determine if the association is conducting


intrastate business in violation of the Alabama foreign corporation
registration statute.
The court is unable to perceive the substantial bearing of the
membership list. As matters stand in the state court that the petitioner
has (1) admitted its presence and conduct of activities in Alabama since
1918 (2) has offered to comply in all respects with the qualification
statute, although preserving its contention that the statute does not
apply to it, and (3) has apparently complied satisfactorily with the
production order except from its membership list.
Whatever interest the state may have in obtaining the names of the
ordinary members has not been shown to be sufficient to overcome
petitioners constitutional objections to the production order.
The court held that the immunity from state scrutiny of membership
lists which the association claims on behalf of its members is here so
related to the right of its members to pursue their private interests
privately and to associate freely with others in doing so as to come
within the protection of the 14th amendment. The judgment of civil
contempt and the $100000 fine must fall.
Communist Party v. Subversive Activities Control Board
367 US 1, 6 L Ed 2d 625, 81 S Ct 1357 (1961)
The subversive activities control act, as amended, and based on
legislative findings from evidence adduced before various congressional
committees that among others, there exists a world communist
movement, which is a worldwide revolutionary movement whose
purpose it is, by treachery, deceit, infiltration into other groups,
espionage, sabotage, terrorism, and any other means deemed
necessary, to establish a communist totalitarian dictatorship in the
countries throughout the world through the medium of a worldwide
communist organization, required the registration with the Attorney
General of Communist action organization and Communist front
organizations including information about their officers. the front
organizations need not list their non-officer members, however. The
Act also provides that whatever the Attorney General has reason to
believe that an organization which has not registered is an organization
of a kind required to register, or that any individual who has not
registered is required to register, he shall petition the Subversive
Activities Control Board for an order that the organization or individual
register.
After the registration, the act then subjects the organization or its
members to certain prohibitions, restrictions, disabilities and
disqualifications in regard to their communications and
correspondence, as well as, employment, naturalization and their
foreign travel, i.e. their application for US passport.
The attorney general petitioned the subversive activities control board
for an order to require that the communist party register as a
communist action organization. After several hearings, the board found
the party to a communist action organization within the meaning of the
act and ordered it to register as such.
The constitutional contentions raised by the party with respect to the
registration requirement are:
1. That the requirement, in the context of the act , in effect
outlaws the Party and is in the nature of a bill of attainder

5.
6.

That compelling organizations to register and list their


members on a showing merely that they are foreigndenominated and operate primarily to advance the
objectives of the world Communist movement constitutes a
restraint on freedom of expression and association in
violation of the 1st amendment
That requiring Party officers to file registration statements
for the Party subjects them to self-incrimination forbidden
by the 5th amendment
That the act violates due process by legislative
predetermination of facts essential to the communist party
within the definition of a communist action organization,
and that the evidentiary elements prescribed for
consideration by the board bear no rational relation to that
definition
That, in several aspects the act is unconstitutionally vague
That the subversive activities control board is so necessarily
biased against the communist party as to deprive it of fair
hearing

in threats to public safety, the congress meets the threat by the


requirement of registration or disclosure.
The congress may entail restraints on speech and association to require
publicity demanded by rational interests high in scale of national
concern.
The requirement that officers or members at anytime during the year
preceding the registration must be listed is a reasonable means of
assuring that the obligation to list present members and officers may
not be evaded. Also to include their aliases must be sustained.
Disclosure of the financial transactions and of the identity of the organs
of publication which it controls is necessary to bring foreigndenominated organizations out into the open, where the public can
evaluate their activities informedly against the revealed background of
the character, nature, and connections. The obligation to identify
presses without more and as applied to foreign-denominated
organizations, does not fetter constitutionally protected free
expression.
The subversive activities control act applies only to foreign
denominated organizations which work primarily to advance the
objectives of a world movement controlled by the government of a
foreign country. There is no attempt here to impose stifling obligations
upon the proponents of a particular political creed as such, or even to
check the importation of particular political ideas abroad for
propagation here. Organizations are subject to it only when shown
after administrative hearing subject to judicial review, to be dominated
by the foreign power or its organs and to operate primarily to advance
its purposes. The registration requirement therefore on its face and as
here applied, does not violate the 1st amendment.
People v Ferrer
48 SCRA 382 (1972)
Petitioners Feliciano Co and Nilo Tayag, together with 5 others, were
separately charged with violation of the anti subversion act. They
moved to quash, assailing the constitutionality of the act on the
grounds that, among others, it is a bill of attainder, it is vague and it
denies them the equal protection of the laws. Resolving the
constitutional issues raised, the trial court declared the statute void on
the grounds that it is a bill of attainder and that it is vague and overbroad, and dismissed the information against the two accused.
..due process..
Before enacting the statute in question, congress conducted careful
investigations and stated the requirements in their preamble..
The constitutionality of the act would be open to question if, instead of
making these findings in enacting the statute, congress omitted to do
so.
79 | P

LATON

Co and Tayag both filed motions for reconsideration


In saying that by means of the act, congress has assumed judicial
magistracy, the trial court failed to take proper account of the
distinction between legislative fact and adjudicative fact.
Legislative fact- those facts which are relevant to the legislative
judgment.
Adjudicative fact- those which tie the legislative enactments to the
litigants.
The test formulated in Nebbia v New York is that laws are seen to have
a reasonable relation to a proper legislative purpose, and are neither
arbitrary nor discriminatory; the requirements of due process are met.
The government has a right to protect itself against subversion.
Even though the purpose is legitimate, it cannot be achieved by means
that broadly stifle fundamental personal liberties be more narrowly
achieved. The requirement of knowing membership as distinguished
from nominal membership has been held as sufficient basis for
penalizing membership in a subversive organization. (one who knows
that the organization is of an unlawful purpose and accepts or retains
membership
The act is not overbroad. The argument was that the use of the word
overthrow is broad for it could be achieved by peaceful means.
It is not. The first where as clause says overthrow not only by force and
violence but also by deceit, subversion and other illegal means. Also
Section 4 mentions overt acts. Knowingly, willfully and by overt acts.
Also the word overthrow connotes violent and illegal means.
..the act and the guarantee of free expression.
The act is aimed against conspiracies to overthrow the government.
Whatever interest in the freedom of speech and freedom of association
is infringed by the prohibition against knowing membership in the
communist party of the Philippines, is so indirect and so insubstantial as
to be clearly and heavily outweighed by the overriding considerations
of national security and the preservation of democratic institutions in
this country.
.. Conclusion and guidelines

Co filed a reiteration of his earlier arguments


Tayag seeks the inclusion in the guidelines set forth in the decision of a
requirement that in prosecutions under the act, the state must prove
that the defendant joined or remained a member of the CPP or of the
subversive organization, knowing its subversive character and with
specific intent to further its basic objectives as shown by direct
participation in the organizations unlawful activities
This submission would nullify the legislative policy embodied in the Act
and frustrate prosecutions under it.
To require proof of direct participation would render the conspiracy
device ineffective in Penal Law.
Arresto Mayor joins
Prision Mayor to death if as well takes arm against the government.
It would also run counter to another established principle regarding
conspiracy that the act of one is the act of all.
The requirement that membership is proved by overt acts is enough.
Shown by taking oath of membership or signing affiliation m\papers in
a subversive organization, knowing its illegal purposes. Also may be
inferred through concert of action. May also be satisfied by non
criminal and relatively minor acts such as signing membership papers,
attending meetings and the like.
The function of the overt act in conspiracy prosecution is simply to
manifest that the conspiracy is at work and neither a project still resting
solely in the minds of the conspirators nor a fully completed operation
no longer in existence. The existence of the conspiracy is itself danger
to national security. ..if the ingredients are present, we
cannot bind the Government to wait until the catalyst is added
In the first place, there is no reason why one who actively and
knowingly works in the ranks of the organization, intending to
contribute to the success of its specific illegal activities should be any
more immune from prosecution than one to whom the organization
has assigned or entrusted the task of carrying out the substantial
criminal acts.

Upheld the validity of the act


Guidelines to be observed in the prosecution of the act.
Elements of the crime
In case of subversive organizations other than the communist party of
the Philippines
a.
Purpose of the organization is to overthrow the
present government of the Philippines and to establish
in this country a totalitarian regime under the
denomination of foreign power
b. That the accused joined such organization
c.
That he did so knowingly, willfully, and by overt acts
In case of the communist party of the Philippines
a.
That the CPP continues to pursue the objectives which
led Congress in 1957 to declare it to be an organized
conspiracy for the overthrow of the Government by
illegal means for the purpose of placing the country
under the control of a foreign power.
b. That the accused joined CPP
That he did so knowingly, willfully, and by overt acts
Resolution on Motions for Reconsiderations
56 SCRA 793 (1974)

In the second place, the requirement of proof of specific intent


precisely limits the operation of the statute only to illegal conduct.
The statute provides that a defendant must be proven to have
knowledge of the proscribed advocacy before he may be convicted.
Thus the member whom the organization is a vehicle for the
advancement of legitimate aims and policies does not fall within the
ban of the statute; he lacks the requisite of specific intent to overthrow
of the government as speedily as circumstances would permit. Such a
person may be foolish, deluded, or perhaps merely optimistic, but he is
not by this statute made a criminal.
The 2 motions for reconsiderations are denied. Our decision of Dec 27,
1972 is hereby declared final and executory.
National Association for the Advancement of Colored People v Button
371US415, (L Ed 2d 405, 83 S Ct 328 (1963)
.Chapter 33 of the Virginia Acts of Assembly forbidding solicitation of
legal business by a runner or capper includes in the definition of runner
or capper an agent for an individual or organization which retains a
lawyer in connection with an action to which it is not a party and in
which it has no pecuniary right or liability..
The case originated in companion suits by the NAACP which seeks to
restrain Chapters 31,32,33,35,and 36 of the Virginia Acts of Assembly
Three-judge court struck down 31,32 and 35 but abstained in 33 and 36
80 | P

LATON

Circuit court held the chapters both constitutional.


Supreme Court of Appeals reversed as to 36 and sustains 33
..
The NAACP is not a conventional political party, but the litigation it
assists, while serving to vindicate the legal rights of members of the
American Negro community, at the same time and perhaps more
importantly, makes possible the distinctive contribution of a minority
group to the ideas and beliefs of our society. For such a group,
association for litigation may be most effective form of political
association.
Chapter 33, as authoritatively construed by the Supreme Court of
Appeals, a person who advises another that his legal rights have been
infringed and refers to him to a particular atty or group of attys for
assistance has committed a crime, as the atty who knowingly renders
assistance under such circumstances. There thus inheres in the statute
the gravest danger of smothering all discussion looking to the eventual
institution of litigation on behalf of the rights of members of unpopular
minority.
It is enough that a vague and broad statute lends itself to selective
enforcement against unpopular cases. We cannot close our eyes to the
fact that the militant Negro civil rights movement has engendered the
intense resentment and opposition of the politically dominant
community of Virginia; litigation assisted by the NAACP has been
bitterly fought. In such circumstances, a statute broadly curtailing
group activity leading to litigation may easily become a weapon of
oppression, however evenhanded its terms appear. Its mere existence
could freeze out all such activity on behalf of the civil rights of Negro
citizens.
The association and its members are advocating lawful means of
vindicating legal rights.
We hold that Chapter 33 violates the 14th amendment by unduly
inhibiting protected freedom of expression and association. As
construed by the court, Chap 33, at least potentially, prohibits every
cooperative activity that would make advocacy of litigation meaningful.
Precision of regulation must be the touchstone in an area so closely
touching our precious freedoms.
A state may not, under the guise of prohibiting professional
misconduct, ignore constitutional rights.
Malicious intent was of the essence of the common law offenses of
fomenting or stirring up litigation. The exercise in our own, as in this
case, of 1st amendment rights to enforce constitutional rights through
litigation, as a matter of law, cannot be deemed malicious. There has
been no showing of a serious danger of professionally reprehensible
conflicts of interests which rules against solicitation frequently seek to
prevent. This is so partly because no monetary stakes are involved, and
so there is no danger that the atty will desert or subvert the paramount
interests of his client to enrich himself or an outside sponsor. And the
aims and interests of NAACP have not been shown to conflict with
those of its members and nonmember Negro litigants.
The state failed to advance any substantial regulatory interest, in the
form of substantive evils flowing from petitioners activities, which can
justify the broad prohibitions which it had imposed.

Intimate Associations
Marriage and family and other personal relationships could
very well illustrate the freedom of intimate association
The Court has long recognized hat, because the Bill of Rights is
designed to secure individual liberty, it must afford the
formation and preservation of certain kinds of highly personal
relationships a substantial measure of sanctuary from
unjustified interference by the state

Grisworld v Connecticut
381 US 497, 14 L Ed 2d 510, 85 S Ct 1678 (1965)
A statute makes it a crime for any person to use any drug or article to
prevent conception. Appellants, the Executive Director and the Medical
Director of the Planned Parenthood League of Connecticut, were found
guilty as accessories for giving married people information and medical
advice on how to prevent conception and prescribing a contraceptive
device or material for the wifes use. They were fined $100 each. They
claim that the accessory statute, as so applied violated the 14th
amendment.
The appellants have standing to raise the constitutional rights of the
married people with whom they had a professional relationship.
Certainly the accessory should have standing to assert the offense
which he is charged is assisting is not, or cannot constitutionally be a
crime.
The right to association is more than the right to attend a meeting; it
includes the right to express ones attitudes or philosophies by
membership in a group or by affiliation with it or by other lawful
means. The present case, then, concerns a relationship lying within the
zone of privacy created by several fundamental constitutional
guarantees. And it concerns a law which, in forbidding the use of
contraceptives rather than regulating their manufacture or sale, seeks
to achieve its goals by means having a maximum destructive impact
upon that relationship. A governmental purpose to control or prevent
activities constitutionally subject to state regulation may not be
achieved by means which sweep unnecessarily broadly and thereby
invade the area of protected freedoms. NAACP v Alabama. Would we
allow the police to search the sacred precincts of marital bedrooms for
telltale signs of use of contraceptives? The very idea is repulsive to the
notions of privacy surrounding the marital relationship.
Marriage is an association that promotes a way of life, not causes; a
harmony in living, not political faiths; a bilateral loyalty, not commercial
or social projects. Yet it is an association for as noble a purpose as any
involved in our prior decisions.
..
Mr. Justice Black with whom Mr. Justice Stewarts joins dissenting..
The right of privacy as a comprehensive substitute against
unreasonable search and seizures Privacy is broad, abstract and
ambiguous concept which can easily be shrunken in meaning but which
can also, on the other hand, easily be interpreted as a constitutional
ban against many things other than searches and seizures.
The government has a right to invade the right to privacy unless
prohibited by some specific constitutional provision.
There is no provision in the constitution that gives the court blanket
power to exercise supervisory veto over the wisdom and value of
legislative policies and to hold unconstitutional those laws which they
believe is unwise or dangerous. The use of federal courts of such a
formula or doctrine or whatnot to veto federal laws simply takes away
the power to make laws from congress and transfers the power to this
court for ultimate determination.
It is not too much to say that there is no legislative body ever does pass
laws without believing that they will accomplish a sane, rational, wise
and justifiable purpose.

Associations, Conformity and Dissension


It is generally the rule that the collective might of like-minded
persons could achieve much more than any force or pressure
contributed by each member acting on his own. Like any other
right, this is also subject to limitations, one of which applies to
those in the government service
81 | P

LATON

The right of government employees to organized is limited to


the formation of unions or associations only, without including
the right to strike
While members may exercise their right to speak and disagree
within the organization, they may also exercise their right to
dissociate if they could no longer reconcile their own personal
feelings, principles or objectives with those of the group
Be that as it may, however, there may also be instances where,
even if one desires to exercise his right not to associate with
others, he may still be compelled to contribute his share for the
upkeep of an organization whose existence is required by
public interest

In contrast to other fundamental powers, the power of eminent


domain may be exercised not only by the government itself but
even by private corporations, such as public utilities involved in
supplying electricity, water, telecommunications, rail and air
transportation. The authority to exercise the power is indicated
in their franchises
Public Use
The public use requirement for a valid exercise of the power
of eminent domain is a flexible and evolving concept influence
by changing conditions
When land has been acquired for public use in fee simple
unconditionally, either by the exercise of eminent domain or by
purchase, the former owner retains no rights in the land, and
the public use may be abandoned, or the land may be devoted
to a different use, without any impairment of the estate or title
acquired, or any reversion to the former owner

Expressive Association
This right to group together people with similar ideas or views
on life and its varied aspects may mean also the right to choose
whom to accept into, or reject from, the group. To that extent,
therefore, the right to associate may be considered as an
aspect of the right to express oneself
The right to associate for expressive purposes is not, however,
absolute. Infringements on that right may be justified by
regulations adopted to serve compelling state interests,
unrelated to the suppression of ideas, that cannot be achieved
through means significantly less restrictive of associational
freedoms

Heirs of Juancho Ardona v. Reyes


125 SCRA 220 (1983)
Philippine Tourism Authority (PTA) expropriated certain properties in
Cebu to be converted to a Golf Course in a Sports Complex (basketball
courts, tennis courts, volleyball courts, track and field, baseball and soft
ball diamonds, and swimming pools), clubhouse, childrens playground
and a nature area for picnics, horseback riding for the use of the public.
The heirs said that it is not for public use, for that taking was not
impressed with public use.
No specific constitutional provision allowing the taking for tourism
purposes.
What the petitioners want the court to adopt was a strict construction
of PUBLIC USE.
Eminent domain is inherent in sovereignty and exists in a sovereign
state without any recognition in the constitution

Additional Cases
(J) Right of Association [1]

Chapter 11
Eminent Domain
Private property shall not be taken for public use without just
17
compensation.
It requires that if the government were to take any private
property, it may do so provided it is for public use and that
there be payment for it, which compensation must be one that
is just
It is only where the owner is unwilling to sell, or cannot accept
the price or other conditions offered by the vendee, that the
power of eminent domain will come into play to assert the
paramount authority of the State over the interests of the
property owner. Private rights must then yield to the irresistible
demands of the public interest in the time-honored
justification, as in the case of the police power, that the welfare
of the people is the supreme law

The restrictive view may be appropriate for a Nation which


circumscribes the scope of the government activities and public
concern and which concerns big and correctly located public lands that
obviate the need to take private property for public purposes. Neither
applies to the Philippines. We have never been a laissez faire country.
And the necessities which impel the exertion of sovereign power are all
too often found in areas of scarce public land or limited government
resources.
The constitutional restraints are public use and just compensation
HELD: public use can now be understood as for public interest or
welfare (Those that advance the interest of society)
The public respondents have stressed that the development of the 808
hectares includes plans that would give the petitioners and other
displaced persons productive employment, higher incomes, decent
housing, water and electric facilities, and better living standards. Our
dismissing this petition is, in part, predicated on those assurances. The
right of PTA to proceed with the expropriation of the 282 hectares
already identified as fit for the establishment of a sports complex to
promote tourism is, therefore, sustained.
Manosca v. CA
252 SCRA 412 (1996)
A piece of land located at P. Burgos St. Calzada Taguig, Metro Manila,
the birth place of Felix Manalo (founder of IglesiaNiCristo), is
expropriated to be converted to a National Historical Landmark.

17

CONSTITUTION, Art. III, 9

82 | P

LATON

Public use= expansive meaning; public advantage; general benefit


Violative of Religion Clause? NO.
This attempt to give some religious perspective to the case deserves
little consideration, for what should be significant is the principal
objective of, not the casual consequences that might follow from the
exercise of the police power.
The purpose was to recognize the distinctive contribution of the late F.
Manalo to the culture of the Phil., rather than to commemorate his
pounding of INC.
The practical reality that greater benefit may be derived by the
members of INC is merely secondary/ incidental. That the fact that only
a few would actually benefit does not necessarily diminish the essence
and character of public use.

Local governments have the authority to seize private land and


turn the property over to private developers for economic
development
Just Compensation
The word just is used to intensify the meaning of the word
compensation to convey the idea that the equivalent to be
rendered for the property to be taken shall be real, substantial,
full, or ample
To compensate is to render something which is equal in value
to that taken or received
It is well-settled that just compensation means the equivalent
for the value of the property at the time of its taking. Anything
beyond that is more, and anything short of that is less, than just
compensation
The market value of the land taken is the just compensation to
which the owner of condemned property is entitled, the market
value being that sum of money which a person desirous, but not
compelled to buy, and an owner, willing, but not compelled to
sell, would agree on as a price to be given and received for such
property
The property owner is entitled to compensation only for what
he actually loses, and what he loses is only the actual value of
the property at the time of taking
Judicial Determination
Just compensation would be determined in accordance with
the market value declared by the owner or administrator or
anyone having legal interest in the property, or such market
value as determined by the assessor, whichever is lower
Determination of just compensation is not as simple as childs
arithmetic--it is a judicial function that requires the judicious
minds of men and women!

declared by the owner or as determined by the assessor, whichever is


lower.
Prior to the promulgation of P.D. Nos. 76, 464, 794 and 1533, this Court
has interpreted the eminent domain provisions of the Constitution and
established the meaning, under the fundamental law, of just
compensation and who has the power to determine it. Thus, in the
following cases, wherein the filing of the expropriation proceedings
were all commenced prior to the promulgation of the aforementioned
decrees, we laid down the doctrine on just compensation:
Municipality of Daet v. Court of Appeals (93 SCRA 503, 516) xxx xxx xxx
And in the case of J.M. Tuason & Co., Inc. v. Land Tenure
Administration, 31 SCRA 413, the Court, speaking thru now Chief justice
Fernando, reiterated the 'well-settled (rule) that just compensation
means the equivalent for the value of the property at the time of its
taking. Anything beyond that is more and anything short of that is less,
than just compensation. It means a fair and fun equivalent for the loss
sustained, which is the measure of the indemnity, not whatever gain
would accrue to the expropriating entity
Garcia v. Court ofappeals (102 SCRA 597, 608) xxx xxx xxx Hence, in
estimating the market value, afl the capabilities of the property and all
the uses to which it may be applied or for which it is adapted are to be
considered and not merely the condition it is in the time and the use to
which it is then applied by the owner. And the facts as to the condition
of the property and its surroundings, its improvements and capabilities
may be shown and considered in estimating its value.
Republic v. Santos (141 SCRA 30, 35-36), according to section 8 of Rule
67, the court is not bound by the commissioners' report. It may make
such order or render such judgment as shall secure to the plaintiff the
property essential to the exercise of his right of condemnation, and to
the defendant just compensation for the property expropriated. This
Court may substitute its own estimate of the value as gathered from
the record (Manila Railroad Company v. Velasquez, 32 Phil. 286).
However, the promulgation of the aforementioned decrees practically
set aside the above and many other precedents hammered out in the
course of evidence-laden, well argued, fully heard, studiously
deliberated, and judiciously considered court proceedings. The decrees
categorically and peremptorily limited the definition of just
compensation thus:
P.D. No. 76: For purposes of just compensation in cases of private
property acquired by the government for public use, the basis shall be
the current and fair market value declared by the owner or
administrator, or such market value as determined by the Assessor,
whichever is lower.
P.D. No. 464, 794: Section 92. Basis for payment of just compensation in
expropriation proceedings. In determining just compensation which
private property is acquired by the government for public use, the basis
shall be the market value declared by the owner or administrator or
anyone having legal interest in the property, or such market value as
determined by the assessor, whichever is lower.
P.D. No. 1533: Section 1. In determining just compensation for private
property acquired through eminent domain proceedings, the
compensation to be paid shall not exceed the value declared by the
owner or administrator or anyone having legal interest in the property
or determined by the assessor, pursuant to the Real Property Tax Code,
whichever value is lower, prior to the recommendation or decision of
the appropriate Government office to acquire the property.

Export Processing Zone Authority V. Dulay


149 SCRA 305 (1987)

Held: PD 1533 which eliminates the court's discretion to appoint


commissioners pursuant to Rule 67 of the Rules of Court, is
unconstitutional and void.

The question raised in this petition is whether or not Presidential


Decrees Numbered 76, 464, 794 and 1533 have repealed and
superseded Sections 5 to 8 of Rule 67 of the Revised Rules of Court,
such that in determining the just compensation of property in an
expropriation case, the only basis should be its market value as

We are convinced and so rule that the trial court correctly stated that
the valuation in the decree may only serve as a guiding principle or one
of the factors in determining just compensation but it may not
substitute the court's own judgment as to what amount should be
83 | P

LATON

awarded and how to arrive at such amount.


It is violative of due process to deny to the owner the opportunity to
prove that the valuation in the tax documents is unfair or wrong.
The determination of "just compensation" in eminent domain cases is a
judicial function. The executive department or the legislature may
make the initial determinations but when a party claims a violation of
the guarantee in the Bill of Rights that private property may not be
taken for public use without just compensation, no statute, decree, or
executive order can mandate that its own determination shag prevail
over the court's findings. Much less can the courts be precluded from
looking into the "just-ness" of the decreed compensation

Timely and Prompt Payment


Determination of just compensation would have to be
reckoned from the value of the property either at the time of
the taking or the filing of the complaint for condemnation,
whichever comes first
Property cannot simply be taken in the meantime and payment
to follow some years or decades later. It is not fair that the
owner, while immediately divested of his property, is made to
wait for the equivalent value for some time
Without prompt payment, compensation cannot be considered
just inasmuch as the property owner is made to suffer the
consequences of being immediately deprived of his land while
being made to wait for a decade or more before actually
receiving the amount necessary to cope with his loss. Payment
of just compensation should follow as a matter of right
immediately after the order of expropriation is issued. Any
delay in payment must be counted from said order. However,
the delay to constitute a violation of due process must be
unreasonable and inexcusable; it must be deliberately done by
a party in order to defeat the ends of justice
Where the payment of compensation does not accompany the
taking of property for public use but is postponed to a later
date, the owner of the property is ordinarily entitled to the
award of an additional sum which will compensate for delay or
which will, in other words, produce the full equivalent of the
value of the property paid contemporaneously with the taking
Where private property is acquired by the Government and all
that remains is the payment of the price, the owners action to
collect the price must be brought within ten years otherwise it
would be barred by the statute of limitations. However, where
private property is taken by the Government for public use
without first acquiring title thereto either through
expropriation or negotiated sale, the owners action to recover
the land or the value thereof does not prescribe
Commissioner of public highways v burgos
96 SCRA 831 (1980)
Govt took land for right-of-way purpose. Year 1924
Petitioner wanted to recover, Respondent says, it was either sold or
donated, cannot be recovered.
Dismissed by trial court 1959.
Appealed to SC. Reversed. And the case was remanded to the court of
origin for the compensation plus attorneys fees plus the determination
of just compensation.

Would article 1250 of NCC apply regarding extraordinary inflation?


Peso to dollar rate at the time of hearing was P6.775=$1
NO. 1. There is no contractual obligation 1250 applies if there was
contract or agreement. Obviously there can be no agreement to the
contrary to speak of because the obligation sought by the government
did not originate from contract but from law, which is generally not
subject to the will of the parties.
Also the unusual delay in bringing up the present action (a period of
almost 25 years) bars her claim.
Therefore, the correct amount to be paid was P14615 (based on the
time of taking in 1924, and not adjusted to P49459.34)
Interest of 145410.44 at the rate 0f 6% from 1924 should be reduced.
The solicitor general said that the award of damages should be from
1924 (in the decision in Feb 29 1972) citing the case of raymunda s.
digsan v auditor general, et al ruling that it should be from the time
when the claim for compensation was filed. Whether the ruling in this
case cited is still the prevailing doctrine, what was said in the decision
of this court in the above-cited case involving the same on the instant
matter has become the law of the case. No motion for reconsideration
having been filed by the solicitor general before the decision has
become final.
Wherefore, judgment appealed from was reversed; value of the land
should be from the time of taking P14615 (at P2.37 per sqm) with
interest at 6% per annum from 1924 plus Atty fees of 5000

Form or Mode of Compensation


GR: To satisfy the requirement of just compensation payment
for property taken should be in the form of cash
XPN: If the exercise of such power would entail expansive
acquisitions to carry out an important public policy, such as
agrarian reform
In relation to the agrarian reform program, the Government
has provided for the payment partly in cash and partly in
government financial instruments, as well as payment by
means of shares of stock in government-owned or controlled
corporations, Land Bank of the Philippines preferred shares,
physical assets or other qualified investments, tax credits, and
Land Bank bonds
Parties Entitled to Compensation
In the American jurisdiction, the term owner when employed
in statutes relating to eminent domain to designate the persons
who are to be made parties to the proceeding, refers, as is the
rule in respect to those entitled to compensation, to all those
who have lawful interest in the property to be condemned,
including a mortgagee, a lessee and a vendee in possession
under an executory contract
Government Grants, Easement and Compensation
It may happen that a property that is now held private used to
be government-owned and that it was just granted to citizens
by way of free patent, homestead or any other mode. The issue
that might be raised is whether the government would have to
pay if it takes back part of it for use as an easement. The
answer to this is to be determined in accordance with whatever
reservations there might have been in the grant itself
84 | P

LATON

GR: Non-payment of just compensation does not entitle the


private landowners to recover possession of their expropriated
lots
Elements of Taking
(1) Entry by the expropriator into a private property
(2) The entrance into private property must be for more than a
momentary period
(3) Such entry should be under warrant or color of legal
authority
(4) The property must be devoted to public use or otherwise
informally appropriated or injuriously affected
(5) The utilization of the property for public use must be in such
a way as to oust the owner and deprive him of all beneficial
enjoyment of the property

XPN: In cases where the government failed to pay just


compensation within five (5) years from the finality of the
judgment in the expropriation proceedings, the owners
concerned shall have the right to recover possession of their
property. This is in consonance with the principle that the
government cannot keep the property and dishonour the
judgment
Expropriation, Agrarian Reform and Socialized Housing
While the powers of the government have to march together
harmoniously, like in implementing the agrarian reform
program, there may be instances, however, where the power of
expropriation may come into conflict with such program. In the
event of a conflict, the latter must give way to the former

Questions of Necessity and Judicial Oversight


While basically the determination of which property to be
expropriated is left to the discretion of the authority seeking to
exercise the power, the issue of necessity and arbitrariness
may, however, come to the fore, especially if it is a local
authority or other agency that is trying to exercise such
delegated power. This would then be for the courts to look
into, determining from the facts and the circumstances of each
case whether the power sought to be utilized indeed exists, and
whether it is being applied in a manner consistent with the
authority given
The authority to condemn is to be strictly construed in favor of
the owner and against the condemnor. When the power is
granted, the extent to which it may be exercised is limited to
the express terms or clear implication of the statute in which
the grant is contained

Stages or Phases of Expropriation Proceedings


The first one determines the authority of the condemning
agency and also of the propriety of its exercise of the power in
the context of the facts obtaining in the suit. The second phase
is the determination of the amount of just compensation
The second stage which involves the issue of just compensation
is as important, if not more, than the first stage which refers to
the issue of public purpose
Parenthetically, while entry and possession in favor of
expropriator may be effected at once with the filing of
complaint and the deposit of the assessed value of
property, transfer f ownership would only be made after
payment of the just compensation

the
the
the
full

Non-Payment of Just Compensation and Recovery of Property

Province of Camarines Sur v. Court of Appeals


222 SCRA 173 (1993)
The Province of Camarines Sur filed two separate cases for
expropriation against San Joaquin and was subsequently authorized to
take possession of the property to (1) establish a pilot farm for nonfood and non-traditional agricultural crops and (2) a housing project
for provincial government employees. The San Joaquins filed a motion
to relief and a motion to admit an amended motion to dismiss, both
were denied by the trial court. They then assailed before the Court of
Appeals. The CA ordered the suspension of expropriation proceedings
until after the province shall have secured the approval of the
Department of Agrarian Reform of the plan to expropriate the lands of
petitioners for the use of housing project.
There has been a shift from the literal to broad interpretation of public
purpose or public use for which the power of eminent domain may
be exercised. Under the new concept, public use means public
advantage, convenience or benefit, which tends to contribute to the
general welfare and the prosperity of the whole community. The
expropriation of the property in question is for public purpose. The
establishment of the pilot development center would make available to
the community the invaluable information and technology on
agriculture, fishery, and cottage industry. The housing project also
satisfies the public purpose requirement of the Constitution. As held in
Sumulong v. Guerrero (154SCRA461), Housing is a basic human need.
Shortage in housing is a state concern since it directly and significantly
affects the general welfare.
Sec. 9 of BP Blg. 337 does not intimate in the least that local
government units must first secure the approval of DAR for the
conversion of lands from agricultural to non-agricultural use, before
they can institute the expropriation proceedings. Likewise, there is no
provision in the Comprehensive Agrarian Reform Law which expressly
subjects the expropriation of agricultural lands by the local government
units to the DAR. In Heirs of Juancho Ardana v. Reyes (125SCRA220),
the power of expropriation is superior to the power to distribute lands
under the land reform program.

Acquisition and Transfers of Municipal Properties by National


Government
Local government may have properties of their own. Depending
on the nature of property, whether held as patrimonial or
private property or one that is communal, the national
government may take the same either for consideration or not.
In the former, since the property is really privately owned, then
the national government if it wants it would have to pay for it.
In the latter, the property may be considered as held in trust by
85 | P

LATON

the local government for the benefit of the people and the
national authority
City of Baguio v. National Waterworks and Sewerage Authority
106 Phil. 144 (1959)
RA 1383 created NAWASA, a public corporation, the purpose of which
is to consolidate and centralize all waterworks, sewerage, and
drainage systems in the Philippines under one control, direction, and
general supervision. The law also provided that all existing government
owned waterworks and sewerage systems in the Philippines are
transferred to NAWASA, and that the net book value of the properties
and assets of said entities shall be received by the Authority in payment
for an equal value of the assets of NAWASA.
A complaint was filed contending that RA 1383 does not include the
Baguio Waterworks System. Also, assuming that it does, said Act is
unconstitutional because it has the effect of depriving the plaintiff the
ownership, control, and operation of said waterworks system without
compensation and without due process of law. NAWASA contended
that RA 1383 is a proper exercise of police power of the State and
assuming that the said act contemplates an act of expropriation, it is
still a constitutional exercise of the power of eminent domain and that
at any rate BWS is not a private property but public works for public
service over which the Legislature has control.
The contention that RA 1383 constitutes valid exercise of police power
rather than a directive to expropriate by the exercise of power of
eminent domain cannot be entertained. The Act does not confiscate,
destroy nor appropriate property. It merely directs that all waterworks
belonging to cities, municipalities and municipal districts in the
Philippines be transferred to the NAWASA.
Baguio Waterworks system is not a property held in trust by a
municipal corporation for the benefit of the public but it is rather a
property owned in a proprietary character. The property held by a
municipal corporation in its private capacity is not subject to
unrestricted control of the legislature, and the municipality cannot be
deprived of such property against its will, except by the exercise of
eminent domain with payment of full compensation.
Salas v. Jarencio
46 SCRA 734 (1972)
RA 4118 was never intended to expropriate the property involved but
merely to confirm its character as a communal land of the State and to
make it available for disposition by the National Government. The
subdivision of the land and the conveyance of the resulting subdivision
lots to the occupants by Congressional authorization do not operate as
an exercise of the power of eminent domain without just compensation
in violation of the Constitution, but simply as a manifestation of its right
and power to deal with state property.

Constructive Expropriation and Consequential Damages


While taking ordinarily connotes physical taking, the same need
not always be. There could be taking even if there is no physical
entry or dispossession of the owner. In this instance of
constructive taking, considering that the owner suffers some
loss, he ought to be compensated. Taking occurs not only
when the government actually deprives or dispossesses the
property owner of his property or of its ordinary use, but also
when there is a practical destruction or material impairment of
the value of his property.
United Sates v. Causby
328 U.S. 256, 90 L Ed 1206, 66 S Ct 1062 (1946)
Respondents owned 2.8 acres near an airport outside of Greensboro,
North Carolina. It has on it a dwelling house, and also various
outbuildings which were mainly used for raising chickens. Various

military aircrafts used the airport. They frequently came close to


respondents property; the noise was startling, and the glare of the
landing lights lighted the place up brightly at night. As a result of the
noise, respondents had to give up their chicken business. The result
was destruction of the use of property as a commercial chicken farm.
Respondents were also frequently deprived of sleep, and the family had
become nervous and frightened. They sued the in the Court of Claims
to recover for an alleged taking of their property and for damages to
their poultry business.
The Court of claims held that there was a taking. The damages were
not merely consequential. They were product of a direct invasion of
the respondents domain. Flights over private lands are not taking,
unless they are so low and so frequent as to be a direct and immediate
interference with the enjoyment and the use of the land. The findings
of the court of claims established that there was a diminution in value
of the property, and that the frequent low-level flights were the direct
and immediate cause.
Dissenting Opinion: When flights are made within the navigable
airspace without any physical invasion of the property of the
landowners, there has been no taking of property. It says that, at most,
there was merely incidental damage occurring as a consequence of
authorized air navigation.
Republic v. PLDT
26 SCRA 620 (1969)
In 1947, soon after its creation, the Bureau of Telecommunications set
up its own Government Telephone System by utilizing its own
appropriation and equipment and by renting trunk lines of PLDT, to
enable government offices to call private parties
In March 1958, the plaintiff, through the Director of
Telecommunications, entered into an agreement with RCA
Communications, Inc. (RCA), for a joint overseas telephone service
whereby the Bureau would convey -radio-telephone overseas calls
received by RCAs station to and from local residents. Thereafter, in
April 1958, PLDT complained to the Bureau of Telecommunications that
said bureau was violating the conditions under which Private Branch
Exchange (PBX) is interconnected with the PLDTs facilities for the
bureau had used the trunk lines not only for the use of government
offices but even to serve private persons of the general public, in
competition with the business of the PLDT. It gave notice that if said
violations were not stopped, it would sever the telephone connections
They did not arrive at mutually acceptable terms, however
After trial, the lower court rendered judgment that it could not compel
the PLDT to enter into an agreement with the Bureau because the
parties were not in agreement
Held: Yes. The court a quo has apparently overlooked that while the
Republic may not compel the PLDT to celebrate a contract with it, the
Republic may, in the exercise of the sovereign power of eminent
domain, require the telephone company to permit interconnection of
the government telephone system and that of the PLDT, as the needs
of the government service may require, subject to the payment of just
compensation to be determined by the court
The Bureau of telecommunications, under Section 78(b) of Executive
Order No. 94, may operate and maintain wire telephone or radio
telephone communications throughout the Philippines by utilizing
existing facilities in cities, towns, and provinces under such terms and
conditions or arrangement with present owners or operators as may be
agreed upon to the satisfaction of all concerned; but there is nothing in
this Section that would exclude resort of condemnation proceedings
where unreasonable or unjust terms and conditions are exacted, to the
extent of crippling or seriously hampering the operations of said bureau

The concept of public use is no longer confined in the


traditional notion of use by the public, but held synonymous
86 | P

LATON

with public interest, public benefit, public welfare, and public


convenience
Elections and the Power of Eminent Domain
The Constitution directs the Commission on Elections during
the election period to supervise and regulate the enjoyment or
utilization of all franchise or permits for the operation of media
of communication or information aimed at ensuring equal
opportunity, time and space, and the right to reply, including
reasonable, equal rates therefor, for public information
campaigns and forums among candidates in connection with
the objective of holding free, orderly, honest, peaceful, and
credible elections. In implementation of this mandate, the
COMELEC was directed by law to procure from the print media
the so-called COMELEC Space, and from the broadcast media,
the COMELEC Time. The issue of compensation came up in two
cases, with contrasting results
Philippine Press Institute, Inc. v. COMELEC
244 SCRA 272 (1995)
On March 2 1995, the COMELEC promulgated Resolution No. 2772,
which provided in part as follows: Sec. 2. Comelec Space. The
Commission shall procure free print space of not less than one half (1/2)
page in at least one newspaper of general circulation in every province
or city for use as "Comelec Space" from March 6, 1995 in the case of
candidates for senator and from March 21, 1995 until May 12, 1995. In
the absence of said newspaper, "Comelec Space" shall be obtained
from any magazine or periodical of said province or city.
In the instant Petition for Certiorari and Prohibition, PPI asks the Court
to declare Comelec Resolution No. 2772 unconstitutional and void on
the ground that it violates the prohibition against the taking of private
property for public use without just compensation
Held: Yes. To compel print media companies to donate Comelec
space of the dimensions specified in Section 2 of resolution No. 2772
(not less than one-half Page), amounts to taking of private personal
property for public use or purposes
The threshold requisite for a lawful taking of private property for
public use need to be examined here: one is the necessity for the
taking; another is the legal authority to effect the taking. The element
of necessity for the taking has not been shown by respondent Comelec
Section 2 of Resolution No. 2772 does not, however, provide a
constitutional basis for compelling publishers, against their will, in the
kind of factual context here present, to provide free space for Comelec
purposes. Section 2 does not constitute a valid exercise of the power of
eminent domain
Section 2 of Resolution No. 2772 is a blunt and heavy instrument that
purports, without a showing of existence of a national emergency or
other imperious public necessity, indiscriminately and without regard
to the individual business condition of particular newspaper or
magazines located in different parts of the country, to take private
property of newspaper and magazine publishers
Telecommunications and Broadcast Attorneys of the Philippines, Inc.
v. COMELEC 289 SCRA 337 (1998)
Petitioners challenge the validity of 92 of B.P. Blg. 881 (Omnibus
Election Code) on the ground, among others, that it takes property
without due process of law and without just compensation
It will be noted that while 90 of B.P. Blg. 881 requires the COMELEC to
procure print space which as we have held, should be paid for, 92
states that air time shall be procured by the COMELEC free of charge
Petitioners claim that the primary source of revenue of the radio and

television stations is the sale of air time to advertisers and that to


require these stations to provide free air time is to authorize a taking
which is not a de minimis temporary limitation or restraint upon the
use of private property
Held: No. All broadcasting, whether radio or by television stations, is
licensed by the government
They are merely given temporary privilege of using them. Since a
franchise is a mere privilege, the exercise of the privilege may
reasonably be burdened with the performance by the grantee of
some form of public service
Under 92 of B.P. Blg. 881, the COMELEC does not take over the
operation of radio and television stations but only the allocation of air
time to the candidates for the purpose of ensuring, among other things,
equal opportunity, time, and the right to reply as mandated by the
Constitution
This right of the people is paramount to the autonomy of broadcast
media. To affirm the validity of 92, therefore, is likewise to uphold the
peoples right to information on matters of public concern

Public-Interest Business, Takeovers and Compensation


With regard to public utilities, businesses imbued with public
interest, or vital industries in which exigencies may at times
dictate that the Government take over either momentarily or in
a more permanent manner, the Constitution provides:
Section 17. In times of national emergency, when the public interest so
requires, the State may, during the emergency and under reasonable
terms prescribed by it, temporarily take over or direct the operation of
any privately-owned public utility or business affected with public
interest.
Section 18. The State may, in the interest of national welfare or
defense, establish and operate vital industries and, upon payment of
just compensation, transfer to public ownership utilities and other
private enterprises to be operated by the Government.

While the provision on transfer of utilities is explicit that there


be payment of just compensation, that on temporary takeover
does not say so
The temporary takeover by the government extends only to the
operation of the business and not to the ownership thereof. As
such the government is not required to compensate the private
entity-owner of the said business as there is not transfer of
ownership, whether permanent or temporary
Miscellaneous Considerations
Under the so-called concept of inverse condemnation, it is
the owner who brings suit to recover the compensation on
account of physical intrusion into his property by the
authorities without initiating the appropriate condemnation
proceeding. This may be referred to as regulatory taking,
such as when, by certain regulations affecting private property
like land, there is effectively put in place a permanent
restriction or prohibition on its beneficial use. In such an
instance, the same might come under the concept of taking
for which the owner may claim and be entitled to
compensation
When there is only an incidental taking as a result of certain
public works, such as those caused by the construction of
roads, bridges, underpass or flyovers, then the impairment of
the value of ones property may not be considered
87 | P

LATON

compensable. It is just one of the things one would have to live


up in an ever changing and dynamic society
The provision (Article III, Section 9) is the most important
protection of property rights in the Constitution. This is a
restriction on the general power of the government to take
property. The constitutional provision is about ensuring that
the government does not confiscate the property of some to
give it to others.
There are two different kinds of taking that can be identified. A
possessory taking occurs when the government confiscates or
physically occupies property. A regulatory taking occurs when
the governments regulation leaves n reasonable economically
viable use of the property
A regulation which denies all economically beneficial or
productive use of land will require compensation under the
taking clause
Any arm of the State that exercises the delegated power of
eminent domain must wield that power with circumspection
and utmost regard for procedural requirements. A government
instrumentality that fails to observe the constitutional
guarantees of just compensation and due process abuses the
authority delegated to it, and is liable to the property owner for
damages
Familara v. JM Tuason Co., Inc
49 SCRA 338 (1973)
Petitioner, barrio captain of Barrio Tatalon, seeks to enjoin the
respondent Corporations from bulldozing and fencing any portion of
the Tatalon Estate and/or from selling any of the lots therein, and/or
from filing ejectment cases against the bona-fide occupants of the
estate...Reliance is thus placed by petitioner upon the provisions of
Section 4 of Republic Act No. 2616
Held: This Court has also ruled that Section 4 thereof, which places a
restraint upon the exercise and enjoyment by the owner of certain
rights over its property, is justifiable only if the government takes
possession of the land and is in a position to make a coetaneous
payment of just compensation to its owner
For it must be realized that in a condemnation case, it is from the
condemnors taking possession of the property that the owner is
deprived of the benefits of ownership, such as possession,
management and disposition thereof. Before that time, the proprietary
right of the owner over his property must be recognized
It was definitively ruled that until the proceedings for condemnation
have been fully instituted and possession of the property is taken
over by the condemnor, the enforcement of final decrees of eviction
may not be lawfully suspended

In the acquisition of private property, the Government may


either act through persuasion or negotiation, but if all else fails
and it is rendered helpless on that score, it may then resort to
its coercive power of expropriation, subject always, of course,
to the constitutional guidelines that the property be for public
use and that there be payment of just compensation. Thus, it
could be seen that the tension and accommodation between
private rights and public interests continue to play on
Additional Cases
(K) Eminent Domain [9]
K1. Manotok Realty Inc v CLT realty development corporation

582 scra 583 (2009)


*from Constitutional Law Updates and Teasers (bar 2010) by Atty
Gorospe
Here the court spoke of cleansing effect of the expropriation
proceedings. The fact of expropriation is extremely significant, for
titles acquired by the state by way of expropriation are deemed
cleansed of whatever previous flaws may have attended these titles . . .
. in an in rem proceeding, condemnation acts upon the property. After
condemnation, the paramount title is in the public under a new and
independent title; thus, by giving notice to all claimants to a disputed
title, condemnation proceedings provide a judicial process for securing
better title against all the world than may be obtained by voluntary
conveyance
In this particular case, the court noted that in annulling the manotok
titles, focus was laid on the alleged defects of TCT No. 4211 issued in
September of 1918. However, TCT No. 4211 was issued decades before
the property was expropriated. Thus, any and all defects that may have
attended that particular title would have been purged when the
property covered by it was subsequently acquired by the State through
Eminent Domain.
SPOUSES CIRIACO and ARMINDA ORTEGA vs CITY OF CEBU
In this case, the sangguniang panlunsod of the City of Cebu enacted
ordinance 1519 for the expropriation of the land of the spousesOiriaco
and Armada Ortega also appropriating the sum of P3,284,400.00. The
city filed a case in court for the expropriation. The court fixed the value
of the land to 11000 per square meter or the sum of P31,416,000.00.
this decision of the rtc became final and executory because of Cebu
Citys failure to appeal and a writ of execution was issued to enforce
the courts judgment. The appropriated amount was now subject for
execution or garnishment for the same is no longer exempt from
execution.
The city of Cebu filed an Omnibus Motion to Stay Execution,
Modification of Judgment and Withdrawal of the Case, contending that
the price set by the RTC as just compensation is way beyond the reach
of its intended beneficiaries for its socialized housing program. The
motion was denied and the Motion for Reconsideration was likewise
denied.
Pursuant to the order by the RTC, Sheriff Benigno B. Reas served a
Notice of Garnishment to Philippine Postal Bank, P. del Rosario and
Junquera Branch Cebu City, garnishing [Cebu Citys] bank deposit
therein.
Hence, [Cebu City] filed the instant Petition for Certiorari before the CA
During its pendency, [Cebu City] filed before the [RTC] a Motion to
Dissolve, Quash or Recall the Writ of Garnishment, contending that
Account No. 101-8918-334 mentioned in Ordinance No. 1519 is not
actually an existing bank account and that the garnishment of [Cebu
Citys] bank account with Philippine Postal Bank was illegal, because
government funds and properties may not be seized under writ of
execution or garnishment to satisfy such judgment, on obvious reason
of public policy. The RTC denied said motion. [Cebu Citys] Motion for
Reconsideration was also denied.
[The Spouses Ortega] filed an Ex-Parte Motion to Direct the New
Manager of Philippine Postal Bank to Release to the Sheriff the
Garnished Amount, which was granted by the [RTC]. [Cebu City] filed a
Motion for Reconsideration, but the same was denied.
Hence, [Cebu City] filed another Petition for Certiorari (CA-G.R. SP NO.
00147) [with the Court of Appeals].
-Partially Granted. The assailed Orders of the [RTC] [Assailed Orders
dated March 11, 2002 and July 2, 2003, respectively, in CA-G.R SP NO.
80187] are hereby ANNULLED AND SET ASIDE insofar as they denied
[Cebu Citys] Motion to Stay Execution, but they are hereby AFFIRMED
insofar as they denied [Cebu Citys] Motion to Modify Judgment and
Withdraw from the Expropriation Proceedings. Furthermore, the
assailed Orders of the [RTC dated March 8, 2004 in CA-G.R. SP NO.
00147] are hereby ANNULLED AND SET ASIDE. Let the Decision of the
[RTC] be executed in a manner prescribed by applicable law and
jurisprudence.
Hence, these consolidated appeals by petitioners Cebu City and the
Spouses Ortega positing the following issues:
1. Whether the CA erred in affirming the RTCs denial of Cebu Citys
88 | P

LATON

Omnibus Motion to Modify Judgment and to be Allowed to Withdraw


from the Expropriation Proceedings.
2. Whether the deposit of Cebu City with the Philippine Postal Bank,
appropriated for a different purpose by its Sangguniang Panglungsod,
can be subject to garnishment as payment for the expropriated lot
covered by City Ordinance No. 1519.
We deny both petitions.
On the first issue, the CA did not err in affirming the RTCs Order that
the expropriation case had long been final and executory.
Consequently, both the Order of expropriation and the Order fixing just
compensation by the RTC can no longer be modified. In short, Cebu City
cannot withdraw from the expropriation proceedings.
Section 4, Rule 67 of the Rules of Court on Expropriation provides:
SEC. 4. Order of expropriation. If the objections to and the defenses
against the right of the plaintiff to expropriate the property are
overruled, or when no party appears to defend as required by this Rule,
the court may issue an order of expropriation declaring that the
plaintiff has a lawful right to take the property sought to be
expropriated, for the public use or purpose described in the complaint,
upon the payment of just compensation to be determined as of the
date of the taking of the property or the filing of the complaint,
whichever came first.
A final order sustaining the right to expropriate the property may be
appealed by any party aggrieved thereby. Such appeal, however, shall
not prevent the court from determining the just compensation to be
paid.
An order of expropriation puts an end to any ambiguity regarding the
right of the petitioner to condemn the respondents properties.
Because an order of expropriation merely determines the authority to
exercise the power of eminent domain and the propriety of such
exercise, its issuance does not hinge on the payment of just
compensation. After all, there would be no point in determining just
compensation if, in the first place, the plaintiffs right to expropriate the
property was not first clearly established.
Conversely, as is evident from the foregoing, an order by the trial court
fixing just compensation does not affect a prior order of expropriation.
As applied to the case at bar, Cebu City can no longer ask for
modification of the judgment, much less, withdraw its complaint, after
it failed to appeal even the first stage of the expropriation proceedings.
As regards the second issue raised by the Spouses Ortega, we quote
with favor the CAs disquisition thereon, to wit:
While the claim of [the Spouses Ortega] against [Cebu City] is valid, the
[RTC] cannot, by itself, order the City Council of [Cebu City] to enact an
appropriation ordinance in order to satisfy its judgment.
The proper remedy of [the Spouses Ortega] is to file a mandamus case
against [Cebu City] in order to compel its Sangguniang Panglungsod to
enact an appropriation ordinance for the satisfaction of [the Spouses
Ortegas] claim. This remedy is provided in the case of Municipality of
Makati v. Court of Appeals, which provides:
Nevertheless, this is not to say that private respondent and PSB are left
with no legal recourse. Where a municipality fails or refuses, without
justifiable reason[s], to effect payment of a final money judgment
rendered against it, the claimant may avail of the remedy of mandamus
in order to compel the enactment and approval of the necessary
appropriation ordinance, and the corresponding disbursement of
municipal funds therefor
It is a settled rule that government funds and properties may not be
seized under writs of execution or garnishment to satisfy judgments,
based on obvious consideration of public policy. Disbursements of
public funds must be covered by the corresponding appropriation as
required by law. The functions and public services rendered by the
State cannot be allowed to be paralyzed or disrupted by the diversion
of public funds from their legitimate and specific objects, as
appropriated by law.
WHEREFORE, the petitions in G.R. Nos. 181562-63 and 181583-84 are
hereby DENIED. The Decision of the Court of Appeals in CA-G.R. SP Nos.
80187 and 00147 is AFFIRMED. No pronouncement as to costs.
K2. Ortega v. City of Cebu
602 scra 601 (2009)
*from Constitutional Law Updates and Teasers (bar 2010) by Atty
Gorospe.. )

Where the expropriation case had long been final and executory, both
the order of expropriation and the order fixing just compensation can
no longer be modified-the expropriator can no longer withdraw from
the expropriation proceedings.
Also, the expropriation court cannot, by itself, order the expropriating
local government to enact an appropriation ordinance in order to
satisfy its judgment--the land owner must file a separate mandamus
case for that purpose.

K7. Masikip v. City of Pasig


479 SCRA 391 (2006)
Lourdes Dela Paz Masikip is the registered owner of a parcel of land,
which the City of Pasig sought to expropriate a portion thereof for the
sports development and recreational activities of the residents of
Barangay Caniogan. Masikip refused.
W/N there was genuine necessity to expropriate the property.
Judicial review of the exercise of eminent domain is limited to the ff.
areas of concern: (a) adequacy of just compensation; (b) necessity of
the taking; (c) public use character of the purpose of the taking.
There is already an established sports development and recreational
activity center at Rainforest Park in Pasig City. Evidently, there is no
genuine necessity to justify the expropriation. The records show that
the Certification issued by the Caniogan Barangay Council which
became the basis for the passage of Ordinance No. 4, authorizing the
expropriation, indicates that the intended beneficiary is the
Melendres Compound Homeowners Association, a private, nonprofit organization, not the residents of Caniogan. Where the taking by
the State of a private property is done for the benefit of small
community, such taking cannot be considered for public use.
The right to own and possess property is one of the most cherished
rights of men. Unless the genuine necessity for the expropriations is
clearly established, it shall be the duty of the courts to protect the
rights of individuals to their private property.
K8. National Power Coprporation v. Tiangco
514 SCRA 674 (2007)
Respondents are owners of a parcel of land with an area of 152,187
square meters at Barangay Sampaloc, Tanay, Rizal. NPC requires 19,423
square meters of the respondents aforementioned property, across
which its 500Kv Kalayaan-San Jose Transmission Line Project will
traverse. NPCs Segregation Plan for the purpose shows that the
desired right-of-way will cut through the respondents land. Within the
portion sought to be expropriated stand fruit-bearing trees, such as
mango, avocado, jackfruit, casuy, santol, calamansi, sintones and
coconut trees. After repeated unsuccessful negotiations, NPC filed an
expropriation complaint against the land of the respondent in the RTC
of Tanay, Rizal. The RTC issued a writ of possession in favor of NPC after
paying the deposit requirement. The trial court rendered its decision on
the value of the property using the 1984 tax declaration. (which is
incorrect as stated in the decision of the supreme court) The
respondents filed a motion for reconsideration but it was denied by
RTC. So they filed an appeal and the CA gave merit to the contention of
the respondents and made its revised valuation using the 1993 tax
declaration (increasing the value of the property). The case went up to
the SC.
W/N the property should be valued using the 1984 or the 1993 tax
89 | P

LATON

declarations.
W/N Sec. 3-A of R.A. No. 6395, as amended by P.D. 938 will apply.
In eminent domain cases, the time of taking is the filing of the
complaint, if there was no actual taking prior thereto. Hence, in this
case, the value of the property at the time of the filing of the complaint
on November 20, 1990 should be considered in determining the just
compensation due the respondents. Normally, the time of taking
coincides with the filing of complaint for expropriation as ruled in the
case of Power Corporation v. Court of Appeals, et al. The expropriation
proceedings in this case having been initiated by NPC on November 20,
1990, property values on such month and year should lay the basis for
the proper determination of just compensation.
It should not apply in the case at bar, the acquisition of such easement
is not gratis. The limitations on the use of the property taken for an
indefinite period would deprive its owner of the normal use thereof.
For this reason, the latter is entitled to payment of a just compensation,
which must be neither more nor less than the monetary equivalent of
the land taken.

Chapter 12
Contract Clause
No law impairing the obligation of contracts shall be passed

18

The clause, according to Corwin, is lately of negligible


importance, and might be well stricken from the Constitution.
For most practical purposes, in fact, it has been
It has to give way to the superior and legitimate exercise by the
State of the police power to promote the health, morals, peace,
education, good order, safety, and general welfare of the
people, especially so as statutes in the exercise of valid police
power must be read into every contract

limit the general right to legislate on the subject on


divorce...Nor are judgments, though rendered upon contracts,
deemed to be within the provision...Nor does a general law,
giving consent of a State to be sued, constitute a contract
According to Black, any statute which introduces a change into
the express terms of the contract, or its legal instruction, or its
validity, or its discharge, or the remedy for its enforcement,
impairs the contract. The extent of the change is not material. It
is not a question of degree or manner or cause, but of
encroaching in any respect on its obligation or dispensing with
any part of its force
It should not be overlooked, however, that the prohibition to
impair the obligation of contracts is not absolute and
unqualified. The prohibition is general, affording a broad
outline and requiring construction to fill in the details
The policy of protecting contracts against impairment
presupposes the maintenance of a government by virtue of
which contractual relations are worthwhile--a government
which retains adequate authority to secure the peace and good
order of society. The contract clause of the Constitution must,
therefore, be not only in harmony with, but also in
subordination to, in appropriate instances, the reserved power
of the State to safeguard the vital interests of the people
Indeed, since contracts of employment are impressed with
public interest, the provision of applicable statutes are deemed
written into such contracts and the parties are not at liberty to
insulate themselves and their relationships from the impact of
labor laws and regulations by simply contracting with each
other

The impairment clause is no longer inviolate; in fact, there are


many who now believe it is an anachronism in the present-day
society

Related to the foregoing, the Constitution also expressly


provides, in what is referred to as Reservation Clause, that no
franchise or right for the operation of a public utility shall be
granted except under the condition that it shall be subject to
amendment, alteration, or repeal by the Congress when the
common good so requires

In truth, the Contract Clause has never been thought as a


limitation on the exercise of the States power of taxation save
only where a tax exemption has been granted for a valid
consideration

As to the phrase obligations and contracts, the same imports


a legal duty to perform the specified obligation of that contract,
not to substitute and perform, against the will of one of the
parties, a different, albeit equally valuable obligation

Contractual tax exemptions, in real sense of the term and


where the non-impairment clause of the Constitution can
rightly be invoked, are those agreed to by the taxing authority
in contracts, such as those contained in government bonds or
debentures, lawfully entered into by them under enabling laws
in which the government, acting in its private capacity, sheds its
cloak of authority and waives its governmental immunity. Truly,
tax exemptions of this kind may not be revoked without
impairing the obligations of contracts. These contractual tax
exemptions, however, are not to be confused with tax
exemptions granted under franchises. A franchise partakes the
nature of a grant which is beyond the purview f the nonimpairment clause of the Constitution

What the guarantee prohibits is the passage of a law which


enlarges, abridges or in any manner changes the intention of
the contracting parties

With regard to contracts covered, it has been said that the


term embraces those that are executed, that is, grants, as well
as those that are executory...They embrace the charters of
private corporations...But not the marriage contract, so as to

FACTS: Appellant contests the validity of the Minnesota Mortgage


Moratorium Law, as being t to the contract clause and the due process
and equal protection clauses of the Fourteenth Amendment of the
Federal Constitution. Invoking the relevant provisions of the statue,
appellees applied to the District Court for an order extending the
period of redemption from a foreclosure sale. Their petition stated that

18

CONSTITUTION, Art. III, 10

As to intrusion by private persons into contractual obligations,


the Court has said that this is governed by statutory
enactments not by impairment clause of the Constitution. The
sole purpose of this provision is to safeguard the integrity of
valid contractual agreements against unwarranted interference
by the State in the form of laws
HOME BUILDING & LOAN ASSOCIATION V. BLAISDELL
290 US 398, 78 L Ed 413, 54 S Ct 231 (1934)

90 | P

LATON

they owned a lot in Minneapolis which they had mortgaged to


appellant; by reason of their default, the mortgaged had been
foreclosed and sold to appellant in May 1932; that, because of the
economic depression appellees had been unable to obtain a new loan
or to redeem, and that, unless the period of redemption were
extended, the property would be irretrievably lost, and that the
reasonable value of the property greatly exceeded the amount due on
the mortgage. The District Court of Minnesota entered judgment
extending the period of redemption, subject to the condition that the
appellees should pay the appellant $40 a month through the extended
period. The SC of the State sustained the judgment, upholding the
statute as an emergency measure, namely, the public economic
emergency.

more so if the credits are unsecured. And the injustice is more patent
when the debtor is not even required to pay interest during the
operation of the relief, unlike similar statutes in the United States.
Official pronouncements of the Chief Executive states that the
nation has already recovered from the war, and that the peso is one of
the most stable currencies of today.

ISSUE: Whether or not the Minnesota Mortgage Moratorium Law


violates the Contract Clause of the Federal Constitution.

FACTS: The respondent bank, which was created by a special law (RA
3518), was placed under receivership by the Monetary board of the
Central Bank. Subsequently, the monetary board ordered the
liquidation of the Bank after finding that the Bank had incurred an
outstanding liability of P540, 835, 860.79. The petitioners claim that as
the bank was created by a special law, a contractual relationship now
exists between the Government and the stockholders of the bank that
cannot be disturbed without violation of the impairment clause.

THE COURTS RULING: The Minnesota statute as here applied, does not
violate the contract clause of the Federal Constitution.
REASON:
The conditions upon which the period of redemption is extended do
not appear to be unreasonable. The integrity of the mortgage
indebtedness is not impaired; interest continues to run; the validity of
the sale and the right of a mortgagee-purchaser to title or to obtain a
deficiency judgment if the mortgagor fails to redeem within the
extended period are maintained, and the conditions of redemption
stand as they were under the prior law.
The mortgagor, during the extended period, is not ousted from
possession, but he must pay the rental value of the premises as
ascertained in judicial proceedings, and this amount is applied to the
carrying of the property and to interest upon indebtedness.
An emergency existed in Minnesota which furnished a proper
occasion for the exercise of the reserved power of the state to protect
the vital interests of the community.
The legislation was addressed to a legitimate end, the legislation was
not for the mere advantage of particular individuals, but for the
protection of a basic interest of society.
*Not only are existing laws read into contracts in order to fix the
obligation as between the parties, but the reservation of ESSENTIAL
ATTRIBUTES OF SOVEREIGN POWER is also read into contracts as a
postulate of the legal order.

*Moratorium- postponement of fulfilment of obligations decreed by


the state through the medium of courts or the legislature.
PHILIPPINE VETERANS BANK EMPLOYEES UNION-NUBE V.PHILIPPINE
VETERANS BANK
189 SCRA 14 (1990)

ISSUE: Whether or not the liquidation of Veterans Bank would


constitute a violation of the impairment clause.
HELD:
Even if it be conceded that the charter of the Bank constitutes a
contract between the government and the stockholders of the bank, it
would not follow that the relationship cannot be altered without
violating the impairment clause.
The contract may be altered validly if it involves the public interest,
to which private interests must yield as a postulate of the existing
social order.
The need in the case at bar is no less compelling, to wit, the
preservation of the integrity and stability of our banking system. It is
the duty of the Central bank in such an event to step in and salvage the
remaining resources of the bank so that they may not continue to be
dissipated or plundered by those entrusted with their management.

Additional Cases
(L) Contract Clause [1]

RUTTER V. ESTEBAN
93 Phil. 68 (18 May 1953)
FACTS: Rutter sold to Esteban two parcels of land situated in the City of
Manila for the sum of P9,600, of which P4,800 was paid outright and
the balance was made payable as follows: P2,400 on or before August
7, 1942 and P2,400 on or before August 27, 1943, with interest at rate
of 7 percent per annum. Esteban failed to pay the two instalments as
agreed upon, as well as the interest that had accrued thereon, and so
Rutter instituted on 1949 an action to recover the balance, the interest
due thereon and the attorneys fees stipulated in the contract. Esteban
set up as a defense the moratorium clause embodied in RA 342. He
claims that this is a pre-war obligation and as a war sufferer, payment
of his obligation cannot be enforced until after the lapse of eight years
from the settlement of his claim by the Philippine War Damage
Commission, and this period has not yet expired.
THE COURTS RULE: Judgment is hereby rendered ordering the
defendant to pay the plaintiff the sum of P4,800 with interest thereon
at the rate of 7 percent per annum from August 27, 1942, until its full
payment, plus 12 percent as attorneys fees. Failure to pay this
judgement as stated, the properties mortgaged will be sold at public
auction and the proceeds applied to its payment in accordance with
law. The continued operation and enforcement of RA 342 is
unreasonable and oppressive, hence, NULL and VOID.
REASON:
The period in the case at bar seems to us unreasonable, if not
oppressive. The hope to effect collection becomes extremely remote,

Chapter 13
Poverty and Legal Protection
Free access to the courts and quasi-judicial bodies and
adequate legal assistance shall not be denied to any person by
19
reason of his poverty
The right to litigate us an escape valve to relieve the pressures
of personal disagreements that might otherwise explode into
physical confrontation
While authorities are not required to relieve the accused of his
poverty, they have the obligation not to take advantage of
indigence in the administration of justice
Collective activity undertaken to obtain meaningful access to
the courts is the fundamental right within the protection of the
First Amendment. However, the right would be a hollow
promise if courts could deny associations of workers or others
the means of enabling their members to meet the costs of legal
representation

19

CONSTITUTION, Art. III, 11

91 | P

LATON

It is not enough to say that all pauper litigants should be


assured of legal representation. They deserve quality
representation as well
Obviously, the constitutional guaranty of access to the courts
refers only to courts with appropriate jurisdiction as defined by
law. It does not mean that a person can go to any court for
redress of his grievances regardless of the nature or value of his
claim. If the petitioner is barred from filing his complaint before
our courts, it is because they are not vested with the
appropriate jurisdiction under the Warsaw Convention, which is
part of the law of our land
PEOPLE V. HASSAN
157 SCRA 261 (1988)
FACTS: Usman Hassan was accused of murder for stabbing to death
Ramon Pichel, in Zamboanga City on July 23, 1981. The lone eyewitness
for the prosecution, Jose Samson, in a confrontation at La Merced
Funeral Homes, positively identified Hassan as the very person who
stabbed the victim. The Supreme Court, in ACQUITTING the accused
based on insufficiency of evidence, also delved on the fact of his
poverty.
THE COURTS RULE: Wherefore, the decision is hereby reversed, and
the accused Usman Hassan is ACQUITTED of the crime charged. His
release from confinement is here by ordered, unless he i held for
another legal cause.
REASON:
The element of doubt, if reasonable in this case, must operate
against the inference of guilt the prosecution would draw from its
evidence.
Both Samson and the accused testified clearly and unequivocably
that Usman was alone when presented to Samson by Carpio (police
officer). The confrontation arranged by the police investigator between
the self-proclaimed eyewitness and accused did violence to the right of
the latter to counsel in all stages of the investigation into the
commission of a crime. As it turned out, the method of identification
became just a confrontation.
The rest of the investigation of the crime and the preparation of the
evidence for prosecution were done haphazardly, perfunctorily and
superficially.
The records of the case do not show any attempt on the part of
Corporal Carpio, or any other police officer, to investigate or question
Benhar Isa, (a notorious and a deadly police character in Zamboanga
City, with a long record of arrests), in connection with the killing of
Pichel jr.
The trial judge did not propound any single question to the accused,
and only three to his mother. Taking into account their poverty and
illiteracy, the mother and son needed as much, if not more, help, than
the trial judge extended to the prosecution witnesses during their
examination by asking them clarificatory and mostly leading questions.
In that sense and to that extent, the ACCUSED was DISADVANTAGED.
PEOPLE V. RIO
201 SCRA 702 (1991)
FACTS: Ricardo Rio was convicted of rape and sentenced to reclusion
perpetua by the trial court. He filed an appeal and as such, the records
of the case were forwarded to the Court of Appeals who then promptly
sent it to the Supreme Court in view of the penalty imposed upon him.
In the course of events, Rio filed a motion to withdraw his appeal due
to his poverty. The Court denied Rio's motion and appointed a counsel
de oficio for him.
ISSUE: This is a rape case. The main issue here is obviously irrelevant to
our discussion. Instead, we focus on the constitutional mandate set
forth in Article III, Sec. 11.

can appoint a counsel de oficio to prosecute the latters appeal


pursuant to Rule 122 of the Rules of Court (the rules governing appeal)
and Art. III, Sec. 11.
Lets have a little backgrounder (based on Mr. Justice Malcolms
writings):
Two of the basic privileges of the accused in a criminal prosecution are
(1) the right to the assistance of counsel and (2) the right to a
preliminary examination. Pres. McKinley made the first right a part of
the US Organic Law by imposing the inviolable rule that [the accused]
shall enjoy the right to have assistance of counsel for the defense. Said
right, described by Judge Cooley as perhaps the privilege most
important to the person accused of crime is now enshrined in our
Constitution as Sec. 11 of Art. III.
In criminal cases there can be no fair hearing unless the accused be
given an opportunity to be heard by counsel. The right to be heard
would be of little meaning if it does not include the right to be heard by
counsel.
The right to counsel de oficio does not cease upon the conviction of an
accused by a trial court. It continues, even during appeal, such that the
duty of the court to assign a counsel de oficio persists where an accused
interposes intent to appeal. Even in a case where the accused has
signified his intent to withdraw his appeal, the court is required to
inquire into the reason for the withdrawal. Where it finds the sole
reason for the withdrawal to be poverty, as in this case, the court must
assign a counsel de oficio, for despite such withdrawal, the duty to
protect the rights of the accused subsists and perhaps, with greater
reason.
In this spirit, the Court ordered the appointment of a counsel de oficio
for Rio. (It did not turn out well.)

Chapter 14
Rights of Suspects
Any person under investigation for the commission of an
offense shall have the right to be informed of his right to
remain silent and to have competent and independent counsel
preferably of his own choice. If the person cannot afford the
services of counsel, he must be provided with one. These rights
cannot be waived except in writing and in the presence of
counsel.
No torture, force, violence, threat, intimidation, or any other
means which vitiate the free will shall be used against him.
Secret detention places, solitary, incommunicado, or other
similar forms of detention are prohibited.
Any confession or admission obtained in violation of this or
Section 17 hereof shall be inadmissible in evidence against him.
The law shall provide for penal and civil sanctions for
violations of this section as well as compensation to the
rehabilitation of victims of torture or similar practices, and their
20
families.
As now guaranteed by the Bill of Rights, a suspect, or one under
custodial investigation, is accorded the following basic rights,
viz: (a) right to remain silent; (b) right to have competent and
independent counsel, preferably of his own choice; and, (c)
right to be informed of such rights. He is also entitled to have
counsel appointed for him if he could not afford one. And, to
make his rights more meaningful and not simply forfeited
through ignorance or inattention, the Constitution throws in

RULING: The Court opined that Rio seemed unaware that the former
20

CONSTITUTION, Art. III, 12 (1), (2), (3), and (4)

92 | P

LATON

the additional safeguard that waiver of such rights may only be


had if it is in writing and in the presence of counsel
The Miranda Rights -- Background and Development
The Fifth Amendment of the U.S. Constitution guarantees the
privilege against self-incrimination. The Sixth Amendment, on
the other hand, assures the assistance of counsel in criminal
prosecutions. These were understood basically as referring to
proceedings already instituted in court, not to custodial
investigations
ESCOBEDO V. ILLINOIS
378 U.S. 478, 12 L Ed 2d 977, S4 S Ct 1758 (1974)
FACTS: Danny Escobedo's brother-in-law, Manuel, a convict from
Chicago, was shot and killed on the night of January 19, 1960. Danny
Escobedo was arrested without warrant early the next morning and
interrogated. However, Escobedo made no statement to the police and
was subsequently released that afternoon. Subsequently, Benedict
DiGerlando, who was in custody and considered another suspect told
the police that indeed Escobedo fired the fatal shots because the victim
had mistreated Escobedo's sister. On January 30, the police arrested
Escobedo again this time with his sister, Grace. While transporting
them to the police station, the police explained that DiGerlando had
implicated Escobedo, and urged him and Grace to confess. Escobedo
again declined. Escobedo asked to speak to his attorney, but the police
refused, explaining that although he was not formally charged as of yet,
he was in custody and could not leave. His attorney went to the police
station and repeatedly asked to see his client, but was repeatedly
refused access, saying that Escobedo did not want to see him. Police
and prosecutors proceeded to interrogate Escobedo for fourteen-anda-half hours and refused his request to speak with his attorney with the
same consistency throughout that time. While being interrogated,
Escobedo made statements implicating his knowledge of the crime.
After conviction for murder, Escobedo appealed on the basis of being
denied the right to counsel.
Escobedo appealed to the Illinois Supreme Court, which initially held
the confession inadmissible and reversed the conviction. Illinois
petitioned for rehearing and the court then affirmed the conviction.
Escobedo appealed to the U.S. Supreme Court.
ISSUE: WON, under the circumstances, the refusal by the police to
honor petitioners request to consult with his lawyer during the course
of an interrogation constitutes a denial of the Assistance of Counsel
in violation of the Sixth Amendment.
RULING: YES. When Escobedo requested, and was denied, an
opportunity to consult with his lawyer, the investigation had ceased to
be a general investigation of an unsolved crime. Escobedo had
become the accused, and the purpose of the interrogation was to get
him to confess his guilt despite his constitutional right to do so. In such
a scenario, with Escobedo requesting and being denied an opportunity
to consult with his lawyer, and with the police not having effectively
warned him of his absolute constitutional right to remain silent, the
Supreme Court ruled that Escobedo has been denied the Assistance of
Counsel in violation of the Sixth Amendment as made obligatory
upon the States by the Fourteenth Amendment (as stated in Gideon v.
Wainwright). As such, no statement elicited by the police during the
interrogation may be used against him in a criminal trial.
NOTE: The above ruling was later implicitly overruled by the landmark
case, Miranda v. Arizona.
MIRANDA V. ARIZONA
384 U.S. 436, 16 L Ed e2 692, 86 S Ct 1602 (1966)
FACTS: Police arrested Miranda and took him to a special interrogation
room where they secured a confession. The inculpatory admission was
admitted at trial. In the end, Miranda was convicted for kidnapping and

rape, a decision affirmed on appeal.


ISSUE: WON the privilege set in the Fifth Amendment is fully applicable
during a period of custodial investigation.
RULING: Due to the coercive nature of the custodial interrogation by
police (Mr. Chief Justice Warren cited several police training manuals
which had not been provided in the arguments), no confession could be
admissible under the Fifth Amendment self-incrimination clause and
Sixth Amendment right to an attorney unless a suspect had been made
aware of his/her rights and the suspect had then waived them.
The person in custody must, prior to interrogation, be clearly
informed that he has the right to remain silent, and that anything he
says will be used against him in court; he must be clearly informed
that he has the right to consult with a lawyer and to have the lawyer
with him during interrogation, and that, if he is indigent, a lawyer will
be appointed to represent him.
If the individual indicates in any manner, at any time prior to or during
questioning, that he wishes to remain silent, the interrogation must
cease ... If the individual states that he wants an attorney, the
interrogation must cease until an attorney is present. At that time, the
individual must have an opportunity to confer with the attorney and to
have him present during any subsequent questioning.
DICKERSON V. UNITED STATES
530 U.S. 428, 152 L Ed 2d 1069, 122 S Ct 2315 (2000)
FACTS: In the wake of the ruling on Miranda v. Arizona, Congress
enacted a law which in essence makes the admissibility of a suspects
statements on a custodial interrogation turn solely on whether they
were made voluntarily. Dickerson was indicted for bank robbery,
conspiracy to commit bank robbery, and using a firearm in the course
of committing a crime of violence. He moved to suppress a statement
he had made to the FBI, on the ground that he had not received
Miranda warnings before being interrogated. The District Court
granted his petition, and the Government took an interlocutory appeal;
the Fourth Circuit reversed, acknowledging that even though Dickerson
had not received Miranda rights, the law was satisfied because his
statement was voluntary concluding that Miranda was not a
constitutional holding and that Congress could by statute have the final
say on the question of admissibility.
ISSUE: WON the Miranda court announced a constitutional rule or
merely exercised its supervisory authority to regulate evidence in the
absence of congressional direction.
HELD: A little backgrounder: in the past, a suspect's confession had
always been inadmissible if it had been the result of coercion, or
otherwise given involuntarily. This was true in England, and American
law inherited that rule from England.
However, as time went on, the US Supreme Court recognized that the
Fifth Amendment was an independent source of protection for
statements made by criminal defendants in the course of police
interrogation. Custodial police interrogation by its very nature "isolates
and pressures the individual" so that he might eventually be worn
down and confess to crimes he did not commit in order to end the
ordeal. In Miranda, the Court had adopted the now-famous four
warnings to protect against this particular evil.
Congress, in response, enacted 3501. That statute clearly was
designed to overrule Miranda because it expressly focused solely on
voluntariness of the confession as a touchstone for admissibility. As
regards Congress' authority to pass such a law: on the one hand, the
Court's power to craft non-constitutional supervisory rules over the
federal courts exists only in the absence of a specific statute passed by
Congress. But if on the other hand the Miranda rule was constitutional,
Congress could not overrule it, because the Court alone is the final
arbiter of what the Constitution requires. As evidence of the fact that
the Miranda rule was constitutional in nature, the Court pointed out
that many of its subsequent decisions applying and limiting the
93 | P

LATON

requirement arose in decisions from state courts, over which the Court
lacked the power to impose supervisory non-constitutional rules.
Furthermore, although the Court had previously invited legislative
involvement in the effort to devise prophylactic measures for
protecting criminal defendants against overbearing tactics of the police,
it had consistently held that these measures must not take away from
the protections Miranda had afforded.
In conclusion, the Court held that Miranda announced a constitutional
rule that Congress may not supersede legislatively.

Miranda Rights, Adoption and Adaptation in Philippine Milieu


A perusal of the Philippine provision on rights of suspects
would readily show that it has accorded more rights than found
in Miranda itself
The present Charter has broadened the guarantee in these
respects: (1) the right to counsel means not just any counsel,
but a competent and independent counsel, preferably his own
choice; (2) the right to remain silent and to counsel can only
be waived in writing and in the presence of counsel; and (3) the
rule on inadmissibility expressly includes admission, not just
confessions
People v. Mahinay
Mahinay, a houseboy was accused of rape with homicide. In the police
station he executed a extrajudicial confession with a counsel on how
the crime happened. After trial he was convicted and sentenced to
death. Upon automatic review of the Supreme court, he claims, among
others, that his extrajudicial confession was secured under duress and
that he was assisted by the counsel only when he was forced to sign the
same. He contends that militates against appellant is his constitutional
right to counsel. But his contention was belied by the records as well as
the testimony of the lawyer who assisted, warned and explained to him
his constitutionally guaranteed pre-interrogatory and custodial rights.
The court held that the heavy penalty of death is to be ensured that
such evidence is obtained by lawful means. The court, as guardian of
the rights of the people lays down the procedure, guidelines and duties
which arresting, detaining, inviting, or investigating officer must do and
observe at the time of making arrest and again at and during the time
of the custodial interrogation in accordance with constitution,
jurisprudence and republic act no. 7438. Miranda rights must be
updated by the court because of its insufficiency in the light of legal
developments.

Investigation, Custody and Interrogations


What was said on Escobado might be an appropriate starting
yardstick, i.e., where the investigation has developed from a
general inquiry into an unsolved crime to a focused inquiry
directed at a particular person as a suspect, then the trigger
for the operation of the guaranteed rights must already be
deemed to have been activated. As the Court said in People v.
Bravo, [t]he mantle of protection under this constitutional
provisions covers the period from the time a person is taken
into custody for investigation of his possible participation in the
commission of a crime or from the time he is singled out as a
suspect in the commission of a crime although not yet in
custody...Courts are not allowed to distinguish between
21
preliminary questioning and custodial investigation proper
when applying the exclusionary rule
Custodial investigation has been defined as any questioning initiated by law
enforcement officers after a person has been taken into custody or otherwise
deprived of his freedom of action in any significant way. (Sebastian, Sr. v.
Garchitorena, 343 SCRA 463 [2000])
21

In Illinois v. Perkins, the U.S. Supreme Court said that the


Miranda warnings are not required when the suspect is
unaware that he is speaking to a law enforcement officer, and
gives a voluntary statement which incriminates himself. In said
case an undercover government agent was placed in the cell of
Perkins, who was incarcerated on charges unrelated to the
subject of the agents investigation. He made statements that
implicated him in the crime that the agent sought to solve
Miranda forbids coercion, not mere strategic deception by
taking advantage of a suspects misplaced trust in one he
supposes to be a fellow prisoner...Ploys to mislead a suspect or
lull him into a false sense of security that do not rise to the level
of compulsion or coercion to speak are not within Mirandas
concerns
The tactic employed here to elicit a voluntary confession from a
suspect does not violate the Self-Incrimination Clause
People vs. Marra
SPO3 De Vera with three other policemen went in a crime scene. Upon
arriving the said victim Tandoc was already brought to the hospital and
they went to investigate. As they were investigating, the person that
the witnesses was telling was pointing to a security guard of a lindas
ihaw-ihaw who is Samuel Marra. The policeman saw him eating in a
nearby eatery and asked him series of questions. And even went to
their house to get the issued gun for investigation and upon seeing that
there was a missing bullet he asked Marra point-blank why he shot
Tandoc. At first he denied the accusation then when a witness pointed
at him he said it was a self-defense. De Vera picked up Din to and
brought him to the police station and identified Marra as assailant.
The was held that under Article III, Sec.12(1) provides that any person
under investigation for the commission of an offense shall have the
right to be informed of his right to remain silent and to have a
competent and independent counsel preferably his choice the
inquiry here is whether or not Marra was under custodial investigation
when he admitted the killing but invoked self-defense. The court
believed that it was not so situated.
He was not under custodial investigation when he made the admission.
There was no coercion whatsoever to compel him to make such
statement. He could have refused to answer the questions from the
very start
People vs. Bravo
Appellant was the last person seen with the victim, a nine year old
child, before her disappearance and the subsequent decomposing
body. She was found to be raped. Alexander Mico, Chief of the
intelligence section of the Santiago Police Department was the person
in-charge of the investigation came up with the investigation that
Bravo(appellant) was the suspect. Mico found the suspect at his work
place. And brought him to the police station were the appellant
admitted he was with the girl and was drunk and does not remember
anything that happened. Mico admitted that he did not inform the
appellant of his constitutional right to remain silentMico said it was
an informal interviewing when he made the custodial interrogation
proper.
The court held to acquit Bravo.
The exclusionary rule will apply, sprang from the recognition that police
interrogatory procedures lay fertile grounds for coercion, physical, and
psychological, of the suspect to admit responsibility for the crime under
investigation. It was not intended as a deterrent to the accused from
confessing guilt, if he voluntarily and intelligently so desires but to
protect the accused from admitting what he is coerced to admit
although untrue. Any statement allegedly made by him pertaining to
his possible complicity in the crime without prior notification of his
94 | P

LATON

constitutional right is inadmissible in evidence.

Counsel Competent, Effective and Vigilant


People vs. Sequino
Eugenio Godinez, overseer of the Hacienda Jose Ancajas in Medellin,
Cebu, went to the Medellin Rural Bank to withdraw 50,557.17 to pay
the wages for the workers in the hacienda. The banks cashier
instructed Jimmy the janitor and motorcycle driver of the bank to drive
Godinez and Broniola behind him. As they were nearing the hacienda
the accused, armed with gun block their way and as they past the
accused there was a gunshot and from the motorcycle Broniola has
fallen. The accused was noticed by Godinez because they were
employees before of the hacienda.

We have constitutionalized the right to counsel because of our


hostility against the use of duress and other undue influence in
extracting confessions from a suspect. Force and fraud tarnish
confessions and render them inadmissible
The Constitution has not simply limited itself to guaranteeing
right to counsel. It has taken pains to qualify the kind of lawyer
that should be assigned to a suspect, namely, a competent and
independent counsel preferably of his own choice

SPO Luna,who was in the headquarters, received of the report and


investigated. Upon investigating he saw a piece of paper in the bushes
were the incident happened ad it pointed out in a name Melvida,
Elpidio. In the house Melvidas they saw the father and ask where was
Elpidio and he told that he wsa in his brothers house. Then they go the
that house and found Elpidio and brought him to the police station
because of the absence of the barangay captain. Melvida was not
assisted by a counsel and investigation was not reduced to writing. In
the course of Lunas investigation, melvida admitted the he kept his
part of the share of loot. After which he also identified his companions
place.

It is noteworthy that the modifiers competent and independent


were terms absent in all organic laws previous to the 1987
Constitution. Their addition in the fundamental law of 1997 was
meant to stress the primacy accorded to the voluntariness of
the choice, under the uniquely stressful conditions of a
custodial investigation, by according the accused, deprived of
normal conditions guaranteeing individual autonomy, and
informed judgment based on the choices given to him by a
competent and independent lawyer

The court held that the evidence was inadmissible to the court as
evidence as a violation of the constitutional rights of the accused.

the assistance of counsel must be independent and competent,


that is, providing full protection to the constitutional rights of
the accused

Meaningful Communication of Rights


It is one thing to simply read. It is an entirely different thing to
comprehend what is read. In the same manner, it is not enough
that the Miranda rights are read to the suspect or even
explained to him. It must be done in a manner that enables the
recipient of the advice to understand what is being sought to
be conveyed. It contemplates the transmission of meaningful
information rather that just ceremonial and perfunctory
recitation of an abstract constitutional principle
The rights are meant to be availed of, not simply tendered as a
formality
When the Constitution requires a person under investigation
to be informed of his right to remain silent and to counsel, it
must be presumed to contemplate the transmission of
meaningful information rather the just the ceremonial and
perfunctory recitation of an abstract constitutional principle
In other words, the right of a person under interrogation to be
informed implies a correlative obligation on the part of the
police investigator to explain, and contemplates an effective
communication that results in understanding what is conveyed.
Short of this, there is a denial of his rights, as it cannot truly be
said that the person has been informed of his rights
In people v. Compil, the Court found it highly improbable for a
CLAO lawyer to have fully explained in less than ten (10)
minutes to the suspect who did not even finish Grade One the
latters constitutional rights and the consequences of
subscribing to an extrajudicial confession. In People v. Suela, an
interview of the suspect for five minutes by his lawyer before
the former gave his extrajudicial statement was considered
insufficient. In another case, the Court also expressed
misgivings on whether a farmer who only reached the fourth
grade, and who only understand Ilocano, could have
understood his constitutional rights conveyed to him by
counsel in English and Tagalog

As a corollary to having a lawyer who is competent, he must


also be one who is effective and vigilant. He must be one
devoted to his clients cause in a manner that really protects
him and not one who throws in only a lackadaisical effort. His
assistance must be continuous, from beginning to end
To be an effective counsel, a lawyer need not challenge all the
questions being propounded to his client. The presence of a
lawyer is not intended to stop an accused from saying anything
which might incriminate him but, rather, it was adopted in our
Constitution to preclude the slightest coercion as would lead
the accused to admit something false
Parenthetically, the counsel who is to help a suspect in the
execution of an extrajudicial confession must be one whose
assistance is not limited to the written waiver only. As to one
whose assistance would be needed in order that the right to
counsel may be waived, he must be a real lawyer, i.e., a
member of the bar
What all of these boil down to is this: an extrajudicial
confession, even if gospel truth, but executed by a suspect who
was assisted by a lawyer who failed to meet the exacting
standards of an independent and competent counsel is deemed
an uncounselled confession and, therefore, inadmissible in
evidence
Post-Facto Validation of Confession
In People v. Rous, the Third Division of this Court held that an
extrajudicial confession may be admitted in evidence even if
obtained without the assistance of counsel provided that it was
read and explained to confessant by counsel before it was
signed
***At the time a person is arrested, it shall be the duty of the
arresting officer to inform him of the reason for the arrest and
he must be shown the warrant of arrest, if any. He shall be
95 | P

LATON

informed of his constitutional rights to remain silent and to


counsel, and that any statement he might make could be used
against him. The person arrested shall have the right to
communicate with his lawyer, a relative or anyone he chooses
by the most expedient means--by telephone if possible--or by
letter or messenger. It shall be the responsibility of the
arresting officer to see to it that this is accomplished. No
custodial investigation shall be conducted unless it be in the
presence of counsel engaged by the person arrested, by any
person on his behalf, or appointed by the court upon petition
either of the detainee himself or by anyone on his
behalf.***Any statement obtained in violation of the procedure
herein laid down, whether exculpatory or inculpatory, in whole
or in part, shall be inadmissible in evidence
Fruit of the Poisonous Tree--According to this rule, once the
primary source (the tree) is shown to have been unlawfully
obtained, any secondary or derivative evidence (the fruit)
derived from it is also inadmissible
Independent Counsel of Choice
The Constitution likewise requires that in order for counsel to
be really of service to the needs of the suspect for which
counsel is guaranteed, he must be one whose interest do not
run counter to the intended representation
The following have been considered not independent counsel:
municipal mayor, City Legal Officer, or a Municipal Attorney, a
barangay captain, a police station commander, a lawyer
a[plying for a job at the National Bureau of Investigation, or, an
associate of the private prosecutor in the case in which the
suspect is implicated
And, as further assurance that the counsel is really
independent, the person under investigation is given the
opportunity to make the choice--i.e., the lawyer to be
preferably of his own choice
While the initial choice of the lawyer in cases where a person
under custodial investigation cannot afford the services of a
lawyer is naturally lodged in the police investigators, the
accused really has the final choice as he may reject the counsel
chosen for him and ask for another one
Confessions and Admissions
In contrast to the 1973 Constitution, the 1987 Constitution
explicitly includes in its guarantee extrajudicial confessions and
admissions. In People v. Agustin, the Court called attention to
the difference between the two:
In a confession, there is an acknowledgment of guilt. Admission
is usually applied in criminal cases to statements of fact by the
accused which do not directly involve an acknowledgment of
guilt of the accused or of the criminal intent to commit the
offense with which he is charged
Kinds of Coerced Confessions
In People v. Obrero, the Courts spoke of two kinds of coerced
confessions, i.e., (1) those which are the product of third
degree methods such as torture, force, violence, threat, of
intimidation, and which are dealt with in paragraph 2 12, and

(2) those which are given without the benefit of Miranda


warnings, which are the subject of paragraph 1 of the same 12
Now, under the first paragraph of this provision, it is required
that the suspect in custodial investigation must be given the
following warnings: (1) He must be informed of his right to
remain silent; (2) he must be warned that anything he says can
and will be used against him; and (3) he must be told that he
has a right to counsel, and that if he is indigent, a lawyer will be
appointed to represent him
Voluntary and Spontaneous Admissions and Confessions to
Private Persons
The guarantee is basically designed to prevent an involuntary
and uninformed admission or confession hat might otherwise
be obtained by law enforcers either through the application of
force, threats, coercion and other underhanded means.
Accordingly, even if the Miranda warnings have not been given
to the suspect but he voluntarily made any admissions or
confessions without any prodding on the part of the police,
then the same would be admissible
People v. Cayago
312 SCRA 623 (1999)
Facts: Accused-appellant Rolando Cayago reported to the Pasig City
police that he found the decomposing body of his wife in an abandoned
barangay hall.
When Cayago was giving his statement at the police headquarters, the
police noticed certain inconcistencies, hence, they advised him to
undergo a polygraph test. When Cayago was about to be brought to
Camp Crame for polygraph test, he requested to go to a nearby church,
accompanied by SPO2 Delos Reyes. Thereat, Cayago admitted to Delos
Reyes that he killed his wife and that he was willing to give his
statement relative to said killing.
Atty. Campanilla conferred with Cayago at the Office of the
Investigation Division. After apprising him of his constitutional rights,
Cayago admitted that he killed his wife. Atty Campanilla then advised
Cayago to personally write down his confession.
He was indicted for parricide and at trial his extrajudicial confession
was admitted. He was found guilty. On appeal, Cayago contends that
his extrajudicial confession should be inadmissible because the same
was given without affording him the right to counsel guaranteed by the
Constitution.
Issue: Whether or not Cayagos confession constitute as an admissible
as evidence.
Held: In the case at bar, it is clear that appellant strangulated his wife
resulting to her death. This is supported by appellants own testimony,
his confession to the police and the medical findings corroborating that
she died of asphyxia by strangulation.
Appellants contention that the statement he gave to the police is
inadmissible in evidence because it was given without affording him the
right to counsel guaranteed by the Constitution has no merit. The right
to counsel is afforded by Section 12(1), Article III of the 1987
Constitution only to person(s) under investigation for the commission
of an offense. Custodial rights of a person are not available whenever
he volunteers statements without being asked. He was not investigated
by the authorities. In fact, after appellant admitted to the police officer
that he killed his wife, the officer told him that he will be provided with
a lawyer to assist him, namely, Atty. Campanilla. At the, trial, the latter
testified that he talked to appellant, advised him of his constitutional
rights and was present when the latter wrote his extrajudicial
statement admitting that he killed his wife.
96 | P

LATON

People v. Maqueda
242 SCRA 565 (1995)
Facts: Accused-appellant Maqueda was charge with robbery with
homicide and serious physical injuries in relation to the slaying of
Horace William Barker and the infliction of serious injuries on his wife,
Teresita Mendoza, on the occasion of a robbery on 27 August 1991.
Maqueda was then fetched from Quezon and brought to the Benguet
Provincial Jail. Its commanding officer directed on his men to get
Maquedas statement. He did so and according to him, he informed
Maqueda of his rights under the Constitution. Meanwhile, in April 1992,
while he was under detention, Maqueda filed a Motion to Grant Bail,
stating therein that he is willing and volunteering to be a State
witness in the above entitled case, it appearing that he is the least
guilty among the accused in this case.
In the meantime, Ray Dean Salvosa arrived at the Office of Prosecutor
Zarate and obtained permission from the latter to talk to Maqueda.
Salvosa then led Maqueda toward the balcony. Maqueda narrated to
Salvosa that Salvamante then brought him to the Barker house and it
was only when they were at the vicinity thereof that Salvamante
revealed to him that his real purpose in going to Baguio City was to rob
the Barkers.
He also told other details of the incident. After trial, appellant was
convicted. Although the trial court had doubts on the identification of
Maqueda by prosecution witnesses and thus disregarded their
testimonies on this matter, it decreed a conviction based on the
confession and the proof of corpus delicti as well as on circumstantial
evidence.
Issue: Whether or not Maquedas admission constitute as an admissible
as evidence.
Held: The exercise of the rights to remain silent and to counsel and to
be informed thereof under Section (12), Article III of the Constitution
are not confined to that period prior to the filing of a criminal complaint
or information but are available at that stage when a person is under
investigation for the commission of an offense.
It was therefore, wrong for the trial court to hold that Section 12(1),
Article III of the Constitution is strictly limited to custodial investigation
and that it does not apply to a person against whom a criminal
complaint or information has already been filed because after its filing
he loses his right to remain silent and to counsel.
The Sinumpaang Salaysay of Maqueda taken by SPO2 Mollena after the
formers arrest was taken in palpable violation of his rights under
Section 12(1), Article III of the Constitution. As disclosed by a reading
thereof, Maqueda was not even told of any of his constitutional rights
under the said section. The statement was also taken in the absence so
counsel. Such uncounselled Sinumpaang Salaysay is wholly inadmissible
pursuant to paragraph 3, Section 12, Article III of the Constitution.
However, the extrajudicial admissions of Maqueda to prosecutor Zarate
and to Ray Dean Salvosa stand on a different footing. These are not
governed by the exclusionary rules under the Bill of Rights. Maqueda
voluntarily and freely made them to Prosecutor Zarate not in the
course of an investigation, but in connection with Maquedas plea to be
utilized as a state witness; and as to the other admission, it was given
to a private person.
In the light of his admission to Prosecutor Zarate and Ray Dean Salvosa
and his willingness to be a state witness, Maquedas participation in the
commission of the crime charged was established beyond moral
certainty. His guilt was, as correctly ruled by the trial court, established
beyond doubt by circumstantial evidence.

Confessions and Admissions to the Media

Those in the media are always in search of what is news, a


substantial amount of which is about crimes. They try to report
on news while still hot, or otherwise look for scoops which
would interest their viewers, listeners, or readers. And they
would come across suspects, especially in sensational cases,
invariably they end up interviewing them. In these instances,
they may get answers which are downright incriminating.
Because media practitioners are private persons and they are
not in law enforcement, any admissions or confessions made to
them by unsuspecting suspects are deemed admissible even if
the Miranda warnings were not given before the incriminating
answers were elicited
PEOPLE V. ANDAN
269 SCRA 95 (1997)
The Facts: Accused-appellant Pablito-Andan was indicted for the rape
slaying of Marianne Guevarra, a 20-year-old nursing student, in Baliuag,
Bulacan. Her gruesome death drew public attention and prompted the
town mayor to form a crack team of police officers to look for the
criminal. The police investigated the scene of the crime and the
surrounding areas, including appellants nearby house where they
found bloodstains on the wall of the pigpen in the backyard. A police
team led by the town mayor himself traced appellant in his parents
house. They took him aboard a patrol jeep and brought him to the
police headquarters where he was interrogated. Initially, appellant
denied any knowledge of Marrianes death.
By this time, people and media representatives were already gathered
at the police headquarters awaiting the results of the investigation. The
mayor arrived and proceeded to the investigation room. Upon seeing
the mayor, appellant approached him and whispered a request that
they talk privately. The mayor led appellant to the office of the Chief of
Police and there, appellant broke down and said Mayor, patawarin mo
ako! I will tell you the truth. I am the one who killed Marianne. The
mayor opened the door of the room to let the public and media
representatives witness the confession. The mayor first asked for a
lawyer to assist appellant but since no lawyer was available he ordered
the proceedings photographed and videotaped. In the presence of the
mayor, the police, representatives of the media and appellants own
wife and son, appellant confessed his guilt. His confession was captured
on videotape and covered by the media nationwide. He was found
guilty and sentenced to death. On automatic review to the Supreme
Court, he raises as one of the issues the admission of his extrajudicial
confessions.
The Issue: Whether or not the admission of accused-appellant
constitute an admissible evidence.
Held: When appellant talked with the mayor as a confidant and not as a
law enforcement officer, his uncounselled confession to him did not
violate his constitutional rights. Appellants confessions were made in
response to questions by news reporters, not by police or any other
investigating officer. We have held that statements spontaneously
made by a suspect to news reporters on a televised interview are
deemed voluntary and are admissible in evidence.
We rule that appellants verbal confessions to the newsmen are not
covered by Section 12(1) and (3) of Article III of the Constitution. The
Bill of Rights does not concern itself with the relation between a private
individual and another individual. It governs the relationship between
the individual and the State. The prohibitions therein are primarily
addressed to the State and its agents.
IN VIEW WHEREOF, the decision of the Regional Trial Court, Branch 15,
Malolos, Bulacan in Criminal Case No. 1109-M-94 is affirmed and
accused-appellant Pablito Andan y Hernandez is found guilty of the
special complex crime of rape with homicide.

Even as admissions or confessions made to media men may be


considered admissible, the Court has cautioned against
97 | P

LATON

indiscriminate acceptance of the same, however, in view of the


possibility of abuse
Because of the inherent danger in the use of television as a
medium for admitting ones guilt, and the recurrence of this
phenomenon in several cases, it is prudent that trial courts are
reminded that extreme caution must be taken in further
admitting similar confessions
Reenactments, Police Line-ups and Showups
The counsel requirement also applies to situations where the
suspect may be in a situation where his privilege against selfincrimination would otherwise be compromised. This would be
illustrated in the case of a reenactment. By showing how the
crime was allegedly committed, the suspect practically
confesses his guilt. Thus, evidence or admissions obtained
through uncounselled reenactments, such as pictures taken on
said occasion, would be inadmissible
in so far as the police line-ups are concerned, however, the
general and prevailing rule is that no counsel is required since
there is no custodial investigation or interrogation conducted
yet. Instead, if there is anybody who is subjected to
questioning, it is the witness who is called to identify who
among those in the line-up might be the culprit
In resolving the admissibility of and relying on out-of-court
identification of suspects, courts have adopted the totality of
circumstances test where they consider the following factors,
viz: (1) the witness opportunity to view the criminal at the time
of the crime; (2) the witness degree of attention at that time;
(3) the accuracy of the prior description given by the witness;
(4) the level of certainty demonstrated by the witness at the
identification; (5) the length of time between the crime and the
identification; and (6) the suggestiveness of the identification
procedure
In regard to the suggestive identification, Justice Brennan spoke
of the freezing effect of a pre-trial identification:
Due to the freezing effect recognized in United States v.
Wade, once suggestion has tainted the identification, its mark is
virtually indelible. For once a witness has made a mistaken
identification, he is not likely to go back on his word later on.
As a result, any effort of the accused to duplicate the initial
photographic display will almost necessarily lead to a
reaffirmation of the initial misidentification
GAMBOA V. CRUZ
162 SCRA 642 (1988)
The Facts: The petitioner was arrested for vagrancy without a warrant
of arrest. The following day, during the lineup of five (5) detainees,
including petitioner, complainant Erlinda B. Bernal pointed to petitioner
and said, that one is a companion. On 23 July 1979, an information
for robbery was filed against the petitioner. After the prosecution had
rested its case, petitioner, instead of presenting his defense, filed a
Motion to Acquit or Demurrer to Evidence, predicated on the ground
that the conduct of the line-up, without notice to, and in the absence
of, his counsel violated his constitutional rights to counsel and to due
process.
The Issue: Whether or not there has been a violation of petitioners
constitutional rights to counsel and to due process.

assisted by counsel.
Under the 1973 and 1987 Philippine Constitutions, the right to counsel
attaches at the start of investigation against a respondent and,
therefore, even before adversary judicial proceedings against the
accused have begun.
Given the clear constitutional intent in the 1973 and 1987 Constitutions
to extend those under police investigation the right to counsel, this
occasion may be better than any to remind police investigators that,
while the Court finds no real need to afford a suspect the services of
counsel during a police line-up, the moment there is a move or even an
urge of said investigators to elicit admissions or confessions or even
plain information which may appear innocent or innocuous at the time,
from said suspect, he should then and there be assisted by counsel,
unless he waives the right, but the waiver shall be made in writing and
in the presence of counsel.

Receipts and Samples


The need for counsel would also be implicated where a person
may be asked to sign receipts whereby he acknowledges
certain items, which are likely objects or fruits of the crime,
taken from him by the police. Again, this is similar to confessing
ones guilt. thus, if the suspect signed them in the absence of
counsel, it would be inadmissible

Exclusionary Rule
Just like the sanction for violation of the proscription against
unreasonable search and seizures, violation of the rule on
Miranda warnings also carries the penalty of exclusion.
However, there is one difference. In the former, the
Constitution says that evidence obtained in violation of the said
provision shall be inadmissible for any purpose in any
proceeding. In the latter, a confession or admission obtained
in violation of the Miranda safeguards shall be inadmissible in
evidence against him.
Under the Constitution and existing law and jurisprudence, a
confession to be admissible must satisfy the following
requirements: 1) confession must be voluntary; 2) the
confession must be made with the assistance of competent and
independent counsel; 3) the confession must be express; and 4)
the confession must be in writing. Thus, if the confession or
admission does not comply with said requisites, it will have to
be excluded
Failure to administer Miranda warnings creates a presumption
of compulsion

Waiver
Related to the voluntary admission and the exclusionary rule is
the rule on waiver. If the suspect knowingly, voluntarily and
intelligently waives his right to silence and counsel, then that is
his own lookout

Held: Any person under investigation must among other things, be


98 | P

LATON

In People v. Obrero, the Court declared: It does not matter


that accused-appellant failed to object to the introduction of
these constitutionally proscribed evidence. The lack of
objection did not satisfy the heavy burden of proof which
rested on the prosecution

Protecting the privacy expectations and liberty interests of


individuals need not necessarily be antagonistic to the goal of a
well-ordered society under the Rule of Law
Additional Cases
(M) Rights of Suspects [3]

Lately, however, the Court has come to the doctrine that if no


timely objection is made to the introduction of such tainted
confessions or admissions, the same would be admissible on
the ground that the accused had thereby waived his right not to
have the same considered as part of the evidence against him
People V. Samus
(389 SCRA 93)
FACTS: Appellant was fund guilty of murder for the killing of Dedicacion
Balisi, 62 yrs old and her 6 year old grandson, John Arden Balisi in
Calamba Laguna. The police, 8 days after the commission of the crime,
arrested the appellant who was sighted in Sta. Rosa, Laguna. According
to the prosecution, appellant was brought to Camp Vicente Lim where
he was informed of his constitutional rights. The following morning,
appellant, assisted by Atty. Juliano, gave his statement admitting the
killings. The appellant denied the charges, as well as the version of the
police as to how he was arrested, He alleged that he was arrested
without warrant, was tortured and forced to admit the killing of the
victims, and that he did not know Atty. Juliano nor did he talk to him.
On appeal, he raises among the issues the admission of his extrajudicial
confession.
HELD: While it is true that the confessions of appellant were made
without benefit of counsel, they are still admissible in evidence because
of the appellants failure to make timely objections before the trial
court. If only the defense had proffered them on time, the prosecution
could have been warned of the need to present additional evidence to
support its case. To disregard a major portion of the prosecutions case
at a late stage during an appeal goes against the norms of fundamental
fairness.
The apprehending officers contends that the constitutional rights of
appellant were not violated, since they were not the ones who had
investigated and elicited evidentiary matters from him. The Court is not
persuaded. The events narrated by the law enforcers in court are too
good to be true. Their sworn statements given a day after the arrest
contradict their testimonies and raise doubts on their credibility.
Evidence to be believed, must not only proceed from the mouth of a
credible witness, but must be credible in itself-such as the common
experience of mankind can approve as probable under the
circumstances.

Duty of Subscribing Authorities


In some instances, the extrajudicial confessions may have to be
sworn to before authorities, such as public prosecutors or
investigating judges. To ensure that no force, torture or
violence was inflicted on the confessant, the Court required
said judges and prosecutors to take some steps to assure
themselves that there was no maltreatment leading to the
confessions
Administrative Investigation
The Miranda rights inhere only in custodial investigations.
Accordingly, they may not be invoked in administrative
investigations conducted by an employer
While others may view the rights accorded to a suspect as an
added difficulty imposed on police work in solving crimes, the
same should be seen in the republican state over string-arm
tactics which might be better suited for totalitarian countries.

PEOPLE V. LAUGA
615 SCRA 548
FACTS: Lagua, being the father of AAA with lewd design, with the use of
force and intimidation, did then and there, willfully, unlawfully and
criminally have carnal knowledge with aforementioned daughter, a 13
year[s]old minor against her will. After the deed was done, AAA
recounted what happened with her brother BBB, and then told her
grandfather and uncle, and after which they sought the assistance of
Moises Boy Banting, a bantay bayan.
Moises Boy Banting found appellant in his house wearing only his
underwear. He invited appellant to the police station, to which
appellant obliged. At the police outpost, he admitted to him that he
raped AAA because he was unable to control himself said confession
being something which he denied in his testimony. The RTC found him
guilty and convicted him, and upon appeal said conviction was modified
by the CA (by rendering Lagua ineligible for parole and increasing the
indemnity and moral damages to be paid), hence this petition.
ISSUE: WON his alleged confession with a bantay bayan is admissible
in evidence.
RULING: NO. This Court is convinced that barangay-based volunteer
organizations in the nature of watch groups, as in the case of the
"bantay bayan," are recognized by the local government unit to
perform functions relating to the preservation of peace and order at
the barangay level. Thus, without ruling on the legality of the actions
taken by Moises Boy Banting, and the specific scope of duties and
responsibilities delegated to a "bantay bayan," particularly on the
authority to conduct a custodial investigation, any inquiry he makes has
the color of a state-related function and objective insofar as the
entitlement of a suspect to his constitutional rights provided for under
Article III, Section 12 of the Constitution, otherwise known as the
Miranda Rights, is concerned, therefore finding the extrajudicial
confession of appellant, which was taken without a counsel,
inadmissible in evidence. (This does not change anything either way as
Laguas guilt was not deduced solely from said confession but from
confluence of evidence showing his guilt beyond reasonable doubt
but thats another issue.)

Chapter 15
Rights of the Accused
No person shall be held to answer for a criminal offense
without due process of law.
In all criminal prosecutions, the accused shall be presumed
innocent until the contrary is proved, and shall enjoy the right
to be heard by himself and counsel, to be informed of the
nature and cause of the accusation against him, to have a
speedy, impartial, and public trial, to meet the witnesses face
to face, and to have compulsory process to secure the
attendance of witnesses and the production of evidence in his
behalf. However, after arraignment, trial may proceed
notwithstanding the absence of the accused: Provided, that he

99 | P

LATON

has been duly notified and his failure to appear is


22
unjustifiable.
All persons, except those charged with offenses punishable
by reclusion perpetua when evidence of guilt is strong, shall,
before conviction, be bailable by sufficient sureties, or be
released on recognizance as may be provided by law. The right
to bail shall not be impaired even when the privilege of the writ
of habeas corpus is suspended. Excessive bail shall not be
23
required.
Once a criminal investigation develops to one where there is a
person charged in court, a different set of rights sets in
Criminal Due Process
The basic difference between the two provisions lies in the fact
that while Section 1 speaks of due process in general, both in its
substantive and procedural aspects, Section 14 refers to the
procedural component only, i.e., the manner in which the
finding of guilt or innocence would be had, which basically
means the presence of a court of competent jurisdiction, notice
and hearing, acquisition of jurisdiction over the person of the
accused, and judgment rendered after trial. Section 14
catalogues the essentials of due process in a criminal
prosecution
General Considerations
The fundamental principle of due process necessarily means
that a person must be heard before being condemned. The due
process requirement is part of a persons basic rights, not a
mere formality that may [be] dispensed with or performed
perfunctorily. In its broader aspect, Due process prohibits
criminal stability from shifting the burden of proof to the
accused, punishing wholly passive conduct, defining the crimes
in vague or overbroad language and failing to grant fair warning
of illegal conduct
In criminal proceedings then, due process is satisfied if the
accused is informed as to why he is proceeded against and
what charge he has to meet, with his conviction being made to
rest on evidence that is not tainted with falsity after full
opportunity for him to rebut it and the sentence being imposed
in accordance with a valid law. It is assumed, of course, that the
court that rendered the decision is one of competent
jurisdiction
While that right is statutory rather than constitutional in its
fundament, since it has in fact been established by statute, it is
a component part of due process in criminal justice. The right
to have a preliminary investigation conducted before being
bound over to trial for a criminal offense and hence formally at
risk of incarceration or some other penalty, is not a mere
formal or technical right; it is a substantive right
Part of the guarantee of criminal due process is the assurance
that persons are not simply made to respond to any criminal
proceeding on the basis of flimsy or insubstantial grounds. For
as long as no sufficient basis exists for summoning a person to
answer a criminal charge, he should not be vexed, annoyed or
harassed or otherwise unduly compelled to put up with a
baseless suit.
22
23

CONSTITUTION, Art. III, 14(1) and (2); Sixth Amendment


CONSTITUTION, Art. III, 13; Eight Amendment

Salonga V. Cruz Pano


134 SCRA 438 (1985)
FACTS: There was a rash of bombings occurred in Metro Manila from
August to October 1980. On September 6, Victor Lovely, Jr, almost
killed himself and injured his younger brother, Romeo as a result of an
explosion of a small bomb inside his room at YMCA building in Manila.
Found in his possession were several pictures taken sometime in May
1980 at the birthday party of former Congressman Raul Daza held at
the latters residence in LA. Petitioner Jovito Salonga and his wife were
among the guests, including lovely. Lovely was brought by military and
police authorities, as a result of his injuries, at the AFP Medical Center
where he was placed in the custody and detention of Col. Roman
Madella, under the overall direction of General Fabian Ver, Head of
National Intelligence and Security Authority. Shortly, Lovely and his 2
brothers, Romeo and Baltazar, were charged with subversion, illegal
possession of explosives and damage to property. The petitioner had
been linked to the various bombings in MM. Arrest, Search and Seizure
orders (ASSO) were issued against persons implicated by Lovely, among
them, was herein petitioner. On October 21, while confined in a
hospital, the arresting officer showed petitioner the ASSO form which
however did not specify the charges against him. His lawyers were also
not permitted to visit him in his hospital room and on November 2, he
was transferred against his objections from his hospital arrest to an
isolation room without windows in an army prison camp, in Fort
Bonifacio, Makati. The petitioner was not informed why he was
transferred and detained, nor was he ever investigated or questioned
by any military or civil authority. On or around March 26, the counsel
for petitioner was furnished a copy of an amended complaint signed by
Gen. Prospero Olivas dated march 12, 1981, charging the petitioner,
along with 39 others with violation of RA 1700, as amended. Hearings
for preliminary investigation were conducted. On October 15, the
counsel for petitioner filed a motion to dismiss the charges against
petitioner for failure of the prosecution to establish a prima facie case
against him. The respondent judge denied the motion and on January 4
1982, he issued a resolution ordering the filing of an information for
violation of the revised Anti-subversion Act, as amended, against 40
people, including herein petitioner. the resolutions of the respondent
judge are now the subject of the petition. It is the contention of the
petitioner that no prima facie has been established by the prosecution
to justify the filing of an information against him.
HELD: The petitioner invokes the constitutionally protected right to life
and liberty guaranteed by the due process clause, alleging that no
prima facie has been established to warrant the filing of an information
for subversion against him. Petitioner asks this court to prohibit and
prevent the respondents from using the iron arm of the law to harass,
oppress and persecute him, a member of the democratic opposition in
the Philippines. In this case, respondents agree with our earlier finding
that the prosecution evidence miserably fails to establish a prima facie
case against the petitioner, either as a co-conspirator of a
destabilization plan to overthrow the government or as an officer or
leader of any subversive organization. They have taken the initiative of
dropping the charges against the petitioner. We reiterate the rule,
however, that this Court will not validate the filing of an information
based on the kind of evidence against the petitioner found in the
records. WHEREFORE, the petitioner is DISMISSED for having become
moo and academic.

In Allado v. Diokno, the Court declared: The sovereign power


has the inherent right to protect itself and its people from
vicious acts which endanger the proper administration of
justice; hence, the State has every right to prosecute and
punish violators of the law. This is essential for its selfpreservation, nay, its very existence. But this does not confer a
license for pointless assaults on its citizens
Olivas v. Office of the Ombudsman
239 SCRA 283 (1994)
Petitioner was Commanding General of the PC Metrocom and was
100 | P

LATON

retired from the Armed Forced of the Philippines effective Feb 26,
2986.He assails the order of respondent requiring him to submit his
affidavit and those of his witnesses at the preliminary investigation of a
case for unexplained wealth despite the fact that the PCGG as
complainant had not reduced its evidence in the form of affidavits and
submitted supporting documents. In August 1986, petitioner, was
informed that there was a freeze order issued covering his bank
accounts and a hold order issued against him by PCGG.
Olivas V. Office of the Ombudsman
239 SCRA 283 (1994)
FACTS: Petitioner assails the order of respondent requiring him to
submit his affidavit and those of his witnesses at the preliminary
investigation of a case for unexplained wealth despite the facts the
PCGG, as complainant, had not reduced its evidence in the form of
affidavits and submitted supporting documents. Shortly after petitioner
retired last February 26, 1986, anonymous letters were sent to PCGG
charging him with violations of the Anti Graft and Corrupt Practices
Act and Unexplained Wealth Act. He is among those AFP personnel
being investigated. The issue is whether the petitioner may be
compelled to file his counter affidavit notwithstanding the fact the no
sworn complaint or affidavit has been field against him.
HELD: We find for the petitioner. The lack of complaint and affidavits
cannot be excused on the plea that this case originate in anonymous
letters sent to the PCGG. Because of leads furnished by those letters, it
would seem that the PCGG has found sufficient evidence justifying its
demand to the petitioner to explain. It is incumbent upon the
complainant to reduce the evidence into affidavits. This is a
requirement not only of Rule II, S4 (a) of the Rules of Procedure but
also of due process in an adversary proceedings.
Alonte v. Savellano Jr.
287 SCRA 245 (1998)
Alonte, then the Mayor of Binan, Laguna was charged with rape
together with Concepcion, who was alleged to have brought the
complainant, Punongbayan, a minor, to Alonte. The case was originally
raffled to a branch of RTC in Binan, but complainant sought a change in
venue. While awaiting action on her request, the complainant, assisted
by her parents and counsel, executed an affidavit of desistance,
adverting to the delay and inconvenience to her and to her family. The
petitioners then moved for the dismissal of the case which was
opposed by the public prosecutor. Shortly thereafter, the SC approved
the request of change in venue and was raffled to the respondent
judge. The complainant reiterated before the trial court her decision to
abide by her affidavit of desistance. In the meantime, the judge found
probable cause for the issuance of the warrants of arrests of the
petitioners. On arraignment, both pleaded not guilty. According to the
judge, both parties agreed to proceed while Alonte said that the judge
allowed the prosecution to proceed to present evidence relative only to
the question of voluntariness and validity of the affidavit of desistance.
The parents and the complainant affirmed their decision to desist. The
prosecutor then manifested that in light of the decision of the
complainant and her parents not to pursue the case, the State had no
further evidence against the accused to prove the guilty of the accused,
and moved for the dismissal of the case against both petitioner. Then,
the trial judge promulgated his decision, convicting both accused. They
filed the instant petition, assailing the decision, for among others, being
violative of the guarantee of due process.
HELD: The Court must admit that it is puzzled by the somewhat strange
way the case has proceeded below. Per Judge Savellano after the
waiver by the parties of the pre-trial stage: The trial of the case did
proceed on the merits but thatthe two accused did not present
countervailing evidence during the trail. They did not take the witness
stand to refute or deny under oath the truth of the contents of the
private complainants aforementioned affidavit which she expressed
affirmed and confirmed in court, but instead, thru their respective
lawyers, they rested and submitted the case for decision merely on the
basis of the private complainants so called desistance which, to
them, was sufficient enough for their purposes. They left everything to

the so called desistance of the private complainant.


Jurisprudence acknowledges that the due process in criminal
proceedings, in particular, require a) that the court or tribunal trying
the case is properly clothed with judicial power to hear and determine
the matter before it. b) that jurisdiction is lawfully acquired by it over
the person of the accused c) that the accused is given an opportunity to
be heard and d) that judgment is rendered only upon lawful hearing.
The above constitutional and jurisprudential postulates, by now
elementary and deeply imbedded in our own criminal justice system
are mandatory and indispensable. The principles find acceptance and
are tersely expressed in the oft-quoted statement that procedural due
process cannot possibly be met with out a law which hears before it
condemns, which proceeds upon inquiry and renders judgment only
after trial.
Hildawa v. Enrile
138 SCRA 146 (1985)
Facts: Petitioners Hildawa and Valmonte in these civil actions for
declaration of Nullity of Executive/ Administrative Order creating Secret
Marshals formed to conduct a concentrated campaign against criminal
elements preying on passengers of jeepneys, buses, taxis and all forms
of public conveyance. They allege that such secret marshals which has
the authority to kill thieves, hold uppers, robbers, pickpockets and
other same kind are violative of the several provisions of the
constitution such as those on due process in regards of the accuse.
Issue: Whether or not such formation of secret marshals is violative
against the due process which is vested by the constitution.
Held: According to Relova J.. there is nothing wrong in the creation and
deployment of special operation teams to counter the resurgence of
criminality. The problem is the allege use of violence in the
implementation of objectives of the special squads. Due process of law
is requires that the accused must be heard under the orderly process of
law. Wherefore the respondents are directed to exercise strict
supervision and control over these special operation teams, ensuring
that these teams will not use unnecessary force and will comply strictly
with the law. If should death or injury result, responded is hereby
immediately report matter to their superior officers and the National
Police Commission (NAPOLCOM) for investigation and proper action

Informed Participation in the Proceedings


For the purpose of satisfying the due process requirements, it is
also necessary that the accused have an understanding of what
the proceeding is all about. Accordingly, he would have to be
assisted and informed in such a language and in a manner that
he can understand and comprehend what is being conveyed.
This means that he has to be given, for instance, a counsel with
whom he could communicate and a qualified and competent
interpreter to assist him. Also, an accused should be proceeded
against with a particular understanding of his state or condition
of mind. One exhibiting signs of unstable mental condition
should not be treated like any other sane person if the
guarantee of due process us to be accorded substance and
meaning
People v. Crisologo
150 SCRA 653(1987)
Facts: Appellant, a deaf mute, was charged with robbery with
homicide committed on May 1, 1976. The accused was allegedly
informed of the charge against him through sign language by Special
Policeman Alejandro Munoz, a childhood acquaintance. Subsequently
Munoz entered a plea of guilty on behalf of the accused. The pleas was
disregarded upon the objection of the counsel and arraignment was
rescheduled until such time the court could avail of the service of an
expert in the sign language from the school of the deaf and dumb. In
1979 through another presiding judge the court reset arraignment as
101 | P

LATON

no expert in sign language was available. In 1982 after five years under
another presiding judge directed the school of the deaf and dumb Bago
Gallera, Talmo District, Davao City be availed of to the enable the
accused to intelligently express his understanding of the plea of guilty
or not guilty. Finally in February 10, 1986 without the service of the
expert in sign language, accused was found guilty and sentence to die in
electrocution.
Issue: Whether or not rendering the decision without the aid of an
expert in sign language deprives the accused of due process of law.
Held: Absence of an interpreter in sign language deprived the accused
of the full and fair trial and reasonable opportunity to defend himself.
The absence of a qualified interpreter in sign language and of any other
means, whether in writing of otherwise, to inform the accused of the
charges against him denied the accused his fundamental right to due
process of a law. Aside from the unfair setting of circumstance in which
the accused was convicted, insufficiency of evidence to warrant a
finding guilty beyond reasonable doubt also led the court to set aside
the conviction.
People v. Estrada
333 SCRA 699(2000)
Facts: On December 27, 1994 a confirmation was being held at the St.
Johns Cathedral, Dagupan City, when the appellant Roberto Estrada sat
on the Bishops chair. Some churchgoers summoned Mararac, the
security guard of the Cathedral to request the appellant to vacate the
Bishops Chair. Mararac tapped appellants hand with a night stick but
the appellant did not badge. When Mararac attmpt to tapped the
appellants hands again, the appellant drew a knife from his back and
stubbed the gurad. After the commotion appellant got up, went to the
mocirophone and shouted in the local dialect that no one can beat him
there. Mararac expired a few minutes after arrival in the hospital.
Appellant was charged with murder.
Counsel for the appellant filed an Urgent Motion to suspend
arraignment and to commit the accused to the Psychiatric Ward at
Baguio General Hospital because he was suffering mental illness.
Motion was opposed for the reason that the appellant answered the
questions intelligently. Jail warden of Dagupan City wrote the trial
judge informing him of the accused appellants unsual behaviour and
requesting that he be examined at the hospital to determine whether
he should remain in jail or to other institution, judge ignored the letter.
One year later accused appellant counsel filed a Motion to Confine
Accused for Physical , Mental and Psychiatric Examination. Attached to
this motion was a second letter by the new Jail Warden of Dagupan City
accompanied by letter complaint of the Bukang Liwayway Association
City Jail. The judge still ignored the motion. The trial Court instead
found the appellant guilty of murder and sentenced hin to death.
Issue: Whether or not the appellant was deprived of a fair trial, and a
violation to the basic requirements to due process.
Held: The trial court took solely upon itself to determine the sanity of
the accused appellant. The trial judge is not a psychiatrist or
psychologist or some of the persons mental health. By depriving the
appellant of a mental examination, the trial court effectively deprived
appellant for a fair trial. The trial courts negligence was a VIOLATION
OF THE BASIC REQUIREMNTS OF THE DUE PROCESS. There is a need to
reconstitute the records in accordance with the procedure outlined in
the law, in order to dispel and doubt as to the integrity of the records
that were lost and recovered in the office of the Solicitor General. Every
aspect of the right to due process must be afforded the accused
appellant, and this includes the right to examine and assail the veracity
of every piece of evidence contained in the records. Sentence to death
is vacated and the case is remanded to the court a quo for the conduct
proper mental examination on the accused appellant.

Administrative and Military Tribunals

Courts are the proper fora before which criminal proceedings


are supposed to be heard. It is before the courts that certain
minimum guarantees are assured, not the least of which is the
presence of an impartial magistrate steeped in the delicate task
of determining guilt established by proof beyond reasonable
doubt. Administrative bodies could not qualify for such task,
especially so as they only require substantial evidence for the
purpose of supporting their determinations. As for military
tribunals, they may be better fitted for and attuned to the
disciplinary needs within the military structure but not
necessarily for civilians outside. However, during the time when
martial law was in effect, and when former President Marcos
was still in power, the Court did not find anything wrong with
trial of civilians by military tribunals, only to shed off that
doctrine the moment Marcos was cast away in Hawaii
Scotys Department Store v. Micaller
99Phiul.762(1956)
Facts: Nena Micaller a salesgirl of Scotys Department Store in Scolta
Manila organized a union among her co employees which was later
affiliated to NLU. NLU sent petition to the store with several demands,
the petitioners subjected Micaller and her com employees to
questioning with respect to their union membership activities. Micaller
hired temporary employees equal in number to the old. Latter was
charged with for alleged threats and slander before being dismissed for
insulting the owner of the store and for talking to the girls inside the
store during business hours. Micaller filed charges of unfair labor
practice with the Court of Industrial Relations (CIR), alleging that she
was dismissed because of her union activities. CIR found the petitioners
guilty of unfair labor practice, and ordered them to pay a fine of
100php. In their appeal, the petitioners raise as one of the issues
whether they can be legally punished by a fine of 100php.
Issue: Whether or not petitioners can be legally punished by a fine of
100php
Held: Court reach the conclusion that word cannot refer to the Court of
the Industrial Relations for to give that meaning would be a violation of
the safeguards guaranteed to every accused by our constitution,
referring to those which postulate that no one shall be held to answer
for a criminal offense without due process of law. Procedure laid
down to be observed by the CIR in dealing with unfair labor practice
cases negates those constitutional guarantees to the accused. Rules
and evidence prevailing to the court of the law or equity shall not be
controlling and it is the spirit and intention of the act the court and its
members shall use all reasonable means to ascertain the facts and
without the regards to technicalities of law and procedure. Court shall
not be bound solely by evidence presented during the hearing but may
avail itself of all other means such as ocular inspection and questioning
of well informed persons.
The power to impose penalties provided for section 25 of Republic Act
No. 875 (the fine of 100php) is lodged in ordinary courts and not in CIR.
The decision appealed from is modified by eliminating the fine of
100php in all other respects
OLAGUER V. MILITARY COMMISSION NO. 34
150 SCRA 144 (1987)
Habeas Corpus
In 1979, Olaguer and some others were detained by military personnel
and they were placed in Camp Bagong Diwa. Logauer and his group are
all civilians. They were charged with (1) unlawful possession of
explosives and incendiary devices; (2) conspiracy to assassinate
President and Mrs. Marcos; (3) conspiracy to assassinate cabinet
members Juan Ponce Enrile, Francisco Tatad and Vicente Paterno; (4)
conspiracy to assassinate Messrs. Arturo Tangco, Jose Roo and Onofre
Corpus; (5) arson of nine buildings; (6) attempted murder of Messrs.
Leonardo Perez, Teodoro Valencia and Generals Romeo Espino and
Fabian Ver; and (7) conspiracy and proposal to commit rebellion, and
102 | P

LATON

inciting to rebellion. On August 19, 1980, the petitioners went to the SC


and filed the instant Petition for prohibition and habeas corpus.
ISSUE: Whether or not the petition for habeas corpus be granted.
HELD: The petition for habeas corpus has become moot and academic
because by the time the case reached the SC Olaguer and his
companions were already released from military confinement. When
the release of the persons in whose behalf the application for a writ of
habeas corpus was filed is effected, the Petition for the issuance of the
writ becomes moot and academic. 18 Inasmuch as the herein
petitioners have been released from their confinement in military
detention centers, the instant Petitions for the issuance of a writ of
habeas corpus should be dismissed for having become moot and
academic. But the military court created to try the case of Olaguer
(and the decision it rendered) still continues to subsist.
ISSUE2: The issue is then shifted to: Whether or not a military tribunal
has the jurisdiction to try civilians while the civil courts are open and
functioning.
HELD: The SC nullified for lack of jurisdiction all decisions rendered by
the military courts or tribunals during the period of martial law in all
cases involving civilian defendants. A military commission or tribunal
cannot try and exercise jurisdiction, even during the period of martial
law, over civilians for offenses allegedly committed by them as long as
the civil courts are open and functioning, and that any judgment
rendered by such body relating to a civilian is null and void for lack of
jurisdiction on the part of the military tribunal concerned.

Res inter alios judicatae nullum aliis praejudicium faciunt.


Matters adjudged in a cause do not prejudice those who were
not parties to it. (54 C.J. 719) It is a cardinal rule of procedure
that a courts judgment of order in a case shall not adversely
24
affect persons who were not parties to the self same case
Due Process for the Prosecution and the Offended Party
While the language of the due process provision seems to be a
guarantee only for the accused, the assurance of fair play
actually runs both ways. There could be no due process if only
the accused is treated fairly for, in the ultimate analysis, that
would hardly be considered fair or just at all
A day in court is the touchstone of the right to due process in
criminal justice. It is an aspect of the duty of the government to
follow a fair process of decision-making when it acts to deprive
a person of his liberty. But just as an accused is accorded this
constitutional protection, so is the State entitled to due process
in criminal prosecutions. It must similarly be given the chance
to present its evidence in support of a charge
GALMAN V. SANDIGANBAYAN
144 SCRA 43 (1986)
Facts: Assassination of former Senator Benigno "Ninoy" Aquino, Jr. He
was killed from his plane that had just landed at the Manila
International Airport. His brain was smashed by a bullet fired pointblank into the back of his head by an assassin. The military investigators
reported within a span of three hours that the man who shot Aquino
(whose identity was then supposed to be unknown and was revealed
only days later as Rolando Galman) was a communist-hired gunman,
and that the military escorts gunned him down in turn.
President was constrained to create a Fact Finding Board to investigate
due to large masses of people who joined in the ten-day period of
national mourning yearning for the truth, justice and freedom.

24

See Pages 285-288

The fact is that both majority and minority reports were one in
rejecting the military version stating that "the evidence shows to the
contrary that Rolando Galman had no subversive affiliations. Only the
soldiers in the staircase with Sen. Aquino could have shot him; that
Ninoy's assassination was the product of a military conspiracy, not a
communist plot. Only difference between the two reports is that the
majority report found all the twenty-six private respondents abovenamed in the title of the case involved in the military conspiracy; "
while the chairman's minority report would exclude nineteen of them.
Then Pres. Marcos stated that evidence shows that Galman was the
killer.
Petitioners pray for issuance of a TRO enjoining respondent court from
rendering a decision in the two criminal cases before it, the Court
resolved by nine-to-two votes 11 to issue the restraining order prayed
for. The Court also granted petitioners a five-day period to file a reply
to respondents' separate comments and respondent Tanodbayan a
three-day period to submit a copy of his 84-page memorandum for the
prosecution.
But ten days later, the Court by the same nine-to-two-vote ratio in
reverse, resolved to dismiss the petition and to lift the TRO issued ten
days earlier enjoining the Sandiganbayan from rendering its decision.
The same Court majority denied petitioners' motion for a new 5-day
period counted from receipt of respondent Tanodbayan's
memorandum for the prosecution (which apparently was not served on
them).
Thus, petitioners filed a motion for reconsideration, alleging that the
dismissal did not indicate the legal ground for such action and urging
that the case be set for a full hearing on the merits that the people are
entitled
to
due
process.
However, respondent Sandiganbayan issued its decision acquitting all
the accused of the crime charged, declaring them innocent and totally
absolving them of any civil liability. Respondents submitted that with
the Sandiganbayan's verdict of acquittal, the instant case had become
moot and academic. Thereafter, same Court majority denied
petitioners' motion for reconsideration for lack of merit.
Hence, petitioners filed their motion to admit their second motion for
reconsideration alleging that respondents committed serious
irregularities constituting mistrial and resulting in miscarriage of justice
and gross violation of the constitutional rights of the petitioners and
the sovereign people of the Philippines to due process of law.
Issues:
(1) Whether or not petitioner was deprived of his rights as an accused.
(2) Whether or not there was a violation of the double jeopardy clause.

Held: Petitioners' second motion for reconsideration is granted and


ordering a re-trial of the said cases which should be conducted with
deliberate dispatch and with careful regard for the requirements of due
process.
Deputy Tanodbayan Manuel Herrera (made his expose 15 months later
when former Pres. was no longer around) affirmed the allegations in
the second motion for reconsideration that he revealed that the
Sandiganbayan Justices and Tanodbayan prosecutors were ordered by
Marcos to whitewash the Aquino-Galman murder case. Malacaang
wanted dismissal to the extent that a prepared resolution was sent to
the Investigating Panel. Malacaang Conference planned a scenario of
trial where the former President ordered then that the resolution be
revised by categorizing the participation of each respondent; decided
that the presiding justice, Justice Pamaran, (First Division) would
personally handle the trial. A conference was held in an inner room of
the Palace. Only the First Lady and Presidential Legal Assistant Justice
Lazaro were with the President. The conferees were told to take the
back door in going to the room where the meeting was held,
presumably to escape notice by the visitors in the reception hall waiting
to see the President. During the conference, and after an agreement
103 | P

LATON

was reached, Pres. Marcos told them 'Okay, mag moro-moro na lamang
kayo;' and that on their way out of the room Pres. Marcos expressed his
thanks to the group and uttered 'I know how to reciprocate'.
The Court then said that the then President (code-named Olympus) had
stage-managed in and from Malacaang Palace "a scripted and
predetermined manner of handling and disposing of the AquinoGalman murder case;" and that "the prosecution in the Aquino-Galman
case and the Justices who tried and decided the same acted under the
compulsion of some pressure which proved to be beyond their capacity
to resist. Also predetermined the final outcome of the case" of total
absolution of the twenty-six respondents-accused of all criminal and
civil liability. Pres. Marcos came up with a public statement aired over
television that Senator Aquino was killed not by his military escorts, but
by a communist hired gun. It was, therefore, not a source of wonder
that President Marcos would want the case disposed of in a manner
consistent with his announced theory thereof which, at the same time,
would clear his name and his administration of any suspected guilty
participation in the assassination. such a procedure would be a better
arrangement because, if the accused are charged in court and
subsequently acquitted, they may claim the benefit of the doctrine of
double jeopardy and thereby avoid another prosecution if some other
witnesses shall appear when President Marcos is no longer in office.
More so was there suppression of vital evidence and harassment of
witnesses. The disappearance of witnesses two weeks after Ninoy's
assassination. According to J. Herrera, "nobody was looking for these
persons because they said Marcos was in power. The assignment of the
case to Presiding Justice Pamaran; no evidence at all that the
assignment was indeed by virtue of a regular raffle, except the
uncorroborated testimony of Justice Pamaran himself. The custody of
the accused and their confinement in a military camp, instead of in a
civilian jail. The monitoring of proceedings and developments from
Malacaang and by Malacaang personnel. The partiality of
Sandiganbayan betrayed by its decision: That President Marcos had
wanted all of the twenty-six accused to be acquitted may not be
denied. In rendering its decision, the Sandiganbayan overdid itself in
favoring the presidential directive. Its bias and partiality in favor of the
accused was clearly obvious. The evidence presented by the
prosecution was totally ignored and disregarded.
The record shows that the then President misused the overwhelming
resources of the government and his authoritarian powers to corrupt
and make a mockery of the judicial process in the Aquino-Galman
murder cases. "This is the evil of one-man rule at its very worst." Our
Penal Code penalizes "any executive officer who shall address any order
or suggestion to any judicial authority with respect to any case or
business coming within the exclusive jurisdiction of the courts of
justice."
Impartial court is the very essence of due process of law. This criminal
collusion as to the handling and treatment of the cases by public
respondents at the secret Malacaang conference (and revealed only
after fifteen months by Justice Manuel Herrera) completely disqualified
respondent Sandiganbayan and voided ab initio its verdict. The courts
would have no reason to exist if they were allowed to be used as mere
tools of injustice, deception and duplicity to subvert and suppress the
truth. More so, in the case at bar where the people and the world are
entitled to know the truth, and the integrity of our judicial system is at
stake.
There was no double jeopardy. Courts' Resolution of acquittal was a
void judgment for having been issued without jurisdiction. No double
jeopardy attaches, therefore. A void judgment is, in legal effect, no
judgment at all. By it no rights are divested. It neither binds nor bars
anyone. All acts and all claims flowing out of it are void.
Motion to Disqualify/Inhibit should have been resolved ahead. In this
case, petitioners' motion for reconsideration of the abrupt dismissal of
their petition and lifting of the TRO enjoining the Sandiganbayan from
rendering its decision had been taken cognizance of by the Court which
had required the respondents', including the Sandiganbayan's,
comments. Although no restraining order was issued anew, respondent

Sandiganbayan should not have precipitately issued its decision of total


absolution of all the accused pending the final action of this Court. All
of the acts of the respondent judge manifest grave abuse of discretion
on his part amounting to lack of jurisdiction which substantively
prejudiced the petitioner.
With the declaration of nullity of the proceedings, the cases must now
be tried before an impartial court with an unbiased prosecutor.
Respondents accused must now face trial for the crimes charged
against them before an impartial court with an unbiased prosecutor
with all due process.
The function of the appointing authority with the mandate of the
people, under our system of government, is to fill the public posts.
Justices and judges must ever realize that they have no constituency,
serve no majority nor minority but serve only the public interest as they
see it in accordance with their oath of office, guided only the
Constitution and their own conscience and honor.

Demurrer to Evidence, Acquittal and Civil Liability


When a criminal case is filed, it generally carries with it the
institution of the civil liability arising from the offense. Thus,
when the accused is found guilty, he is also adjudged civilly
liable. It may happen, however, the he may get exonerated
from the criminal liability due to reasonable doubt and yet may
still be found liable for damages. Ordinarily, this would mean
that the judgment comes after both the prosecution and the
defense would have presented their respective evidence
In Salazar v. People, the accused was acquitted in a case for
estafa (swindling) arising from her endorsement of a check
which bounced. She was, however, order to pay the amount
represented by the check. She questioned that part of the
judgment, claiming that she was denied due process since she
was not given an opportunity to adduce evidence to prove that
she was not civilly liable. The Supreme Court agreed:
If the demurrer is granted and the accused is acquitted by the
court, the accused has the right to adduce evidence on the civil
aspect of the case unless the court also declares that the act or
omission from which the civil liability may arise did not exist. If
the trial court issues an order or renders judgment not only
granting the demurrer to evidence of the accused and
acquitting him but also on the civil liability of the accused to the
private offended party, said judgment on the civil aspect of the
case would be a nullity for the reason that the constitutional
right of the accused to due process is thereby violated
Right to Bail
Once a person has been charged he may find himself already
the object of a warrant of arrest with the consequent threat of
momentary loss of his liberty. To secure his freedom, he would
have to post bail or otherwise obtain the same through
recognizance. Bail is the security given for the release of a
person in custody of the law. It may be in the form of corporate
surety, property bond, cash deposit, or recognizance
A recognizance is an obligation of record entered into before
some court or magistrate duly authorized to take it, with the
condition to do some particular act, the most usual condition in
criminal cases being the appearance of the accused for
trial...[A] contract between the sureties and state for the
production of the principal at the required time
General Considerations
104 | P

LATON

In order for one to be entitled to apply for bail, he must already


be in the custody of the law or otherwise deprived of his
liberty. If he is not, then there could hardly be any basis for the
court in granting him liberty as the court would not yet have
acquired any jurisdiction over him. Jurisdiction acquired may be
by reason of an arrest, or simply by surrendering himself to the
court
One who is detained though not yet formally charged may
apply for bail; otherwise, to afford bail to one already charged
and yet deny it to one who has not yet been determined by the
authorities whether to charge or not would result in an
anomalous and absurd situation. On the other hand, the
accused need not be arraigned first before being granted bail
Bail is generally available only in criminal proceedings.
Accordingly, it may not be availed of in administrative
proceedings, like deportation or extradition proceedings
The Court also held that the right to bail has traditionally not
been recognized and is not available to those in the armed
forces given the unique structure of the military
Right to Bail in General
GR: A person charged with a crime has the right to bail
XPN: If he is charged with a crime punishable by reclusion
perpetua or life imprisonment, or death, and the evidence
against him is really strong
In regard to appeals from convictions by the Regional Trial
Court respecting offenses not punishable by death, reclusion
perpetua or life imprisonment, bail may also be availed of, but
then not anymore as a matter of right. It becomes discretionary
with the court
PEOPLE V. DONATO
198 SCRA 130 (1991)
Private respondent Salas, chairman of the communist party of the
Philippines and head of the military arm of the NPA, with co-accused
was charged of rebellion. The information was filed and they were in
military custody. Private respondent filed a petition for bail.
Whether or not the right to bail may, under certain circumstance be
denied to a person with an otherwise bailable offense, and whether
such right may be waived.
Held: Bail is a matter of right even in cases of capital offenses, unless
the proof of guilt is evident or the presumption thereof is great. The
prosecution does not have the right to oppose the bail where bail is a
matter of right. However, in cases where the grant of bail is
discretionary, due process requires that the prosecution must be
allowed to present evidence, within a reasonable time, the evidence
presented should resolve the motion for bail.
Private respondent has waived his right to bail when he entered into an
agreement that he will remain in legal custody and face trial before the
court having custody of his person. Custody has been held to mean
nothing less than actual imprisonment. The right to bail is another of
the constitutional rights which can be waived. It is a right which is
personal to the accused.
OBOSA v. CA
266 SCRA 281 (1997)

Obosa was charges with 2 counts of murder, a capital offense for the
ambush slaying of Sec of Interior and Local Govt Jaime Ferrer and his
driver Jesus Calderon. On the day the trial court promulgated its
decision, Obosa manifested his intention to appeal and ask the court to
allow him to post bail.
Is petitioner entitled to bail as a matter of right?
Held: Before conviction, every person is bailable except if charged with
capital offenses when the evidence of guilt is strong.
We rule that bail cannot be granted as a matter of right even after an
accused, who is charged with the capital offense, appeals his convition
for a non-capital crime. In ruling against bail, the lower court found that
treachery attended the killing thereby justifying its action. Hereby
petition is denied.
Paderanga v CA
247 SCRA 741(1995)
Petitioner was belatedly charged in an amended information as coconspirator in a crime of multiple murder in Cagayan de oro for the
killing of bucag family of which petitioner was mayor at that time.
Felizardo was implicated in the crime. He hired petitioner as counsel
but later during preliminary investigation, he implicated petitioner as
the mastermind of the massacre. Before arrest warrant could be
granted to him, his counsel filed a motion to bail. RTC granted bail.CA
reversed.
The main purpose of bail is to relieve an accused from the rigors of
imprisonment until hi conviction and yet secure his appearance at the
trial. The same cannot be posted before custody over him has been
acquired either by lawful arrest or voluntary surrender. The rationale of
this rule is that it discourages and prevents resort to the former
pernicious practice whereby an accused could just send another in his
stead to post his bail. As a paramount requisite, only those who have
been arrested detained or deprived of liberty can post bail. This
however is subject to the limitation that the applicant is in the custody
of the law. He was constructively in the custody of the law even if he
was not physically arrested because he was hospitalized.
Section Section 13, Article III of the Constitution lays down the rule that
before conviction, all indictees shall be allowed bail, except only those
charged with offenses punishable by reclusion perpetua when the
evidence of guilt is strong. In pursuance thereof, Section 4 of Rule 114,
as amended, now provides that all persons in custody shall, before
conviction by a regional trial court of an offense not punishable by
death, reclusion perpetua or life imprisonment, be admitted to bail as a
matter of right. The right to bail, which may be waived considering its
personal nature 21 and which, to repeat, arises from the time one is
placed in the custody of the law, springs from the presumption of
innocence accorded every accused upon whom should not be inflicted
incarceration at the outset since after trial he would be entitled to
acquittal, unless his guilt be established beyond reasonable doubt.
Thus, the general rule is that prior to conviction by the regional trial
court of a criminal offense, an accused is entitled to be released on bail
as a matter of right, the present exceptions thereto being the instances
where the accused is charged with a capital offense or an offense
punishable by reclusion perpetua or life imprisonment 23 and the
evidence of guilt is strong. Under said general rule, upon proper
application for admission to bail, the court having custody of the
accused should, as a matter of course, grant the same after a hearing
conducted to specifically determine the conditions of the bail in
accordance with Section 6 (now, Section 2) of Rule 114. On the other
hand, as the grant of bail becomes a matter of judicial discretion on the
part of the court under the exceptions to the rule, a hearing,
mandatory in nature and which should be summary or otherwise in the
discretion of the court, is required with the participation of both the
defense and a duly notified representative of the prosecution, this time
to ascertain whether or not the evidence of guilt is strong for the
provisional liberty of the applicant. Of course, the burden of proof is on
the prosecution to show that the evidence meets the required
quantum.
105 | P

LATON

SC reinstated bail.

Bail Hearing
Part of the requirement in the grant or denial of bail, especially
in non-bailable cases, is the necessity for a hearing. The hearing
is summary in nature, meaning such brief and speedy method
of receiving and considering the evidence of guilt as is
practicable and consistent with the purpose of the hearing
which is merely to determine the weight of evidence for
purposes of bail
It has also been pointed out that: A bail application does not
only involve he right of the accused to temporary liberty, but
likewise the right of the State to protect the people and the
peace of the community from dangerous elements. These two
rights must be balanced by a magistrate in the scale of justice,
hence, the necessity for hearing to guide his exercise of
discretion
Further, arraignment is not a prerequisite to the conduct of
hearings on a petition for bail. A person is allowed to petition
for bail as soon as he is deprived of his liberty by virtue of his
arrest or voluntary surrender--he need not await his
arraignment
RE RELEASE BY JUDGE MANUEL T. MURO OF AN ACCUSED IN A NONBAILABLE OFFENSE
367 SCRA 285
Yu yuk lai is one of the accused for sale and delivery of 3 kg of shabu,
was reasrested bby PAOCTF while playing baccarat without jail guards
at the holiday inn casino. At that time he was supposed to be detained
at the manila city jail since his bail was denied. It was published in
Philippine star that Yu yuk lai has obtained an order signed by muro
allowing her hospital confinement for medical and humanitarian
reasons. Muro was directed to submit pertinent documents showing
that the order was regularly issued. He complied. He was suspended
and investigated.
Trial judge may rely on statements of someone knowledgeable in the
subject, it should not be to the extent that such reliance would amount
to a surrender of his authority to decide.
As shown by the records, respondent judge was guilty of gross
misconduct constituting violations of the code of judicial conduct for
being utterly inefficient and for manifesting partiality. Despite request
that he be confined in PGH, judge preferred Manila Doctors on the
ground that it is yu yuks choice. He subjected the length of medical
confinement to the will of her physicians. And he did not make incisive
inquiry to ascertain the true state of facts despite the opposition by
another prosecutor. He threw all his cautions in the wind in favor of yu
yuk
Judge was dismissed.
Lavides v Court of appeals
324 SCRA 321 (2000)
Petitioner was arrested for child abuse. His arrest was made without
warrant as a result of entrapment conducted by police. He was caught
with 16-year old complainant. Within a month, nine more informations
were filed against him by complainant and 3 others who had been
exploited in prostitution given money as payment for sexual
intercourse. He was entitled to bail with conditions that 1. He shall not
be entitled to waiver of appearance and must be present at all hearings
2. If he did not, his bail shal be automatically forfeited, warrants shall
be immediately issued and cases will proceed trial in absentia 3. hold
departure-order stands 4. Approval of bail shall be done after
arraignment for courts to acquire jurisdiction over him.

In requiring that petitioner be first arraigned before he could be


granted bail, the trial court apprehended that if petitioner were
released on bail he could, by being absent, prevent his early
arraignment and thereby delay his trial until the complainants got tired
and lost interest in their cases. Hence, to ensure his presence at the
arraignment, approval of petitioner's bail bonds should be deferred
until he could be arraigned. After that, even if petitioner does not
appear, trial can proceed as long as he is notified of the date of hearing
and his failure to appear is unjustified, since under Art. III, 14(2) of the
Constitution, trial in absentia is authorized. This seems to be the theory
of the trial court in its May 16, 1997 order conditioning the grant of bail
to petitioner on his arraignment.
This theory is mistaken. In the first place, as the trial court itself
acknowledged, in cases where it is authorized, bail should be granted
before arraignment, otherwise the accused may be precluded from
filing a motion to quash. For if the information is quashed and the case
is dismissed, there would then be no need for the arraignment of the
accused. In the second place, the trial court could ensure the presence
of petitioner at the arraignment precisely by granting bail and ordering
his presence at any stage of the proceedings, such as arraignment.
Under Rule 114, 2(b) of the Rules on Criminal Procedure, one of the
conditions of bail is that "the accused shall appear before the proper
court whenever so required by the court or these Rules," while under
Rule 116, 1(b) the presence of the accused at the arraignment is
required.
On the other hand, to condition the grant of bail to an accused on his
arraignment would be to place him in a position where he has to
choose between (1) filing a motion to quash and thus delay his release
on bail because until his motion to quash can be resolved, his
arraignment cannot be held, and (2) foregoing the filing of a motion to
quash so that he can be arraigned at once and thereafter be released
on bail. These scenarios certainly undermine the accused's
constitutional right not to be put on trial except upon valid complaint
or information sufficient to charge him with a crime and his right to
bail.8
It is the condition in the May 16, 1997 order of the trial court that
"approval of the bail bonds shall be made only after arraignment,"
which the Court of Appeals should instead have declared void. The
condition imposed in the trial court's order of May 16, 1997 that the
accused cannot waive his appearance at the trial but that he must be
present at the hearings of the case is valid and is in accordance with
Rule 114. For another condition of bail under Rule 114, 2(c) is that
"The failure of the accused to appear at the trial without justification
despite due notice to him or his bondsman shall be deemed an express
waiver of his right to be present on the date specified in the notice. In
such case, trial shall proceed in absentia."
Petitioner could delay the proceedings by absenting himself from the
arraignment. But once he is arraigned, trial could proceed even in his
absence. So it thought that to ensure petitioner's presence at the
arraignment, petitioner should be denied bail in the meantime. The fly
in the ointment, however, is that such court strategy violates
petitioner's constitutional rights
The Court of Appeals declared conditions (a) and (b) invalid but
declined to pass upon the validity of condition (d) on the ground that
the issue had become moot and academic. Although this condition is
invalid, it does not follow that the arraignment of petitioner on May 23,
1997 was also invalid. Contrary to petitioner's contention, the
arraignment did not emanate from the invalid condition that "approval
of the bail bonds shall be made only after the arraignment." Even
without such a condition, the arraignment of petitioner could not be
omitted. In sum, although the condition for the grant of bail to
petitioner is invalid, his arraignment and the subsequent proceedings
against him are valid.
WHEREFORE, conditions are valid, with the exception of condition (d)
(making arraignment a prerequisite to the grant of bail to petitioner),
which is hereby declared void.
106 | P

LATON

In Serapio v. Sandiganbayan, the Court held that no


inconsistency exists between application of an accused for bail
and his filing of a motion to quash. These two reliefs have
objectives which are not necessarily antithetical to each other.
Certainly, the right of an accused to seek provisional liberty
when charged with an offense not punishable by death,
reclusion perpetua or life imprisonment, or when charged with
an offense punishable by such penalties but after due hearing,
evidence of his guilt is found not to be strong, does not
preclude his right to assail the validity of the Information
charging him with such offense
Reasonableness of Bail
The directive not to require excessive bail could thus be read to
mean that bail should be just and equitable and not the
product of capricious or whimsical exercise of discretion
DE LA CAMARA V ENAGE
41 SCRA 1
Petitioner, Ricardo de la Camara, a town mayor in Misamis Oriental,
was arrested and detained Magsaysay, Misamis Oriental was arrested
and detained at the provincial jail for his alleged participation in the
killing of 14 and wounding off 12 other laborers.(multiple frustrated
murder and multiple murder) petitioner filed for bail premised on the
assertion that there was no evidence to link him with such incident.
Respondent judge granted bail admitting that the prosecution failed to
prove that petitioner would flee even if he had an opportunity but fixed
1.195 M +.
The bail of petitioner is repugnant to the constitutional mandate
prohibiting excessive bail. The case is moot and academic since
petitioner escaped. However for the guidance of the lower courts, here
are the doctrines that should be observed in fixing the amount of bail:
1. Before conviction, every person is bailable except if charged with
capital offenses when the evidence of guilt is strong. Such right flows
from the presumption of innocence.
2. Where the right of bail exist, it should not be rendered nugatory by
requiring a sum that is excessive. The sole permissible function of
money bail is to assure the accuseds presence in trial and bail set at
higher figure than amount reasonably calculated to fulfil this purpose is
excessive.
3. The guidelines in fixing bail:
1. Ability of the accused to give bail
2. Nature of offense
3. Penalty for offense
4. Character and reputation of accused
5. Character and strength of evidence
6. Probability of appearing in trial
7. Forfeiture of other bonds
8. WON the accused was fugitive when arrested
9. If accused is under bond for other cases

Extradition and Bail


As noted earlier, the right to bail has been considered as
referring only to criminal proceedings and not extending to
administrative cases. In extradition proceedings, the Court said
that the right to avail of bail would be only in exceptional
circumstances. Thus, after a potential extradite has been
arrested or placed under the custody of the law, he may apply
for, and be granted, bail as an exception only upon a clear and
convincing showing that (1) once granted bail, he will not be a
flight risk or a danger to the community, and (2) there exist
special, humanitarian and compelling circumstances

Facts: In accordance to the existing RP-US Extradition Treaty, the US


Government sent to the RP documents requesting the extradition of
Mark B. Jimenez a.k.a. Mario Batacan Crespo. SFA transmitted the
documents to the SOJ for appropriate action. Then the DOJ filed with
the RTC the appropriate petition for Extradition in relation to charges
involving conspiracy to defraud the US, tax evasion, wire fraud, false
statements, and illegal campaign contributions. To prevent the flight of
Jimenez, the petition prayed for an order for his immediate arrest.
Nevertheless, prior to such petition, the Jimenez had already filed and
urgent manifestation or ex-parte motion which prayed that the
application for his arrest warrant be set for hearing. RTC granted the
motion and eventually issued a warrant for Jimenezs arrest and fixed
bail at P1 Min cash. After he surrendered his passport and posted the
required cash bond, Jimenez was granted provisional liberty by the
court.
Issues: WON prospective extraditees are entitled to notice and hearing
before warrants for their arrest can be issued
WON they are entitled to the right to bail and provisional liberty while
the extradition proceeding s are pending
Held: Conviction is only applicable when a person has been arrested
and detained for violation of Phil. criminal laws. It is not applicable to
extradition proceedings because extradition courts do not render
judgments of conviction or acquittal. The constitutional provision on
bail is also not applicable to extradition proceeding s because
presumption of innocence is not an issue therein. Extradition
proceedings are separate and distinct from the trial for the offense for
which Jimenez is charged. He should apply for bail before the courts
trying the criminal cases against him, not before the extradition court.
Exceptions to the No Bail Rule: Upon a clear and convincing showing (1)
that once granted bail, the applicant will not be a flight risk or a danger
to the community, and (2) that there exist special, humanitarian and
compelling circumstances including those cited by the highest court in
the requesting state when it grants provisional liberty in extradition
cases therein.
Regarding the alleged disenfranchisement that Jimenez contends, the
Court is not persuaded since it was already of public knowledge that
the US was requesting for his extradition prior to his running for and
winning of a congressional seat in Manila. His constituents were or
should have been prepared for the consequences of his extradition
case. Also, the disenfranchisement argument was already debunked by
the SC in People v. Jalosjos.
In response to the contention of the respondent about anticipated
delay, the court says that extradition proceedings are summary in
nature and they are only resorted to determine WON the petition and
its annexes conform to the Extradition treaty law.
Although the SC admits that respondent has not actually fled during the
preliminary stages of the request for his extradition, it does not mean
that he will not flee as the process moves forward to its final decision.
Petition is GRANTED; the RTC order is declared NULL and VOID, the
challenge order which granted bail to Jimenez, is SET ASIDE, and the
bail posted by him is CANCELLED.

Presumption of Innocence
The starting point in any criminal prosecution is the
presumption of innocence. This presumption stands as a
fundamental principle of both constitutional and criminal law,
imposing a rule of evidence, a degree of proof that demands no
less than total compliance. The accused goes before the court
without any prejudgment that he is guilty, or any obligation to
establish his innocence. Instead, it is the responsibility of the
State to prove with the requisite amount of evidence--proof
beyond reasonable doubt--that he is guilty

Government of the USA v. Purganan


389 SCRA 623 (2002)
107 | P

LATON

Then also, the presumption of innocence has never been


intended as evidence of innocence of the accused but only to
shift the burden of proof that he is guilty to the prosecution. If
accusation is not synonymous with guilt, so is the presumption
of innocence not a proof thereof. It is one thing to say that the
accused is presumed to be innocent in order to place on the
prosecution the burden of proving beyond reasonable doubt
that the accused is guilty. It is quite another thing to say that he
is innocent and if he is convicted that he has been unjustly
convicted
Proof Beyond Reasonable Doubt
A primordial consideration of due process when it comes to
criminal proceedings is the requirement that the presumption
of innocence could only be overcome by a quantum of evidence
that is based on proof beyond reasonable doubt. Any other
lesser standard of proof would run counter to the due process
guarantee. This higher requirement is basically dictated by the
fact that more substantial and important societal values are
involved--liberty and even, possibly, life
Proof beyond reasonable doubt, while indeed requiring more
than the other standards of proof, does not, of course, mean
such degree of proof that would produce absolute certainty.
Moral certainty only is required, or that degree of proof which
produces conviction in an unprejudiced mind
The sea of suspicion has no shore, and the court embarks
upon it is without rudder compass

if done by an adult would constitute the crime or crimes of Larceny.


The judge relied on the NY Family Court Act which required a standard
of proof based on a preponderance of evidence and not on the
appellants contention that proof must be established beyond a
reasonable doubt as required by the 14th amendment. The appellant
was then ordered to be placed in a training school for an initial period
of 18 months subject to annual extension of his commitment until his
18th birthday (6 yrs in his case). NY CA affirmed and sustained the
constitutionality of the provision of the NY Family Court Act.
Issue: WON the standard of proof beyond a reasonable doubt is
required in adjudicatory stage of delinquency proceedings
Held: The observance of the standard of proof beyond a reasonable
doubt will not compel the State to abandon or displace any of the
substantive benefits of the juvenile process. The Court rejects the CAs
suggestion that there is only a questionable difference between
reasonable doubt and preponderance standards. The constitutional
safeguard of proof beyond a reasonable doubt is also required during
the adjudicatory stage of delinquency proceedings because those are
constitutional safeguards. The Court agrees with Chief Judge Fulds
dissenting opinion that where a 12-yr old is charged with an act of
stealing which renders him liable to confinement for as long as 6 yrs,
then as a matter of due process, the case against him must be proved
beyond a reasonable doubt.
Estrada v. Sandiganbayan
369 SCRA 394 (2001)
Facts: Petitioner seeks the invalidation of the Anti-Plunder Law and
contends that the law allegedly dispensed with the requirement of
proof beyond reasonable doubt for conviction.
Issue: WON the Anti-Plunder Law violates the Due Process Clause

It must necessarily involve genuine and irreconcilable


contradictions based, not on suppositional thinking, but on the
hard facts constituting the elements of the crime. It is not mere
possible doubt, because everything relating to human affairs is
open to some possible or imaginary doubt. It should not be
vague, speculative or whimsical, but intelligent, reasonable
and impartial and based on a careful examination and
conscious consideration of all the evidence in the case. A
reasonable doubt is not such a doubt as any man may start by
questioning for the sake of a doubt; nor a doubt suggested or
surmised without foundation in facts of testimony, for it is
possible always to question any conclusion derived from
testimony, but such questioning is not what is reasonable doubt

Held: The use of the reasonable doubt standard is indispensable to


command the respect and confidence of the community in the
application of criminal law. Such standard has acquired such high
stature in the realm of constitutional law since it gives life to the Due
Process Clause which protects the accused against conviction except
upon proof beyond reasonable doubt of every fact necessary to
constitute the crime with which he is charged.

In an American case, it was held that an instruction to the jury


which equated reasonable doubt with a grave uncertainty
and an actual substantial doubt required less proof than was
consistent with the due process guarantee. It is plain to as that
the words substantial and grave, as they are commonly
understood, suggest a higher degree of doubt that is required
for acquittal under the reasonable doubt standard. When those
statements are then considered with the reference to moral
certainty, rather than evidentiary certainty, it becomes clear
that a reasonable juror could have interpreted the instruction
to allow a finding of guilt based on a degree of proof below that
required by the Due Process Clause

People v. Mendoza
231 SCRA 264 (1994)

In Re Winship
397 US 358, 25 L Ed 2d, 90 S Ct 1068 (1970)
Facts: Sec 12 of the NY Family Court Act defines a juvenile delinquent as
a person over 7 and less than 16 yrs. of age who does any act which, if
done by an adult, would constitute a crime. During a hearing in 1967,
the judge found that appellant, then a 12-yr old boy had entered a
locker and had stolen $112 from a womans pocketbook, an act which

What the prosecution needs to prove beyond reasonable doubt is only


a number of acts necessary to form a combination or series which
would constitute a pattern and involving an amount of at least P50 M.
There is no need to prove each and every other act alleged in the
Information. Thus, the prosecution is not required to make a deliberate
and conscious effort to prove pattern.

Facts: The Office of the Provincial Fiscal of Malaybalay, Bukidnon filed


the instant petition for certiorari and mandamus assailing the judgment
of respondent judge Mendoza because of the latters acquittal of the
accused Juan Magalop of the crime of robbery with force upon things
despite his plea of guilt. Based on the evidence, on Jan. 20, 1987, the
storeroom of Bukidnon National School of Home Industries in
Maramag, Bukidnon was ransacked. Since there was a loss of identity of
the culprit/s, responsibility was laid on the accused Juan Magalop,
Petronilo Fernandez, and Ricarte Dahilan. Magalop pleaded guilty,
Fernandez pleaded non guilty, Dahilans arraignment was deferred
because he was not mentally well. The court a quo conducted a hearing
and acquitted Fernandez and Magalop because the prosecution failed
to prove that the 3 accused were responsible even if some of the stolen
articles were found in the possession of Barbie Tan to whom they were
allegedly sold and such person refused to testify. Also, it was not shown
how Magalop and Fernandez conspired. The RTC denied the motion for
reconsideration.
Issue: WON respondent judge was correct in acquitting the accused
108 | P

LATON

Held: The petition is defective since only the OSG may bring and defend
actions on behalf of the People of the Philippines once such actions are
brought before the CA or the SC. However, in this case, petitioner filed
it directly with the Provincial Fiscal of Malaybalay, Bukidnon without
coursing it first through the OSG. It is evident that from the start that
the case of the prosecution against the 3 accused was non-existent
because the stolen articles were found in possession of Barbie Tan and
yet the prosecution did not summon her to the witness stand. Although
the respondent judge erred in the process, since it can be fairly
concluded that there was no standing plea at the time the court
rendered its judgment of acquittal, such acquittal was a nullity.
However, the Court cannot allow such procedural error to prevail over
the constitutional right of the accused to be presumed innocent until
the contrary is proved. Since there was no evidence against Magalop,
his acquittal must be sustained. Petition is DISMISSED for lack of merit
and the acquittal of the accused is sustained.
PEOPLE V. FRAGO
232 SCRA 653 (1994)
Facts: Appellant, an ice cream vendor, was charged with attempted
rape and rape. According to the version of the prosecution, the accused
went to the residence of nine-year-old Ronalyn Pastera. The accused
entered her bedroom, fanned her face with his handkerchief then lifted
her. He was about to take her out of the room when she suddenly woke
up and screamed for help. As a consequence, the accused drop Ronalyn
on the floor and run out of the house. The prosecution would seem to
infer that after running, he went to the boarding house of Jicelyn. He
carried Jicelyn to a nearby house where he had carnal knowledge while
she was deeply asleep. When she narrated of the harrowing
experience, Jicelyn and her mother went to the hospital where she was
examined. The results indicated the presence spermatozoa and
physical virginity lost. On the same day, Ronalyns father and Jicelyn
reported the incidents to the police authorities then filed their formal
complaints against the accused. Appellant interposed denial and alibi.
After trial, the court a quo found the accused guilty of rape but
acquitted him in the attempted rape case. In his appeal, he imputes
error to the trial courts conviction on the basis of an identification
made without a counsel and according to Jicelyns story, which he
believes fantastic, thereby denying his constitutional right to be
presumed innocent until proven beyond reasonable doubt.
Issue: WON Orlando Fragos conviction in the rape case was proved
beyond reasonable doubt.
Ruling: The testimony of Jicelyn shows that she has no reliable basis for
pointing to the accused as the person who raped her. The only
evidence of sexual intercourse is the result of the medical examination.
There was no positive identification of Orlando Frago by Jicelyn.
There seems to be no question that, on the part of the Pastera sisters,
they may have recognized appellant positively because their room was
lighted with a lamp and he was not wearing anything on his face. Since
they were neighbors, there is the possibility of Jicelyn conferring with
the Pastera sisters the identity of the accused. Thus, the Courts
conclusion that Jicelyns identification of Orlando Frago was merely
patterned after the identification made by the Pastera sisters. This is
then a derivative, not positive, identification. The identification then
of appellant by Jicelyn is doubtful.
Appellants denial and alibi are inherently weak, but the prosecution
cannot rely on their frailty to enhance its cause. The prosecution must
draw its strength from its own evidence.
Wherefore, Orlando Fargo is acquitted as his guilt has not been
proved beyond reasonable doubt.

THE INFORMED PLEA


When an accused is asked to state whether he pleads guilty or not
guilty, the same must be done in a manner that ensures that he actually
know what he is talking about. And this aspect becomes more critical

when the case is one where the supreme penalty of death may be
imposed. Courts should take extra efforts to assure themselves that the
accused, if ever he pleads guilty, does so with the all the attendant
safeguards by which he could give an informed, intelligent, knowing
and voluntary acknowledgment of his guilt.

The Informed Plea


When an accused is asked to state whether he pleads guilty or
not guilty, the same must be done in a manner that ensures
that he actually know what he is talking about. And this aspect
becomes more critical when the case is one where the supreme
penalty of death may be imposed. Courts should take extra
efforts to assure themselves that the accused, if ever he pleads
guilty, does so with all the attendant safeguards by which he
could give an informed, intelligent, knowing and voluntary
acknowledgment of his guilt
PEOPLE V. CHUA
366 SCRA 283 (2001)
Facts: Appellant was charged by his 13-year-old daughter, Chenny, of
rape. At arraignment, he pleaded not guilty but at the pretrial
conference appellant, through counsel, manifested that he was
withdrawing his plea and changing it to guilty as charged. The pretrial
was then reset to another date, on which resumption the court
propounded to appellant several questions to determine the
voluntariness of appellants change of plea and of his comprehension of
its consequences. Satisfied with appellants response, the court ordered
his rearraignment. Appellant, with the assistance of counsel, withdrew
his plea of not guilty and entered a plea of guilty as charged.
Thereafter, the court ordered the prosecution to present its evidence.
The prosecution presented Chenny who testified on how the alleged
rape was committed.
The trial court found appellant guilty and sentenced him to death.
Issue: WON the trial court observed the mandatory procedure in
ascertaining the guilt of the accused.
Ruling: When the accused enters a plea of guilty to a capital offense,
the trial court must do the following: (1) conduct a searching inquiry
into the voluntariness of the plea and the accuseds full comprehension
of the consequence thereof; (2) require the prosecution to present
evidence to prove the guilt of the accused and the precise degree of his
culpability; and (3) ask the accused if he desires to present evidence in
his behalf and allow him to do so if he desires. This procedure is
mandatory and a judge who fails to observe it commits grave abuse of
discretion.
The essence of a plea of guilty is that the accused admits his guilt freely,
voluntarily and with full knowledge of the consequence and meaning of
his act and with a clear understanding of the precise nature of the
crime charged in the complaint or information. The court must conduct
a searching inquiry. The inquiry is not a simple question and answer
exercise; it must be searching. To search means to look into or over
carefully or thoroughly in an effort to find something.
The judge should, first of all, consider the age, personality, educational
background, socio-economic status and other personal circumstances
of the accused had been coerced or placed under a state of duress
either by actual threats of physical harm coming from malevolent or
avenging quarters, or by mistaken impressions given, wittingly or
unwittingly, by authorities or parties; whether the accused had the
assistance of competent counsel during the custodial and preliminary
investigations; and whether he understood the charges against him. In
all cases, the bottom line is that the judge must fully convince himself
that: (1) the accused, in pleading guilty, is doing so voluntarily; and (2)
he, in so doing, is truly guilty, and that there exists a rational basis for a
finding of guilt, based on his testimony.

109 | P

LATON

It is clear that the court a quo did not probe carefully and thoroughly
into the reasons for applelants change of plea and his comprehension
of the consequences of said plea. Further, the trial court did not
bother to explain the essential elements of the crime with which
appellant was charged. The trial court did not only neglect to make
the searching inquiry, it also failed to inquire from appellant whether
he desired to present evidence in his behalf.

and possessed a firearm, described in the indictment, which firearm


had theretofore been shipped and transported in interstate
commerce. The Circuit Court of Appeals reversed the conviction on
each count.

There is also another element of the crime overlooked by the trial


court. There is no law that specifically defines and penalizes incest. The
case involves rape. Force or intimidation is an element of the crime of
rape. There must, therefore, be proof beyond reasonable doubt that
the victim did not resist her defloration due to the moral ascendancy
of the accused.

It shall be unlawful for any person who has been convicted of a crime
of violence or is a fugitive from justice to receive any firearm or
ammunition which has been shipped or transported in interstate or
foreign commerce, and the possession of a firearm or ammunition
was shipped or transported or received, as the case may be, by such
person in violation of this Act.

Wherefore, the case is remanded to the trial court for rearraignment.


(PUNO, J.)

The Government seems to argue that there are two alternatives tests
of the validity of a presumption created by statute. The first is that
there be a rational connection between the facts proved and the fact
presumed; the second, that of comparative convenience of producing
evidence of the ultimate fact. These are not independent tests, but the
first is controlling, and the second but a corollary.

PRESUMPTIONS OF INNOCENT AND GUILT


The Constitution presumes innocence but this not necessarily
incompatible with the presumption of guilt arising from facts and
circumstances proven and from which culpability could be inferred.
When it comes to presumptions, there must be a rational connection
between the facts proven and the ultimate facts presumed so that the
inference of one from proof of the other is not irrational and arbitrary
because of lack of connection between the two in common experience.
As the Supreme Court reminded lower courts: The courts,
nonetheless, are advised to take caution in applying the presumption of
regularity. It should not by itself prevail over the presumption of
innocence and the constitutionally-protected rights of the individual.
The constitutional presumption of innocence can only be overturned by
competent and credible proof and never by mere disputable
presumptions.

Presumptions of Innocence and Guilt


The Constitution presumes innocence but this is not necessarily
incompatible with a presumption of guilt arising from the facts
and circumstances proven and from which culpability could be
inferred
When it comes to presumptions, there must be a rational
connection between the facts proven and the ultimate facts
presumed so that the inference of one from proof of the other
is not irrational and arbitrary because of lack of connection
between the two in common experience
The constitutional presumption of innocence can only be
overturned by competent and credible proof and never by
mere disputable presumptions
TOT V. UNITED STATES
319 U.S. 463, 87 L Ed 1519, 63 Ct 1241 (1943)
Facts: The two accused in these cases were prosecuted under section
2(f) of the Federal Firearms Act.
In No. 569, Tot was convicted upon an indictment which charged that
he, having been previously convicted of two crimes of violence, a
burglary and an assault and battery, with intent to beat, wound, and illtreat, at Newark, New Jersey, knowingly, unlawfully, and feloniously
received a described firearm which had been shipped and transported
in interstate commerce to the said City of Newark. The Circuit Court of
Appeals affirmed the judgment.
In the other case, No. 636, Delia the respondent, was convicted upon
two counts. The first alleged that, he was a person previously convicted
of a crime of violence robbery while armed and that he received

Ruling: These cases involve the construction and validity of section 2(f)
of the Federal Firearms Act, which is:

Doubtless the defendants in these cases knew better than anyone else
whether they acquired the firearms or ammunition in interstate
commerce. It would, therefore, be a convenience to the Government to
rely upon the presumption and cast on the defendants the burden of
coming forward with evidence to rebut it. (Justice Roberts)
HIZON V. COURT OF APPEALS
265 SCRA 517 (1996)
FACTS: In response to the reports of illegal fishing operations in the
coastal waters received by the Philippine National Police Maritime
Command of Puerto Princesa City, Palawan, the city mayor organized
Task Force Bantay Dagat to assist the police in the detection and
apprehension of violations of the laws on fishing. There was a report
that a boat and several small craft were fishing by muro ami within
the shoreline of Puerto Princesa. The police proceeded to the area and
found several men fishing in motorized sampans and a big fishing boat.
They boarded and inspected the boat. The police saw two foreigners
having only photocopied passports and discovered a large aquarium full
of live lapu-lapu and assorted fish. The license of the boat and its
fishermen were in order, but nonetheless they were brought to Puerto
Princesa for further investigation. The crew and fishermen were
charged with several violations including conducting fishing operations
without mayors permit, employing excess fishermen on board, and
having two Hongkong nationals on board without original passports.
The following day, the police directed the boat captain to get random
samples of fish for laboratory examination. It was found that they
contained sodium cyanide. The PNP Maritime Command of Puerto
Princesa City filed the complaint against the owner and operator
Richard Hizon, the boat captain, Silverio Gargar, the boat engineer,
Ernesto Andaya, two other crew members, the two Hongkong nationals
and 23 fishermen for the offense of illegal fishing with the use of
obnoxious or poisonous substance penalized under Sections 33 and 38
of P.D. No. 704, the Fisheries Decree of 1975. After trial, they were
found guilty and affirmed by the Court of Appeals.
On appeal to the Supreme Court, petitioners question the admissibility
of the evidence which were seized without warrant, and the appellates
courts failure to hold that the statutory presumption of guilt under
Section 33 of P.D. 704 cannot prevail over the constitutional
presumption of innocence.
Ruling: Given the evidence admitted by the trial court, the next
question now is whether petitioners are guilty of the offense of illegal
fishing with the use of poisonous substances. Again, the petitioners,
joined by the Solicitor General, submit that the prosecution evidence
cannot convict them. SC agreed.
The offense of illegal fishing is committed when a person catches, takes
or gathers or causes to be caught, taken or gathered fish, fishery or
110 | P

LATON

aquatic products in Philippine waters with the use of explosives,


electricity, obnoxious or poisonous substances. The law creates a
presumption that illegal fishing has been committed when: (a)
explosives, obnoxious or poisonous substances or equipment or device
for electric fishing in a fishing are found in a fishing boat or in the
possession of a fisher-man; or (b) when fish caught or killed with the
use of explosives, obnoxious or poisonous substances or by electricity
are found in a fishing boat. Under these instances, the boat owner,
operator or fishermen are presumed to have engaged in illegal fishing.
The validity of laws establishing presumptions in criminal cases is a
settled matter.
The prosecution failed to explain the contradictory findings on the fish
samples and this omission raises a reasonable doubt that the one ton of
fishes in the cage were caught with the use of sodium cyanide. The
apprehending officers who boarded and searched the boat did not find
any sodium cyanide nor any poisonous or obnoxious substance. The
only basis for the charge of fishing with poisonous substance is the
result of the first NBI laboratory test on the four fish specimens.
Petitioners are acquitted.
People v. Godoy
250 SCRA 676 (1995)
Accused-appellant Danny Godoy, a married high school teacher, was
charged in two separate informations with rape and kidnapping with
serious illegal detention of Mia Taha allegedly committed in January
1994. The complainant testified as to how the accused allegedly raped
her several times and how she was kept for three days and nights in a
hotel. Godoy denied the alleged rape and kidnapping. He also
presented evidence that during his detention following Mia`s filing of a
complaint, Mia`s cousin delivered to him on different occasions two
letters from Mia. Said letters reaffirmed her love for him. After the trial,
the court rendered judgement finding appellant guilty of the crimes and
sentenced him to the maximum penalty in both cases
It frequently happens that in a particular case two or more
presumptions are involved. Sometimes the presumptions conflict, one
tending to demonstrate the guilt of the accused and the other his
innocence. In such case, it is necessary to examine the basis for each
presumption and determine what logical or social basis exists for each
presumption, and then determine which should be regarded as the
more important and entitled to prevail over the other. It must however
be remembered that the existence of a presumption indicating guilt
does not in itself destroy the presumption against innocence unless the
inculpating presumption together with all of the evidence, or the lack
thereof, is sufficient to overcome the presumption of innocence by
proving the defendants guilt beyond a reasonable doubt. Until the
defendants guilt is shown in this manner, the presumption of
innocence continues.The presumption of innocence is founded upon
the first principles of justice, and it is not a mere form but a substantial
part of the law. It is not overcome by mere suspicion or conjecture, a
probability that the defendant committed the crime, nor by the fact
that he had the opportunity to do so. Its purpose is to balance the
scales in what would otherwise be an uneven contest between the lone
individual pitted against the People and all the resources at their
command. Its inexorable mandate is that, for all the authority and
influence of the prosecution, the accused must be acquitted and set
free if his guilt cannot be proved beyond the whisper of a doubt. This is
in consonance with the rule that conflicts in evidence must be resolved
upon the theory of innocence rather than upon a theory of guilt when it
is possible to do so,
WHEREFORE, the judgement appealed from is hereby REVERSED and
SET ASIDE, and accused-appellant DANNY GODOY is hereby ACQUITTED
of the crimes.

Equipoise Rule

prosecution and the accused, the latter should be acquitted.


This is the so-called equipoise rule. Under this rule, where the
evidence on an issue of fact is in equipoise, or there is doubt on
which side the evidence preponderates, the party having the
burden of proof loses.
Right to Be Heard and to Counsel
The right to be heard would be a farce if it did not include the
right to counsel. This right to have assistance of a lawyer is
principally designed to afford an accused an opportunity to
present his side as well as to protect his interests
Right to Be Heard
Basic and fundamental to the idea of due process is the
opportunity to be heard in defense of oneself before he is
deprived of his life, liberty or property
Rock v. Arkansas
483 U.S. 44, 97 L Ed 2d 37, 107 S Ct 2704 (1987)
Rock got into an argument with her abusive husband. She called the
police, who arrived at the scene to find the husband dead, and Rock
holding a gun, completely hysterical.Rock professed that she could not
remember what happened. Her attorney took her to a hypnotist, and
under hypnosis, she stated that she was holding the gun, but never
actually pulled the trigger. A gun expert was brought in, and he found
that the gun was defective and prone to firing when knocked around.
At trial, Rock attempted to introduce the hypnotically-derived
testimony.The prosecution objected on the grounds that the testimony
was inadmissible.The prosecution argued that a defendant's testimony
is limited to matters that they can prove were remembered before
hypnosis. The Trial Judge excluded the testimony, but allowed the gun
expert's analysis into evidence. Arkansas has a State law that excluded
hypnotically-derived testimony on the grounds that it was inherently
unreliable.
The Trial Court convicted Rock of manslaughter. She appealed. The
Arkansas Supreme Court affirmed. Rock appealed.Rock argued that the
limitations on her testimony violated her right to present a defense.
However, the Arkansas Supreme Court found that "the dangers of
admitting this kind of testimony far outweigh whatever probative value
it may have."
The US Supreme Court reversed and remanded for a new trial.
The US Supreme Court found that under the 6th Amendment a
defendant has a right to testify in their own defense. The Court found
that the Arkansas Supreme Court failed to perform a constitutional
analysis.While the US Supreme Court noted that hypnotically-derived
testimony is often incorrect, it isn't always incorrect, and so Arkansas'
absolute bar to hypnotically-derived testimony was an impermissible
and arbitrary violation of the 6th Amendment right to testify in your
own defense.The Court noted that in this case, the gun expert's findings
corroborated Rock's hypnotically-derived memory, which is a good
argument for admissibility.
In a dissent, it was argued that an individual's right to represent
evidence is always subject to reasonable restrictions and that the right
to present evidence is not absolute. There is a convincing argument
that hypnotically-derived testimony is inherently unreliable, and
therefore it is reasonable for Arkansas to bar it from admissibility.

Right to Assistance of Counsel


The right to be heard may not really be of much help if the
accused himself were the only one who managed his own
defense. He is not steeped in the intricacies of criminal
courtroom practice and procedure, not to speak of the nuances
and other complexities in the law itself

With the presumption of innocence favouring the accused, it


follows that if the evidence is equally balanced between the
111 | P

LATON

It is said that a little learning is a dangerous thing; and he who


acts as his own lawyer has a fool for a client (In Re: Joaquin T.
Borromeo, 241 SCRA 405 [1995])
It is likewise noteworthy that while the right to assistance of
counsel in regard to custodial investigation may basically be to
help the suspect in keeping intact his right to remain silent so
as not to incriminate himself, the right to counsel as part of the
trial proper is meant to assist the accused get his version of the
story across for consideration and appreciation by the court
It is also elementary that the accused himself has the primary
right to choose his own counsel. However, considering the
States and the offended partys right to speedy and adequate
justice, the court may restrict his option to retain a private
counsel if the accused insists on an attorney he cannot afford,
or if the chosen counsel is not a member of the bar, or if the
attorney declines to represent the accused for a valid reason
In any event, the right guaranteed by the Constitution is to an
effective counsel, not necessarily an intelligent one
In regard to line-ups, while, as discussed in Chapter 14 on rights
of the suspects, there is no need for giving the Miranda
warning before conducting such identification process, it has
been held in the United States that counsel should be present
in such a proceeding, as the same constitutes a critical stage
in the criminal prosecution, specially in regard to the possibility
of suggestiveness
In assessing the effectiveness of counsels assistance, the
Strickland standard...is too stringent for application in
Philippine judicial setting. Strickland only seeks to ensure that
the adversarial testing process is present in a case by requiring
that the assistance rendered by counsel be effective. The
presence of an adversarial testing process, in other words,
ensues that the trial is fair by according the accused due
process through the effective assistance of counsel
UNITED STATES v. ASH
413 U.S. 300
A robbery was committed at a bank by a man wearing a stocking mask,
waiving a pistol, and another entered, gathering the money. An
informer told authorities that he had discussed the robbery with Ash
and photos, including one of Ash, were shown to four witnesses. All
four made uncertain identifications. In preparation for trial, the
witnesses were shown a photographic display to determine whether
the witnesses could make in-court identifications.
The Court of Appeals held that Ashs right to counsel was violated when
his attorney was not given the opportunity to be present at the
photographic displays.
Did the Court of Appeals err in finding that Ashs Sixth Amendment
rights had been violated?
Yes. The Court found that Wade recognized times where the
subsequent trial would cure a one-sided confrontation between
prosecuting authorities and an uncounseled defendant. Such stages
were not critical. Since the accused himself is not present at the time
of the photographic display, no liability arises that the accused might be
misled by his lack of familiarity with the law or overpowered by his
professional adversary. We are not persuaded that the risks inherent in
the use of photographic displays are so pernicious that an extraordinary
system of safeguards is required.
The Court reversed and remanded the judgment of the Court of

Appeals.
SAYSON v. PEOPLE
166 SCRA 680 (1988)
In March 1972, petitioner was charged with the crime of attempted
Estafa through Falsification of a Commercial Document. When
arraigned in December 1972, he pleaded not guilty. In October 1974,
after several postponements, the prosecution rested its case. At the
hearing of 9 December 1974, when the defense was scheduled to
present its evidence, only the petitioner appeared. He said that his
counsel had another case in a different court. In the morning of the said
day, his lawyer also sent a telegram to the court requesting cancellation
of the hearing because he was sick. The court denied the motion for
postponement and the case was considered submitted for decision
without petitioners evidence. The trial court rendered judgment in
January 1975, finding the accused guilty. The Court of Appeals affirmed
but modified the penalty by imposing a lower imprisonment term and
eliminating the fine.
Here, the trial court had been liberal in granting the postponements
secured by the petitioner himself, at the same time admonishing the
latter to be ready with his present counsel or another counsel [Original
Records, p.430.] Notwithstanding this admonition, the petitioner kept
on attending the hearings without securing another lawyer to
substitute his present counsel who was constantly absent during the
hearings. Still, as admitted by petitioner in his memorandum, the trial
court, at the December 9, 1974 hearing, allowed him to look for a
lawyer but no one was available at the time [Rollo, p.94.] These steps
undertaken by the trial court removes any doubt that its order was
tainted with grave abuse of discretion.
People v. Santoclides, Jr.
321 SCRA 310 (1999)
Appellant was charged with and found guilty of the crime of rape. On
appeal, he assigns as one of the errors his being represented at trial by
a person not authorized to practice law, amounting to denial of due
process.
W/N the misrepresentation of his counsel not authorized to practice
would amount to deprivation of his right to counsel and thus result to
his acquittal.
Judgment is SET ASIDE; case is REMANDED to the trial court for new
trial.
When an accused was represented by not a member of the Philippine
Bar during trial, the judgment should be set aside and case must
remanded to the trial court for a new trial. The person who
misrepresents himself as a lawyer shall be held liable for indirect
contempt.
The right to counsel is of such primordial importance that even if an
accused was represented by three successive counsels from PAO, the
court has ordered the remand of a rape case when it found that
appellant was not properly and effectively accorded the right to
counsel.
The presence and participation of counsel in criminal proceedings
should never take lightly. Even the most intelligent or educated man
may have no skill in the science of the law, particularly on rules of
procedure and without a counsel he may convicted not because he is
guilty but because he does not know how to establish his innocence.
The right of an accused to counsel is guaranteed to minimize the
imbalance in the adversarial system where the accused is pitted against
the awesome prosecutory machinery of the State. Such a right
proceeds from the fundamental principle of due process which basically
means that a person should be heard before being condemned.
Jurisprudence has also held that the right to practice law is not a
natural right or constitutional right but is in the nature of a privilege or
franchise. It is limited to persons of good moral character with special
112 | P

LATON

qualifications duly ascertained and certified. The right does not


presuppose in its possessor integrity, legal standing and attainment, bit
also the exercise of a special privilege, highly personal and partaking of
the nature of a public trust.
People v. Liwanag
363 SCRA 62 (2001)
Complainant Hernandez was on her way home at around 1:00 am,
when she was offered by tricycle terminal driver Vendibil a special
trip. She would be brought about to leave, when Simbulan and
Liwanag also rode the same tricycle behind the driver. Then, Liwanag
then entered the sidecar and sat beside the complainant, pointed an
instrument at the side of her neck and declared hold-up. The
complainant told that she only has P60,00 on her pocket. Liwanag
told her since they cant get anything from her, she would submit
herself to them. The malefactors succeeded on raping her. After
satisfying their lust, they talked of killing her. She pleaded for her life
and in desperation, offered money in exchange for her life. From initial
demand of P10,000, they finally agreed on P2,000, and instructed
her to deliver the money at Guadalupe, Makati at 11:30 am the same
morning. After she was released, she went home and told the whole
incident to her mother and proceeded to the police station, to devise
an entrapment operation. The police succeeded on arresting the
suspects and thus accused of highway robbery with multiple rape.
During arraignment, all the suspects pleaded not guilty on the crime
charged. Simbulan and Vendabil were earlier released on recognizance,
but then re-arrested for failure to appear during the trial. The trial
convicted Liwanag. On appeal, he contends, among others, that he was
deprived of his constitutional right to effective and competent counsel.
He claims that from the time he was arrested up to the time of his
conviction, he was deprived of his other constitutional rights,
particularly his right to be secure in his person against unreasonable
search and seizures, his right to preliminary investigation, and his right
to bail. In addition, he claims that the assistance extended to him by his
former counsel was ineffective to the extent that the private
complainant and two prosecution witnesses were hardly crossexamined, while the medico-legal officer was not cross-examined at all.
He changed counsels three times from the time of the investigation
until his trial.
W/N he was denied his constitutional guaranteed right to be heard by
himself and counsel.
There was no dispute that accused-appellant was provided with
counsel de officio who assisted him during the arraignment and
conducted the cross-examination of all prosecution witnesses as well as
his direct examination. Thereafter, from the time of he was crossexamined up to the presentation of other defense witnesses; he was
assisted by a counsel of his choice.
Jurisprudence defined the meaning of effective counsel only in the
light of Art. III, Sec. 12(1) of the Constitution, which refers to the right
of persons under custodial investigation. The rationale was elucidated
on People v. Lucero:
The 1987 Constitution requires that a person under investigation for
commission of a crime should be provided with counsel. We have
constitutionalized the right to counsel because of our hostility against
the use of duress and other undue influence in extracting confessions
from a suspect. Force and fraud tarnish confessions and render them
inadmissible. In providing for said right, this Court has held in the same
case that when the Constitution requires right to counsel, it did not
mean any kind of counsel but effective and vigilant counsel. The
requirements of effectiveness and vigilance of counsel during the stage
before arraignment were for the purposes of guarding against the use
of duress and other undue influence in extracting confessions which
may taint them and render them inadmissible.
Art. III, sec. 14(2) of the 1987 Constitution requires that the accused
shall enjoy the right to be heard by himself and counsel. The ratio was
explained in People v. Holgado:

In essence, the right to be heard by counsel simply refers to the right to


be assisted by counsel for the purpose of ensuring that an accused is
not denied the collateral right to due process, a fundamental right
which cannot be waived by an accused. The underlying basis for due
process is the concept of fairness, without which there can be no
justice. In other words, there can be no due process accorded an
accused if he is not given the right to be heard through counsel or
assisted by counsel. It follows that in order to be heard, and therefore
be accorded due process, the assistance given by counsel must be
effective as implied in the rationale of Art. III, Sec. 14(2). In this sense,
this court subscribes to American jurisprudence when it held that the
right of an accused to counsel is beyond question of fundamental right.
Without counsel, the right to a fair trial itself would be of little
consequence, for it is through counsel that the accused secures his
other rights. In other words, the right to counsel is right to effective
assistance of counsel
Contention of the accused that his counsel de officio failed to safeguard
his right to be secure against unreasonable search and seizures. He was
arrested without warrant and his counsel should contested. Argument
is not well-taken. It was reiterated in People v. Costelo:
Appellants failure to quash the information, his participation in the
trial and presenting evidence in his behalf, placed him in estoppel to
make such challenge. He has patently waived any objection or
irregularities and is deemed as having submitted himself to the
jurisdiction of the court. It should be noted that the legality of arrest
affects only jurisdiction of the court over the person of the accused.
Consequently, if objection on such ground is waived, the illegality of the
arrest is not sufficient reason foe setting aside an otherwise valid
judgment rendered after the trial, free from error. The technicality
cannot render the subsequent proceedings void and deprived the State
of its right to convict the guilty when the facts on the record point to the
culpability of the accused.
Any objection involving a warrant of arrest must be made before he
enters his plea, otherwise the objection is deemed waived.
Contention of the accused that he was deprived of right to bail. If his
counsel should be effective, he would filed for proper motion. This is
without merit. It was ruled in People v. Manes:
The issue of bail has been rendered academic by the conviction of the
accused. When an accused is charged with capital offense, or an
offense punishable by Reclusion Perpetua, or life imprisonment or
death, and evidence of guilt is strong, bail must be denied, as it is
neither a matter of right nor discretion.
The contention of the accused that his counsels show ineffectiveness is
not convincing. As noted by the Office of the Solicitor General:
The pertinent transcripts of stenographic notes would show that
appellants counsel de officio, Atty. Uy, cross-examined the private
complainant extensively as well as two other prosecution witnesses.
That said counsel opted not to cross-examine the prosecution expert
witness, Dr. Nario, is of no moment because said witness merely
explained in court her findings and conclusions that she had arrived at
after conducting the medical examination on the private complainant.
In fact, at one point, Atty. Uy raised an objection to the private
prosecutors question on how private complainants genital injuries
were sustained for being incompetent to answer, which objection was
impliedly sustained by the trial judge.
The accused invoke the Strickland standard which is too stringent for
application in Philippine judicial setting. The standard seeks to ensure
that the adversarial testing process is present in a case by requiring that
the assistance rendered by counsel be effective. The presence of an
adversarial testing process, in other words, ensures that the trial is fair
by according the accused due process through effective assistance of
counsel.
Fairness is likewise the object of Art. III, Sec. 14(2) of the Philippine
Constitution, assistance afforded by counsel to an accused in light of
113 | P

LATON

the Philippine constitutional requirement need only be in accordance


with the pertinent provisions of Rules of Court, Code of Professional
Responsibility and Canons of Professional Ethics.

violated when upon the acquittal of the former crime does not
preclude him convicted on the crime of attempted estafa through
falsification of official and commercial documents.

The presumption that counsels performance was reasonable, as long


as the trial was fair, accorded the accused of due process by means of
effective assistance of a counsel, the constitutional requirement that an
accused shall have the right to be heard by himself and counsel is
satisfied. The only instance when the quality of counsels assistance
cane be questioned is when an accused is deprived of due process.

1st issue:

Based on the findings, accused-appellant was not deprived of his right


to heard by himself and counsel.

Contention without merit. Such right, an ancient bulwark of the


liberties of men, has its origin in the Bill of Rights which the people of
Great Britain demanded and received from the Prince and Princess of
Orange on Feb. 13, 1688. it was adopted by the US Constitution and
extended to Philippines by Jones Law and enshrined in 1935, 1973 and
1987 Constitution. Has its objectives:
1.

Right to Be Informed
An element of fair play in the prosecution of any person would
be the requirement that he be adequately informed of what he
is really charged with, of what acts or omission he may have
done or failed to do which amounted to a crime
General Considerations
Aside from providing an accused with information as to his
alleged criminal misconduct, the right to be informed also
affords him an opportunity to set up any defense that might
have been brought about by a prior acquittal, conviction, or
earlier proceeding arising from the same facts or offense. At
the same time, it also informs the court of the facts alleged so
that such tribunal may determine if they are sufficient in law to
support a conviction
Relevant herein is the so-called variance doctrine under which,
in spite of the difference between the crime that was charged
and that which was eventually proved, the accused still may be
convicted of whatever offense that was proved even if not
specifically set pout in the Information provided it was included
in what was charged

2.
3.

To furnish the accused with such description of the charge


against him as will enable him to make defense.
To avail himself of his conviction or acquittal for protection
against further prosecution for the same cause.
To inform the court pf the facts alleged, so that it may
decide whether they are sufficient in law to support a
conviction, if one should had.

Rules of Court has prescribed the appropriate rules. It was provided on


Sec. 4, Rule 120 of Rules of Court.
What determines the real nature and cause of accusation against an
accused is the actual recital of facts stated in the information or
complaint and not the caption or preamble of the information or
complaint nor the specification of the provision of law alleged to have
been violated, they being conclusions of law. An incorrect caption is not
a fatal mistake.
2nd issue:
There is no sufficient circumstantial evidence to prove conspiracy
between the petitioner and Catre to commit the crime. Neither there is
active participation in the commission of the crime. The concordant and
cumulative effect of the acts of the petitioner as proven by the
prosecutions evidence fails to satisfy the requirements of Sec. 4, Rule
133 of the Rules of Court.
Motion for reconsideration is granted. Acquitted.

Pecho v. People
262 SCRA 518 (1996)

Right to be Informed and the Death penalty


Petitioner was charged with violation of a special law, RA 3019 in
relation to an attempted underdeclaration of items to be taken out
from Customs, automotive diesel engines were attempted to be passed
off as agricultural disc blades and irrigation water pumps. Petitioner
was convicted by Sandiganbayan. On appeal, Supreme Court modified
the judgment by holding him guilty of the complex crime of attempted
estafa through falsification of official and commercial documents. Court
said that the special law penalizes only consummated offenses and the
offense charge was not consummated. Petitioner files for motion to
reconsideration that since he has been acquitted of violation of the
special law, he could not be convicted anymore a crime punishable by
the RPC, a general law, otherwise the constitutional provision on
double jeopardy would be violated. The acquittal of the crime charged
precludes conviction for attempted estafa through falsification of
official and commercial documents. The Assistant Solicitor General and
a Solicitor disagreed with the petitioner of his asserted double
jeopardy, considering that no new information for estafa through
falsification of public document was filed against the petitioner, but
agrees for his plea of acquittal of the latter case for another ground,
insufficiency of evidence. When the court directed the Sol Gen to
inform the court whether he agreed or not, to the recommendation of
the Asst. Sol Gen, he not only expressly agree, but added that there was
no overt act conclusively attributed to petitioner.
W/N the evidence adduced by the prosecution had established beyond
reasonable doubt the guilt of the petitioner.
W/N the petitioners constitutional right against double jeopardy was

The imperative to provide the accused knowledge of what he is


being charged with, together with the possible consequence of
conviction, carries more urgency when the crime charged is
capital. The punishment being irrevocable and irreversible, the
Court has to be careful and certain that the accused really
understood what he was told, and that if ever he pleaded
guilty, then it should have been under the safeguards that
would provide the assurance that he really knew what he was
responding to
People v. Alicando
251 SCRA 293 (1995)
Appellant was accused of raping and killing a four year old girl. At
arraignment, he pleaded guilty. After his plea of guilt, the trial court
ordered the prosecution to present its evidence. It also set the case for
reception of evidence for the appellant, if he so desired. The evidence
shows that after a witness pointed to the appellant as one who
committed the crime, he was arrested and interrogated by the police.
He verbally confessed his guilt without the assistance of counsel. On
the basis of this uncounseled verbal confession and follow up
interrogations, the police came to know and recovered from appellants
house, the victims green slippers, a pair of gold earrings, a buri mat, a
stained pillow, and a stained t-shirt all of which were presented as
evidence for the prosecution. After trial, he was convicted and
114 | P

LATON

sentenced to death. In his brief, appellant assails the decision of the


trial court as a travesty of justice.

Concepcion, this court held that no valid judgment can be rendered


upon an invalid arraignment.

We find the decision of the trial court sentencing the appellant to death
is shot full of errors, both substantive and procedural. The conviction is
based on an amalgam of inadmissible and incredible evidence
supported by scoliotic logic.

The arraignment is void, the judgment is also void. There is no


philosophy that allows the State to kill without any semblance of
fairness and justice.

1.

The arraignment was null and void.

Section1a of rule 116 on arraignment


Section 1, Arraignment and plea, how made.(a) The accused must be arraigned before the court where
the complaint or information has been filed or
assigned for trial. The arraignment must be made in
open court by the judge or clerk by furnishing the
accused a copy of the complaint or information with
the list of witnesses, reading the same in the language
or dialect known to him and asking him whether he
plead guilty or not guilty. The prosecutor may,
however, call at the trial witness other than those
named in the complaint or information.
Records do not reveal that the information was read against the
appellant in the language or dialect known to him. The information was
written in English, it is unknown if appellant understands English nor is
it known what dialect is understood by the appellant.
Note: after reading the information to the accused, the accused pleads
guilty.
2.

The plea of guilt is likewise null and void.

Section 3 of rule 116


Section 3. Plea of guilty to capital offense; reception of evidence.When the accused pleads guilty to a capital offense, the court shall
conduct a searching inquiry into the voluntariness and full
comprehension of the consequences of his plea and require the
prosecution to prove his guilt and precise degree of culpability. The
accused may also present evidence in his behalf.
The bottom line of the rule is that the plea of guilt must be based on a
free and informed judgment. Thus the questions should focus on:
1. Voluntariness of the plea
2. Full comprehension of the consequences of the plea.
Neither was satisfied during the searching inquiry. The records do not
reveal any information about the personality profile of the appellant
which can serve as a trustworthy index of his capacity to give a free and
informed plea of guilt. The age, socio-economic status and educational
background were not plumbed by the trial court. The questions were
framed in English yet there is no inkling that he had a nodding
acquaintance of English. It must also be noted that the trial court did
not bother to explain to the appellant the essential elements of the
crime rape with homicide.
Also the trial court did not ask when he was arrested, by whom, how
and where he was interrogated, whether he was medically examined
before and after the examination.
It warned the appellant that he would get the mandatory penalty of
death without explaining the word mandatory. It did not inform the
appellant of the indemnity he has to pay for the death of the victim. It
cautioned the appellant that there will be some effects to his civil rights
without telling him what those effects are.
The appellants plea of guilt is void and the trial court erred in using it
to sentence the accused to death.
3.

Some prosecution evidence, offered independently of the


plea of guilt of the appellant were inadmissible yet, were
considered by the trial court in convicting the appellant

In binabay vs people, et al., a ponencia of Mr. Chief Justice R.

People V. Ramos
296 SCRA 559 (1998)
Appellant was accused of having raped Elizabeth T. Ramos, 14-year old
minor through force, violence and intimidation.
A re-reading of the accusatory portion of the information reveals that
appellant was charged with rape on its simple form with the additional
allegation that the victim was 14 years old. Such act is punishable by
reclusioin perpetua under article 335 of rpc .
In RA 7659 if the offender is the parent of the victim, the penalty is
death.
The trial court arrive at the conclusion that the penalty for that kind of
rape is reclusion perpetua to death and considering the relationship as
a generic aggravating circumstance, imposed him a penalty of death
(the higher of the two indivisible penalties.
The case of people vs bayot gave the reminder that a qualifying
circumstance or an inherent aggravating circumstance should not be
mistaken for a generic aggravating circumstance. The court
distinguishes the two by stating that a generic aggravating
circumstance, not offset by any mitigating circumstance, increases the
penalty which should be imposed to the maximum period without
exceeding the limit prescribed by law, while a qualifying circumstance
not only gives the crime its proper and exclusive name but also imposes
the author thereof no other penalty but that specially prescribed by law
for the said crime.
Held: since the relationship was in a nature of a qualifying
circumstance, and not merely a generic aggravating circumstance, it
must be alleged in the information.

In People v. Aquino, the Court said that the words aggravating/


qualifying, qualifying, qualified by, aggravating, or
aggravated by need not be expressly stated as long as the
particular attendant circumstances are specified in the
Information
Right to be Informed and Aggravating Circumstances
Under the Revised Rules of Criminal Procedure which took
effect on 1 December 2000, it is now required that aggravating
circumstance, whether generic or qualifying must be alleged in
the Information or Complaint in order to be appreciated
Right to Speedy Trial
To ensure that justice be done, it is necessary that there be no
unwarranted, vexatious, capricious, and oppressive delays in
the criminal proceedings. Otherwise, it would be animate the
familiar dictum that justice delayed is justice denied
Delay in proceedings not only could give rise to denial of justice
in its general application. It could also work undue vexation and
oppression upon the accused and other characters involved,
from the government itself to the private offended parties
If he needs to be declared innocent and free, then he should
have it at the earliest opportunity instead of being harassed by
prolonging his hardships. If he is to be convicted, then it would
115 | P

LATON

also be to his advantage that the matter be determined soonest


in order that he may be able to take whatever legal steps are
available to him, or to enable him to simply accept the verdict
instead of being agonized by a sword of Damocles hanging over
his head

trial. He also argues that the CA erred in not sustaining the submission
of the Solicitor General for his acquittal. Required to comment, the
Office of the Solicitor General adopted its Motion and Manifestation
filed before the CA and recommended the grant of the petition for
Certiorari averring that the findings and conclusions of the CA are not
supported by substantial evidence.

Further, even as the right to a speedy trial is guaranteed to the


accused, the same should not be utilized to deprive the State of
a reasonable opportunity of fairly indicting and prosecuting
criminals

In his first assignment of error, he claims lack of due process due to the
unusual speed of the trial court.
Applicable to this case is General Order no. 39 amending General Order
no 12, datedSept.30, 1972.

Conde vs Rivera
45 Phil 650 (1924)

23. Crimes where the offended party is a tourist or a transient.

Aurelia Conde, formerly a municipal midwife in Lucena, Tayabas. She


was forced to respond to no less the five information for various crimes
and misdemeanors, and appeared with her witnesses and counsel at
hearings no less than on 8 different occasions only to see the cause
postponed.

The civil court shall have concurrent jurisdiction with the military
tribunals over the said crimes, provided that civil courts shall dispose of
such cases w/in 24 hrs after the filing thereof by the arresting officer.
The court or tribunal that first assumes jurisdiction shall exercise
jurisdiction shall exercise jurisdiction to the exclusion of all others.

She then filed this present petition for mandamus and prohibition.

He was not denied due process, the trial court merely deferred such
conference till after the prosecution had presented its witnesses. (it is
to be noted further that defense counsel was not totally unprepared for
the trial for it was ready with 2 witnesses when asked by the court).
Moreover, after the prosecution had rested its case, trial was resumed
the next day, thereby giving the defense enough time to prepare for
the presentation of its direct evidence.
The trial was not indicative of inordinate haste. On the contrary, they
should be commended for their punctilious compliance with the explicit
mandate of the law.

Philippine organic and statutory law expressly guarantee that in all


criminal prosecutions the accused shall enjoy the right to speedy trial in
order that if innocent, she may go free, and she has been deprived of
that right in defiance of law. Dismissed from her humble position, and
compelled to dance attendance on courts while investigations and trials
are arbitrarily postponed without her consent, is palpably and openly
unjust to her and a detriment to the public. By the use of reasonable
diligence, the prosecution could have settled upon the appropriation of
the information, could have attended the formal preliminary
examination, and could have prepared the case for trial free from
vexations, capricious and oppressive delays.
We lay down the legal prosecution that, where a prosecuting officer,
without good cause, secures postponements of the trial of the
defendant against his protest beyond a reasonable period of time, as in
the instance for more than a year, the accused is entitled to relief by a
proceeding in mandamus to compel a dismissal of the information, or if
hr be restrained of his liberty, by habeas corpus to obtain his freedom.

That the complainant was not a transient visitor, his having been in
Olongapo for a number of times does not make him any less than a
transient or one whose stay is of uncertain duration. Also,
notwithstanding the number of times he has been to the country, he is
still a tourist, or one who travels from place to place for pleasure and
culture.

Dismissed, cost against respondent fiscal.

Also, on Solicitor Generals position that the guilt was not established
beyond reasonable doubt, the complainant was categorical in his
identification of the petitioner-accused and emphatic as to the latters
direct and active involvement in the robbery.

Magat v. CA
116 SCRA 283 (1982)

DACANAY V. PEOPLE
240 SCRA 490 (1995)

Petitioner was charged with robbery for divesting of his money an


American businessman who went to Olongapo as a tourist.

FACTS : In 1985 , petitioner was the vice-president of the National


Sugar Trading Corporation (NASUTRA) . In 1986, a criminal complaint
for economic sabotage through smuggling, with regard to the
importation of raw sugar by NASUTRA, was filed with the Tanodbayan
against the principal officers of the said corporation including the
petitioner. In 1988, a resolution was issued by Special Prosecutors
Gervacio and Kallos recommending the dismissal of the complaint
against petitioner and his co-accused for lack of sufficient evidence and
the withdrawal of the information filed in court. In 1989, the resolution
was reviewed by special prosecutor Orencia, who recommended its
disapproval and such recommendation was approved by Acting Special
Prosecutor Guerrero and by the Ombudsman. The motion for
reconsideration filed by co-accused Unson and adopted by petitioner
was denied. In 1991, petitioner filed a motion for immediate and
separate trial invoking his constitutional right to a speedy trial.
Respondent People opposed the said motion on the ground that a
separate trial for petitioner would entail a lengthy and repetitious
proceeding . The petitioner filed a motion for reconsideration setting
forth as grounds therefor his advanced age and protection of his
reputation. The Sandiganbayan denied petitioners motion for
reconsideration .

The information was filed July 26 1979 at 3pm, a week after the
incident complained of.
Magat was charged together with four others. The case was
immediately raffled and set for arraignment and trial. Only the
petitioner was arraigned at 4:35 the same afternoon.
After arraignment, it proceeded to trial. Counsel asked for time, an
hour at least, to confer with his client, the fiscal said he had no
objection to having a conference between counsel and his witness
provided it be done after he would have already submitted the Peoples
case . The court said it would allow counsel to consult his client before,
and from time to time, during the cross-examination.
The following day, trial was resumed at 830 am. At 1135, judgement of
conviction was promulgated.
He appealed to CA. CA promulgated its decision August 21 1981
affirming but reduced the penalty.
In his appeal to the SC, petitioner raises the issues of having been
denied due process, specifically the right to be heard, including ample
opportunity for the accused and the counsel to confer and prepare for

ISSUE: whether or not petitioner is entitled to a separate trial.


DECISION: The respondents are ORDERED to proceed with the trial of
petitioner in Criminal Case No. 11957, separately if it need be.
116 | P

LATON

REASON: the main objective of respondent is that such a procedure


would entail a repetitive presentation of evidence. But the resulting
inconvenience and expense on the part of the Government cannot be
given preference over the right to speedy trial and the protection to a
persons life, liberty or property accorded by the constitution. A
separate trial is in consonance with the right of an accused to a speedy
trial as guaranteed to him by the 1987 constitution under Art. III sec.
14(2). The primordial purpose of this constitutional right is to prevent
the oppression of an accused by delaying criminal prosecution for an
indefinite period of time. It has been eight years since the information
against petitioner was filed, but the case against him has yet to be
tried. The long delay has clearly prejudiced petitioner, who is now more
than seventy-three years of age.
GUERRERO V. CA
257 SCRA 703 (1996)
FACTS: In 1971, an information for triple homicide through reckless
imprudence was filed against petitioner before the Court of First
Instance in Caloocan, alleging that on or about May 13 1969, the
petitioner, then the pilot of a non-commercial aircraft, without taking
the necessary care and precaution to avoid accidents or injuries to
persons, and without ascertaining as to whether the quantity of fuel in
the tanks of his aircraft was sufficient for the flight, flew said aircraft
with four passengers on board. When the engine quit twice indicating
that there was no more fuel, accused made an emergency landing and
in doing so the aircrafts landing gear collided with a dike and trees near
a fishpond in Malabon, resulting in the death of three passengers.
Due to several postponements, all filed by the petitioner, the
prosecution was only able to start presenting its evidence in 1972. In
1975, the prosecution finally rested its case while the defense did so in
1978. In 1989, court administrator ordered the re-raffling of the case
from the RTC of Caloocan City to the RTC of Navotas-Malabon which
had jurisdiction over the case. In March 1990, Judge Aquino ordered
the parties to follow-up and complete the transcript of stenographic
notes within 30 days considering that the same was found to be
incomplete. When the parties were not able to complete the transcript
of stenographic notes, the court ordered the retaking of the
testimonies of the witnesses.

the original setting that the accused suddenly became zealous of


safeguarding his right to speedy trial and disposition.
While this court recognizes the right to speedy disposition quite
distinctly from the right to a speedy trial, and although this court has
always zealously espouse protection from oppressive and vexatious
delays not attributable to the party involved, at the same time, we hold
that a partys individual rights should not work against and preclude the
peoples equally important right to public justice. Three people died as
a result of the crash of the airplane that the accused was flying. It
appears to us that the delay in the disposition of the case prejudiced
not just the accused but the people as well. Since the accused has
completely failed to assert his right seasonably and inasmuch as the
respondent judge was not in a postion to dispose of the case on the
merits due to the absence of factual basis, we hold it proper and
equitable to give the parties fair opportunity to obtain substantial
justice in the premises.
PEOPLE V. TEE
395 SCRA 419 (2003)
FACTS: Appellant was charged with violation of the Dangerous Drugs
Act. After his conviction, he raised as one of the issues the several
postponements allowed by the trial court by reason of the repeated
absence of the prosecutions witness in the person of Abratique.
Appellant insists that such lapse on the prosecutions part violated the
speedy trial act of 1998. On record, the trial court found that
prosecution witness Danilo Abratique failed to appear in no less than
twenty hearings. No less than five warrants of arrest were issued
against him to compel him to testify. The NBI agent who supposedly
had him in custody was found guilty of contempt of court for failing to
produce Abratique at said hearings and sanctioned. Nothing on the
record discloses the reason for Abratique aforecited absences.
Appellant now stresses that the failure of Abratique to appear and
testify on twenty hearing dates violated appellants constitutional and
statutory right to a speedy trial.
ISSUE: whether or not failure of Abratique to appear and testify on
twenty hearing dates violated appellants constitutional and statutory
right to a speedy trial.
DECISION: no. his right was not violated

In October 1990 the presiding judge set the retaking on the 24th of the
month, however, on the said date the retaking was reset to Nov. 9 due
to the petitioners failure to appear on the scheduled hearing. On Nov.
7, petitioner filed a motion to dismiss on the ground that his right to
speedy trial has been violated. The judge denied the motion to dismiss
and reconsideration and reset the retaking. The petitioner then filed a
petition for certiorari, prohibition and mandamus with the court of
appeals, anchored on the alleged violation of his right to speedy trial. In
its decision in 1991, the appellate court dismissed the petition and also
denied the motion for resconsideration. Before the SC, petitioner
complains of undue prejudice as a result of the protracted litigation and
of violation of his right to speedy trial.
ISSUE: does the constitutional right to speedy trial include the right to
prompt disposition and judgment
DECISION: the petition is DENIED
REASON: In the determination of whether or not the right to a speedy
trial has been violated and on whether or not a person officially
charged with the administration of justice has violated the speedy
disposition of case guarantee, certain factors may be considered and
balanced against each other such as length of delay, reason for the
delay, assertion of the right or failure to assert it, and prejudice caused
by the delay. In this case, the petitioner merely sat and waited after the
case submitted for resolution in 1979. It was only in 1989 when the
case was re-raffled and only after respondent trial judge ordered the
parties to follow-up and complete the transcript of stenographic notes
that matters started to get moving towards the resolution of the case.
More importantly, it was only after the new trial judge reset the
retaking of the testimonies because of the petitioners absence during

REASON: the speedy trial act of 1998, provides that the trial period for
criminal cases in general shall be 180 days. However, in determining the
right of an accused to speedy trial, courts should do more than a
mathematical computation of the number of postponements of the
scheduled hearings of the case. The right to speedy trial is deemed
violated only when: (1) the proceedings are attended by vexatious,
capricious, and oppressive delays; or (2) when unjustifiable
postponements are asked for and secured; or (3) when without cause
or justifiable motive a long period of time is allowed to elapse without
the party having his case tried.
In the present case, although the absence of prosecution witness
Abratique totaled twenty hearing days, there is no showing that
prosecution capriciously caused Abratiques absences so as to vex or
oppress appellant an deny him his rights. Nor do we find a delay of
twenty hearing days to be an unreasonable length of time. Delay of less
than two months has been found, in fact, to be not an unreasonably
lengthy period of time. Moreover, nothing on the record shows that
appellant objected to inability of the prosecution to produce its
witness. Under the Rules, appellant could have moved the trial court to
require that witness Abratique post bail to ensure that the latter would
testify when required and also he could have moved to have Abratique
found in contempt and duly sanctioned but he did neither. It is a bit too
late in the day for appellant to invoke now his right to speedy trial.
No persuasive reason supports appellants claim that his constitutional
right to speedy trial was violated. One must take into account that a
trial is always subject to postponements and other causes of delay. But
in the absence of a showing that delays were unreasonable and
capricious, the State should not be deprived of a reasonable
117 | P

LATON

opportunity of prosecuting an accused.

Right to Impartial Trial and Tribunal


It is a basic requirement of due process that the judge or
tribunal hearing a case should be impartial. The case would
have to be determined in accordance with the evidence
presented and not by any personal bias or prejudice on the part
of the court. Lady Justice is supposed to be above the
importunities of the parties and blindfolded as well in order
that she can decide cases based strictly on the merits and not
on the personalities of the parties involved
The guiding rule is that a judge must not only render a just,
correct and impartial decision but should also do so in such a
manner as to be free from any suspicion as to his fairness,
impartiality and integrity
PEOPLE V. OPIDA
142 SCRA 295 (1986)
FACTS: several persons ganged up on Fabian Galvan, stoned and hit him
with beer bottles until finally one of the stabbed him to death. The
actual knife-wielder was identified as Mario del Mundo. Nonetheless,
appellants Opida and Marcelo was charged with murder as conspirators
and, after trial, sentence to death. The basis of their conviction by the
trial court was the testimony of two prosecution witnesses, neither of
whom positively said that the accused were at the scene of the crime,
their extrajudicial confessions which were secured without the
assistance of counsel, and corroboration of the allege conspiracy under
the theory of interlocking confessions.
What was striking about this case is the way the trial judge conducted
his interrogation of the two accused and their lone witness, Lilian
Layug. Reading the transcripts, one gathers the impression that the
judge had allied himself with the prosecution to discredit at the outset
the credibility of witnesses for the defense. Opida is a police character,
admittedly a member of the Commando gang and with a string of
convictions for robbery, theft and vagrancy. It is worth noting that the
judge took special interest in his tattoos, required him to remove his
shirt so they could be examined, and even describe them in detail for
the record. The interrogation of Virgilio Marcelo, the other accused,
was conducted almost wholly by the judge who started cross-examining
the witness even before the defense counsel could ask his first
question, and took over from the prosecatuion the task of impeaching
Marcelos credibility. The judge asked him about his drug addiction, his
membership in the Commando gang, his tattoos, his parentage, his
activities, his criminal record all when he was supposed to be under
examination by his own lawyer.

because of the constitutional presumption of innocence. Needless to


stress, this right is available to every accused, whatever his present
circumstances and no matter how dark and repellent his past.
In any event, convictions are based not on the mere appearance of the
accused but on his actual commission of the crime, to be ascertained
with pure objectivity of the true judge who must uphold the law for all
without favor or malice and always with justice. Considering the way
they were tried, we now declare that they should not be detained in jail
in a minute longer. While this is not to say that the accused are not
guilty, it does mean that, because their constitutional rights have been
violated, their guilt, if it exists, has not been established beyond
reasonable doubt and so cannot be pronounced.
Tabuena v. Sandiganbayan
268 SCRA 332 (1997)
In January 1986, President Marcos instructed over the phone, Luis
Tabuena, the general manager of Manila International Airport
Authority (MIAA), to pay directly to the Presidents Office what the
MIAA owed the Philippine National Construction Corporation (PNCC). A
week later, Tabuena received from Mrs. Fe Roa-Gimenez, the private
secretary of the President, the same instruction given by Marcos over
the phone. Tabuena, with the help of co-accused Dabao, assistant
manager of MIAA, then caused the release of 55 million pesos, went to
the office of Gimenez and gave the money. No receipt was issued.
Appellants were charged of malversation. For the prosecution, they
said that there were no outstanding obligations in favour of PNCC at
the time of the disbursement of 55 Million. As for the defense, they
said that they acted in good faith, that they were merely complying
with the Presidents instructions.
Appellants were charged before the Sandiganbayan with malversation.
After trial, they were convicted, the court disregarding their defense of
having acted in good faith.
Held: Good faith is a defense in a prosecution for malversation.
However, what amounted to acquittal is the violation of the accuseds
basic constitutional right to due process.
The SC is strucked by the way Sandiganbayan actively took part in the
questioning of a defense witness and of the accused
themselves.Questions from the court after Tabuenas crossexamination totalled 67 questions, this is more than 5 times the
prosecutors questions, more than double the total of direct
examination and cross examination.

ISSUE: violation of the right to impartial trial and tribunal and


inadmissibility of the extrajudicial confessions of the accused under the
due process guarantee of the constitution

The SC acknowledged the right of a trial judge to question witnesses,


but this is limited to clarificatory questions. In this case, this limitation
was not observed.The cold neutrality of an impartial judge requirement
of due process was certainly denied Tabuena and Peralta when the
court, with its overzealousness, assumed the dual role of magistrate
and advocate.

DECISION: the conviction of Opida and Marcelo is reversed and they


are hereby ordered released immediately.

Marcos v. Sandiganbayan
297 SCRA 95 (1998)

REASON: Given the obvious hostility of the judge toward the defense, it
was inevitable that all the protestations of the accused in this respect
would be, as they in fact were, dismissed. And once the confessions
were admitted, it was easy enough to employ them as corroborating
evidence of the claimed conspiracy among the accused.

Petioner Imelda Marcos was the minister of Human Settlement while


Jose P. Dans, Jr. Was the minister of Transporation and
Communication.The two served as ex officio chairman and vice
chairman of Light Rail Transport Authority. Marcos was also the
chairman of Philippine General Hospital Foundation. The two entered in
a lease agreement, Marcos acting as the chairman of PGHF and Dans as
vice chairman of LRTA, LRTA leased to PGHF a lot at a monthly rental of
102,760 for 25 years.. On june 1984, Marcos entered a sublease
agreement wherein Transnational Construction Corporation
represented by President Ignacio Gimenez, wherein said lessee rented
the same area for 734,000 a month for 25 years. Petitioner and Dans
were indicted in 1992 for conspiring and confederating with each other
in entering into lease agreement alleged to be manifestly and grossly
disadvantageous to the government, in violation of Anti Graft and

The accused are admittedly notorious criminals who were probably


even proud of their membership in the Commando gang even as they
flaunted their tattoos as a badge of notoriety. Nevertheless, they were
entitled to be presumed innocent until the contrary was proved and
had a right not to be held to answer for a criminal offense without due
process of law. The judge disregarded these guarantees and was in fact
all too eager to convict the accused. The scale of justice must hang
equal and, in fact, should even be tipped in favor of the accused

118 | P

LATON

Corrupt Practices Act.


The case was raffled to First Division of Sandiganbayan, with presiding
Justice Francis Garchitorena as Chairman, Justices Balajadia and Atienza
as members. Failing to comply with the requirement of unanimity with
the dissent of Justice Narcisio Atienza, Garchitorena requested to have
a Special Division of 5, designating Justices amores and del Rosario as
additional members. Justice Amores wrote Garchitorena that he be
given 15 days to send his manifestation. However, on the same day, the
two original members of the division of 3 agreed with the opinion of
Justice Atienza. Garchitorena then issued an order dissolving the
division of 5. Said agreement was arrived over lunch in an unnamed
Quezon City restaurant that day among Justices Garchitorena,
Balajadia, Del Rosario, and the other who was not a member of the
Special Division. The three conviceted the petitioner and Dans,JR.
Petitioner appealed to the SC.
Held: The procedural flaws committed by the Sandiganbayan are fatal
to the validity of its decision convicting the petitioner for the
following reasons: That sessions of the Sandiganbayan shall be held in
the principal office, that rules of Sandiganbayan do not allow
unscheduled discussion of cases, that they do not allow informal
discussion of cases, that they do not allow the presence of a nonmember in the deliberation of cases, and that they do not allow
exclusion of a member of a division. These irregularities violated the
right of the petitioner to be tried in a collegial court. Moreover, Justice
Francisico observed the court questions were so numerous which as
per petitioner Dans count totalled 179 compared to prosecutor
Querubins questions which numbered merely 73. More noteworthy is
that the court propounded leading, misleading, and baseless
hypothetical questions rolled into one.
Marcoss motion for reconsideration was granted and was acquitted.
Webb v. People
Petitioners were charged with the crime of rape with homicide for
allegedly raping Carmela Vizconde, killing carmela herself, her mother,
and her sister. The case was raffled to RTC of Paranaque presided by
respondent Tolentino. Webb and his co-accused, Bion, sought the
disqualification of respondent judge. The respondent judge allegedly
told the media that failure to surrender following the issuance of the
warrant of arrest is an indication of guilt. Respondent judge denied the
motion. In September 1995, Biong filed another motion to disqualify
respondent judge on the ground of bias and impartiality. This was
likewise denied by the respondent judge. The petitioners filed separate
petitions for bail. On nov. 9, 1995, petitioners filed a motion to
disqualify the judge due to bias and prejudice, the judge denied the
motion for lack of merit. The respondent judge denied petitioners
petitions for bail.
The present petition principally anchored on the alleged error of the CA
in declaring that no sufficient ground existed for the disqualification of
respondent judge and on the issue of lack of fairness and impartiality
on the part of the respondent judge.
Held: To disqualify a judge on the basis of bias and prejudice, the
movant must prove the same by clear and convincing evidence. This is a
heavy burden and petitioners failed to discharge their burden of proof.
To be disqualifying, the bias and prejudice must be shown to have
stemmed from an extrajudicial sources and result in an opinion on the
merits on some basis other than what the judge learned from his
participation in the case. Extrinsic evidence is required to establish bias,
bad faith, malice or corrupt purpose, in addition to the palpable error
which may be inferred from the decision or order. Although the
decision may seem so erroneous as to raise doubts concerning the
judges integrity, absent extrinsic evidence, the decision itself would be
insufficient to establish a case against the judge. Petioners simply lean
on the alleged series of adverse rulings of the respondent judge which
can be characterized as palpable errors. This is not enough.
Bracy v. Gramley

Right to Public Trial


While it is part of the guarantee of criminal due process that
there be a public trial in order that there be an assurance that
the case may not be railroaded or otherwise unfairly dealt with
in secrecy, too much publicity may also create its own problems
and difficulties
What constitutes a public trial? A trial possesses that character
when anyone interested in observing the manner a judge
conducts the proceedings in his courtroom may do so, be he a
relative, a friend or a complete stranger to the proceedings.
However, the mere fact that trial is held in the air-conditioned
chambers of the judge does not violate the right to public trial.
And if the trial is so held in the judges chamber, then the
accused is entitled to be inside the chamber and not simply by
the door
General Considerations
Secrecy may suggest something sinister, or suppression and
oppression. It might as well be if trials were held in private, far
from the view and observation of the public. For by then,
nobody really knows what goes on within the walls of silence
except those inside
This does not preclude the courts from excluding the public,
however, where the subject matter is sensitive or otherwise
dictated by the nature of the proceedings, as in rape cases
where the audience might be more interested as salacious
voyeurs than disinterested spectators

Publicity and Prejudice


When it comes to the issue of impartiality of trials, the matter f
the adverse and contagious effect of publicity may also come
into the picture. Publicity could have a distorting effect on trial
such that judges may not see the case for what it really is but as
distilled through the noise outside, or the pens and lenses of
the ubiquitous mass media. It is in this area that there comes
again the need to carefully weigh conflicting values of society-the right of the people to know and of the press to disseminate
newsworthy materials, on one hand, and the right of the
accused to a fair and impartial trial, on the other hand
WEBB v. DE LEON
247 SCRA 652 (1995)
Hubert Webb, Michael Gatchalian, Antonio Lejano and six (6) other
persons, were charged with the crime of Rape with Homicide in relation
to rape and killing of Carmela Vizconde, her mother Estrellita, and her
sister Jennifer in their home in Paranaque. The DOJ Panel conducted a
preliminary investigation and, in an issued Resolution, found probable
cause to hold respondents for trial. Information for rape with homicide
was filed against the petitioners with Paranaque RTC. Warrants of
Arrests were issued against them. Petitioners Webb, Gatchalian, and
Lejano voluntarily surrendered to the police authorities and filed
petitions questioning the proceeding against them and the issuance of
warrants, assailing, the prejudicial publicity that attended their
preliminary investigation considering the highly sensationalized nature
of the case.
Conflicting demands of freedom of speech and of the press, the publics
right to information, and the accuseds right to a fair and impartial trial
119 | P

LATON

collide and compete for prioritization. The possibility of media abuses


and their threat to fair trail notwithstanding, criminal trials cannot be
completely closed to the press and the public.
To warrant a finding of judicial publicity there must be allegation and
proof that the judges have been unduly influenced, not simply that they
might be, by the barrage of publicity. (Martelino, et al. Vs. Alejandro et
al.) In the case at bar, nothing in the records will prove that the tone
and content of publicity that attended the investigation of petitioners
fatally infected fairness and impartiality of DOJ panel. DOJ panel is
composed of an Asst. Chief State Prosecutor and Senior State
Prosecutors, their long experience in criminal investigation is a factor to
be considered whether they can easily be blinded by the klieg lights of
publicity. The 26-page Resolution carries no indubitable indicia of bias
for it does not appear that they considered any extra-record evidence
except evidence properly adduced by the parties. Also, the length of
time the investigation was conducted despite its summary nature and
generosity in accommodating the motions of petitioners speak well of
their fairness.
Petitions are dismissed for lack of showing of grave abuse of discretion
on the part of respondents.
RE: REQUEST FOR RADIO-TV COVERAGE OF THE TRAIL IN THE
SANDIGANBAYAN OF THE PLUNDER CASES AGAINST THE FORMER
PRESIDENT JOSEPH E. ESTRADA
360 SCRA 248 (2001)
The Kapisanan ng mga Brodkaster ng Pilipinas (KBP) requested the SC to
allow media coverage via live radio and television broadcast of the
anticipated trial before the Sandiganbayan of the trial of the plunder
cases against former president Joseph E. Estrada, in order to assure the
public full transparency in the proceedings of an unprecedented case in
our history.Sec. of Justice filed the instant petition seeking live radio
and television coverage of the proceedings grounded on the right of the
people to be informed on matters of public concern as well as desire
for transparency in administration of justice. Estrada, expressed his
opposition, and so did the IBP.
The proprietary of granting or denying the petition involve the weighing
out of constitutional guarantees of freedom of the press and the right
to public information, on the other hand, and the fundamental rights of
the accused, on the other hand, along with the constitutional power of
the court to control its proceeding in ensuring a fair and impartial trial.
When these rights race against one another, jurisprudence tells us that
the right of the accused must be preferred to win.
A public trial is not synonymous with publicized trial; it only implies that
the court doors must be open to those who wish to come, sit in the
available seats, conduct themselves proper decorum and observe the
trial process.
The television does not simply mirror or reflect events as they unfold.
The images transmitted onscreen are the end products of a series of
technical modifications employed by TV editors and cameramen;
limiting the publics perception of the events being covered.
With intense coverage, the media becomes less of a defensive force
against injustice and more of an offensive force by intimidating
witnesses, distracting the lawyers and distorting the unfolding drama in
the courtroom. Thus, until and unless the media can secure the rights
of the accused and eliminate all the adverse effects, specifically on the
general public, the TV should remain outside of the courtroom.
DISSENTING:
Puno, J.
It will to a large degree, throttle the right of the press to access to
information and choke the flow of knowledge to the people. it is the
people who govern in a democracy and they can only govern well if
they are fully informed. A people kept in the dark by the blindfold of
ignorance will only govern with mistakes.
By outlawing the TV in the trial of former President Estrada, the
majority has denied our people the opportunity to know completely

and accurately W/N his trial will be fair and impartial.


Panganiban, J.
It is technologically possible to uphold the right of the people to public
information without violating the right of the accused to due process
and without impeding the orderly administration of justice.
Media and judiciary are natural partners, because they are bound
together by the same reasons for being- the search for truth, the
protection of the peoples rights, and the defense for the basic norms
of society. By proscribing live coverage of trials and hearings, the Court
has lost an indispensable teammate in discovering, processing and
reporting the raw, unadulterated and unvarnished truth.
RESOLUTION ON MOTION FOR RECONSIDERATION
365 SCRA 62 (2001)
The Secretary of Justice filed a motion for reconsideration arguing that
there is really no conflict between the right of the people to public
information and the freedom of the press, on one hand, and on the
other, the right of the accused to a fair trial; that if there is a clash
between these rights, it must be resolved in favour of the right of the
people and the press because the people, as the repository of
sovereignty, are entitled to information; and, that live media coverage
is a safeguard against attempts by any party to use the courts as
instruments for the pursuit of selfish interests. Estrada reiterates his
objection to the live coverage of his trial. Based on his experience with
the impeachment trial. Live media coverage will only pave way for socalled expert commentary which can trigger massive demonstrations
aimed at pressuring the Sandiganbayan. He further contends that the
right of people to information may be served through other means less
distracting, degrading, and prejudicial than live coverage.
After due deliberation, SC finds no reason to alter or modify its decision
on prohibiting live coverage of the trial of the former president. By a
vote of 9-6, the court denies the MR of the Sec. of Justice.
In lieu of the live TV and radio coverage, the court resolved, by the vote
of 8-7, to order the audiovisual recording for documentary purposes.
Reasons for televised recording: 1)the hearings are of historic
significance. 2)Estrada cases involve matters of vital concern to our
people who have a fundamental right to know how their government is
conducted. 3)audio-visual presentation is essential for the education
and civic training of people.
Conditions of ausio-visual recording: a)trial shall be recorded in its
entirety, excepting such portions thereof as the Sandiganbayan
determine should not be held public; b)cameras shall be installed
inconspicuously inside the courtroom and the movement of TV crews
shall be regulated; c)audio-visual recordings shall be madefor
documentary purposes only and shall be made without comment
except such annotations of scenes depicted therein as may be
necessary to explain them; d)live broadcast of the recordings before
the Sandiganbayan shall have rendered its decision in cases against
former president Estrada shall be prohibited under the pain of
contempt of court and other violations; e)audio-visual recording of the
proceedings shall be made under the supervision and control of the
Sandiganbayan or its Division; f)simultaneously with the release of the
audio-visual recordings for public broadcast, the original thereof shall
be deposited in the National Museum and the Records Management
and Archive Office for preservation and exhibition in accordance with
law.

Related to the issue of prejudicial publicity is the prohibition of


certain fictionalized representation of matters which might be
the subject of on-going court proceedings
Right of Confrontation
Another element of fairness is the right of whoever is charged
to have the opportunity to confront whoever it is whose
120 | P

LATON

testimony or evidence may lead the former to lose his liberty or


even life
It is the province of [cross-examination] to test the credibility of
the witnesses, expose falsehood or half-truth, uncover the
truth which rehearsed direct-examination testimonies may
successfully suppress, and demonstrate inconsistencies on
substantial matters which create reasonable doubt. In short,
cross-examination is an indispensable instrument of criminal
justice to give substance and meaning to the Constitutional
right of the accused to confront the witnesses against him and
to show that the presumption of innocence has remained
steadfast and firm
It has also been held that the admission of hearsay evidence in
criminal cases would be a violation of the constitutional
guarantee of the right of confrontation
Mere opportunity and not actual cross-examination is the
essence of the right

(d) The Confrontation Clause commands that reliability be assessed in a


particular manner: by testing in the crucible of cross-examination.
Roberts allows a jury to hear evidence, untested by the adversary
process, based on a mere judicial determination of reliability, thus
replacing the constitutionally prescribed method of assessing reliability
with a wholly foreign one.
(e) Roberts' framework is unpredictable. Whether a statement is
deemed reliable depends on which factors a judge considers and how
much weight he accords each of them. However, the unpardonable vice
of the Roberts test is its demonstrated capacity to admit core
testimonial statements that the Confrontation Clause plainly meant to
exclude.
(f) The instant case is a self-contained demonstration of Roberts'
unpredictable and inconsistent application. It also reveals Roberts'
failure to interpret the Constitution in a way that secures its intended
constraint on judicial discretion. The Constitution prescribes the
procedure for determining the reliability of testimony in criminal trials,
and this Court, no less than the state courts, lacks authority to replace
it with one of its own devising.

CRAWFORD V. WASHINGTON
541 U.S. 36, 158 L Ed 2d 177, 124 Ct 1354 (2004)

PEOPLE V. PIDO
200 SCRA 45 (1991)

Petitioner was tried for assault and attempted murder. The State
sought to introduce a recorded statement that petitioner's wife Sylvia
had made during police interrogation, as evidence that the stabbing
was not in self-defense. Sylvia did not testify at trial because of
Washington's marital privilege. Petitioner argued that admitting the
evidence would violate his Sixth Amendment right to be "confronted
with the witnesses against him." Under Ohio v. Roberts, 448 U. S. 56,
that right does not bar admission of an unavailable witness's statement
against a criminal defendant if the statement bears "adequate `indicia
of reliability,'" a test met when the evidence either falls within a "firmly
rooted hearsay exception" or bears "particularized guarantees of
trustworthiness." The trial court admitted the statement on the latter
ground. The State Supreme Court upheld the conviction, deeming the
statement reliable because it was nearly identical to, i. e., interlocked
with, petitioner's own statement to the police, in that both were
ambiguous as to whether the victim had drawn a weapon before
petitioner assaulted him.

Accdg. To Prosecution: Teresita was awakened at around 1-2:00 by an


intruder who she identified as the appellant. Holding a sharp bladed
instrument in his right hand he threatened to kill Teresita if she would
shout. He told her to remove her panty and afterwards placed himself
on top of her, and with his left hand inserted his penis into his genital
organ. Thereafter, he left through the window, leaving Teresita crying.
Meanwhile, Lydia, who was also staying in the same house came out of
her room and when she switched on the light, she saw Teresita shaking
and crying. She later told Lydia that she was raped by the appellant.
Accdg. To the Appellant: While admitting that he had sexual intercourse
with Teresita, claimed that their tryst was with her consent. He
declared that Teresita was his former girlfriend and on the night in
question, he attended the despedida party of the husband of Lydia at
their backyard. In the course of their drinking spree, he saw Teresita
looking out the window and waving her hand at him, asking him to
come up. They simultaneously took off their clothes and made love,
and while they were in the act, Lydia came out of her room and saw
them. Both were embarrassed, and because of their embarrassment,
she cried. He claimed that Teresita filed the complaint for rape because
Lydia threatened that she will report what she saw to Teresitas
common-law husband.
After trial, appellant was found guilty. The TC characterized the
testimony of the complainant that appellant raped her as positive and
categorical, and clear, positive, and convincing. It concluded that the
credibility of the declaration of the complainant and her witness, Lydia,
have not been shaken much less destroyed by the searching questions
of the cross-examiner
Upon reviewing the case, the Court find that the prosecution failed to
discharge its duty to establish the guilt of the accused beyond
reasonable doubt. The TC merely hurried itself to the sweeping
conclusion that the credibility of their declarations have not been
shaken much less destroyed by the searching questions of the crossexaminer. This is not supported by facts unfurled during the crossexamination.
This case is an exception to the rule that appellate courts will generally
not disturb the factual findings of the trial court considering that it is in
better position to decide the question , having heard the witnesses
themselves and observed their deportment and manner of testifying.
Two special considerations in the case justify such departure: 1) it was
another judge (Judge Lazaro) who heard and received the whole
testimony . Judge Makasinar, who decided the case, did not have
sufficient base=is to form an opinion as to the complainants
deportment an manner of testifying; 2)the TC had ignored or
overlooked substantial facts and circumstances:
D
By a direct, positive and unequivocal statement, complainant
categorically asserted on direct examination that:

The State's use of Sylvia's statement violated the Confrontation Clause


because, where testimonial statements are at issue, the only indicium
of reliability sufficient to satisfy constitutional demands is
confrontation.
(a) The Confrontation Clause's text does not alone resolve this case, so
this Court turns to the Clause's historical background. That history
supports two principles. First, the principal evil at which the Clause was
directed was the civil-law mode of criminal procedure, particularly the
use of ex parte examinations as evidence against the accused. The
Clause's primary object is testimonial hearsay, and interrogations by
law enforcement officers fall squarely within that class. Second, the
Framers would not have allowed admission of testimonial statements
of a witness who did not appear at trial unless he was unavailable to
testify and the defendant had had a prior opportunity for crossexamination. English authorities and early state cases indicate that this
was the common law at the time of the founding. And the "right . . . to
be confronted with the witnesses against him," Amdt. 6, is most
naturally read as a reference to the common-law right of confrontation,
admitting only those exceptions established at the time of the
founding.
(b) This Court's decisions have generally remained faithful to the
Confrontation Clause's original meaning.
(c) However, the same cannot be said of the rationales of this Court's
more recent decisions. The Roberts test departs from historical
principles because it admits statements consisting of ex
parte testimony upon a mere reliability finding.

121 | P

LATON

When he [accused] was on top of me, he inserted his private organ to


my private organ.
with the use of his left hand, which she repeated in detail at one point
of the cross-examination, thus:
Q
After you have removed your panty, what did the accused
do?
A
He removed his hand . . . left hand on my neck and moved
his body a little to the back and he held his private organ and inserted it
into my private organ.
Q
Which hand did he use in inserting his organ to yours?
A
He used his left hand.
However, in another point during cross-examination, she affirmed that
the accused was beside her, and she was the one who took hold of his
penis and put it into her private organ:
Q
Have you felt his body when he was beside you at that right?
A
Yes, sir.
Q
For how many minutes have (sic) you felt his body when he
was beside you?
A
Just for a short while, sir.
Q
Because you immediately took hold of his penis and put into
your vagina?
A
Yes, sir.
which she did for the following reason:
Q
And so that you can immediately finish his intercourse?
A
Yes, sir.
The lame excuse for such conduct offered by her on redirect
examination was because she was nervous as she was threatened by
accused who told her "you insert it otherwise I will kill you including
your children." He resorted to this because "at first he was not able to
insert it"; yet, on direct examination, she categorically stated that when
he was on top of her he inserted his organ, and although she did not
see how, she knew that it was already inside.
E
While on direct examination, she wanted to convey that she was
reluctant to remove her panty, and in an earlier cross-examination, she
wanted the court to believe that she spread her legs preparatory to the
intercourse because she was allegedly ordered by the accused; yet
subsequently, she was candid enough to admit that she immediately
removed her panty and raised her dress for the intercourse, thus:
Q
So, when this intruder allegedly told you to remove your
panty, you immediately removed your panty?
A
Yes, sir, I obeyed him because I was afraid.
Q
And after that you immediately raised your dress so that this
intruder can immediately have sexual intercourse?
A
Yes, sir.
Q
And immediately this intruder immediately (sic) penetrated
his penis over (sic) your vagina?
A
Yes, sir.
Q
And while he was penetrating (sic) his over your vagina, he is
(sic) making love with you?
A
No sir, straight.
However, in another portion of the cross-examination, complainant, by
slip of her tongue, admitted that she in fact voluntarily removed her
panty, thus:
Q
And you never even bothered to go out of the mosquito net
and shout?
A
I stood up, sir.
Q
You stood up because you want (sic) to wear again your
panty which you voluntarily removed?
A
Yes, sir.
Q
You never even bothered to go to the window and asked
(sic) help from your neighbors?
A
No. sir. (Emphasis supplied)
F
On cross-examination, the cross examiner succeeded in showing that
the sexual liaison was the product of mutual consent, or the
complainant and the accused were "making love." The following
questions and answers are enlightening:
Q
Now, you said that after less than 5 minutes making
love with this alleged intruder, he left you at the sala, am I right?
A
Yes sir, he went away but I could not recall if he left because
I was crying.
Q
Are (sic) you crying aloud?

A
Not so loud, sir.
Q
You are (sic) crying because of happiness, am I right?
A
No, sir.
Q
After making love with you, do you know if this intruder
dressed-up?
A
I do not know anymore sir, I was crying because of fear.
xxx xxx xxx
Q
So, you want to impress this Honorable Court that while you
were making love with the accused this Lydia Sulit passed by?
A
Yes, sir.
xxx xxx xxx
Q
While you were making love with the accused, do you
remember if the party is (sic) still going on downstairs?
A
Yes, sir.
Q
So, there are (sic) still many people at the ground floor of
your house during that night when you were making love with the
accused?
A
No sir, that is not true.
Q
Which is not true?
A
The people were already asleep, sir.
Q
After making love with the accused, do you know where the
accused passed thru in going down?
A
I do not know, sir.
Q
Is it not true that the accused jumped over the window?
A
I do not know sir, I was crying because I was afraid.
This claim that she did not then know where the accused passed
through is of course inconsistent with her assurance on direct
examination that the accused used the window for his exit:
A
After he put on his panty or shorts, he left through the
window. This window is at the second floor of the house where he
jumped.
xxx xxx xxx
A
He went out of the window where the stairs is just nearby
and he stepped on it. There he already went down.
xxx xxx xxx
A
Well, I don't know how he entered, Your Honor, but he went
out through the window.
then she confirmed on such cross-examination, thus:
Q
So, you were then lying when you testified on January 18,
1982 that the accused jumped over the window?
A
It was not our window that opened in front of where we are
(sic) sleeping, sir.
Q
But you are telling that the window is (sic) still open since
you slept and making (sic) love with the accused?
A
That window was really open because it is (sic) warm, sir.
If the references to "making love" were without basis, or that the
questions based thereon were misleading, the prosecuting fiscal should
have objected. None was made, and there is no showing that the
prosecuting fiscal is incompetent. After this "love-making" rendezvous,
the accused promised to return to make love again.
Q
And after finishing the intercourse what did this intruder do
if he did anything?
A
He told me that he will return, sir.
Q
To make love again with you?
A
Yes, sir.
Q
And you said "yes" because you like it?
A
No sir, I was crying.
Q
Because you are (sic) afraid that your husband might know
it?
A
No sir, I was still startled because of my fear.
G
This fear, however, seemed to be more imagined than real in the light
of the above findings clearly demonstrating her consent to the act. As a
matter of fact, there is enough evidence to show that she signaled to
the accused to spend time with her and that she prepared for the tryst.
For reasons only known to the prosecution, it opted not to rebut the
damaging testimony of the accused that among others, complainant
signaled to him to come up; she thereafter took a bath; when he came
up, she ordered him to get inside the mosquito net, and to give him
space beside her, she even moved her youngest child; and then they
simultaneously took off their clothes and made love. While they were
in the act, Lydia Sulit came out of her room and saw them. Because of
embarrassment, complainant cried. He then dressed up and returned
122 | P

LATON

to the party.
What then seems to be clear is that Lydia did not arrive after the act,
but as testified by the accused and corroborated by complainant
herself, she saw both inside the mosquito net. The complainant was so
embarrassed that she had to concoct the story of rape.
PEOPLE V. NARCA
275 SCRA 696 (1997)
Accused-appellants were charged with murder fo the killing of Mauro
Reglos, Jr. They filed a motion to quash the information, and when this
failed, they filed a motion for bail. During the bail hearing, the victims
wife, Elizabeth Reglos, who was with him on that fateful night, testified
on direct examination. Defense counsel requested the court that his
cross-examination of Elizabeth be conducted on the next hearing, the
following month. However, such cross-examination never took place
because Elizabeth died before the hearing. After hearing, lower court
denied bail, and after trail, convicted the appellants.
Issue: W/N Elizabeths testimony be given credence considering that
she was nit cross-examined.
Held: Where death prevents cross-examination under such
circumstances that no responsibility of any sort can be ascribed to the
plaintiff or the witness, it seems a harsh measure to strike out all that
has obtained in the direct examination.
Besides, mere opportunity and not actual cross-examination is the
essence of the right to cross-examine. Right to cross-examination is a
personal one which may be waived expressly or impliedly by conduct
amounting to a renunciation of the right of cross-examination. Thus,
where a party has had the opportunity to cross-examine a witness but
failed to avail himself of it, he necessarily forfeits the right to crossexamine and the testimony given on direct examination of witness will
be received or allowed to remain in record.
Sec. 1 (f) of Rule 115 provides that either party may utilize as part of
its evidence the testimony of a witness who is deceased ***given in
another case or proceeding and under Sec 8, Rule 114 as amended by
circular 12-94, evidence presented during the bail hearings, like the
testimony of the deceased witness Elizabeth, are considered
automatically reproduced at trial.
PEOPLE v. ORTIZ-MIYAKE
279 SCRA 180 (1997)
Accused appellant Lanie Ortiz-Miyake was charged with illegal
recruitment in large scale in RTC of Makati on a complaint initiated by
Elenita Marasigan, Imelda Generillo and Rosamar del Rosario. In
addition, she was indicted for estafa by means of pretenses in the same
court, the offended party being Elenita Marasigan alone.
Of the three complainants in the case for illegal recruitment in large
scale, Marasigan was the only one who testified at the trial. The two
other complainants, Generillo and Del Rosario, were unable to testify as
they were then abroad. in lieu of their testimonies, the prosecution
presented as witnesses Lila Generillo, the mother of Imelda, and Victor
Amin, the sister of Del Rosario. The final witness for the prosecution
was Riza Balberte, POEA representative, testified that appellant was not
authorized or licensed to recruit workers for overseas employment. The
prosecution sought to prove that although two of the three
complainants were unable to testify, appellant was guilty of committing
the offense against all three and, therefore, be convicted as charged.
The TC convicted appellant of both crimes as charged. In convicting
appellant of illegal recruitment in large scale, the lower court adopted a
previous decision (conviction for estafa in 1993 wherein complainats
Generillo and Del Rosario charged the appellant) of MTC of Paranaque
as a basis for the judgment. RTC adopted the facts and conclusions
established in the earlier decision as its own findings of facts and as its
rationale for the conviction of the case before the court.

The accused in a criminal case is guaranteed the right of confrontation.


Such right has two purposes: first, to secure the opportunity of crossexamination; and, second, to allow the judge to observe the
deportment and appearance of the witness while testifying. This right,
however, is not absolute as it is recognized that it is sometimes
impossible to recall or produce a witness who has already testified in a
previous proceeding, in which event his previous testimony is made
admissible as a distinct piece of evidence, by way of exception to the
hearsay rule.
The adoption by the Makati TC of the facts stated in the decision of the
Paranaque TC does not fall under the exception to the right of
confrontation as the exception contemplated by law covers only the
utilization of testimonies of absent witnesses made in previous
proceedings, and does not include utilization of previous decisions or
judgments.
Prosecution did not offer the testimonies made by complainants.
Instead, what was offered, admitted in evidence, and utilized as a basis
for conviction in the case for illegal recruitment in large scale was the
previous decision in the estafa case. A previous decision or judgment,
while admissible in evidence may only prove that an accused was
previously convicted of a crime. It may not be used to prove that the
accused is guilty of a crime charged in a subsequent case.
For insufficiency of evidence and in the absence of the third element of
illegal recruitment in large scale: the offense is committed against
three or more persons, the court cannot affirm the conviction for
illegal recruitment in large scale but agrees that appellant illegally
recruited Marasigan, for which she ust be held liable for the lesser
offense of simple illegal recruitment.

Relevant to the right of an accused to confront the witness


against him is the issue as to whether to disclose the identity of
confidential informers or to present them at the witness stand.
This necessarily involves again the balancing of conflicting
societal values
The Court is sharply aware of the compelling considerations
why confidential informants are usually not presented by the
prosecution. One is the need to hide their identity and preserve
their invaluable service to the police. Another is the necessity
to protect them from objects or targets of revenge by the
criminals they implicate once they become known. All these
considerations, however, have to be balanced with the right of
an accused to a fair trial
It was held that the scope of the privilege is limited by its
underlying purpose. Thus, where the disclosure of the contents
of the communication will not tend to reveal the identity of an
informer, the contents are not privileged. Likewise, once the
identity of the informer has been disclosed to those who would
have cause to resent the communication, the privilege is no
longer applicable
A further limitation on the applicability of the privilege, which
arises form the fundamental requirements of fairness was
emphasized. Where the disclosure of an informers identity, or
the contents of his communication, is relevant and helpful to
the defense of an accused, or is essential to a fair determination
of a cause, the privilege must give way
In sum, there is no fixed rule with respect to disclosure of the
identity of an informer. The problem has to be resolved on a
case to case basis and calls for balancing the state interest in
protecting people from crimes against the individuals right to
prepare his defense. The balance must be adjusted by giving
due weight to the following factors, among others: (1) the
123 | P

LATON

crime charged, (2) the possible defenses, (3) the possible


significance of the informers testimony, and (4) other relevant
factors
Right of Compulsory Process
This guarantee assures the accused an opportunity to compel
the production of evidence and the attendance of witnesses on
his behalf in order that he may be adequately equipped to
present his side. Also, compared to the 1935 Constitution which
only guaranteed compulsory process to secure the attendance
of witnesses, the 1973 and 1987 Constitutions expanded the
same by now including the right to secure the production of
evidence in ones behalf as well. Accordingly,
By analogy, U.S. v. Ramirez which laid down the requisites for
compelling the attendance of witnesses, may be applied to this
expanded concept. Thus, the movant must show: (a) that the
evidence is really material; (b) that he is not guilty of neglect in
previously obtaining the production of such evidence; (c) that
the evidence will be available at the time desired; and (d) that
no similar evidence could be obtained
In Rolito Go v. Court of Appeals, the right to have a preliminary
investigation conducted before being bound over for trial for a
criminal offense, and hence formally at risk of incarceration or
some other penalty, is not a mere or technical right; it is a
substantive right
In the 1963 watershed case of Brady v. Maryland the United
States Supreme Court held that suppression of evidence
favorable to an accused upon request violates due process
where the evidence is material to guilt or punishment,
irrespective of the good faith or bad faith of the prosecution
The right to the production of all evidence at a criminal trial
similarly has constitutional dimensions. The Sixth Amendment
explicitly confers upon every defendant in a criminal trial the
right to be confronted with the witnesses against him and to
have compulsory process for obtaining witnesses in his favor.
Moreover, the Fifth Amendment also guarantees that no
person shall be deprived of liberty without due process of law.
It is the manifest duty of the courts to vindicate those
guarantees, and to accomplish that it is essential that all
relevant and admissible evidence be produced
We conclude that, when the ground for asserting privilege as to
subpoenaed materials sought for use in a criminal trial is based
only on the generalized interest in confidentiality, it cannot
prevail over the fundamental demands of due process of law in
the fair administration of criminal justice. The generalized
assertion of privilege must yield to the demonstrated, specific
need for evidence in a pending criminal trial
US v. SCHEFFER
523 U.S. 303
While defending himself before a military court martial on, among
other things, substance abuse charges, airman Edward G. Scheffer
sought to introduce his polygraph examination results. The results
indicated there was "no deception" in Scheffer's denial that he used
drugs while enlisted. Relying on Military Rule of Evidence 707 ("Rule
707"), prohibiting the use of polygraph results in court-martial
proceedings, the military judge refused Scheffer's request to admit his
results into evidence. On successive appeals, following his conviction on
all charges, the Air Force Court of Appeals affirmed but the Court of

Appeals for the Armed Forces reversed, finding the evidentiary


exclusion to be unconstitutional. The United States appealed and the
Supreme Court granted certiorari.
Does Military Rule of Evidence 707, excluding the admission of
polygraph results into evidence, violate a defendant's Sixth Amendment
right to present a fair defense?
No. In an 8-to-1 decision, the Court held that Rule 707 was consistent
with the legitimate interest of state and federal authorities to admit
only reliable evidence. In addition to noting the even-handed scope of
Rule 707, excluding from evidence both favorable and unfavorable
polygraph results, the Court emphasized the poor reliability of
polygraph evidence as a whole. In the absence of sounder detection
methods, the Court noted that the fundamental premise of the criminal
justice system is that juries are the ultimate and most reliable
evaluators of credibility and truthfulness.

Trials In Absentia--Waiver of Appearance


While the accused has the right to be heard, he cannot hold the
proceedings hostage, however, by absenting himself or refusing
to participate. Thus, the Constitution provides that after
arraignment, trial may proceed notwithstanding the absence of
the accused provided that he has been duly notified and his
failure to appear is unjustifiable
Nor is it only the due process guarantee that calls for the
accused being duly arraigned. As noted, it is at the stage where
in the mode and manner required by the Rules, an accused, for
the first time, is granted the opportunity to know the precise
charge tat confronts him. It is imperative that he is thus made
fully aware of possible loss of freedom, even of his own life,
depending on the nature of the crime imputed to him
Clearly, the innovation introduced by the present Constitution
goes no further than to enable a judge to continue with the trial
even if the accused is not present under the conditions therein
specified. It does not give him the right to jump bail.
People v. Salas
143 SCRA 163
Mario Abong was originally charged with homicide in the Court of First
Instance of Cebu but before he could be arraigned the case was
reinvestigated on motion of the prosecution. As a result of the
reinvestigation, an amended information was filed, with no bail
recommended, to which he pleaded not guilty. Trial commenced, but
while it was in progress, the prisoner, taking advantage of the first
information for homicide, succeeded in deceiving the city court of Cebu
into granting him bail and ordering his release; and so he escaped. The
respondent judge, learning later of the trickery, cancelled the illegal bail
bond and ordered Abong's re-arrest. But he was gone. Nonetheless, the
prosecution moved that the hearing continue in accordance with the
constitutional provision authorizing trial in absentia under certain
circumstances. The respondent judge denied the motion, however, and
suspended all proceedings until the return of the accused. The order of
the trial court is now before us on certiorari and mandamus. The judge
erred. He did not see the woods for the trees. He mistakenly allowed
himself to be tethered by the literal reading of the rule when he should
have viewed it from the broader perspective of its intendment
The trial judge is directed to investigate the lawyer who assisted Mario
Abong in securing bail from the city court of Cebu on the basis of the
withdrawn information for homicide and to report to us the result of
his investigation within sixty days.
The order of the trial court dated December 22, 1983, denying the
motion for the trial in absentia of the accused is set aside. The
respondent judge is directed to continue hearing the case against the
124 | P

LATON

respondent Mario Abong in absentia as long as he has not reappeared,


until it is terminated. No costs

It could thus be seen that the Bill of Rights has set up a lot of
guarantees and safeguards by which life and liberty, as well as
property of individuals are protected from being taken by the
State, or otherwise lost. A persons entitlement to his freedom
and to his life and what other things he may own or possess
could not simply be forfeited without having to satisfy the
constitutional shield and requirements, starting with the
presumption of innocence to the quantum of proof needed to
secure a conviction. In between are the other procedural
hurdles designed to ensure fairness in order that only the
guilty, as much as humanly possible, be punished and that the
innocent be set free to continue to enjoy life and liberty and to
bask in the wide open and exhilarating field of freedom
Additional Cases
(N) Rights of Suspects [7]

Re: Conviction of Judge Adoracion G. Angeles, RTC, Br. 121, Caloocan


City in Criminal Cases Nos. Q-97-69655 to 56 for Child Abuse
543 SCRA 196 (2008)
Does the presumption of innocence come to an end when there is
conviction by the trial court, even if there is an appeal taken? In this
case, the Court echoed what was said Trillanes the presumption lost
only when there is a final judgment. Accordingly, the fact of [the
judges] conviction by the RTC does not necessarily warrant her
suspension. We agree with [her] argument that since her conviction the
crime of child abuse is currently on appeal before the CA, the same has
not yet attained finality. As such, she still enjoys the constitutional
presumption of innocence. It must be remembered that the existence
of a presumption indicating the guilt of the accused does not in itself
destroy the constitutional presumption of innocence unless the
inculpating presumption, together with all the evidence, or the lack of
any evidence or explanation, proves the accuseds guilt beyond a
reasonable doubt.
Andaya v. People
493 SCRA 539
In this case, the accused was charged with Falsification of Private
Documents but no damage to the offended party was proved. The
accused was still convicted, though, for intent to cause damage to
government through evasion of tax. Despite the trial courts
concession that the allegedly offended party (a savings and loan
association) suffered no damage, it still convicted the accused,
reasoning out that the third essential element of falsification of private
document was present because the falsification of the voucher was
done with criminal intent to cause damage to the government
considering that its purpose was to lower the tax base of the recipient
of the money, allowing him to evade payment of taxes.
The Court saw the act done by the judge unjustified. We find ourselves
unable to agree with this ratification of the trial court because it
violates the constitutional right of petitioner to be informed of the
nature and cause of the accusation against him. It must be
remembered that [n]o matter how conclusive and convincing the
evidence of guilt may be, an accused cannot be convicted of any
offense unless it is charged in the information on which he is tried or is
necessarily included therein. To convict him of a ground not alleged
while he is concentrating his defense against the ground alleged would
plainly be unfair and underhanded. The rule is that a variance between

the allegation in the information and proof adduced during trial shall be
fatal to the criminal case if it is material and prejudicial to the accused
so much so that it affects his substantial rights.
No matter the victim, accused still guilty! That might have been the
mindset of the trial court judge.

Chapter 16
Privilege of the Writ of Habeas Corpus
The privilege of the writ of habeas corpus shall not be
suspended except in cases of invasion or rebellion, when the
25
public safety requires it.
Liberty being the rule instead of the exception, there must be
an effective and speedy manner by which any deprivation of
such freedom be swiftly looked into and remedied
Indeed, it has been called the great writ of liberty by which
the legality of ones detention or deprivation of freedom of
movement may be inquired into. Vindication of due process, it
has been well said, is precisely the historic office of the Great
Writ. In a nutshell, it is a writ directed to a person detaining
another, commanding the former to produce the body of the
latter at a designated time and place. The objective of the writ
is to determine whether the confinement or detention is valid
and lawful. If it is, the writ cannot be issued
More specifically, its vital purposes are to obtain immediate
relief from illegal confinement, to liberate those who may be
imprisoned without sufficient cause, and to deliver them from
unlawful custody. It is then essentially a writ of inquiry and is
granted to test the right under which a person is detained
General Considerations
The writ of habeas corpus extends to all cases of illegal
confinement by which any person is deprived of his liberty.
The extraordinary writ of habeas corpus has long been a haven
of relief for those seeking liberty from any unwarranted denial
of freedom of movement. Indeed, habeas corpus embraces so
broad a dimension--aside from being thorough and complete-it affords prompt relief from unlawful imprisonment of any
kind, and under all circumstances
Further, it has also been held that [i]t is not physical restraint
alone which is inquired into by the writ of habeas corpus.
Reservation in the form of restrictions attached to the
temporary release of a detainee constitutes restraints on his
liberty and limits his freedom of movement of petitioner
Habeas Corpus could also be utilized to invoke the beneficial
effects of a law which was subsequently enacted. Hence, where
the decision convicting the accused is already final, the
appropriate remedy of the convict who invokes the retroactive
application of a statute is to file a petition for habeas corpus,
not a motion for reconsideration with modification of sentence
The privilege of the writ could also be availed of to secure ones
liberty from the restraint imposed by a private person, as
25

CONSTITUTION, Art. III, 15

125 | P

LATON

exemplified in the case of Caunca v. Salazar. It is likewise


available in regard to custody of children but not for the
purpose of compelling a spouse to live with the other
In relation to other procedural remedies, in Velasco v. Court of
Appeals, the Court said, relative to the availability of both
certiorari and habeas corpus:
While ordinarily, the writ of habeas corpus will not be granted
when there is an adequate remedy by writ of error or appeal or
by writ of certiorari, it may, nevertheless, be available in
exceptional cases, for the writ should not be considered
subservient to procedural limitations which glorify over
substance
Gumabon v Director of Prisons
37 SCRA 420
Petitioners Mario Gumabon, Blas Bagolbagol, Gaudencio Agapito,
Epifanio Padua and Paterno Palmares were charged and convicted of
the complex crime of rebellion with murder. They were imposed the
penaltyof Reclusion Perpetua. At the time of the petition each suffered
more than 13 years of imprisonment.Subsequently, the Court ruled in
PEOPLE VS. HERNANDEZ that the information against the accused in
thatcase for rebellion complexed with murder, arson and robbery was
not warranted under Article 143 of theRPC, there being no such
complex offense. Petitioners thus invoke that the ruling in Hernandez
be applied tothem. Petitioners contend that he has served more than
the maximum penalty that could have been imposedupon them and is
thus entitled to freedom, his continued detention being illegal.
WON Habeas Corpus can be granted to convicted inmates.
Petition for Habeas Corpus Granted, petitioners ordered released. Once
a deprivation of a constitutional right is shown to exist, the court that
rendered the judgment is deemed ousted of jurisdiction and Habeas
Corpus is the appropriate remedy to assail the legality of the detention.
The essential purpose of the writ of habeas corpus is to inquire into all
manner of involuntary restraint and to relieve therefore if such
restraint is illegal. The exception of jurisdiction, excluding habeas
corpus to sentenced prisoners, is not found here because of the
retroactivity of the Hernandez rulings. What is required under the
equal protection of law is the uniform operation of legal norms so that
all persons under similar circumstances would be accorded the
same treatment both in the privileges conferred and the liability
imposed. For the principle that equal protection and security shall be
given to every person under circumstances, which if not identical are
analogous. If law be looked upon in terms of burdens or charges, those
that fall within a class shall be treated in the same fashion, whatever
restrictions cast on some in the group equally binding on the rest.

Ilusorio v Bildner
332 SCRA 169
The wife of Potenciano Ilusorio filed a petition for Habeas Corpus of his
86 year old husband. After living for 30 years under the same house,
the couple separated in 1972. The younger daughter of the two filed
apetition for custody of the father because of his failing physical and
mental health. Later on, the daughter allegedly prohibited the mother
from seeing the father and living with her. Potenciano filed a motion to
enjoin a previous motion granted for visitation rights.
WON Writ of Habeas Corpus can compel a husband to live with his wife.
No court is empowered as a judicial authority to compel a husband
to live with his wife. In order to justify the grant of the writ of habeas
corpus, the restraint of liberty must be in the nature of an illegal and
involuntary deprivation of freedom of action. The illegal restraint of
liberty must be actual and effective, not merely nominal or moral.
The law provides that the husband and the wife are obliged to live
together, observe mutual love, respect and fidelity. The sanction
therefore is the "spontaneous, mutual affection between husband and
wife and not any legal mandate or court order" to enforce consortium.
Obviously, there was absence of empathy between spouses Erlinda
and Potenciano, having separated from bed and board since 1972.
Resolution on Motion for Reconsideration
Erlinda K. Ilusorio claimed that she was not compelling Potenciano to
live with her in consortium and that Potenciano's mental state was not
an issue. One reason why Erlinda K. Ilusorio sought custody of her
husband was that respondents Lin and Sylvia were illegally restraining
Potenciano Ilusorio to fraudulently deprive her of property rights out of
pure greed.
Motion denied Again
Clearly, Erlinda cannot now deny that she wanted Potenciano Ilusorio
to live with her.
The fact of illegal restraint has not been proved during the hearing at
the Court of Appeals on March 23, 1999.16 Potenciano himself declared
that he was not prevented by his children from seeing anybody and
that he had no objection to seeing his wife and other children whom he
loved. We were not convinced that Potenciano Ilusorio was mentally
incapacitated to choose whether to see his wife or not. Again, this is a
question of fact that has been decided in the Court of Appeals.

Suspension of the Privilege of the Writ, Bail and Damages

Subayno v Enrile
145 SCRA 282

This particular Bill of Rights guarantee is meant to delimit the


circumstances in which the privilege of the writ may be
suspended, namely, in cases of invasion or rebellion when the
public safety requires it

Ybanez, an officer of the education arm of the BAYAN movement was


abducted in Cebu City. Alleging tha this abductors was the military,
Subayno who is Ybanezs lover, filed a petition for Habeas Corpus to the
court. A line up was conducted on the military of Camp Lapu-Lapu
which yielded no identification of the perpetrators.

This should also be taken in conjunction with the other


restrictions contained in Article VII regarding the strictures in
place whenever the President suspends the privilege or
declares martial law. It states:

WON a motion for Habeas Corpus can be granted by mere accusation


of illegal detention.

Section 18. The President shall be the Commander-in-Chief of all armed


forces of the Philippines and whenever it becomes necessary, he may
call out such armed forces to prevent or suppress lawless violence,
invasion or rebellion. In case of invasion or rebellion, when the public
safety requires it, he may, for a period not exceeding sixty days,
suspend the privilege of the writ of habeas corpus or place the
Philippines or any part thereof under martial law. Within forty-eight
hours from the proclamation of martial law or the suspension of the
privilege of the writ of habeas corpus, the President shall submit a
report in person or in writing to the Congress. The Congress, voting
jointly, by a vote of at least a majority of all its Members in regular or

The petition must be dismissed for lack of proof, without prejudice to


the filing of another petititon. Under the foregoing circumstances, the
return of the writ must be taken on its face value considering that,
unless it is in some way traversed or denied, the facts stated therein
must be taken as true. Moreover, a writ of habeas corpus should not
issue where it is not necessary to afford the petitioner relief or where it
would be ineffective. The writ of Habeas corpus cannot be used as
a means of obtaining evidence.

126 | P

LATON

special session, may revoke such proclamation or suspension, which


revocation shall not be set aside by the President. Upon the initiative of
the President, the Congress may, in the same manner, extend such
proclamation or suspension for a period to be determined by the
Congress, if the invasion or rebellion shall persist and public safety
requires it.
The Congress, if not in session, shall, within twenty-four hours following
such proclamation or suspension, convene in accordance with its rules
without need of a call.
The Supreme Court may review, in an appropriate proceeding filed by
any citizen, the sufficiency of the factual basis of the proclamation of
martial law or the suspension of the privilege of the writ of habeas
corpus or the extension thereof, and must promulgate its decision
thereon within thirty days from its filing.
A state of martial law does not suspend the operation of the
Constitution, nor supplant the functioning of the civil courts or
legislative assemblies, nor authorize the conferment of jurisdiction on
military courts and agencies over civilians where civil courts are able to
function, nor automatically suspend the privilege of the writ of habeas
corpus.
The suspension of the privilege of the writ of habeas corpus shall apply
only to persons judicially charged for rebellion or offenses inherent in,
or directly connected with, invasion.
During the suspension of the privilege of the writ of habeas corpus, any
person thus arrested or detained shall be judicially charged within three
days, otherwise he shall be released.

Related to this is the declaration in the Bill of Rights itself that


the right to bail shall not be impaired even when the privilege
26
of the writ of habeas corpus is suspended
The only significant consequence of the suspension of the writ
of habeas corpus is to divest the courts of the power to issue
the writ whereby the detention of the person is put in issue. Or,
in the language of the Court in Aberca v. Ver, What is
suspended is merely the right of the individual to seek release
from the detention through the writ of habeas corpus as a
speedy means of obtaining his liberty
The suspension is supposed to enable the Government to deal
more effectively with the rebellion or invasion that might be
afoot. Even this advantage to the authorities has been limited,
however, by the requirement that those arrested or detained
pursuant to such suspension be judicially charged within three
days or otherwise they should be released
Finally, the suspension of the privilege could not be seized
upon, however, as a license to violate the rights of the persons
who might be affected thereby. There must still be adherence
to the Rule of Law even as the privilege might not be fully
available in the meantime
ABERCA v. VER
160 SCRA 590 (1988)
FACTS: This case stems from alleged illegal searches and seizures and
other violations of the rights and liberties of plaintiffs by various
intelligence units of the Armed forces of the Philippines, known as Task
Force Makabansa (TFM), ordered by General Fabian Ver to conduct
pre-emptive strikes against known communist-terrorist underground
houses in view of increasing reports about CT plans to sow disturbances
in Metro Manila. The defendants through their counsel, filed a motion
to dismiss which was favorably acted upon by the trial court.
26

CONSTITUTION, Art. III, 13

ISSUES:
(1) Whether or not the suspension of the privilege of the writ of
habeas corpus bars a civil action for damages for illegal searches
conducted by the military personnel and other violations of rights
and liberties guaranteed under the Constitution.
(2) Who can be held liable for such violations: only the military
personnel directly involved and/or their superiors as well?
THE COURTS RULE: Accordingly, we grant the petition and ANNUL and
SET ASIDE the resolution of the respondent court. Let the case be
remanded to the respondent court for further proceedings.
REASON: (1) We find merit in the petitioners contention that the
suspension of the privilege of the writ of habeas corpus does not
destroy petitioners right and cause of action for damages for illegal
arrest and detention and other violations of their constitutional rights.
The suspension does not render valid an otherwise illegal arrest or
detention. What is suspended is merely the right of the individual to
seek release from detention through the writ of habeas corpus as a
speedy means of obtaining his liberty.
(2)Doctrine of state immunity from suit cannot be construed as a
blanket license or a roving commission untrammelled by any
constitutional restraint, to disregard or transgress upon the rights and
liberties of the individual citizen enshrined in and protected by the
Constitution.
Article 32 of the Civil Code which renders any public officer or
employee or any private individual liable in damages for violating the
Constitutional rights and liberties of another, does not exempt
respondents from responsibility.

It is also noteworthy that in the United States, the suspension


of the privilege of the writ is vested in the Congress and not in
the President, through the former may authorize the latter to
do so. Only in the rarest of circumstances has Congress seen fit
to suspend the writAt all other times, it has remained a
critical check on the Executive, ensuring that it does not detain
individuals except in accordance with law
If society is really to value liberty and freedom, it must provide
the means and the method by which the validity of any
deprivation of liberty, any detention may be challenged and
scrutinized at the earliest opportunity and in the most
expeditious manner. Here, the office is filled up by the writ of
habeas corpus
Additional Cases
(O) Habeas Corpus [7]
Martinez v. Mendoza
FACTS: Petitioners are mother and wife of Michael Martinez, who was
allegedly abducted and taken away by 7 persons last November 19,
2001 at Sun Valley, Paranaque City. Martinez, is being implicated in the
killing of Dorothy Jones, aka Nida Blanca. It appears that on the evening
on November 19, 2001, the CIDG presented before the media a certain
Philip Medel, Jr who allegedly executed a statement confessing to his
participation in the killing, naming Michael Martinez as the person who
introduced him to Rod Lauren Strunk, the husband of Nida Blanca and
alleged mastermind in the killing. In a televised interview, Medel
narrate that he saw Michael Martinez at the CDG at Cam Crame where
he was being detained. The respondents vehemently deny any
participation or involvement in the alleged abduction of Martinez and
said that the alleged victim was never confined and detained by them
or in their custody at any given time. The CA did not believe Medels
narration and said that his credibility was highly suspect since, he had
contradicted himself as to material facts, as the Court opined. The issue
here is whether the court erred in reversing the trail court and
dismissing the petition for habeas corpus.
127 | P

LATON

HEL: The SC held that the petitioners anchor for the present case is the
disappearance of Martinez. The matter of his alleged detention is, at
best, merely consequential of his disappearance. The ultimate purpose
for the writ of habeas corpus is to relieve a person from unlawful
restraint. It is a remedy intended to determine whether the person
under detention is held under lawful authority. If the respondents are
neither detaining nor restraining the person on whose behalf the
petition for habeas corpus has been filed, then it should be dismissed.
Considering that respondents have persistently denied having Martinez
in their custody, and absent any decisive proof to rebut their denial, the
SC is restrained to affirm the CAs dismissal of the petition for habeas
corpus.

Republic of the Philippines


SUPREME COURT
Manila
A.M. No. 07-9-12-SC

THE RULE ON THE WRIT OF AMPARO

Manalo V. Calderon
FACTS: On May 125, 2007, 5 unidentified men forcibly entered Bgy.
Pinagbayanan Elementary School in Taysan, Batangas, which served as
a polling area for the 2007 National and Local elections. They entered
polling precinct 76-A and poured gasoline over a ballot box in and set it
ablaze. In the investigation that ensued, several eyewitnesses
identified some of petitioners as the perpetrators of the school burning
The investigation also yielded that all petitioners, who are members of
the PNOP regional special operations group, failed to timely respond to
the incident . Acing on the report, the PNP hierarchy issued 3 successive
memoranda. The petitioners contend that these memoranda defines
and circumscribes the scope of petitioners restrictive custody that
although technically speaking, they are not detained or imprisoned,
their physical movements are however, limited within Camp Vicente
Lim; they cannot go home to their respective families and if they would
leave Camp Vicente Lim, they need to be escorted. They said that such
is degrading and their restrictive custody status is illegal. Without
necessarily giving due course to the petition, the Court required
respondents to comment. In lieu of the comment, the OSG manifested
that 2 of the memoranda be recalled.
HELD: Notwithstanding the mootness of the issues on restrictive
custody and monitoring movements of petitioners, we opt to resolve
them given a) the paramount public interest involved b) their
susceptibility of recurring yet evading review and c) the imperative
need to educate the police community on the matter.
The release of petitioners by respondents in a petition for habeas
corpus does not automatically abate a decision on the case. Similarly, a
recall of the custody order challenged by petitioners will not necessarily
call for a dismissal on the ground of mootness alone. Although, the
general rule is mootness of the issue warrants a dismissal, there are
well defined exceptions. Courts will decided cases, otherwise moot and
academic if 1) there is a grave violation of the constitutions 2)
exceptional character of the situation and paramount public interest is
involved 3) when constitutional issue raised requires formulation of
controlling principles to guide the bench, bar and public and 4) the case
if capable of repetition yet evading review.
There is no illegal restraint in the restrictive custody and monitored
movements of police officers under investigation. A petition for habeas
corpus will be given due course only if it shows that petitioner is being
detained or restrained of his liberty unlawfully. A restrictive custody
and monitoring of movements of whereabouts of police officers under
investigation by their superiors is not a form of illegal detention or
restrained of liberty.
The ultimate purpose of the write of habeas corpus is to relieve a
person from unlawful restrained. The writ cannot and will not issue
absent a showing that petitioners are deprived of their liberty. Neither
can it relive petitioners, who are police officers, from the valid exercise
of prescribed discipline over them by the PNP leadership.

(25 September
2007)

SECTION 1. Petition. The petition for a writ of amparo is a remedy


available to any person whose right to life, liberty and security is
violated or threatened with violation by an unlawful act or omission of
a public official or employee, or of a private individual or entity.
The writ shall cover extralegal killings and enforced disappearances or
threats thereof.
SEC. 2. Who May File. The petition may be filed by the aggrieved
party or by any qualified person or entity in the following order:
a.
b.

c.

Any member of the immediate family, namely: the spouse,


children and parents of the aggrieved party;
Any ascendant, descendant or collateral relative of the
aggrieved party within the fourth civil degree of
consanguinity or affinity, in default of those mentioned in
the preceding paragraph; or
Any concerned citizen, organization, association or
institution, if there is no known member of the immediate
family or relative of the aggrieved party.

The filing of a petition by the aggrieved party suspends the right of all
other authorized parties to file similar petitions. Likewise, the filing of
the petition by an authorized party on behalf of the aggrieved party
suspends the right of all others, observing the order established herein.
SEC. 3. Where to File. The petition may be filed on any day and at any
time with the Regional Trial Court of the place where the threat, act or
omission was committed or any of its elements occurred, or with the
Sandiganbayan, the Court of Appeals, the Supreme Court, or any justice
of such courts. The writ shall be enforceable anywhere in the
Philippines.
When issued by a Regional Trial Court or any judge thereof, the writ
shall be returnable before such court or judge.
When issued by the Sandiganbayan or the Court of Appeals or any of
their justices, it may be returnable before such court or any justice
thereof, or to any Regional Trial Court of the place where the threat,
act or omission was committed or any of its elements occurred.
When issued by the Supreme Court or any of its justices, it may be
returnable before such Court or any justice thereof, or before the
Sandiganbayan or the Court of Appeals or any of their justices, or to any
Regional Trial Court of the place where the threat, act or omission was
committed or any of its elements occurred.
SEC. 4. No Docket Fees. The petitioner shall be exempted from the
payment of the docket and other lawful fees when filing the petition.
The court, justice or judge shall docket the petition and act upon it
immediately.
SEC. 5. Contents of Petition. The petition shall be signed and verified
and shall allege the following:
a.

The personal circumstances of the petitioner;


128 | P

LATON

b.

c.

d.

e.

f.

The name and personal circumstances of the respondent


responsible for the threat, act or omission, or, if the name is
unknown or uncertain, the respondent may be described by
an assumed appellation;
The right to life, liberty and security of the aggrieved party
violated or threatened with violation by an unlawful act or
omission of the respondent, and how such threat or
violation is committed with the attendant circumstances
detailed in supporting affidavits;
The investigation conducted, if any, specifying the names,
personal circumstances, and addresses of the investigating
authority or individuals, as well as the manner and conduct
of the investigation, together with any report;
The actions and recourses taken by the petitioner to
determine the fate or whereabouts of the aggrieved party
and the identity of the person responsible for the threat, act
or omission; and
The relief prayed for.

The petition may include a general prayer for other just and equitable
reliefs.
SEC. 6. Issuance of the Writ. Upon the filing of the petition, the court,
justice or judge shall immediately order the issuance of the writ if on its
face it ought to issue. The clerk of court shall issue the writ under the
seal of the court; or in case of urgent necessity, the justice or the judge
may issue the writ under his or her own hand, and may deputize any
officer or person to serve it.
The writ shall also set the date and time for summary hearing of the
petition which shall not be later than seven (7) days from the date of its
issuance.
SEC. 7. Penalty for Refusing to Issue or Serve the Writ. A clerk of
court who refuses to issue the writ after its allowance, or a deputized
person who refuses to serve the same, shall be punished by the court,
justice or judge for contempt without prejudice to other disciplinary
actions.
SEC. 8. How the Writ is Served. The writ shall be served upon the
respondent by a judicial officer or by a person deputized by the court,
justice or judge who shall retain a copy on which to make a return of
service. In case the writ cannot be served personally on the respondent,
the rules on substituted service shall apply.
SEC. 9. Return; Contents. Within seventy-two (72) hours after service
of the writ, the respondent shall file a verified written return together
with supporting affidavits which shall, among other things, contain the
following:
a.

b.

c.

d.

The lawful defenses to show that the respondent did not


violate or threaten with violation the right to life, liberty and
security of the aggrieved party, through any act or omission;
The steps or actions taken by the respondent to determine
the fate or whereabouts of the aggrieved party and the
person or persons responsible for the threat, act or
omission;
All relevant information in the possession of the respondent
pertaining to the threat, act or omission against the
aggrieved party; and
If the respondent is a public official or employee, the return
shall further state the actions that have been or will still be
taken:
i.
ii.

to verify the identity of the aggrieved party;


to recover and preserve evidence related to the
death or disappearance of the person identified
in the petition which may aid in the prosecution
of the person or persons responsible;

iii.
iv.

v.
vi.

to identify witnesses and obtain statements from


them concerning the death or disappearance;
to determine the cause, manner, location and
time of death or disappearance as well as any
pattern or practice that may have brought about
the death or disappearance;
to identify and apprehend the person or persons
involved in the death or disappearance; and
to bring the suspected offenders before a
competent court.

The return shall also state other matters relevant to the investigation,
its resolution and the prosecution of the case.
A general denial of the allegations in the petition shall not be allowed.
SEC. 10. Defenses not Pleaded Deemed Waived. All defenses shall
be raised in the return, otherwise, they shall be deemed waived.
SEC. 11. Prohibited Pleadings and Motions. The following pleadings
and motions are prohibited:
a.
b.
c.
d.
e.
f.
g.
h.
i.
j.
k.
l.

Motion to dismiss;
Motion for extension of time to file return, opposition,
affidavit, position paper and other pleadings;
Dilatory motion for postponement;
Motion for a bill of particulars;
Counterclaim or cross-claim;
Third-party complaint;
Reply;
Motion to declare respondent in default;
Intervention;
Memorandum;
Motion for reconsideration of interlocutory orders or
interim relief orders; and
Petition for certiorari, mandamus or prohibition against any
interlocutory order.

SEC. 12. Effect of Failure to File Return. In case the respondent fails
to file a return, the court, justice or judge shall proceed to hear the
petition ex parte.
SEC. 13. Summary Hearing. The hearing on the petition shall be
summary. However, the court, justice or judge may call for a
preliminary conference to simplify the issues and determine the
possibility of obtaining stipulations and admissions from the parties.
The hearing shall be from day to day until completed and given the
same priority as petitions for habeas corpus.
SEC. 14. Interim Reliefs. Upon filing of the petition or at anytime
before final judgment, the court, justice or judge may grant any of the
following reliefs:
(a) Temporary Protection Order. The court, justice or judge, upon
motion or motu proprio, may order that the petitioner or the aggrieved
party and any member of the immediate family be protected in a
government agency or by an accredited person or private institution
capable of keeping and securing their safety. If the petitioner is an
organization, association or institution referred to in Section 3(c) of this
Rule, the protection may be extended to the officers involved.
The Supreme Court shall accredit the persons and private institutions
that shall extend temporary protection to the petitioner or the
aggrieved party and any member of the immediate family, in
accordance with guidelines which it shall issue.
The accredited persons and private institutions shall comply with the
rules and conditions that may be imposed by the court, justice or judge.
(b) Inspection Order. The court, justice or judge, upon verified
motion and after due hearing, may order any person in possession or
129 | P

LATON

control of a designated land or other property, to permit entry for the


purpose of inspecting, measuring, surveying, or photographing the
property or any relevant object or operation thereon.
The motion shall state in detail the place or places to be inspected. It
shall be supported by affidavits or testimonies of witnesses having
personal knowledge of the enforced disappearance or whereabouts of
the aggrieved party.
If the motion is opposed on the ground of national security or of the
privileged nature of the information, the court, justice or judge may
conduct a hearing in chambers to determine the merit of the
opposition.
The movant must show that the inspection order is necessary to
establish the right of the aggrieved party alleged to be threatened or
violated.
The inspection order shall specify the person or persons authorized to
make the inspection and the date, time, place and manner of making
the inspection and may prescribe other conditions to protect the
constitutional rights of all parties. The order shall expire five (5) days
after the date of its issuance, unless extended for justifiable reasons.
(c) Production Order. The court, justice or judge, upon verified motion
and after due hearing, may order any person in possession, custody or
control of any designated documents, papers, books, accounts, letters,
photographs, objects or tangible things, or objects in digitized or
electronic form, which constitute or contain evidence relevant to the
petition or the return, to produce and permit their inspection, copying
or photographing by or on behalf of the movant.
The motion may be opposed on the ground of national security or of
the privileged nature of the information, in which case the court, justice
or judge may conduct a hearing in chambers to determine the merit of
the opposition.
The court, justice or judge shall prescribe other conditions to protect
the constitutional rights of all the parties.
(d) Witness Protection Order. The court, justice or judge, upon
motion or motu proprio, may refer the witnesses to the Department of
Justice for admission to the Witness Protection, Security and Benefit
Program, pursuant to Republic Act No. 6981.
The court, justice or judge may also refer the witnesses to other
government agencies, or to accredited persons or private institutions
capable of keeping and securing their safety.
SEC. 15. Availability of Interim Reliefs to Respondent. Upon verified
motion of the respondent and after due hearing, the court, justice or
judge may issue an inspection order or production order under
paragraphs (b) and (c) of the preceding section.
A motion for inspection order under this section shall be supported by
affidavits or testimonies of witnesses having personal knowledge of the
defenses of the respondent.
SEC. 16. Contempt. The court, justice or judge may order the
respondent who refuses to make a return, or who makes a false return,
or any person who otherwise disobeys or resists a lawful process or
order of the court to be punished for contempt. The contemnor may be
imprisoned or imposed a fine.
SEC. 17. Burden of Proof and Standard of Diligence Required. The
parties shall establish their claims by substantial evidence.
The respondent who is a private individual or entity must prove that
ordinary diligence as required by applicable laws, rules and regulations
was observed in the performance of duty.
The respondent who is a public official or employee must prove that
extraordinary diligence as required by applicable laws, rules and
regulations was observed in the performance of duty.
The respondent public official or employee cannot invoke the
presumption that official duty has been regularly performed to evade
responsibility or liability.

SEC. 18. Judgment. The court shall render judgment within ten (10)
days from the time the petition is submitted for decision. If the
allegations in the petition are proven by substantial evidence, the court
shall grant the privilege of the writ and such reliefs as may be proper
and appropriate; otherwise, the privilege shall be denied.
SEC. 19. Appeal. Any party may appeal from the final judgment or
order to the Supreme Court under Rule 45. The appeal may raise
questions of fact or law or both.
The period of appeal shall be five (5) working days from the date of
notice of the adverse judgment.
The appeal shall be given the same priority as in habeas corpus cases.
SEC. 20. Archiving and Revival of Cases. The court shall not dismiss
the petition, but shall archive it, if upon its determination it cannot
proceed for a valid cause such as the failure of petitioner or witnesses
to appear due to threats on their lives.
A periodic review of the archived cases shall be made by the amparo
court that shall, motu proprio or upon motion by any party, order their
revival when ready for further proceedings. The petition shall be
dismissed with prejudice upon failure to prosecute the case after the
lapse of two (2) years from notice to the petitioner of the order
archiving the case.
The clerks of court shall submit to the Office of the Court Administrator
a consolidated list of archived cases under this Rule not later than the
first week of January of every year.
SEC. 21. Institution of Separate Actions. This Rule shall not preclude
the filing of separate criminal, civil or administrative actions.
SEC. 22. Effect of Filing of a Criminal Action. When a criminal action
has been commenced, no separate petition for the writ shall be filed.
The reliefs under the writ shall be available by motion in the criminal
case.
The procedure under this Rule shall govern the disposition of the reliefs
available under the writ of amparo.
SEC. 23. Consolidation. When a criminal action is filed subsequent to
the filing of a petition for the writ, the latter shall be consolidated with
the criminal action.
When a criminal action and a separate civil action are filed subsequent
to a petition for a writ of amparo, the latter shall be consolidated with
the criminal action.
After consolidation, the procedure under this Rule shall continue to
apply to the disposition of the reliefs in the petition.
SEC. 24. Substantive Rights. This Rule shall not diminish, increase or
modify substantive rights recognized and protected by the Constitution.
SEC. 25. Suppletory Application of the Rules of Court. The Rules of
Court shall apply suppletorily insofar as it is not inconsistent with this
Rule.
SEC. 26. Applicability to Pending Cases. This Rule shall govern cases
involving extralegal killings and enforced disappearances or threats
thereof pending in the trial and appellate courts.
SEC. 27. Effectivity. This Rule shall take effect on October 24, 2007,
following its publication in three (3) newspapers of general circulation.

Chapter 17
Speedy Disposition of Cases
All persons shall have the right to a speedy disposition of their
cases before all judicial, quasi-judicial, or administrative
27
bodies.

27

CONSTITUTION, Art. III, 16

130 | P

LATON

In order to ensure the right to expeditious determination of


cases is not limited to trials in criminal proceedings, the
Constitution has explicitly provided for the same also in
proceedings before judicial, quasi-judicial and administrative
bodies. This protection extends to all citizens, including those
in the military and covers the periods before, during and after
trial, affording broader protection that Section 14(2) which
guarantees merely the right to a speedy trial. In any event, the
rationale for both Section 14(2) and Section 16 of Article III of
the Constitution is the same, justice delayed is justice denied
The importance of speedy disposition of cases could not be
overstated. The office of a judge exists for one solemn end--to
promote the ends of justice by administering it speedily and
impartially.Nevertheless, the desirability of speed and
expediency in the disposition of cases should not ride
roughshod on the rights of the litigants
In any event, the guarantee of speedy disposition of cases does
not mean a mechanical act of simply counting days or months.
It is a relative and flexible concept that accommodates and
adapts to the various circumstances that may go into the
proceedings and the issues they bring. What it simply ensures is
freedom from arbitrary, oppressive, vexatious and
unreasonable delays, not absolute right to a specific period.
Expounding on this right which was first explicitly provided for
in the 1973 Constitution, the Court aid in Caballero v. Alfonso,
Jr.:
The guarantee of the right to "a speedy disposition of cases," which the
Constitution expressly provides, recognizes the truism that justice
delayed can mean justice denied. Likewise, the broad sweep that the
guarantee comprehends, when it provides that the right is available
before all judicial, quasi-judicial or administrative bodies, confirms that
the application of the immunity from arbitrary and oppressive delays is
not limited to an accused in a criminal proceeding but extends to all
parties and in all cases. Hence, under the constitutional provision, any
party to a case may demand expeditious action on the part of all who
are officially tasked with the proper administration of justice.
However, "speedy disposition of cases" is a relative term. Just like the
constitutional guarantee of "speedy trial" accorded an accused in all
criminal proceedings, "speedy disposition of cases" is a flexible
concept. It is consistent with delays and depends upon the
circumstances. What the Constitution prohibits are unreasonable,
arbitrary and oppressive delays which render rights nugatory.
In the determination of whether or not the right to a "speedy trial" has
been violated, certain factors may be considered and balanced against
each other. These are length of delay, reason for the delay, assertion of
the right or failure to assert it, and prejudice caused by the delay. The
same factors may also be considered in answering judicial inquiry
whether or not a person officially charged with the administration of
justice has violated the "speedy disposition of cases" guarantee.
To strike down a law on the ground that it violates the guarantee of
"speedy disposition of cases" requires more than a citation of what may
be a misfeasance or malfeasance of a public officer whose duty and
responsibility it is to apply and administer the law. The challenge must
be based on a clear showing that it is the law, or its operation, and not
merely its administration, which invades and impairs constitutionally
protected personal or property rights. In the case at bar, it is true that
the referral of cases to the Department of Agrarian Reform opens the
door to more bureaucratic red tape and, perhaps, more opportunities
for corrupt practices. The defects in the bureaucratic system do not,
however, constitute valid arguments against the merits of legislative
policy intended to protect the legitimate tenant-tiller. Besides, it is not
for this Court to determine the wisdom of PD 1038. This is a matter left
for Congress to re-examine in the exercise of its legislative authority.

TATAD V. SANDIGANBAYAN
159 SCRA 70 (1988)
FACTS: Antonio de los Reyes, charged petitioner, who was then
Secretary and Head of the Department of Public Information, with
alleged violations of RA 3019, otherwise known as the Anti-graft and
Corrupt Practices Act. In January 1980, the resignation of petitioner was
accepted by President Marcos. Four months later, the Tanodbayan
referred the complaint of delos Reyes to the Criminal Investigation
Servive (CIS) for fact-finding investigation. The CIS investigator
submitted his Investigation Report concluding that evidence indicates
that petitioner had violated Sec3(e) and Sec 7 of RA 3019, and
recommended appropriate legal action on the matter. In October 1982,
all affidavits and counter-affidavits were with the Tanodbayan for final
disposition but it was only in July 1985 when the Tanodbayan approved
a resolution recommending that several informations be filed against
the petitioner before the Sandiganbayan. Petitioner filed with the
Sandiganbayan a consolidated motion to quash the information
predicated on several grounds including deprivation of due process of
law and of the right to a speedy disposition of the cases filed against
him.
ISSUE: Whether or not petitioner was deprived of his constitutional
right to due process and the right to speedy disposition of the cases
against him.
THE COURTS RULE: After a careful review of the facts and
circumstances of this case, we are constrained to hold that the
inordinate delay in terminating the preliminary investigation and the
filing of the information in the instant case is violative of the
constitutionally granted right of the petitioner to due process and to a
speedy disposition of the cases against him. Accordingly, the
informations in the Criminal Cases should be DISMISSED.
REASON:
A painstaking review of the facts cannot but leave the impression
that political motivations played a vital role in activating and propelling
the prosecutorial process in this case.
The long delay in resolving the case under preliminary investigation
cannot be justified on the basis of the facts on record. The law (PD 911)
prescribes a ten-day period for the prosecutor to resolve a case under
preliminary investigation by him from its termination.
A delay of close to three years cannot be deemed reasonable or
justifiable in the light of the circumstance obtaining in the case at bar.
CADALIN V. POEAS ADMINISTRATOR
238 SCRA 721 (1994)
FACTS: Petitioners Cadalin, Amul and Evagelista, in their own behalf
and on behalf of 728 other OCWs what instituted a class suit for money
claims before the POEA arising from their recruitment by Asia
International Builders Corporation (AIBC) and overseas employment by
Brown & Root International Inc. (BRII). In 1989, the POEA rendered its
decision. This was appealed to the NLRC which promulgated its decision
more than two years later. One of the issues raised before the SC is the
alleged violation of the right to speedy disposition of cases.
HELD:
Speedy disposition of cases is a flexible concept, what the
Constitution prohibits are unreasonable, arbitrary and oppressive
delays which render rights nugatory.
The final disposition in the administrative level after seven years
from their inception, cannot be said to be attended by unreasonable,
arbitrary and oppressive delays (complaints undergoing several
amendments, 1,767 claimants, fights between lawyers of complainants)
as to violate the constitutional rights to a speedy disposition of the
cases of complainants.

Well, one could only speed up so much without burning


oneself. Depending on the circumstances, each case would
have to be viewed in its own pace and not simply rushed for the
131 | P

LATON

sake of speed. And, as illustrated by Cadalin, delay could be due


to the proceedings themselves but also to in-fighting or
internecine strife among lawyers on the same side of the fence
Chapter 18
Privilege Against Self-Incrimination
No person shall be compelled to be a witness against
28
himself
Respect for, and recognition of, mans individuality and right to
be left alone may be said to be implicated in this right. It is a
form of a civilized protest against the use of torture in
extorting confessions. It prescribes an option of refusal to
answer incriminating questions and not a prohibition of
inquiry.
History, Development and Policy Considerations
The privilege is one of those advances in the progress of man,
from the barbarous methods of extracting confessions from
unwilling lips to the more humane ways of procuring evidence
which do not rely on the person sought to be prosecuted. It
may as well be reflective of the direction by which a
government deals with and treats its citizens and inhabitants
It was established on the grounds of public policy and
humanity--of policy, because if the party were required to
testify, it would place the witness under the strongest
temptation to commit the crime of perjury, and of humanity,
because it would prevent the extorting of confessions by duress
It had its origin in a protest against the inquisitorial methods of
interrogating the accused person, which had long obtained in
the continental system
The privilege against self-incrimination registers an important
advance in the development of our liberty--one of the great
landmarks in mans struggle to make himself civilized

under duress. Thus, it would not be implicated if it is physical


evidence that is sought to be taken from a person or otherwise
sought to be produced, except when these are personal papers.
The seizure of books and documents by means of a search
warrant, for the purpose of using them as evidence in a criminal
case against the person in whose possession they were found,
is unconstitutional because it makes the warrant unreasonable,
and it is equivalent to a violation of the constitutional provision
prohibiting the compulsion of an accused to testify against
himself. What is actually proscribed is the use of physical or
moral compulsion to extort communication from the accused
and not the inclusion of his body in evidence when it may be
material
VILLAFLOR V. SUMMERS
41 Phil. 62 (1920)
FACTS: Emeteria Villaflor and Florentino Souingco are charged with the
crime of adultery. The court ordered the defendant Villaflor, herein
petitioner, to submit her body to the examination of one or two
competent doctors to determine if she was pregnant or not.
ISSUE: Whether or not compelling a woman to submit her body to be
examined by physicians to determine if she was pregnant violates the
provision that no person shall be compelled in any criminal case to be a
witness against himself.
THE COURTS RULE: Although the order of the trial judge, acceding to
the request of the assistant fiscal for an examination on the person of
the defendant by physicians is phrased in absolute terms, it should,
nevertheless, be understood as subject to the limitations herein
mentioned, and therefore, legal. The writ of habeas corpus prayed for
is hereby DENIED.
REASON:
Once again we lay down the rule that the constitutional guaranty,
that no person shall be compelled in any criminal case to be a witness
against himself, is limited to a prohibition against compulsory
testimonial self-incrimination.
It is a reasonable presumption that in an examination by reputable
and disinterested physicians due care will be taken not to use violence
and not to embarrass the patient any more than is absolutely
necessary.

Scope of Privilege
In determining the circumstances in which the privilege may be
invoked, one would have to take into account the nature of the
evidence, the personality of the person invoking it, and the
proceeding involved. Parenthetically, it is available to one who
might be guilty or one who simply professes innocence. Not
because one claims no wrongdoing that he can not invoke the
privilege to remain silent
The right, while ordinarily available only in criminal
prosecutions, extends to all other government proceedings-including civil actions, legislative investigations, and
administrative proceedings that possess a criminal or penal
aspect--but not to private investigations done by private
individuals
Testimonial v. Physical Evidence
The privilege is basically directed at testimonial evidence or any
evidence communicative in nature acquired from the accused
CONSTITUTION, Art. III, 17. In the American Bill of Rights, this is among
those rights guaranteed by the Fifth Amendment, which provides: No
person...shall be compelled in any criminal case to be a witness against
himself,...
28

BELTRAN V. SAMSON
53 Phil. 570 (1929)
FACTS: For the purpose of comparing the petitioners handwriting and
determining whether or not it was he who wrote certain documents
supposed to be falsified, the fiscal petitioned the lower court to order
herein petitioner to appear before the former to take dictation in
petitioners own handwriting. Petitioner then filed a petition for
prohibition seeking to enjoin the order of the lower court.
ISSUE: Whether the writing from the fiscals dictation by the petitioner
for purpose of comparing the latters handwriting and determining
whether he wrote certain documents supposed to be falsified,
constitutes evidence against himself within the scope and meaning of
the constitutional provision under examination.
THE COURTS RULE: Wherefore, we find the present action well taken,
and it is ordered that the respondents and those under their orders
desist and abstain absolutely and forever from compelling petitioner to
take down dictation in his handwriting for the purpose of submitting
the latter for comparison.
REASON:
As to its scope, the privilege is not limited precisely to testimony, but
extends to all giving or furnishing of evidence.
It is the duty of the courts liberally to construe the prohibition in
favor of personal rights, and to refuse permit any steps tending toward
132 | P

LATON

their invasion. Hence, there is the well-established doctrine that the


constitutional inhibition is directed not merely to giving of oral
testimony but embraces as well the furnishing of evidence by other
means than by word of mouth, the divulging, in short, of any fact which
the accused has a right to hold secret.
In the case at bar, the question deals with something not yet in
existence, and it is precisely sought to compel the petitioner to make,
prepare, or produce by this means, evidence not yet in existence; in
short, to create this evidence which may seriously incriminate him.
* The privilege not to give self-incriminating evidence, while absolute
when claimed, may not be waived by any one entitled to invoke it.

Related to the foregoing is the issue as to whether requiring a


suspect to say something in order to hear and identify his voice
may violate the privilege. In United States v. Wade, the U.S.
Supreme Court again held that there was no such violation,
holding thus:
We have no doubt that compelling the accused merely to
exhibit his person for observation by a prosecution witness
prior to trial involves no compulsion of the accused to give
evidence having testimonial significance. It is compulsion of the
accused to exhibit his physical characteristics, not compulsion
to disclose any knowledge he might have
We held in Schmerber,...that the distinction to be drawn
between an accuseds communications, in whatever form,
vocal or physical, and compulsion which makes a suspect or
accused the source of real or physical evidence
Reenactments
A person who is made to re-enact a crime may also right fully
invoke his privilege against self-incrimination, because by his
conduct of acting out how the crime was supposedly
committed, he thereby practically confesses his guilt by action
which is as eloquent, if not more so, than words
PEOPLE V. OLVIS
154 SCRA 513 (1987)
FACTS: Following the report on September 9, 1975 by the siblings of
Deosdedit Bagon that he was missing, an inquiry was conducted by the
police of Polanco, Zamboanga del Norte. After a volunteer informed the
police that Bagon was last seen together with Sorela, one of the
accused appellants, the police went to pick him up. Then, Sorela
admitted having participated together with Romulo Villarojo and
Leonardo Cademas(co-accused) in the killing of the missing Bagon. The
policemen thereafter made the three re-enact the crime. Initial findings
of investigators disclosed that the threesome of Solero, Villarojo and
Cademas executed Bagon on orders of accused Anacleto Olvis, then the
mayor of Polanco. The three appellants, together with Olvis, were all
charged with murder. After trial, Olvis was acquitted and the appellants
were found guilty and sentenced to death.
ISSUE: Whether or not the three accused-appellants EXTRAJUDICIAL
CONFESSIONS are inadmissible in evidence.
THE COURTS RULE: The accused appellants Leonardo Cademas and
Dominador Sorela are ACQUITTED on the ground of reasonable doubt.
The accused-appellant Romulo Villarojo is found guilty of homicide, and
is sentenced to suffer an indeterminate penalty of eight years and one
day of prision mayor as minimum, to fourteen years, eight months and
one day of reclusion temporal as maximum.
REASON:
Forced re-enactments, like uncounselled and coerced confessions
come within the ban against self-incrimination. We hold that evidence
based on such a re-enactment to be in violation of the Constitution and

hence, incompetent evidence.


The constitutional privilege has been defined as a protection against
testimonial compulsion, but this has been since then extended to any
evidence communicative in nature acquired under circumstances of
duress.
*This should be distinguished from mechanical acts the accused is
made to execute not meant to unearth undisclosed facts but to
ascertain physical attributes determinable by simple observation
(extract virus from body, pregnancy test, expectorate morphine from
his mouth, footprinting test). In this case, the accused does not speak of
his guilt.

Diverse Proceedings, Different Roles--Accused, Respondents and


Witnesses
The privilege applies to all kinds of proceedings in which an
incriminating statement may be obtained. Thus, while primarily
it has direct application in criminal proceedings, the privilege
against self-incrimination may also be invoked in civil or
administrative proceedings if the questions asked would tend
to elicit an incriminating answer. It is available to him in a
pending criminal case in court, or, in a preliminary investigation
before the public prosecutors office. Insofar as accused and
witnesses are concerned, the privilege means that the former
may absolutely refuse to take the witness stand while the latter
may only refuse to answer incriminatory questions when asked
An accused when he takes the witness stand correspondingly
waives his right against self-incrimination, at least in so far as
relating to those matters in which he testifies about. Thus, he
may be cross-examined on matters he testified about and he
cannot thereafter refuse to answer questions just because they
may tend to incriminate him. He simply cannot eat his cake and
have it, too. Nevertheless, if he is asked an incriminatory
question in regard to another crime, he may avail of his
privilege against self-incrimination
Another exception to the right of an accused not to take the
witness stand is when he is covered by an immunity statute, in
which case he may not refuse to answer incriminatory
questions
It has also been held that this privilege is something that the
person invoking it is supposed to know. Thus, it is not
incumbent on the judge to remind or warn him about possibly
incriminating questions and answers that may come his way.
The constitutional provision-does not impose on the judge, or other officer presiding over a
trial, hearing or investigation, any affirmative obligation to
advise a witness of his right against self-incrimination. It is a
right that a witness knows or should know, in accordance with
the well known axiom that every one is presumed to know the
law, that ignorance of the law excuses no one. Furthermore, in
the very nature of things, neither the judge nor the witness can
be expected to know in advance the character or effect of a
question to be put to the latter
The right against self-incrimination is not self- executing or
automatically operational. It must be claimed. If not claimed by
or in behalf of the witness, the protection does not come into
play. It follows that the right may be waived, expressly, or
impliedly, as by a failure to claim it at the appropriate time.

133 | P

LATON

early Philippine case, the Court said: [T]o compel a person to


produce his private papers to be used in evidence against him
would be equivalent to compelling him to be a witness against
himself

Corporations and Self-Incrimination

Regulatory Reporting Requirements


In an increasing complex society, the government may require
certain reports to be made every now and then for the more
efficient management and ordering of life for everyone, or
simply out of necessity and inevitable consequence of having
multifarious activities and concerns. Thus, papers and reports
have to [be] filed and made, income and other taxable
transactions and occurrences have to be recorded and
reported, and other studies undertaken and submitted for
monitoring, revision or approval by the appropriate
government agencies. In doing so, some aspect of such
requirements may be perceived as providing incriminating
information

Silence and Guilt


The privilege should be understood to mean that invoking it is
not a sign or admission of guilt. Silence should not be read as an
implied incrimination in itself, otherwise the right would
become meaningless since he means by which one tries to keep
himself away from prison would become the very vehicle which
he is delivered
It has also been held that [e]xercise of such privilege can
neither be equated with guilt nor be treated as a forbidden
failure to cooperate with a proper inquiry and used by
government as the basis for adverse treatment, including denial
of a public benefit
Nevertheless, the foregoing should be distinguished from a
situation in which a person keeps quiet when incriminating
statements are being made by others and there is a failure to
say anything in refutation
Rule 130, Section 32 of the Rules of Court provides that an act
or declaration made in the presence and within the hearing or
observation of a party who does or says nothing when the act
or declaration is such as naturally to call for action or comment
if not true, and when proper and possible for him to do so, may
be given in evidence against him
The problem here is in determining when silence is golden, and
when it is obviously counterproductive. It should ever be
remembered, however, that the privilege of silence is there to
be availed of and not to provide unnecessarily adverse
inferences simply because it was exercised
Self-Incrimination and Unreasonable Searches
The privilege against self-incrimination is also directly related to
the right against unreasonable searches and seizures. In an

It is elementary that the right against self-incrimination has no


application to juridical persons. The basic reason for this is the
fact that corporations are mere creatures by sufferance of the
State. They could have no existence or personality if they were
not allowed and recognized by the government. The U.S.
Supreme Court explained:
Historically, private corporations have been subject to broad
visitorial power, both in England and in this country. And it long
has been established that Congress may exercise wide
investigative power over them, analogous to the visitorial
power of the incorporating state, when their activities take
place within or affect interstate commerce. Correspondingly, it
has been settled that corporations are not entitled to all of the
constitutional protections which private individuals have in
these and related matters. As has been noted, they are not at
all within the privilege against self-incrimination, although this
Court more than once has said that the privilege runs very
closely with the Fourth Amendment's search and seizure
provisions. It is also settled that an officer of the company
cannot refuse to produce its records in his possession upon the
plea that they either will incriminate him or may incriminate it
Immunity Statutes
While the Bill of Rights guarantee may give the impression that
the privilege against self-incrimination is absolute, the courts
have recognized that a person may still be compelled to testify
even if it will incriminate him in the process. In exchange of
allowing the government to obtain necessary evidence which
may lead to an admission by a person of his criminal
wrongdoing, it is required that a protection equivalent or coextensive to that guaranteed by Self-Incrimination Clause be
accorded the witness. This comes in the form of the so-called
immunity statutes, which may either be in the nature of a use
or derivative use immunity or a transactional immunity. The
former simply means that whatever s elicited from the witness,
as well as any other evidence which the investigators were led
to because of the testimony given, would not be admissible in
evidence against the witness. In the latter, the witness is
immunized from prosecution in relation to the crime in which
he was compelled to provide testimony

In Commission on Elections v. Tagle, the Supreme Court upheld


the right of the Commission on Elections to grant immunity to
those who voluntarily give information against those
responsible for vote-buying
Incrimination in Foreign Jurisdictions
134 | P

LATON

May the privilege be invoked if the possibility of incrimination is


in regard to criminal laws of other countries and not within the
jurisdiction of the State in which the proceedings are held? In
United States v. Balsys, the U.S. Supreme Court said no--at
least, for the moment

The Bill of Rights guarantees the freedom to freely speak but


recognizes as well the moment when one may be better off
being silent--for his own protection. Either way, he could pick
on the menu of rights and freedoms with which to feast as he
goes along enjoying or simply living life in a libertarian society
Additional Cases
(P) Self-Incrimination Clause [3]

a) those committed while he was in the service of the Marcos


government; b) those committed in behalf of the Marcos government;
and c) any other act revealed by him in the course of his cooperation
with the PCGG.
2. Yes. EO No. 14 defines the jurisdiction over cases involving the illgotten wealth of former President Ferdinand E. Marcos, Mrs. Imelda R.
Marcos, members of their immediate family, close relatives,
subordinates, close and/or business associates, dummies, agents and
nominees. SECTION 5, as amended authorizes the PCGG to grant
immunity from criminal prosecution. iT also does not provide any
express limitations as to the scope of immunity from criminal
prosecution that the PCGG is authorized to grant. The qualifications
that Section 5 do provide relate to the character of the information or
testimony before the PCGG of the grantee of immunity.
3. No. Section 5 of E.O. 14-A does not make any qualification as to
classes of criminal acts, offenses, or cases, it COMPLETELY immunizes
from prosecution.
P3. Disini v. Sadiganbayan
GR. No. 180564 June 22 2010

P2. Tanchanco v. Sandiganbayan


476 SCRA 202 (2005)
Facts: Tanchanco was the NFA Administrator from 1972-1986 during
the Marcos regime and his co-petitioner, Romeo Lacson was the
Deputy Administrator of the NFA. On May 6, 1988, Tanchanco and the
PCGG entered in to a Cooperation Agreement wherein Tanchanco
would cooperate with the RP in locating and pursuing government
properties stolen by the Marcos family. It was also agreed that RP
would dismiss all cases against Tanchanco pending before the
Sandiganbayan, lift any sequestration orders against Tanchanco's
properties, if any, rescind hold orders it may have issued against his/her
actions, and shall not bring any civil or criminal charges against him
arising from his service for the Marcos government and any other
actions revealed by him. Thereafter, Tanchanco was called upon to
testify in a case against Imelda Marcos for violation of the RICO Act
wherein he elicited concerning the transfer of P10 M rebate obtained
by the NFA from the Philippine National Lines to the Security Bank.
Although he cooperated, a criminal case was filed against him with the
Sandiganbayan for malversation of public funds amounting to P10 M
from the PNB. He filed a Motion for Reinvestigation, wherein he argued
that the case should be dismissed as he had been granted immunity by
the PCGG. The case was dismissed. In 1997, Tanchanco was charged
with Malversation of Public Funds and Failure of Accountable Officer to
Render Accounts. He filed a motion to quash because the Cooperation
Agreement granted immunity to Tanchanco from criminal prosecution.
Motion was denied. Sandiganbayan claimed that Sec 5 of E.O.
14 empowered the PCGG to grant immunity from criminal prosecution
which pertained only to offenses which may arise from the act of a
person testifying or giving information in connection with the recovery
of supposed ill-gotten wealth and that charges of malversation and
failure to render an accounting could not be considered as falling within
the immunity because they were not related or connected to the
testimony or information furnished by Tanchanco in a proceeding
concerning the recovery of the purported ill-gotten wealth of the
Marcoses. Sandiganbayan declared in the motion for reconsideration
that the grant of immunity to crimes to which petitioners were charged
are beyond the authority and mandate of the PCGG.
Issues:
WON the grant of immunity under the Cooperation Agreement
encompassed the malversation and failure to render accounts charges.
Whether the PCGG, in entering into the Cooperation Agreement, acted
within the scope of its statutory authority to extend immunity in the
first place
Whether the available immunity from criminal prosecution relates only
to the prosecution of the grantee in like minded cases.
Held:
1. Yes. The undertakings expressed in the Cooperation Agreement are
quite clear-cut, even if broad in scope. It is composed of 3 classes of
actions committed by Tanchanco:

Facts: In 1989 respondent RP, represented by PCGG wanted petitioner


Jesus P. Disini to testify for his government in its case against
Westinghouse and an arbitration case which the latter and others filed
against RP before the International Chamber of Commerce Court of
Arbitration. Petitioner was an executive in the companies of his 2nd
cousin, Herminio T. Disini, from 1971-1984. RP believed that
Westinghouse's contract for the construction of the Bataan Nuclear
Power Plant, brokered by one of Herminio's companies, had been
attended by anomalies. On Feb 16, 1989, RP and petitioner entered
into an Immunity Agreement wherein the latter undertook to testify for
his gov't and provide its lawyers the necessary documents and
information. RP also guaranteed that it shall not compel Disini to testify
in any other proceeding brought by RP against Herminio. Petitioner
complied with the said agreement however after 18 yrs, or on Feb 27,
2007, upon application of RP, respondent Sandiganbayan, issued a
subpoena duces tecum and ad testificandum against Disini
commanding him to testify and produce documents before such court
on March 6 and 30 2007 in a case filed by RP against Herminio. Disini
filed a motion to quash subpoena invoking his immunity agreement
with the Republic, but respondent Sandiganbayan ignored the motion
and issued a new subpoena, directing him to testify before it on May 6
and 23, 2007. On July 19, 2007 the PCGG issued a resolution revoking
and nullifying the Immunity Agreement between petitioner Disini and
RP since it prohibited the latter from requiring Disini to testify against
Herminio. On August 16, 2007 respondent Sandiganbayan denied
Disini's motion to quash subpoena,which made the latter take recourse
to this Court.
Issues:
WON the PCGG acted within its authority when it revoked and nullified
the Immunity Agreement between respondent RP and petitioner Disini
WON respondent Sandiganbayan gravely abused its discretion when it
denied petitioner Disini's motion to quash the subpoena addressed to
him.
Held: The guarantee given by PCGG covers only immunity from civil or
criminal prosecution and not immunity from providing evidence in
court. The immunity from the need to testify in other ill-gotten wealth
cases would defeat the very purpose of EO 1 which charged the PCGG
with the task of recovering all ill-gotten wealth of former President
Marcos. Moreover, if he refuses to testify in those other cases, he
would face indirect contempt and would be penalized. His refusal to
testify as ordered by the Sandiganbayan would result in prosecution for
criminal contempt.
RP invokes Sec 15 , Art XI of the 1987 Constitution which provides: "The
right of the State to recover properties unlawfully acquired by public
officials or employees from them or from their nominees, or
transferees, shall not be barred by prescription, laches or estoppel."
However, the estoppel which Disini invokes does not have the effect of
denying the state its right to recover whatever ill-gotten wealth
Herminio may have acquired under the Marcos regime. And although
135 | P

LATON

the government cannot be barred by estoppels, such principle cannot


apply to this case since PCGG acted within its authority when it
provided Disini with a guarantee against having to testify in other cases.
Petition is GRANTED and PCGG resolution is ANNULLED.

Chapter 19
Political Prisoners and Involuntary Servitude
No person shall be detained solely by reason of his political
beliefs and aspirations.
No involuntary servitude in any form shall exist except as
punishment for a crime whereof the party shall be duly
29
convicted.
Consistent with the underlying philosophy of a society which
values individual freedom and self-determination and
guarantees freedom of thought and of the mind, it is only
logical that nobody should be imprisoned solely because of
political beliefs and aspirations
And in a society that also values liberty, involuntary servitude
should be anathema. Individuals are free to make their own
decisions, to move about and realize their dreams. They could
not be held in bondage. Slavery and involuntary servitude,
together with their corollary, peonage, all denote a condition
of enforced, compulsory service of one to another.
The only time when they may be forced to do things, in
accordance with the language of the Constitution, is when they
have been found guilty of a crime for which they may rightfully
be compelled to do things in accordance with their punishment.
Another exception would be in regard to service for the
defense of the country, i.e., [t]he Government may call upon
the people to defend the State and, in the fulfillment thereof,
all citizens may be required, under conditions provided by law,
to render personal military or civil service. Related to this is
the concept of posse comitatus, where able-bodied me may be
called upon to contribute their share in services for the
maintenance of peace and order in their own community
In Aclaracion v. Gatmaitan, a court stenographer, after he had
ceased to be a court stenographer, was compelled to transcribe
his stenographic notes despite hi contention that forcing him to
do so would constitute involuntary servitude. The Court
dismissed such argument by declaring that the situation of
enforced, compulsory service of one to another or the
condition of one who is compelled by force, coercion, or
imprisonment, and against this will, to labor for another,
whether he be paid or not, simply does not obtain
From the earliest historical period, the contract of the sailor has
been treated as an exceptional one, and involving, to a certain
extent, the surrender of his personal liberty during the life of
the contract. Indeed the business of navigation could scarcely
be carried on without some guaranty, beyond the ordinary civil
remedies upon contract that the sailor will not desert the ship
at a critical moment or leave her at some place where seamen
are impossible to be obtained--as Molloy forcibly expresses it,
to rot in her neglected brine

CONSTITUTION, Art. III, 18(1) and (2). Its counterpart in the American
Constitution is the Thirteenth Amendment which states: Neither slavery nor
involuntary servitude, except as a punishment for the crime whereof the party
shall have been duly convicted, shall exist within the United States, or any
place subject to their jurisdiction

United States v. Pompeya


31 Phil. 245 (1915)
The Facts: For his alleged failure, without justifiable motive, to render
service on patrol duty, Silvestre Pompeya was charged with the
violation of a municipal ordinance enacted pursuant to Section 40 of
the Municipal Code (Act. No. 82, as amended by Act No. 1309) of Iloilo,
which required each able-bodied male resident of the municipality,
when so required by the municipal president to assist in the
maintenance of peace and good order in the community, by
apprehending ladrones, etc., as well as by giving information of the
existence of such persons in the locality. Upon arraignment, he
presented a demurrer on the ground that the acts charged in the
complaint do not constitute a crime. He argued that the municipal
ordinance alleged to be violated is unconstitutional because it is
repugnant to the Organic Act of the Philippines which guarantee the
liberty of the citizens. The lower court sustained said demurrer and
ordered the dismissal of said complaint. The prosecuting attorney
appealed.
The Issue: Whether or not the ordinance upon which said complaint
was based (paragraph m of section 40 of the Municipal Code) which
was adopted in accordance with the provisions of Act No. 1309 is
constitutional.
Held: To authorize the municipal president to require able-bodied male
residents of the municipality, between ages of eighteen and fifty years,
to assist, for a period not exceeding five days in any one month, in
apprehending ladrones, robbers and other lawmakers and suspicious
characters, and to act as patrols for the protection of the municipality,
not exceeding one day in each week.
The question we have to consider is whether or not the facts stated in
the complaint are sufficient to show a (a) cause of action under the said
law; and (b) whether or not said law is in violation of the provisions of
the Philippine Bill in depriving citizens of their rights therein
guaranteed.
We are of the opinion, and so hold, that the power exercised under the
provisions of Act No. 1309 falls within the police power if the state and
that the state was fully authorized and justified in conferring the same
upon the municipalities of the Philippine Islands, and that, therefore,
the provisions of said act and not in violation nor derogation of the
rights of the persons affected thereby.
With reference to the first question presented by the appeal, relating to
the sufficiency of the complaint, it will be noted that Act No. 1309
authorized the municipal governments to establish ordinances
requiring (a) all able-bodied male residents, between the ages 18 and
55 [50], and (b) all householders, under certain conditions, to do certain
things.

Of course, in the ultimate analysis, men and women are only as


free as they can let themselves be. The Constitution is there to
guarantee the right but they must take the necessary steps to
assert and exercise their freedoms, safeguard their liberties--as
well as pay the price for their own actions and decisions
Chapter 20
Excessive Fines and Cruel Punishments
Excessive fines shall not be imposed, nor cruel, degrading or
inhuman punishment inflicted. Neither shall death penalty be
imposed, unless, for compelling reasons involving heinous
crimes, the Congress hereafter provides for it. Any death
penalty already imposed shall be reduced to reclusion
perpetua.

29

The employment of physical, psychological, or degrading


punishment against any prisoner or detainee or the use of
136 | P

LATON

substandard or inadequate penal facilities under subhuman


30
conditions shall be dealt with by law.
Society, to be viable, has to have order and peace in the
community. For this, purpose, it has to come up with rules of
conduct by which certain acts or activities are either required
to be done, or prohibited from being engaged in. If these rules
are violated, and after the transgressor shall have been
proceeded against in accordance with the proper procedure,
the corresponding punishment may then be imposed on them
as a way of vindicating societys rights, and those of the
offended party, or for the purpose of deterring future wrong
doings or infractions. But not because the State has the right
to criminalize certain acts or omissions and penalize
infractions of its rules and regulations does it mean that it
may impose any and all kinds of penalties or punishments,
irrespective of their reasonableness, severity, or
proportionality to the offense committed. There is also merit
in the view that punishment inflicted beyond the merit of the
offense is so much punishment of innocence. Speaking of the
Eight Amendment, the U.S. Supreme Court said that the basic
concept underlying the constitutional guarantee is nothing less
than the dignity of man x x x
It regulates the power of the State to punish. Thus, it prohibits
the imposition of excessive fines and infliction of cruel,
degrading or inhuman punishments. While it allows the
imposition of the death penalty, it limits it to heinous crimes
only
Origin and Development
"The prohibition in the Philippine Bill against cruel and unusual
punishments is an Anglo-Saxon safeguard against governmental
oppression of the subject, which made its first appearance in
the reign of William and Mary of England in 'An Act declaring
the rights and liberties of the subject, and settling the
succession of the crown,' passed in the year 1689. It has been
incorporated into the Constitution of the United States (of
America) and into most constitutions of the various States in
substantially the same language as that used in the original
statute. The exact language of the Constitution of the United
States is used in the Philippine Bill"
In the 1935 Constitution, there was reference to cruel and
unusual punishment but in the 1973 Constitution it was
modified to cruel or unusual punishment. Then, in the 1987
Constitution it was written as cruel, degrading, or inhuman
punishment. The Bill of Rights Committee of the 1986
Constitutional Commission read the 1973 modification as
prohibiting unusual punishment even if not cruel. It was thus
seen as an obstacle to experimentation in penology.
Consequently, the Committee reported out the present text
which prohibits 'cruel, degrading or inhuman punishment' as
more consonant with the meaning desired and with
jurisprudence on the subject."

punishment rather than its severity in respect of duration or


amount, and apply to punishments which never existed in
America or which public sentiment has regarded as cruel or
obsolete, such as those inflicted at the whipping post, or in the
pillory, burning at the stake, breaking on the wheel,
disemboweling, and the like
Bail, fines, and punishment traditionally have been associated
with the criminal process, and by subjecting the three to
parallel limitations the text of the Amendment suggests an
intention to limit the power of those entrusted with the
criminal-law function of government. An examination of the
history of the Amendment and the decisions of this Court
construing the proscription against cruel and unusual
punishment confirms that it was designed to protect those
convicted of crimes. We adhere to this longstanding limitation
and hold that the Eighth Amendment does not apply to the
paddling of children as a means of maintaining discipline in
public schools
In Zaldivar v. Sandiganbayan, the Court held:
The indefiniteness of the respondent's suspension, far from
being 'cruel' or 'degrading' or 'inhuman' has the effect of
placing, as it were, the key to the restoration of his rights and
privileges as a lawyer in his own hands. That sanction has the
effect of giving respondent the chance to purge himself in his
own good time of his contempt and misconduct by
acknowledging such misconduct, exhibiting appropriate
repentance and demonstrating his willingness and capacity to
live up to the exacting standards of conduct rightly demanded
from every member of the bar and the officer of the courts
In Trop v. Dulles, the U.S. Supreme Court held that expatriation
or denationalization as a punishment is barred by the Eight
Amendment. It declared: There may be involved no physical
mistreatment, no primitive torture. There is, instead, the total
destruction of the individual's status in organized society. It is a
form of punishment more primitive than torture, for it destroys
for the individual the political existence that was centuries in
the development
Weems v. United States
217 U.S. 349, 54 L Ed 793, 30 S Ct 544 (1910)
The Facts: Weems, a disbursing officer of the Coast Guard and
Transportation of the United States Government of the Philippine
Islands was found guilty of falsification of a public and official document
namely, a cash book of the captain of the port of Manila, by entering as
paid out 208 and 408 pesos, as wages of employees of the lighthouse
service of the United States Government of the Philippine Islands. He
was sentenced to the penalty of fifteen years of cadena, together
with the accessories and to pay a fine of 4,000 pesetas. The judgment
and sentence were affirmed by the Philippine Supreme Court. He
appealed to the U.S. Supreme Court. In his fourth assignment of error,
he contends that the penalty imposed on him is violative of the
guarantee of the Philippine Bill of Rights, as contained in the Act of 1
July 1902, against cruel unusual punishments of the Philippine Bill of
Rights.

Excessive Fines and Cruel Punishments


This is primarily addressed to the lawmakers, however, subject
to judicial review. The prohibition of cruel and unusual
punishments is generally aimed at the form or character of the
CONSTITUTION, Art. III, 19(1) and (2). The U.S. counterpart is found in the
Eight Amendment which decrees: Excessive bail shall not be required, not
excessive fines imposed, nor cruel and unusual punishments inflicted
30

The Issue: Whether or not that a punishment of fifteen years of


cadena together with the accessories and to pay a fine of 4,000 pesetas
was a cruel and unusual punishment?
Held: Those sentenced to cadena temporal and cadena perpetua shall
labor for the benefit of the state. They shall aways carry a chain at the
ankle, hanging from the wrists; they shall be employed at hard and
painful labor, and shall receive no assistance whatsoever from without
137 | P

LATON

the institution.

(1)

These provisions are attacked as infringing that provision of the Bill of


Rights of the islands which forbids the infliction of cruel and unusual
punishment.
What constitutes a cruel and unusual punishment has not been exactly
decided. It has been said that, ordinarily, the terms imply something
inhuman and barbarous torture and the like.

(2)

The general rule that a judgment rendered by a court in a criminal


case must conform strictly to the statute, and that any variation from
its provisions, either in the character or the extent of punishment
inflicted, renders the judgment absolutely void.
In cases where the statute makes hard labor a part of the punishment,
it is imperative upon the court to include than in its sentence.
The Philippine Code unites the penalties of cadena temporal, principal
and accessory, and it is not in our power to separate them, even if they
are separable, unless their independence is such that we can say that
their union was not made imperative by the legislature.
It follows from these views that, even if the minimum penalty of
cadena temporal had been imposed, it would have been repugnant to
the Bill of Rights. In other words, the fault is in the law; and, as we are
pointed to no other under which sentence can be imposed, the
judgment must be reversed, with directions to dismiss the proceedings.

Death Penalty
One of the most contentious issues when it comes to
punishments is the constitutionality or perhaps the advisability
and wisdom of the death penalty. The language of the
Constitution itself implies that the supreme penalty may be
provided for by Congress. The issue therefore boils down to the
conditions which must attend the enactment and the
subsequent application of such a law. Before such a law be
enacted, it is required that there be compelling reasons and
that it is for heinous crimes. The problem in determining what
would constitute compelling reasons and defining what are
heinous crimes. Then even after one may have succeeded,
there is still the question as to whether the manner or method
of execution is constitutional, or whether it is within
constitutional limits to apply the death penalty to certain types
of persons
People v. Echegaray
267 SCRA 682 (1997)
The Facts: Accused appellant Leo Echegaray was found guilty if raping
his ten-year-old daughter for he was sentenced to death. The Supreme
Court affirmed the same on appeal. He filed a motion for
reconsideration. Subsequently, he discharged his defense counsel and
retained the services of Anti-Death Penalty Task Force of the Free Legal
Assistance Group (FLAG). Thereafter, FLAG filed a Supplemental Motion
for Reconsideration on behalf of Echegaray. It raised, among the
grounds for reversal of the death sentence, the issue of the
constitutionality of R.A. 7659 (The Death Penalty Law), which took
effect on 31 December 1993, in that it violates the constitutional
prohibition on cruel, inhuman and excessive punishments.
The Issue: Whether or not R.A. 7659 (The Death Penalty Law), which
took effect on 31 December 1993, violates the constitutional
prohibition on cruel, inhuman and excessive punishments.
Held: The penalty complained of is neither cruel, unjust nor excessive.
Punishments are cruel when they involve torture or a lingering death,
but punishment of death is not cruel, within the meaning of that word
as used in the constitution. It implies there something inhuman and
barbarous, something more than the mere extinguishment of life.

that the death penalty law is unconstitutional per se for having


been enacted in the absence of compelling reasons therefore; and
that the death penalty for rape is a cruel, excessive and inhuman
punishment In violation of the constitutional proscription against
punishment of such nature.

We have already demonstrated earlier in our discussion of heinous


crimes that the forfeiture of life simply because life was taken, never
was a defining essence of the death penalty in the context of our legal
history and cultural experience; rather, the death penalty is imposed in
heinous crimes because the perpetrators thereof have committed
unforgivably execrable acts that have so deeply dehumanized a person
or criminal acts with severely destructive effects on the national efforts
to lift the masses from abject poverty through organized governmental
strategies based on a disciplined and honest citizenry.
Capital punishment ought not to be abolished solely because it is
substantially repulsive, if infinitely less repulsive than the acts which
invoke it. Yet the mounting zeal for its abolition seems to arise from a
sentimentalized hyper fastidiousness that seeks to expunge from the
society all that appears harsh and suppressive. If we are to preserve the
humane society, we will have to retain sufficient strength of character
and will to do the unpleasant in order that tranquility and civility may
rule comprehensively. It seems very likely that capital punishment is a
*** necessary, if limited factor in that maintenance of social tranquility
and ought to be retained on its ground. To do otherwise is to indulge in
the luxury of permitting a sense of false delicacy to reign over the
necessity of social survival.
WHEREFORE, in view of all foregoing, the Motion for Reconsideration
and the Supplemental Motion for Reconsideration are hereby DENIED
for LACK OF MERIT.
Echegaray v. Secretary of Justice
297 SCRA 754 (1998)
The Facts: Following the decision in People v. Echegaray, supra.,
petitioner filed a Petition for Prohibition, Injunction and/or Temporary
Restraining Order to enjoin respondents Secretary of Justice and
Director of Bureau of Prisons from carrying out execution of petitioner
by lethal injection.
Contending that these are unconstitutional and void for being, inter
alia, cruel, degrading and inhuman punishment per se as well as
arbitrary, unreasonable and a violation of due process.
Earlier, in 1996, Congress had seen it fit to change the mode of
execution of the death penalty from electrocution to lethal injection,
and passed R.A. No. 8177.
The Issue: Whether or not execution by lethal injection is
unconstitutional and void for being, inter alia, cruel, degrading and
inhuman punishment per se as well as arbitrary, unreasonable and a
violation of due process.
Held:
I. LETHAL INJECTION, NOT CRUEL, DEGRADING OR INHUMAN
PUNISHMENT UNDER SECTION 19, ARTICLE III OF THE 1987
CONSTITUTION
An Act declaring the rights and liberties of the subject, and settling the
succession of the crown
It is well-settled in jurisprudence that the death penalty per se is not a
cruel, degrading or inhuman punishment.
This Court held that [p]unishments are cruel when they involve torture
or a lingering death; but the punishment of death is not cruel, within
the meaning of that word as used in the constitution. It implies there
something inhuman and barbarous, something more than mere
extinguishment of life.
Third paragraph of R.A. No. 8177 which requires that all personnel
138 | P

LATON

involve in execution proceedings should be trained prior to the


performance of such task.
Any infliction of pain in lethal injection is merely incidental in carrying
out the execution of the death penalty and does not fall within the
constitutional proscription against cruel, degrading or inhuman
punishment.
The cruelty against which the Constitution protects a convicted man is
cruelty inherent in the method of punishment, not necessary suffering
involved in any method employed to extinguish life humanely.
We find that the legislatures substitution of the mode of carrying out
the death penalty from electrocution to lethal injection infringes no
constitutional rights of petitioner herein.
II. REIMPOSITION OF THE DEATH PENALTY LAW DOES NOT VIOLATE
INTERNATIONAL TREATY OBLIGATIONS
Second Optional Protocol to the International Covenant on Civil and
Political Rights, Aiming at the Abolition of the death Penalty was
adopted by the General Assembly on December 15, 1989. The
Philippines neither signed nor ratified said document. Evidently,
petitioners assertion of our obligation under the Second Optional
Protocol is misplaced.
WHEREFORE, the petition is DENIED insofar as petitioner seeks to
declare the assailed statute (Republic Act No. 8177) as unconstitutional.
Atkins v. Virginia
536 U.S. 304, 153 L Ed 2d 335, 122 S Ct 2242 (2002)
The Facts: Petitioner, Daryl Renard Atkins, was convicted of abduction,
armed robbery and capital murder, and sentenced to death. Atkins did
not argue before the Virginia Supreme Court that his sentence was
disproportionate to the penalties imposed for similar crimes in Virginia,
but he did contend that he is mentally retarded and thus cannot be
sentenced to death. The Court was not willing to commute Atkins
sentence of death to life imprisonment merely because his IQ score.
The Issue: The American public, legislators, scholars, and judges have
deliberated over the question whether the death penalty should ever
be imposed on a mentally retarded criminal. The
consensus reflected in those deliberation informs our answer to the
question presented by this case: whether such executions are cruel
and unusual punishments prohibited by the Eight Amendment to the
Federal Constitution.
Held: The basic concept underlying the Eight Amendment is nothing
less than the dignity of manThe Amendment must draw its meaning
from the evolving standards of decency that mark the progress of a
maturing society.
Mentally retarded persons frequently know the difference between
right and wrong and are competent to stand trial. Because of their
impairments, however, by definition they have diminished capacities to
understand and process information, to communicate, to abstract form
mistakes and learn from experience, to engage in logical reasoning, to
control impulses, and to understand the reactions of others.
Their deficiencies do not warrant an exemption from criminal
sanctions, but they do diminish their personal culpability.
In light of these deficiencies, our death penalty jurisprudence provides
two reasons consistent with the legislative consensus that the mentally
retarded should be categorically excluded from execution. First, there is
a serious question as to whether either justification that we have
recognized as a basis for the death penalty applies to mentally retarded
offenders.
Mentally retarded defendants may be less able to give meaningful
assistance to their counsel and are typically poor witnesses, and their
demeanor may create an unwarranted impression of lack of remorse

for their crimes.


Our dependent evaluation of the issue reveals no reason to disagree
with the judgment of the legislatures that have recently addressed the
matter and concluded that death is not a suitable punishment for a
mentally retarded criminal. We are not persuaded that the execution of
mentally retarded criminals will measurably advance the deterrent or
the retributive purpose of the death penalty. Construing and applying
the Eight Amendment in the light of our evolving standards of
decency, we therefore conclude that such punishment is excessive and
that the Constitution places a substantive restriction on the States
power to take the life of a mentally retarded offender.

Prison Life and Inmates Rights


Being sent to prison does not necessarily and effectively cut off
all of a persons entitlement to rights and freedom enshrined in
the Constitution. Its most immediate effect would, of course,
be the loss of liberty. As the Court said, in People v. Jalosjos:
More explicitly, "imprisonment" in its general sense, is the
restraint of one's liberty. As a punishment, it is restraint by
judgment of a court or lawful tribunal, and is personal to the
accused. The term refers to the restraint on the personal liberty
of another; any prevention of his movements from place to
place, or of his free action according to his own pleasure and
will. Imprisonment is the detention of another against his will
depriving him of his power of locomotion and it "[is] something
more than mere loss of freedom. It includes the notion of
restraint within limits defined by wall or any exterior barrier."
A prisoner does not shed such basic First Amendment rights at
the prison gate. Rather, he retains all the rights of an ordinary
citizen except those expressly, or by necessary implication,
taken from him by law.
In Wolf v. McDonell, [i]t then declared that the following
safeguards should be present: (1) advance written notice of
charges; (2) written statement by the fact-finders as to the
evidence relied on and reasons for the disciplinary action; and
(3) the prisoner should be allowed to call witnesses and present
documentary evidence in his defense if permitting him to do so
will not jeopardize institutional safety or correctional goals
With respect to prison mail censorship, it was also ruled that
the following criteria be met: First, the regulation or practice in
question must further an important or substantial
governmental interest unrelated to the suppression of
expression. Second, the limitation of First Amendment
freedoms must be no greater than is necessary or essential to
the protection of the particular governmental interest involved.
In this regard, it must be remembered that censorship of
prisoner mail implicates not only the inmates freedom to
communicate but also the addressees own constitutional right
The Courts task then is to formulate a standard of review for
prisoners constitutional claims that is responsive both to the
policy of judicial restraint regarding prisoner complaint and to
the need to protect constitutional rights. It proceeded to
conclude that a lesser standard of review than a strict scrutiny
is appropriate on determining the constitutionality of prison
rules. When a prison regulation impinges on inmates
constitutional rights, the regulation is valid I it is reasonably
related to legitimate penological interests. However, strict
scrutiny is the proper standard of review when it relates to a
policy of prisoner segregation based on race
139 | P

LATON

Procunier v. Martinez
416 U.S. 396, 40 L Ed 2d 224, 94 S Ct 1800 (1974)
The Facts: Appellees, prison inmates, brought this action challenging
prisoner mail censorship regulations issued by the Director of the
California Department of Corrections and the ban against the use of law
students and legal paraprofessionals to conduct attorney-client
interviews with inmates. The regulations also directed inmates not to
write letters in which they unduly complain or magnify grievances,
including contraband writings, namely, those expressing inflammatory,
political, racial, religious or other views of beliefs. The regulations
further provided that inmates may not send or receive letters that
pertain to criminal activity; are lewd, obscene, or defamatory; contain
foreign matter or, are otherwise inappropriate. Prison employees
screened both incoming and outgoing personal mail for violations of
these regulations.
The Issue: The Issue before us is the appropriate standard of review for
prison regulations restricting freedom of speech.
Whether or not prisoner mail censorship regulations constitute a
violation of the Constitutional guaranty against excessive fines and
cruel punishments as well as a violation of any other rights enshrined in
the Constitution.
Held: Applying the teachings of our prior decisions to the instant
context, we hold that censorship of prisoner mail is justified if the
following criteria are met. First, the regulation or practice in question
must further an important or substantial governmental interest
unrelated to the suppression of expression. Second, the limitation of
First Amendment freedoms must be no greater than is necessary or
essential to the protection of the particular governmental interest
involved.
On the basis of this standard, we affirm the judgment of the District
Court. The regulations invalidated by that court authorized, inter alia,
censorship of statements that unduly complain or magnify
grievances, expression of inflammatory political, racial, religious or
other views, and matter deemed defamatory or otherwise
inappropriate. These regulations fairly invited prison officials and
employees to apply their own personal prejudices and opinions as
standards for prisoner mail censorship.
We also agree to the decision of the District Court that the decision to
censor or withhold delivery of a particular letter must be accompanied
by minimum procedural safeguards. The interest of prisoners and their
correspondents in uncensored communication by letter, grounded as it
is in the First Amendment, is plainly a liberty interest within the
meaning of the Fourteenth Amendment even though qualified of
necessity by the circumstances of imprisonment. As such, it is protected
from arbitrary governmental invasion. The District Court required that
an inmate be notified of the rejection of a letter written by or
addressed to him, that the author of that letter be given a reasonable
opportunity to protest that decision, and that complaints be referred to
a prison official other than the person who originally disapproved the
correspondence. These requirements do not appear to be unduly
burdensome, nor do appellants so contend.
The constitutional guarantee of due process of law has a corollary the
requirement that prisoners be afforded access to courts in order to
challenge unlawful convictions and to seek redress for violations of
their constitutional rights. This means that inmates must have a
reasonable opportunity to seek and receive the assistance of attorneys.
Regulations and practices that unjustifiably obstruct the availability of
professional representation or other aspects of the right of access tot
the courts are invalid.
The judgment is Affirmed.

Beyond Prison Walls

Depending on the behavior of the prisoner, he may be made to


serve out the full term of his prison sentence or temporarily be
set free earlier. The latter is accomplished by means of parole.
This temporary liberty, however, is subject to certain conditions
such that if he violates any of them he may soon find himself
back in prison garb. What are the procedural safeguards
attendant to these parole revocation proceedings? The
American Supreme Court dealt with this issue in Morrissey v.
Brewer. In the process at arriving at the answer, it had to
analyze the personal liberty stakes of the parolee as against the
societal interests.
We turn to an examination of the nature of the interest of the parolee
in his continued liberty. The liberty of a parolee enables him to do a
wide range of things open to persons who have never been convicted
of any crime. The parolee has been released from prison based on an
evaluation that he shows reasonable promise of being able to return to
society and function as a responsible, self-reliant person. Subject to the
conditions of his parole, he can be gainfully employed and is free to be
with family and friends and to form the other enduring attachments of
normal life. Though the State properly subjects him to many restrictions
not applicable to other citizens, his condition is very different from that
of confinement in a prison. He may have been on parole for a number
of years, and may be living a relatively normal life at the time he is
faced with revocation. The parolee has relied on at least an implicit
promise that parole will be revoked only if he fails to live up to the
parole conditions. In many cases, the parolee faces lengthy
incarceration if his parole is revoked.
We see, therefore, that the liberty of a parolee, although
indeterminate, includes many of the core values of unqualified liberty
and its termination inflicts a "grievous loss" on the parolee and often on
others. It is hardly useful any longer to try to deal with this problem in
terms of whether the parolee's liberty is a "right" or a "privilege." By
whatever name, the liberty is valuable, and must be seen as within the
protection of the Fourteenth Amendment. Its termination calls for
some orderly process, however informal.
Turning to the question what process is due, we find that the State's
interests are several. The State has found the parolee guilty of a crime
against the people. That finding justifies imposing extensive restrictions
on the individual's liberty. Release of the parolee before the end of his
prison sentence is made with the recognition that, with many prisoners,
there is a risk that they will not be able to live in society without
committing additional antisocial acts. Given the previous conviction and
the proper imposition of conditions, the State has an overwhelming
interest in being able to return the individual to imprisonment without
the burden of a new adversary criminal trial if, in fact, he has failed to
abide by the conditions of his parole.
Yet the State has no interest in revoking parole without some informal
procedural guarantees. Although the parolee is often formally
described as being "in custody," the argument cannot even be made
here that summary treatment is necessary as it may be with respect to
controlling a large group of potentially disruptive prisoners in actual
custody. Nor are we persuaded by the argument that revocation is so
totally a discretionary matter that some form of hearing would be
administratively intolerable. A simple factual hearing will not interfere
with the exercise of discretion. Serious studies have suggested that fair
treatment on parole revocation will not result in fewer grants of parole.
This discretionary aspect of the revocation decision need not be
reached unless there is first an appropriate determination that the
individual has, in fact, breached the conditions of parole. The parolee is
not the only one who has a stake in his conditional liberty. Society has a
stake in whatever may be the chance of restoring him to normal and
useful life within the law. Society thus has an interest in not having
parole revoked because of erroneous information or because of an
erroneous evaluation of the need to revoke parole, given the breach of
parole conditions... And society has a further interest in treating the
parolee with basic fairness: fair treatment in parole revocations will
140 | P

LATON

enhance the chance of rehabilitation by avoiding reactions to


arbitrariness.

The Court then turned its focus on the nature of the process
that is due, bearing in mind that the interest of both State and
parolee will be furthered by an effective but informal hearing.
In analyzing what is due, it referred to two important stages in
the typical process of parole revocation--(2) the arrest of
parolee and preliminary hearing, and (b) the revocation hearing
itself. Each stage has its own set of procedural requirements in
order not to offend the Due Process Clause.
In the first stage, some minimal inquiry in the nature of a "preliminary
hearing" should be conducted ate or reasonably near the place of the
alleged parole violation or arrest and as promptly as convenient after
arrest to determine whether there is probable cause or reasonable
ground to believe that the arrested parolee has committed acts that
would constitute a violation of parole conditions. This determination
should be made by someone not directly involved in the case. This
independent officer need not be a judicial officer. The granting and
revocation of parole are matters traditionally handled by administrative
officers. The parolee should be given notice that the hearing will take
place and that its purpose is to determine whether there is probable
cause to believe he has committed a parole violation. The notice should
state what parole violations have been alleged. At the hearing, the
parolee may appear and speak in his own behalf; he may bring letters,
documents, or individuals who can give relevant information to the
hearing officer. On request of the parolee, a person who has given
adverse information on which parole revocation is to be based is to be
made available for questioning in his presence. However, if the hearing
officer determines that an informant would be subjected to risk of
harm if his identity were disclosed, he need not be subjected to
confrontation and cross-examination. The hearing officer shall have the
duty of making a summary, or digest, of what occurs at the hearing in
terms of the responses of the parolee and the substance of the
documents or evidence given in support of parole revocation and of the
parolee's position. Based on the information before him, the officer
should determine whether there is probable cause to hold the parolee
for the final decision of the parole board on revocation. Such a
determination would be sufficient to warrant the parolee's continued
detention and return to the state correctional institution pending the
final decision.

With respect to the second stage--the probation revocation


hearing--the Court said:
We cannot write a code of procedure; that is the responsibility of each
State. Most States have done so by legislation, others by judicial
decision usually on due process grounds. Our task is limited to deciding
the minimum requirements of due process. They include (a) written
notice of the claimed violations of parole; (b) disclosure to the parolee
of evidence against him; (c) opportunity to be heard in person and to
present witnesses and documentary evidence; (d) the right to confront
and cross-examine adverse witnesses (unless the hearing officer
specifically finds good cause for not allowing confrontation); (e) a
"neutral and detached" hearing body such as a traditional parole board,
members of which need not be judicial officers or lawyers; and (f) a
written statement by the factfinders as to the evidence relied on and
reason for revoking parole. We emphasize there is no thought to
equate this second stage of parole revocation to a criminal prosecution
in any sense. It is a narrow inquiry; the process should be flexible
enough to consider evidence including letters, affidavits, and other
material that would not be admissible in an adversary criminal trial.

Factors in Determining Whether Sanction is Civil or Penal


Relevant to the issue of punishment is the determination as to
whether a particular sanction imposed by aw may be
considered as merely civil or penal

On this matter, the factors or criteria set out in Kennedy v.


Mendoza-Martinez have been generally referred to in
ascertaining whether an act of Congress is penal or regulatory
in nature. These factors are:
Whether the sanction involves an affirmative disability or restraint,
whether it has historically been regarded as a punishment, whether it
comes into play only on a finding of scienter, whether its operation will
promote the traditional aims of punishment - retribution and
deterrence, whether the behavior to which it applies is already a
crime, whether an alternative purpose to which it may rationally be
connected is assignable for it, and whether it appears excessive in
relation to the alternative purpose assigned are all relevant to the
inquiry, and may often point in differing directions. Absent conclusive
evidence of congressional intent as to the penal nature of a statute,
these factors must be considered in relation to the statute on its face.

If any trend could be discerned from the rights and guarantees


accorded accused and convicts, it is the ever-evolving
humanization of the law and of punishments. The
transgressions of the rules of an organized society for its own
protection and self-preservation may call for the imposition of
penalties but these would have to be in accordance with the
mandates of an enlightened body politic. In the future, some
forms of punishments that are taken for granted today may be
treated or seen in different light and thus pose other questions
of constitutionality. As Atkins graphically illustrates, evolving
standards of decency could put on a different gloss to certain
past practices and punishments. And in the same manner that
from what were recognized as rights of the accused developed
constitutionally encrusted rights of suspects, in the not too
distant future, there might also come to be more clearly
defined, and guaranteed rights of convicts
Chapter 21
Non-Imprisonment for Debts
No person shall be imprisoned for debt or non-payment of a
31
poll tax.
Another marker in the advance of man from barbarism to more
civilized and humane treatment of individuals is the
proscription against imprisonment for debt. And, for the
benefit of those less fortunate members of society, there is also
the guarantee against incarceration simply because of their
failure to pay a poll tax
Imprisonment for Debts

Poll Tax
Chapter 22
Double Jeopardy
No person shall be twice put in jeopardy of punishment for the
same offense. If an act is punished by a law and an ordinance,
conviction or acquittal under either shall constitute a bar to
32
another prosecution for the same act.

CONSTITUTION, Art. III, 20


CONSTITUTION, Art. III, 21
In the U.S., the guarantee against double jeopardy is found in the Fifth
Amendment: No person shall be held to answer for a capital, or otherwise
infamous crime, unless on a presentment or indictment of a Grand Jury, except
in cases arising in the land or naval forces, or in the Militia, when in actual
31
32

141 | P

LATON

This guarantee against jeopardy of being tried twice for the


same offense gives a person the freedom and the right not to
be unnecessarily vexed or threatened with repeated
prosecutions. The rule protects the accused not against the
peril of second punishment, but against being again tried for
the same offense. The fundamental philosophy behind the
constitutional proscription against double jeopardy is to afford
the defendant, who has been acquitted, final repose and
safeguard him from government oppression through the
abuse of criminal process. Once prosecuted, whatever result
that ensues should already be a bar to any subsequent attempt
to relitigate the same alleged criminal conduct--a res judicata
dressed in prison grey. As the Court explained in People v.
Ylagan:

In a nutshell, before the defense of double jeopardy may be


invoked, the following requisites must first be present:
(a) first jeopardy which had attached prior to the second;
(b) first jeopardy had been terminated; and,
(c) the prosecution is for the same offense
As to when legal jeopardy is deemed to have attached, the
presence of the following conditions must be shown:
(a) a valid indictment;
(b) a competent court;

Without the safeguard this article establishes in favor of the


accused, his fortune, safety, and peace of mind would be
entirely at the mercy of the complaining witness, who might
repeat his accusation as often as dismissed by the court and
whenever he might see fit, subject to no other limitation or
restriction than his own will and pleasure. The accused would
never be free from the cruel and constant menace of a neverending charge, which the malice of the complaining witness
might hold indefinitely suspended over his head, were it not
that the judiciary is exclusively empowered to authorize, by an
express order to that effect, the repetition of a complaint or
information once dismissed in the cases in which the law
requires that this be done Thanks to this article, the accused,
after being notified of the order rest dismissing the complaint
may, as the case may be, either rest assured that he will not be
further molested, or prepare himself for the presentation of a
new complaint. In either case, the order gives him full
information as to what he may hope or fear, and prevents his
reasonable hopes from being dissipated as the result of an
equivocal and indefinite legal situation. To this much, at least,
one who has been molested, possibly unjustly, by prosecution
on a criminal charge, is entitled.
But as simple as the language of the guarantee appears to be,
the assurance is only against a second or subsequent
prosecution for the same offense. Thus, it is possible that a
single act may give rise to one or more offenses, unless the said
act constitutes a violation of both a national law and a local
ordinance, in which event conviction or acquittal under either
shall constitute a bar to another prosecution under the other
for the same act. Another matter to be considered in regard to
constitutional safeguard is the fact that subsequent
prosecution may be had if the earlier prosecution was not
decided on the merits and therefore did not end in an outright
acquittal or conviction, such as when the dismissal was due to
the motion of the accused himself, or with his express consent

(c) an arraignment had;


(d) a valid plea entered; and
(e) the case dismissed or otherwise terminated without the
express consent of the accused
Valid Indictment
It is a requirement for jeopardy to attach that there must be a
valid complaint or information to start with. This essentially
means that the said indictment must be filed by a duly
authorized person or officer. It is a valid information signed by a
competent officer, among other things, which confers
jurisdiction on the court over the person of the accused and the
subject matter of the accusation. An invalid information is no
information at all and cannot be the basis for criminal
proceedings. Hence, dismissal of the first information would
not be a bar to a subsequent prosecution--jeopardy does not
attach where a defendant pleads guilty to a defective
indictment that is voluntarily dismissed by the prosecution
Arraignment and Valid Plea
It is a requirement that for legal jeopardy to attach, there
must first be an arraignment and a valid plea because without
the same the court cannot have jurisdiction to proceed to
render a valid and binding judgment. A void judgment would
then have no force and effect. Consequently, no jeopardy could
also be deemed to have attached to preclude another
prosecution

General Considerations
Like most of the Bill of Rights guarantees, this protection
against double jeopardy is also of American origin. It was
introduced into the country through the Philippine Bill of 1902
and the Jones Law of 1916. This principle is founded upon the
law of reason, justice and conscience. It is embodied in the
maxim of the civil law non bis in idem, in the common law of
England, and undoubtedly in every system of jurisprudence,
and instead of having specific origin it simply always existed.

Dismissals and Finality of Judgments


Once a criminal case terminates in acquittal, conviction, or is
dismissed without the consent of the accused, he cannot be
prosecuted anew for the same offense. It is important to
determine, however, whether the court that rendered the
decision had jurisdiction throughout, or might have been
ousted of such authority along the way. Further, for an order of
dismissal to be effective, it must first be reduced to writing and

service in time of War or public danger; nor shall any person be subject for the
same offense to be twice put in jeopardy of life or limb;

142 | P

LATON

signed by the Judge. Otherwise, a verbal order of dismissal can


not be the valid basis for claiming double jeopardy
Where there is a valid information and the accused has been
arraigned, an order of dismissal issued by the court, motu
proprio, in the course of a trial of a criminal case, whether
based on the merits or for failure of prosecution witnesses to
appear, has the effect of a judgment of acquittal and double
jeopardy attaches. The order is also immediately executory.
However, this order of dismissal must be written in the official
language, personally and directly prepared by the judge and
signed by him conformably with the provisions of Rule 120,
Section 2 of the Rules of Court. In the instant case, it is very
clear that the order was merely dictated in open court by the
trial judge. There is no showing that this verbal order of
dismissal was ever reduced to writing and duly signed by him.
Thus, it did not yet attain the effect of a judgment of acquittal,
so that it was still within the powers of the judge to set it aside
and enter another order, now in writing and duly signed by
him, reinstating the case
It goes without saying that the dismissal should be one in a
case already pending in court, not one before the public
prosecutors office. Since a preliminary investigation is not
part of the trial, the dismissal of a case by the fiscal will not
constitute double jeopardy and hence there is no bar to the
filing of another complaint for the same offense

or amendment to correct a manifest substantial error, even if


unwittingly committed by the trial court through oversight or an
initially erroneous comprehension, can be made only with the consent
or upon the instance of the accused. Errors in the decision cannot be
corrected unless the accused consents thereto, or himself moves for
reconsideration of, or appeals from, the decision.
It must be stressed, however, that the protection against double
jeopardy in the foregoing rules may be waived by the accused. Thus,
when the accused himself files or consents to the filing of a motion for
reconsideration or modification, double jeopardy cannot be invoked
because the accused waived his right not to be placed therein by filing
such motion. His motion gives the court an opportunity to rectify its
errors or to reevaluate its assessment of facts and conclusions of law
and make them conformable with the statute applicable to the case in
the new judgment it has to render. The raison detre is to afford the
court a chance to correct its own mistakes and to avoid unnecessary
appeals from being taken. In effect, a motion for reconsideration or
modification filed by or with consent of the accused renders the entire
evidence open for the review of the trial court without, however,
conducting further proceedings, such as the taking of additional proof.
Clearly, therefore, appellants cannot dictate upon the trial court which
aspects of the judgment of conviction should be reviewed. Having filed
a timely motion for reconsideration asking the court to acquit, or in the
alternative, convict them of the lesser offense of homicide, appellants
waived the defense of double jeopardy and effectively placed the
evidence taken at the trial open for the review of the trial
courtHence, the court a quo is not only empowered but also under
obligation to rectify its mistake in appreciating the qualifying
circumstance of abuse of superior strength instead of treachery
Bustamante V. Maceren (p. 741)
48 SCRA 155 (1972)

Motions for Reconsideration and Appeals by the Accused


A judgment of conviction could only be reconsidered or
appealed at the initiative of the accused. When the accused
decides to simply serve his sentence or pay his fine, that is the
end of the case. The prosecution may not move for
reconsideration or appeal the judgment as the same would
place the accused in double jeopardy. The prosecution could
not also appeal for the purpose of increasing the penalty
However, when the accused himself appeals, he stands the
chance of having his penalty increased since an appeal in a
criminal case throws open the whole case for review, including
the penalty imposed. Given the rule for appeals, how about if
he simply moves for reconsideration? Can he delimit the extent
to which the court may review the decision for the purposes of
reconsideration so that it may not modify the judgment
adversely against him? This was he thinking of the accused in
People v. Astudillo but the Court disagreed:
Anent the qualifying circumstance of treachery, we find no merit in
appellants contention that the trial cannot validly appreciate the same
in its amended decision because the attendance of treachery was not
one of the issues raised in their motion for reconsideration. Otherwise
stated, appellants posit that the reconsideration of the judgment of
conviction should be limited only to the issues raised in their motion for
reconsideration, i.e., their guilt or innocence and/or the propriety of
appreciating the qualifying circumstance of abuse of superior
strength which was not alleged in the information.
*** The requisite consent of the accused to such motion for
reconsideration or modification is intended to protect the latter from
having to defend himself anew from more serious offenses or penalties
which the prosecution or the court may have overlooked. Accordingly,
once the judgment has been validly promulgated, any reconsideration

FACTS: Petitioner was charged with the crime of murder in Laguna.


Upon arraignment on Dec 14 1970, he entered a plea of guilty and was
sentenced to one year imprisonment. On the very same day was
promulgated, petitioner made an express waiver of his right to appeal.
Accordingly, Judge Coquia who rendered the aforesaid judgment,
issued a commitment order on the same day the petitioner started to
served his sentence. Three days later, petitioner withdrew his plea of
guilty and the judge ordered the trial of the case on merits. Judge
Coquia, subsequently was transferred and the case was reassigned to
the respondent judge who eventually rendered a new judgment against
the petitioner finding him guilty of the crime of homicide and convicting
him to an indeterminate sentence of 6 years and one day to twelve
years and one day. A motion for reconsideration was filed questioning
the jurisdiction of the court to try his case anew after he had fully
served the judgment rendered by Judge Coquia. Petitioner, argued that
the judgment of Judge Coquia against him had already become final
when he started serving his sentence and that therefore, the court lost
jurisdiction over his case and that no amount of waiver or consent on
his part could bestow on said court jurisdiction that it had already lost
The judge said that he could not nullify an order of another judge of
equal rank and that only a higher court had the authority to nullify said
order but that he would hold his ruling in abeyance until petitioner
could file a certiorari before the Supreme court. Hence, this petition.
HELD: We agree and grant the certiorari. Petitioner is entitled to the
relief prayed for. With the judgment of conviction not only
promulgated but actually carried out with petitioner having started to
serve his sentence, no such order reopening the case should have been
issued. That was not in accordance with the controlling doctrine on the
constitutional right against being twice put in jeopardy. It is true that he
had in fact contributed to bringing such judicial deviation from the
correct norm, That did not forfeit though his right to a remedy to which
he is entitled.
People v. Magat (p. 745 Vol. 2)
332 SCRA 517 (2000)
FACTS: Appellant Anotonio Magat was accused of raping his daughter
143 | P

LATON

twice when she was 17 yrs old and when she was 19 yrs. Old. Upon
arraignment on Jan 19, 1997, he pleaded guilty but bargained a for a
lesser penalty for each case. The court issued a sentence of
imprisonment term of 10 years or each crime. After 3 months, the
cases were revived at the instance of he complainant on the ground
that the penalty imposed was too light. As a consequence, he was rearraigned where he entered a plea of not guilty but subsequently he
entered anew a plea of guilty. He was sentenced to death by lethal
injection. On automatic review o the SC, appellant contends that the
trial court erred re-arraigning and proceeding to trail despite the fact
that he was already convicted earlier based on his plea of guilt. He also
argues that when the court rendered judgment convicting him, the
prosecution did not appeal nor more fro reconsideration or took steps
to set aside the order he also posits that the re-arraignment and trial on
the same information violated his right against double jeopardy.
HELD: The January 1997 order of the trial court convicting the accused
appellant on his own plea of guilt is void ab initio on the ground that
accused-appellants plea is not the plea bargaining contemplated and
allowed by law and the rules of procedure. The only instance where a
plea bargaining is allowed under the rules is when an accused pleads
guilty to a lesser offense. It must be emphasized that accused appellant
did not plead to a lesser offense but pleaded guilty to the rape charges
and only bargained for a lesser penalty. In short, he did not plea bargain
but made conditions on the penalty to be imposed. This is erroneous
because by pleading guilty to the offense charged, accused appellant
should be sentenced to the penalty to which he pleaded. Nonetheless,
whatever procedural infirmity in the arraignment of the accused
appellant was rectified he was re-arraigned and entered a new plea.
Accused appellant did not question the procedural errors in the first
arraignment and failing failed to do so, he is deemed to have
abandoned his right to question the same and waived the errors in
procedure.

Waiver and Estoppel


Just like most rights, the right against double jeopardy may be
waived or otherwise barred by estoppel. If the case did not
proceed to either acquittal or conviction because the accused
along the way sought the dismissal of the case in the
meantime, then this may be considered as a waiver, because by
so doing he chose to abort the process. At the same time, if by
his allegations he may have misled the court into dismissing the
case on an invalid ground, such as want of jurisdiction, then he
could not subsequently be heard to assert that the court was
wrong in going along with him. He is estopped to claim
otherwise
People v. Obsania (p. 747 Vol. 2)
23 SCRA 1249
FACTS: A day after the occurrence of the crime of rape with robbery,
Erlina Dollente, 14 year old victim and her parents filed in the municipal
court of Balungao, Pangasinan, a complaint against the defendant.
After the case was remanded to the Court of the First Instance of
Pangasinan for further proceedings, the assistant provincial fiscal filed
an information for rape against the accused, employing the same
allegations with an additional averment that the offense was
committed with lewd designs. Upon arraignment, the accused
pleaded not guilty and moved for the dismissal of the case contending
that the complaint was totally fatally defective for failure to allege lewd
design and that the subsequent information filed did not cure the
jurisdictional infirmity. The Court a quo granted the motion and
ordered the dismissal of the action. The fiscal appealed.
HELD: Two issues are tendered for resolution, namely: First, are lewd
design an indispensable element which should be alleged in the
complaint? Second, does the present appeal place the accused in
double jeopardy? We held that the trial judge erred in dismissing the
case on the grounds that the complaint was defective for failure to
allege lewd design and as a consequence of such infirmity that the

court a quo if not acquire jurisdiction over the case. The error of the
trial judge was in confusing the concept of jurisdiction with that of
insufficiency on substance of an indictment. We also held that the
application of the sister doctrines of waiver and estoppel required two
sine qua non conditions: first, the dismissal must be sought or induced
by the defendant personally or though his counsel and second, such
dismissal must not be on the merits and must not necessarily amount
to an acquittal. Indubitably, the case at bar falls squarely within the
periphery of the said doctrines which have been preserved unimpaired
in the corpus of our jurisprudence. The case is hereby remanded to the
court of origin for further proceedings in accordance with law.

Dismissals, Insufficient Evidence and Speedy Trials


As shown above, the termination of a case other than on
acquittal or conviction of the accused, which dismissal is
brought about by the accused himself, is not a bar to further
prosecution. The defense of double jeopardy could not be
invoked. The exception, however, would be in cases of
demurred to evidence being sustained and dismissal is actually
in the nature of an acquittal. Thus, there could be no more
subsequent prosecution for the same offense
On a demurred to evidence being favorably acted upon, the
acquittal results from the fact that the prosecution has not
really presented that amount of evidence sufficient to
overcome the presumption of innocence. Where the
prosecution evidence could not sustain a conviction, there is no
point in still requiring the accused to present his evidence. On
the right to speedy trial, when this right is upheld it means that
the accused is now free because the prosecution failed to abide
by its duty to prosecute without unwarranted delays. In effect,
the State has failed to prove its case within a reasonable period
of time. Nevertheless, and paradoxically, when there is, in
dismissing a case based on right to speedy trial, an act of
overspeeding on the part of the court, this may lead to a
contrary result
Salcedo V. Mendoza (p. 755)
88 SCRA 811 (1979)
FACTS: A criminal information for homicide through reckless
imprudence against the petitioner was filed. Upon arraignment, he
entered a plea of not guilty and the case was then set for trial. When
the case was called for trial on the scheduled date, the provincial fiscal
asked for and was granted postponement because the accused failed to
appear. On the next scheduled hearing, they once more, moved for
postponement. The 3rd time it was called for trial, no prosecuting fiscal
appeared for the prosecution. The private prosecutor, who withdrew
his appearance from the case and reserved the right to file a separate
civil action, moved for its postponement in order to give the
prosecution a another chance because they intended to request the
Ministry of Justice to appoint a special prosecutor to handle the case.
The trial court denied the same. Whereupon, the petitioner, though
counsel, moved for the dismissal for the criminal case invoking his right
to speedy trial. The respondent judge dismissed the case . A motion to
reconsider the above order was filed the same day but was denied the
following day. A second motion was filed and was set for hearing,
however, the parties failed to submit their pleadings, The respondent
Judge issued an order setting aside the order of dismissal and ordering
that the case be set for trial. Petitioner filed a motion for
reconsideration alleging that the dismissal of the criminal case against
him was equivalent to an acquittal and reinstatement of the same
would place him twice in jeopardy for the same offense. The trial court
denied the motion. In the instant petition for certiorari, the Solicitor
General, in his Comment, agrees with the petitioner that a
reinstatement of this case would operate to violate his right against
double jeopardy.
HELD; The stand of the petitioner and the Solicitor General is well
144 | P

LATON

taken. We have said that the dismissal of the criminal case predicated
on the right of the accused to speedy trial, amounts to an acquittal on
the merits which bars the subsequent prosecution of the accused for
the same offense. In the present case the respondent judge dismissed
the criminal case upon the motion of the petitioner involving his
constitutional right to speedy trial because the prosecution failed to
appear on the day of the trial on March 28, 1978 after it had previously
been postponed twice. The setting aside of the respondent judge of the
order of dismissal of March 28 1978 and thereby reviving criminal case
places the petitioner twice in jeopardy in the same offense. The
respondent judge therefore committed a grave abuse of discretion in
issuing the order of May 9 1978 setting aside the order of dismissal
issued on March 28 1978.
People V. Declaro (p. 757)
170 SCRA 142
FACTS: Private respondent, Edgar Ibabao was charged with slight
physical injuries through reckless imprudence as a result of a traffic
accident. The case was docketed as Criminal case no 1028-N wherein a
certain Crispin Conanan was the offended party. Again, an information,
docketed as Criminal case no 1421 was filed against the same accused
by Eduardo Salido as the offended party. The second case arose from
the same incident. Upon the arraignment of the accused in the first
case , he entered a plea of not guilty. The hearing was set for hearing
but the offended party and the prosecuting fiscal failed to appear at the
scheduled hearing despite due notice. After a series of motion filed, the
court dismissed the case for lack of interest on the part of the
prosecution. Since the case was dismissed, the petitioner filed a motion
to dismiss the second case on the ground that the dismissal for the
prior case is a bar to the prosecution. The court dismissed the second
case on the ground of double jeopardy. Hence, this petition with the
petitioner contending that double jeopardy have not set in because a.)
the dismissal of the first case was at the instance and the express
consent of the accused and his counsel and b.) the second offense
charged is not the same as the fist, nor is it an attempt to commit the
same or a frustration thereof nor does it include in the firs and c.) the
first case is not yet terminated.
HELD; The petitioner is impressed with merit. Although there are
criminal cases which were dismissed upon motion of the accused
because the prosecution was not prepared for trial sine the
complainant or his witnesses did not appear at the trail or where the
court held that the dismissal is equivalent to an acquittal that would bar
further prosecution of the defendant for the same offense. However,
the facts and the circumstance of the present case does not warrant
similar ruling. Double jeopardy will apply even if the dismissal is made
with the express consent to the accuse or upon his own motion only if
it is predicated in either of 2 grounds i.e. insufficiency of he evidence or
denial of the right to a speedy trial. In both cases, the dismissal will
have the effect of the acquittal. Since the dismissal in this case does
not fall under either of these 2 instances, and it was made with the
express consent of the accused, it would not thereby be a bar to
another prosecute for the same offense,

Acquittal, Mistrials and Appeals


An acquittal puts an end to the whole case. The accused can
not anymore be imperilled by the prosecutor either seeking a
reconsideration of the judgment or taking an appeal from it.
Both actions would be considered as violation of the guarantee
against double jeopardy. If there was grave abuse of discretion
which ousted the court of jurisdiction, that would be a different
matter, however. Without jurisdiction, the court cannot
proceed to render a valid judgment. Thus, the prosecution may
elevate the matter to an appellate court by means of the
special civil action of certiorari
In Galman v. Sandiganbayan, the Supreme Court nullified the
acquittal of the accused because of a mistrial brought about by
the acts of then President Marcos in influencing and

manipulating the trial of those accused in the killing of former


Senator Aquino and the supposed assassin, Galman. The Court
explained why there was no double jeopardy:
1. No double jeopardy.--It is settled doctrine that double jeopardy
cannot be invoked against this Court's setting aside of the trial courts'
judgment of dismissal or acquittal where the prosecution which
represents the sovereign people in criminal cases is denied due process.
As the Court stressed in the 1985 case of People vs. Bocar-Where the prosecution is deprived of a fair opportunity to prosecute
and prove its case its right to due process is thereby violated.
The cardinal precept is that where there is a violation of basic
constitutional rights, courts are ousted of their jurisdiction. Thus, the
violation of the State's right to due process raises a serious
jurisdictional issue which cannot be glossed over or disregarded at will.
Where the denial of the fundamental right of due process is apparent, a
decision rendered in disregard of that right is void for lack of
jurisdiction. Any judgment or decision rendered notwithstanding such
violation may be regarded as a "lawless thing, which can be treated as
an outlaw and slain at sight, or ignored wherever it exhibits its head."
Respondent Judge's dismissal order dated July 7, 1967 being null and
void for lack of jurisdiction, the same does not constitute a proper
basis for a claim of double jeopardy.
xxx xxx xxx
Legal jeopardy attaches only (a) upon a valid indictment, (b) before a
competent court, (c) after arraignment, (d) a valid plea having been
entered; and (e) the case was dismissed or otherwise terminated
without the express consent of the accused. The lower court was not
competent as it was ousted of its jurisdiction when it violated the right
of the prosecution to due process.
In effect the first jeopardy was never terminated, and the remand of
the criminal case for further hearing and/or trial before the lower
courts amounts merely to a continuation of the first jeopardy, and does
not expose the accused to a second jeopardy.
More so does the rule against the invoking of double jeopardy hold in
the cases at bar where as we have held, the sham trial was but a mock
trial where the authoritarian president ordered respondents
Sandiganbayan and Tanodbayan to rig the trial and closely monitored
the entire proceedings to assure the pre-determined final outcome of
acquittal and total absolution as innocent of an the respondentsaccused.33
Respondents-accused's contention that the Sandiganbayan judgment of
acquittal ends the case which cannot be appealed or re-opened,
without being put in double jeopardy was forcefully disposed of by the
Court in People vs. Court of Appeals, which is fully applicable here, as
follows: "That is the general rule and presupposes a valid judgment.
As earlier pointed out, however, respondent Courts' Resolution of
acquittal was a void judgment for having been issued without
jurisdiction. No double jeopardy attaches, therefore. A void judgment
is, in legal effect, no judgment at all. By it no rights are divested.
Through it, no rights can be attained. Being worthless, all proceedings
founded upon it are equally worthless. It neither binds nor bars anyone.
All acts performed under it and all claims flowing out of it are void.
xxx xxx xxx
"Private respondent invoke 'justice for the innocent'. For justice to
prevail the scales must balance. It is not to be dispensed for the
accused alone. The interests of the society, which they have wronged
must also be equally considered. A judgment of conviction is not
necessarily a denial of justice. A verdict of acquittal neither necessarily
spells a triumph of justice. To the party wronged, to the society

33

See Page 762

145 | P

LATON

offended, it could also mean injustice. This is where the Courts play a
vital role. They render justice where justice is due.

It is only in very rare instances, however, that an acquittal had


may be allowed to be questioned and reverse. For unless the
violation of the basic dictates is so flagrant and egregious, the
judgment in favor of the accused would stand
People v.Velasco (p. 763)
340 CRA 207
FATS: Alex Vinculado was killed while his twin brother Levi lost
permanently his vision and their uncle was shot. Information for
Homicide and frustrated homicide was filed against Mayor Honorato
Galvez of San Ildefonso, Bulacan and his bodyguard, Godofredo Diaz.
Charges were however, withdrawn and a new one for murder and
frustrated murder was filed against the accused. In addition, the mayor
was charged with violation of PD 1866 for unauthorized carrying of
firearm outside this residence. The court found them guilty . However,
it acquitted Galvez of the same charges due to insufficiency of
evidence. The acquittal of Mayor Galvez is challenged by the
government in this petition of certiorari. Their position is that the
elevating the issue of criminal culpability to the Supreme court despite
acquittal by the trial court should not be considered as violative of the
constitutional right against double jeopardy.\
HELD: The petition at hand seeks to nullify the decision of the trial
court acquitting the accused goes deeply in the courts appreciation
and evaluation in esse of the evidence adduced by the parties. A
reading of the questioned decision shows that respondents judge
considered the evidence received at trial. These consisted among
others of the testimonies relative to the position of the victims visa vis
the accused and the trajectory, location and nature of the gunshot
wounds, and the opinion of the expert witness for the prosecution.
While the appreciation may have resulted in possible lapses in evidence
evaluation, it does not detract from the fact that the evidence was
considered and passed upon. This consequently exempts the act from
the writs limiting requirement of excess or lack of jurisdiction. Hence, it
becomes an improper object and non reviewable by certiorari. To
reiterate, errors of judgment are not to be confused with errors in the
exercise of jurisdiction.
Merciales v. Court of Appeals
379 SCRA 345 (2002)
Facts : On August 12, 1993 six information of rape with homicide in
connection of the death of Maritess Ricafirt Merciales were filed
against private respondents Jeslito Nuada, Patrcik Moral, Adonis
Nieves, Ernesto Lobelete, Domil Grageda and Ramon Frlores before the
regional trial Court in Legaspi City. Although seven witnesses were
presented by the prosecutor, none actually saw the commission of the
crime. Only Nuada one of the accused who came forward and express
willingness to turn as state witness. Trial Court required the prosecutor
to present evidence to justify Nuadas discharge as a state witness. But
the latter insisted that there was no need for such proof since Nuada
had already admitted into the Witness protection program of the
Department of Justice. Trial court denies the motion to discharge
Nuada. In due time the accused filed demurrer of evidence, the trial
court issued an order acquitting all the accused for lack of sufficient
evidence to prove the guilt of the accused beyond reasonable doubt.
Instant petition assails the failure of the court of appeals to nullify the
trial courts order on the ground that the trial judge tolerated or
committed injustice by failing to require prosecution which is present.
Issue: Whether or not the reopening of the case violates the accused
right against double jeopardy.
Held: In the case at Bar the private prosecutor knew that he had not
presented sufficient evidence to convict the accused. In this sense he
remiss his duty to protect the interest of the offended parties. Refusing
to comply to present evidence violates Rule 119 section 9 of Rules of
the Court expressly requires presentation of evidence. Accused would

not be placed in Double jeopardy for the reason that from the very
beginning lower trial tribunal acted without jurisdiction is in legal
contemplation. Case remanded to the Regional Trial court of Legaspi,
Public prosecutor is ordered to complete presentation of available
witness for the prosecution.

Appeals and Civil Liability


The rule that an acquittal cannot be appealed refers only to
criminal aspect of the case. The offended party may appeal the
civil component without violating the guarantee against double
jeopardy. The proceeding assumes the nature already of a civil,
not criminal, litigation
In Manantan v. Court of Appeals, x x x:
Our law recognizes two kinds of acquittal, with different effects on the
civil liability of the accused. First is an acquittal on the ground that the
accused is not the author of the act or omission complained of. This
instance closes the door to civil liability, for a person who has been
found to be not the perpetrator of any act or omission cannot and can
never be held liable for such act or omission. There being no delict, civil
liability ex delicto is out of the question, and the civil action, if any,
which may be instituted must be based on grounds other than
the delict complained of. This is the situation contemplated in Rule 111
of the Rules of Court. The second instance is an acquittal based on
reasonable doubt on the guilt of the accused. In this case, even if the
guilt of the accused has not been satisfactorily established, he is not
exempt from civil liability which may be proved by preponderance of
evidence only. This is the situation contemplated in Article 29 of the
Civil Code, where the civil action for damages is for the same act or
omission. Although the two actions have different purposes, the
matters discussed in the civil case are similar to those discussed in the
criminal case. However, the judgment in the criminal proceeding
cannot be read in evidence in the civil action to establish any fact there
determined, even though both actions involve the same act or
omission. The reason for this rule is that the parties are not the same
and secondarily, different rules of evidence are applicable. Hence,
notwithstanding herein petitioners acquittal, the Court of Appeals in
determining whether Article 29 applied, was not precluded from
looking into the question of petitioners negligence or reckless
imprudence.

Supervening Fact Doctrine


Once a person has been charged and has pleaded, the charge
may not be amended or revised in a substantial way, otherwise
this would also be a violation of his right against double
jeopardy. In this regard, however, one has to consider whether
the fact that is sought to justify the amendment was already
present when the formal accusation and plea were made, or
came about only subsequently. If before, then the amendment
may not be had without violating the constitutional safeguard.
If after, it may be done as the new development is a
supervening fact which could not have been included earlier
Melo v. People
85 Phil. 766 (1950)
Facts: Conrado Melo was charge in the Court of first instance of Rizal
with frustrated homicide, inflicted to Benjamin Obillo with a kitchen
knife with intent to kill. Accused pleaded not guilty of the offense
charge in the same they the victim died. Evidence of death was only
available on January 3, 1950 the next they amended information was
filed charging the accused with consummated homicide. Accused file
for motion to squash the amended information alleging double
jeopardy but motion was denied.
Issue: whether or not the denial of the motion denies the accused the
right against double jeopardy.
146 | P

LATON

Facts: Buling was charge of a crime of less serious physical injuries for
having inflicted wounds on Isidro Balaba, which according to the
complaint requires medical assistance for a period of 10 to 15 days.
Accused pleaded guilty and found guilty, then began to serve his
sentence. Balabas injuries however did not heal. Provincial Fiscal filed
another complaint of serious physical injuries. Information alleges that
the victim requires medical attendance and incapacitated him for a
period of 1 to 2 months. After the trial accused is found guilty of
serious physical injuries as a new sentenced.

second offense charged is exactly the same as the one alleged


in the first information...And, there is identity between the two
offenses when the evidence to support a conviction for one
offense would be sufficient to warrant a conviction for the
other. Under the Rules there is identity between two offenses
not only when the second offense is exactly the same as the
first, but also when the second offense is an attempt to commit
the first or a frustration thereof, or when it necessarily includes
or is necessarily included in the offense charged in the former
complaint or information. In this connection, an offense may
be said to necessarily include another when some of the
essential ingredients of the former as alleged in the information
constitute the latter. And vice-versa, an offense may be said to
be necessarily included in another when all the ingredients of
the former constitute a part of the elements constituting the
latter (Rule 116, sec. 5.) In other words, one who has been
charged with an offense cannot be again charged with the
same or identical offense though the latter be lesser or
greater than the former.

Issue: Whether or not prosecution conviction of Balaba for less physical


injuries is a bar for the second prosecution for serious physical injuries.

People v. City Court of Manila, Branch VI


154 SCRA 175 (1987)

Held: the new fact did not supervened, what happened was no X ray
examination of wounded hand was made during the first examination,
which was merely superficial. If new fact had been disclosed in the
previous examination may be attributed to the incompetence on the
part of the examining physician. If the X ray examination discloses the
existence of a fracture, such fracture should have existed when the first
examination wear made. Therefore there is no supervening fact that
could be said to have arisen. Judgement conviction was set aside and
the defendant was acquitted for serious physical injury.

Facts: Gonzales together with Pangilinan was accused of violating Sec.7


in relation to sec. 11 R.A. 201(3) of RPC which is Consist of indecent or
immoral plays scenes, acts or shows, not just motion pictures. And
Art.3060 which pertains to public exhibition of any motion picture
which has not been previously passed by the motion Board of Censors
for Motion Pictures that constitutes criminal offense. Gonzales moved
to squash on the ground of double jeopardy.

Same Offense v. Same Act

Held: It is evident that the two offenses are different. There is no


identity of the offense here involved for which legal jeopardy in one
may be invoked. Evidence required to prove one offense is not the
same evidence required to prove another. Double jeopardy cannot
prosper.

Held: Right against double jeopardy does not apply when the second
offense was not in existence at the time of the first prosecution for
simple reason that in such case there is no possibility of accused, during
the first prosecution, to be convicted for an offense that was inexistent.
Thus where the accused was charged with physical injuries and after
conviction the injured person dies, the charge for homicide against the
same accused does not put him twice in jeopardy.
People v. Buling
107 Phil. 712 (1960)

The constitutional guarantee contemplates two situations. One


in which the same act may give rise to two or more offenses in
which case the rule against double jeopardy does not apply,
and another where even if the same act produces two offenses
the protection is made available
Our Bill of Rights deals with two (2) kinds of double jeopardy. The first
sentence of clause 20, section 1, Article III of the Constitution, ordains
that "no person shall be twice put in jeopardy of punishment for the
same offense." (Emphasis in the original) The second sentence of said
clause provides that "if an act is punishable by a law and an ordinance,
conviction or acquittal under either shall constitute a bar to another
prosecution for the same act." Thus, the first sentence prohibits double
jeopardy of punishment for the same offense, whereas the second
contemplates double jeopardy of punishment for the same act. Under
the first sentence, one may be twice put in jeopardy of punishment of
the same act provided that he is charged with different offenses, or the
offense charged in one case is not included in or does not include, the
crime charged in the other case. The second sentence applies, even if
the offenses charged are not the same, owing to the fact that one
constitutes a violation of an ordinance and the other a violation of a
statute. If the two charges are based on one and the same act
conviction or acquittal under either the law or the ordinance shall bar a
prosecution under the other. Incidentally, such conviction or acquittal is
not indispensable to sustain the plea of double jeopardy of punishment
for the same offense. So long as jeopardy has attached under one of
the informations charging said offense, the defense may be availed of
in the other case involving the same offense, even if there has been
neither conviction nor acquittal in either case.

Same Offense
Going back to the first situation, the question is: When is there
same offense? Simply put, there is a same offense where the

Issue: whether or not ground of double jeopardy could be invoked.

Perez v. Court of Appeals


168 SCRA 236 (1988)
Facts: Mendoza charged the petitioner Perez before the court of first
instance of Pampanga with consented abduction. Trial court convicted
Perez who the appealed to the Court of Appeals. Court hold that what
he did was seduction and not abduction. Subsequently the complainant
filed another criminal offense against Perez, this time for qualified
seduction. Petitioner filed as motion for squash invoking double
jeopardy. Motion was denied. He file petition for certiorari and
prohibition to the Supreme Court.
Issue: Whether or not the denial of the court violates the right of the
petitioner against double jeopardy.
Held: The plea cannot therefore be accorded with merit as the two
indictments are perfectly distinct from the point of the law. Difference
between the two are in consented abduction taking away of the
offended must be with her consent after cajolery from the offender and
the taking must be in lewd design. While in qualified seduction the
crime committed must be in abuse of authority, confidence and
relationship and the offender had sexual intercourse with the woman.
Nierras v. Dacuycuy
181 SCRA 1 (1990)
Facts: Petitioner a customer of Pilipinas Shell Corporation, purchase oil
products from it. Upon the delivery petitioner issued nine checks. Upon
the presentation of the checks all nine was dishonoured for the reason
that petitioners account was already close. Pilipinas Shell repeatedly
demanded for depository checks or direct payment of the products, but
147 | P

LATON

the petitioner failed to do so. Petitioner was charged of nine violation


of B.P. Blg. 22 or Bouncing checks law. Nine Criminal cases were also
later instituted against him for estafa under Article 315(2-d) of RPC.
Petitoner filed a motion to squash invoking double jeopardy as
elements of estafa under Article 315(2-d) of RPC are also present in
criminal cases in B.P. Blg. 22.

prohibition on appointment as provided for in Sec. 6, Art. IX-B of the


Constitution and Sec. 94 (b) of the Local Government Code, mandating
that a candidate who lost in any election shall not, within one year after
such election, be appointed to any office in the Government?

Issue: Whether or not the petitioner can be held liable for nine criminal
cases for violation of B.P. Blg. 22 and separately be held liable for crime
of estafa under article 315(2-d) of RPC.

The Sandiganbayan, Fourth Division held that the qualifications for a


position are provided by law and that it may well be that one who
possesses the required legal qualification for a position may be
temporarily disqualified for appointment to a public position by reason
of the one-year prohibition imposed on losing candidates. However,
there is no violation of Article 244 of the Revised Penal Code should a
person suffering from temporary disqualification be appointed so long
as the appointee possesses all the qualifications stated in the law.

Held: Petitioner is charged of two distinct and separate offenses.


Petitioner failed to answer that deceit and damage is essential to article
315(2-d) of RPC but to required to B.P. Blg. 22. Hence filing of two sets
of information does not give rise to double jeopardy.

Same Act
As noted earlier, when the same act gives rise to prosecution
under a national law and a municipal or local ordinance,
acquittal or conviction under either would be a bar to the
other. Thus, since the two offenses are really different because
they are penalized by two jurisdictions--national and local--if
the second part of the guarantee were not provided, individuals
could be prosecuted twice for the same act, even if under both
regulations they ,ay already constitute same offense if they
were enacted by the same legislative body

The Court answers in the negative.

There is no basis in law or jurisprudence for this interpretation. On the


contrary, legal disqualification in Article 244 of the Revised Penal Code
simply means disqualification under the law. Clearly, Section 6, Article
IX of the 1987 Constitution and Section 94(b) of the Local Government
Code of 1991 prohibits losing candidates within one year after such
election to be appointed to any office in the government or any
government-owned or controlled corporations or in any of their
subsidiaries.
Villapando's contention and the Sandiganbayan, Fourth Division's
interpretation of the term legal disqualification lack cogency. Article
244 of the Revised Penal Code cannot be circumscribed lexically. Legal
disqualification cannot be read as excluding temporary disqualification
in order to exempt therefrom the legal prohibitions under Section 6,
Article IX of the 1987 Constitution and Section 94(b) of the Local
Government
Code
of
1991.

Administrative Proceedings
The guarantee against double jeopardy refers to criminal
proceedings. It may not, therefore, be invoked elsewhere
Miscellany
A man is entitled to some definite idea as o how a criminal case
should be determined. There should be some closure that he
can rely on, either in his conviction or acquittal. In either case,
he should be entitled to repose knowing that a particular
matter or chapter in his life has come to [an] end, a book that
he may now close and put aside
Additional Cases
(Q) Double Jeopardy [3]
Q1. People V. Sandiganbayan (Fourth Division)
559 SCRA 449 (2008)
During the May 11, 1998 elections, Villapando ran for Municipal Mayor
of San Vicente, Palawan. Tiape (now deceased), a relative of
Villapando's wife, ran for Municipal Mayor of Kitcharao, Agusan del
Norte. Villapando won while Tiape lost. Thereafter, Villapando
designated Tiape as Municipal Administrator of the Municipality of San
Vicente, Palawan. Maagad and Fernandez charged Villapando and Tiape
for violation of Article 244 of the Revised Penal Code before the Office
of the Deputy Ombudsman for Luzon. The complaint was resolved
against Villapando and Tiape charging the two with violation of Article
244 of the Revised Penal Code filed with the Sandiganbayan.
Article 244 of the Revised Penal Code provides: Unlawful
appointments.-Any public officer who shall knowingly nominate or
appoint to any public office any person lacking the legal qualifications...
There appears to be a dispute. This Court is now called upon to
determine whether Orlando Tiape, at the time of [his] designation as
Municipal Administrator, was lacking in legal qualification. Stated
differently, does "legal qualification" contemplate the one (1) year

Although this Court held in the case of People v. Sandiganbayan that


once a court grants the demurrer to evidence, such order amounts to
an acquittal and any further prosecution of the accused would violate
the constitutional proscription on double jeopardy, this Court held in
the same case that such ruling on the matter shall not be disturbed in
the
absence
of
a
grave
abuse
of
discretion.
In this case, the Sandiganbayan, Fourth Division, in disregarding basic
rules of statutory construction, acted with grave abuse of discretion. Its
interpretation of the term legal disqualification in Article 244 of the
Revised Penal Code defies legal cogency. Legal disqualification cannot
be read as excluding temporary disqualification in order to exempt
therefrom the legal prohibitions under the 1987 Constitution and the
Local Government Code of 1991. We reiterate the legal maxim ubi lex
non distinguit nec nos distinguere debemus. Basic is the rule in
statutory construction that where the law does not distinguish, the
courts should not distinguish. There should be no distinction in the
application of a law where none is indicated.
Q2. People V. Hernandez
499 SCRA 688 (2006)
The instant case stemmed from a complaint filed by Aquilino Pimentel,
Jr., a senatorial candidate in the May 1995 elections, against private
respondents for allegedly decreasing Pimentel's votes in the Statement
of Votes per precinct and in the City Certificate of Canvass for Pasig
City. The COMELEC filed for violation of Section 27(b) of R.A. No. 6646
against private respondents.
Section 1, Rule 122 of the Revised Rules of Criminal Procedure provides
that [a]ny party may appeal from a judgment or final order, unless the
accused will be placed in double jeopardy.
As a general rule, the prosecution cannot appeal or bring error
proceedings from a judgment in favor of the defendant in a criminal
case in the absence of a statute clearly conferring that right. Thus,
errors of judgment are not appealable by the prosecution. Appeal by
the prosecution from the order of dismissal of the criminal case by the
trial court may be allowed only on errors of jurisdiction when there was
denial of due process resulting in loss or lack of jurisdiction. This is so as
148 | P

LATON

while it is true that double jeopardy will attach in case the prosecution
appeals a decision acquitting the accused, an acquittal rendered in
grave abuse of discretion amounting to lack or excess of jurisdiction
does not really "acquit" and therefore does not terminate the case as
there can be no double jeopardy based on a void indictment.

souls since they both apply retroactively to prejudice the


person affected

In the case at bar, the trial court dismissed the cases against private
respondents for the denial of their right to speedy trial. In a long line of
cases, we have held that a dismissal on the ground of the denial of the
accused's right to a speedy trial will have the effect of acquittal that
would bar further prosecution of the accused for the same
offense. Thus, we have held that where after such dismissal the
prosecution moved for the reconsideration of the order of dismissal
and the court re-set the case for trial, the accused can successfully
claim double jeopardy as the said order was actually an acquittal, was
final and cannot be reconsidered. Hence, petitioner was correct in
filing a petition for certiorari under Rule 65, alleging that "respondent
judge committed grave abuse of discretion and/or acted without or in
excess of jurisdiction in issuing the order of dismissal dated November
23, 2001 allegedly on account of the speedy trial rule" as an appeal was
not available to it. Where the dismissal of the case was allegedly
capricious, certiorari lies from such order of dismissal and does not
involve double jeopardy, as the petition challenges not the correctness
but the validity of the order of dismissal and such grave abuse of
discretion amounts to lack of jurisdiction which prevents double
jeopardy from attaching.

While an ex post facto law is basically and generally understood


as that which punishes retroactively an act which when
committed was not prohibited, it actually assumes several
forms. Thus, it is one which:

On July 27, 2003 at around 1:00 a.m., more than 300 heavily armed
junior officers and enlisted men of the AFP entered the premises of the
Oakwood Premier Luxury Apartments on Ayala Avenue, Makati City,
where they disarmed the security guards and planted explosive devices
around the building. They then declared their withdrawal of support
from their Commander-in-Chief and demanded that she resign as
President
of
the
Republic.
After much negotiation, the group finally laid down their arms.
Subsequently, an Information for coup detat was filed against them
with the RTC, at the same time that they were tried at court martial for
conduct unbecoming an officer. They question the jurisdiction of the
court martial, contending that the RTC ordered that their act was not
service-connected and that their violation of Art. 96 of the Articles of
War (RA 7055) was absorbed by the crime of coup detat.
Separate Opinion of Justice Tinga:
Double Jeopardy--It is very well-settled that double jeopardy attaches if
one is tried by both a military court and a civilian court over the same
act, notwithstanding the differing natures of both tribunals

Chapter 23
Ex Post Facto Laws and Bills of Attainder

The other part of the guarantee safeguards against legislative


action by which guilt is declared without the benefit of a judicial
proceeding. Ex post facto laws and bills of attainder are closely
related and intertwined. They normally go together like kindred
CONSTITUTION, Art. III, 22

(2) aggravates a crime, or makes it greater than it was, when


committed;
(3) changes the punishment and inflicts a greater punishment
than the law annexed to the crime when committed;
(4) alters the legal rules of evidence, and authorizes conviction
upon less or different testimony than the law required at the
time of the commission of the offense;

(6) deprives a person accused of a crime of some lawful


protection to which he has become entitled, such as the
protection of a former conviction or acquittal, or a
proclamation of amnesty. However, the lawful protection to
which an accused has become entitled is qualified and not
given a broad scope. It does not embrace the mode of
procedure provided for in the statutory right to appeal. Thus, a
requirement for unanimity of justices for the affirmance of a
death sentence can be subsequently changed without violating
the constitutional proscription
Historical Background
The Ex Post Facto Clause refers to criminal laws. However, this
was not always the understanding. Early commentators
understood ex post facto laws to include all laws of
retrospective application, whether civil or criminal... The
constitutional inhibition refers only to criminal laws which are
given retroactive effect. It does not preclude, therefore, the
retroactive application of an extradition treaty since said
international agreement is neither a piece of criminal legislation
nor a criminal procedural statute

34

Another guarantee of fairness, this provision assures that a


person could only be found guilty of a crime if, at the time of
its commission, it was already defined as such by law. The law
can only be prospective, covering events and incidents after its
effectivity, and not one reaching back to punish conduct or acts
which were not considered offenses at the time of their
commission or omission

34

(1) makes criminal an act done before the passage of the law
and which was innocent when done, punishes such an act;

(5) assuming to regulate civil rights and remedies only, in effect


imposes penalty or deprivation of a right for something which
when done was lawful; and

Q3. Gonzales v. Abaya


498 SCRA 445 (2006)

No ex post facto law or bill of attainder shall be enacted.

Ex Post Facto Laws

The essence of the prohibition that the Ex Post Facto Clause


seeks to prevent is the prejudice that an ex post facto law rings
to the accused. Accordingly, if the new law is actually favorable
to him, then it will be given a retroactive effect

Legislative Acts v. Judicial Determinations


Further, as the language of the Clause makes clear, it is
addressed to the legislature prohibiting it from enacting such
149 | P

LATON

kinds of laws. Accordingly, it is not directed at judicial


determinations. But the spirit of due process precludes similarly
the courts from acting unreasonably by retroactively reaching
out to prejudice an accused. The American Supreme Court said
in Marks v. United States:
The Ex Post Facto Clause is a limitation upon the powers of the
Legislature and does not, of its own force, apply to the Judicial Branch
of government. But the principle on which the Clause is based--the
notion that persons have a right to fair warning of that conduct which
will give rise to criminal penalties--is fundamental to our concept of
constitutional liberty. As such, that right is protected against judicial
action by the Due Process Clause of the Fifth Amendment

Bills of Attainder
Bills of attainder are legislative acts which inflicts punishment
on named individuals or members of an easily ascertainable
group without judicial trial. Its essence is the substitution of a
legislative for a judicial determination of guilt
The U.S. Supreme Court gave a concise background of the
constitutional proscription against bills of attainder in United
States v. Brown:
A logical starting place for an inquiry into the meaning of the
prohibition is its historical background. The bill of attainder, a
parliamentary act sentencing to death one or more specific persons,
was a device often resorted to in sixteenth, seventeenth and
eighteenth century England for dealing with persons who had
attempted, or threatened to attempt, to overthrow the government.
In addition to the death sentence, attainder generally carried with it a
'corruption of blood,' which meant that the attainted party's heirs could
not inherit his property. The 'bill of pains and penalties' was identical
to the bill of attainder, except that it prescribed a penalty short of
death, e.g., banishment, deprivation of the right to vote, or exclusion of
the designated party's sons from Parliament. Most bills of attainder and
bills of pains and penalties named the parties to whom they were to
apply; a few, however, simply described them. While some left the
designated parties a way of escaping the penalty, others did not. The
use of bills of attainder and bills of pains and penalties was not limited
to England. During the American Revolution, the legislatures of all
thirteen States passed statutes directed against the Tories; among
these statutes were a large number of bills of attainder and bills of
pains and penalties.

The bill of attainder was a tool for political vendetta in an everchanging landscape of temporal ascendancy: Bills of attainder
were typically directed at once powerful leaders of
government. By special legislative Acts, Parliament deprived
one statesman after another of his reputation, his property,
and his potential for future leadership. The motivation of such
bills was as much political as it was punitive--and often the
victims were those who had been the most relentless in
attacking their political enemies at the height of their own
power

banishment, deprivation of the right to vote, confiscation of


property or disqualification from certain employment or
vocations. However, as intended in the Bill of Rights, it
encompasses both.
Although the prohibition against bills of attainder has been addressed
only infrequently by this Court, it is now settled beyond dispute that a
bill of attainder, within the meaning of Art. I, is by no means the same
as a bill of attainder at common law. The definition departed from the
common law concept very early in our history, in a most fundamental
way. At common law, the bill was a death sentence imposed by
legislative Act. Anything less than death was not a bill of attainder, but
was, rather, "a bill of pains and penalties." This restrictive definition
was recognized tangentially in Marbury v. Madison, 1 Cranch 137, 179
(1803), but the Court soon thereafter rejected conclusively any notion
that only a legislative death sentence or even incarceration imposed on
named individuals fell within the prohibition. Mr. Chief Justice Marshall
firmly settled the matter in 1810, holding that legislative punishment in
the form of a deprivation of property was prohibited by the Bill of
Attainder Clause: A bill of attainder may affect the life of an
individual, or may confiscate his property, or may do both.

Bills of Attainder and the Doctrine of Separation of Powers


It has also been stated that the Bill of Attainder Clause is
related to the doctrine of separation of powers. The best
available evidence, the writings of the architects of our
constitutional system, indicates that the Bill of Attainder Clause
was intended not as a narrow, technical (and therefore soon to
be outmoded) prohibition, but rather as an implementation of
the separation of powers, a general safeguard against
legislative exercise of the judicial function, or more simply
trial by legislature. The concern of the Framers of the U.S.
Constitution that a legislature should not be able unilaterally
to impose a substantial deprivation on one person was
expressed not only in this general allocation of power, but also
in more specific provisions, such as the Bill of Attainder
Clause,... This Clause, and the separation-of-powers doctrine
generally, reflect the Framers concern that trial by a legislature
lacks the safeguards necessary to prevent the abuse of power.
Nevertheless, the Bill of Attainder Clause not only was
intended as one implementation of the general principle of
fractionalized power, but also reflected the Framers' belief that
the Legislative Branch is not so well suited as politically
independent judges and juries to the task of ruling upon the
blameworthiness, of, and levying appropriate punishment
upon, specific persons.
People vs. ferrer
Respondents co and tayag and others were charged with violation of
anti-subversion law which outlaws the communist party of he
Philippines and other subversive associations and punishes those who
affiliates becomes or remains a member pf the party or any subversive
organizations.
Won the act is a bill of attainder

Bills of Pains and Penalties

The essence of bill of attainder is substitution of a legislative for a


judicial determination of guilt. In the case at bar the act was
condemned because it tars and feathers the communist party of the
Philippines as a continuing menace to the freedom and security of the
country.

The proscription against bills of attainder also includes the socalled bills of pains and penalties, As noted above, it its
restrictive sense, the bills of attainder referred to the
imposition of death as a penalty while the bills of pains and
penalties referred to lesser penalties, such as imprisonment,

SC: no. It did not specify the communist party or members to be


punished. What it does is declare the party to be an organized
conspiracy to overthrow the government. Its focus is not on individuals
but on conduct. The court will still be judicially established. Indeed it is
onl when a statute applies either to named individuals or to easily
ascertainable members of a group in such a way as to inflict a
150 | P

LATON

punishment on them without a judicial trial does it become a billof


attainder.nor it is enough that a statute specfy persons or groups in
order that it may fall within the ambit of prohibition against bll of
attainder,. It is also necessary to that it must apply retroactively and
reach past conduct. If a statute is a bill of attainder it is also an expost
facto law.
Nixon V Administrator Of General Services
After appellant had resigned as the president of the united states, he
executed Nixon-sampson agreement with the administrator of general
services that provided the storage of documents and tape recordings
near his home, with this agreement, neither the administrator nor the
president can access the materials without each others consent. After
3 years, appellant could withdraw the documents but not the tapes
which can be withdrawn after 5 years. Following the 5 years, the
administrator would destroy such tapes and or at appellants death or
after expiration of 10 years whichever occurred first. The presidential
recordings and materials preservation act wqs enacted by congress to
abrogate agreement
Appellant argued that it is a bill of attainder. The act was faulted for
singling out appellant as opposed to all the presidents or members of
the government.
Appellants contention of meaning of bill of attainder is too much. By
arguing that an individual or defined group is attained whenever he or
he is compelled to bear burdens which individual or group dislikes, the
appellant removes the anchor that ties the bill of attainder to realistic
conceptions. His view could cripple the very process of legislating, for
every individual tht is made subject of adverse legislation can complain
that the lawmakers should have defined the relevant affected class at a
greater level of generality the specificity does not automatically offend
the bill of attainder clause.
Congress expressed no interest in punishing or penalizing appellant
butu justified the act according to objectives to preserve judicial
evidence and historical materials. In judging the constitutionality of
the act, we may only look to its terms, to the intent expressed by
members of congress and existence of legitinmate explanations.

appeal.
Held: Yes. AO 13 and MO 61 are not ex post facto laws.
The constitutional doctrine that outlaws an ex post facto law generally
prohibits the retrospectivity of penal laws. Penal laws are those acts of
the legislature which prohibit certain acts and establish penalties for
their violations; or those that define crimes, treat of their nature, and
provide for their punishment. The subject administrative and
memorandum orders clearly do not come within the shadow of this
definition. Administrative Order No. 13 creates the Presidential Ad
Hoc Fact-Finding Committee on Behest Loans, and provides for its
composition and functions. It does not mete out penalty for the act of
granting behest loans. Memorandum Order No. 61 merely provides a
frame of reference for determining behest loans. Not being penal laws,
Administrative Order No. 13 and Memorandum Order No. 61 cannot
be characterized as ex post facto laws. There is, therefore, no basis for
the Ombudsman to rule that the subject administrative and
memorandum orders are ex post facto.

Chapter 24
Citizenship
The following are citizens of the Philippines:
(1) Those who are citizens of the Philippines at the time of the
adoption of this Constitution;
(2) Those whose fathers or mothers are citizens of the
Philippines;
(3) Those born before January 17, 1973, of Filipino mothers,
who elect Philippine citizenship upon reaching the age of
majority; and
(4) Those who are naturalized in accordance with law.

If a man is to be punished for what he has done, it must be in


accordance with rules which were already in existence at the
time he acted. A law cannot be passed to reach back to days of
innocence and suddenly, by legal fiat, declare certain acts
which were then considered perfectly legal as badges of guilt.
Much less can the legislature simply create by its own hands,
without any judicial determination, a judgment that certain
persons are guilty and visit upon them the corresponding
penalty, similarly fashioned just then by them. In any society
that abides by the dictates of due process and republicanism,
such legislative determinations could never be considered
consistent with expectations of fairness and justice, as well as
with the overall design that balances States authority with
individuals right to liberty and freedom
Additional Cases
(R) Ex Post Facto Laws [2]
R1. Salvador v. Mapa, Jr
539 SCRA 34 (2007)
President Ramos issued AO13/MO61 creating the Presidential Ad Hoc
Fact-Finding Committee on Behest Loans. Several loan accounts were
referred to the Committee for investigation, among others is that of
Philippine Eagles Mines, Inc. (PEMI) whose officers were known cronies
of then President Marcos. The Committee then filed a complaint
against the respondents before the Ombudsman for violation of AntiGraft and Corrupt Practices Act. The complaint was dismissed due to
prescription and if such orders are to be considered as bases of
charging respondents, they become ex post facto laws. Hence, the

35

Natural-born citizens are those who are citizens of the


Philippines from birth without having to perform any act to
acquire or perfect their Philippine citizenship. Those who elect
Philippine citizenship in accordance with paragraph (3), Section
36
1 hereof shall be deemed natural-born citizens.
Philippine Citizens
Citizens by Election
Application for admission to the Philippine bar, Vicente ching
Vicente Ching,legitimate son of a Chinese citizen and a Filipino was
born 1964.after completing bachelor of laws in st louis baguio he filed
an application to take the 1998 bar examinations. He was permitted to
take the bar subject to the condition that he must submit proof of his
Filipino citizenship. He submitted certification by board of accountancy
that he is a CPA and a voters certification and that he was elected as
sangguniang bayan of la union. He passed the bar exams but he was not
allowed to take the oath unless he submits further proof of citizenship.
In the comment filed by OSG, it states that being the legitimate child of
Chinese father and a Filipino mother bprn under the 1935 constitution
was a chinese citizen and continued to be so until upon reaching the
age of majority he elected Philippine citizenship. Ching has not formally
elected Philippine citizenship and if ever he does it is no longer within
the reasonable time allowed by jurisprudence.
Can a legitimate child born under the 1935 constitution of a Filipino
35
36

CONSTITUTION, Art. IV, 1


CONSTITUTION, Art. IV, 2

151 | P

LATON

mother and an alien father 14 years after he has reached the age of
majority?
The reasonable time has been interpreted to mean that the election
should be made within 3 years from reaching age of majority. But this
period may be extended under certain circumstances as when the
person concerned has always considered himself a Filipino. However
span of 14 years is way beyond the contemplation of the requirements
of electing upon reaching the age of majority. Ching offered no reason
why he delayed his election. Philippine citizenship can never be treated
like a commodity that can be claimed when needed and suppressed
when convenient. One who is privileged to elect Philippine citizenship
has only an inchoate right to such citizenship. He should avail his right
with fervor enthusiasm and promptitude. Application to Philippine bar
denied.

allowed to takenoath of allegiance despite the opposition of republic.


Won a Chinese woman may be lawfully naturalized as a citizen of the
Philippines separately from her husband also a citizen of China.
It is clear that she is a Chinese bgy birt, parentage and by marryiage to
a Chinese. And commonwealth act also declares that a woman loses
her citizenship if married to an alien if by vitue of the laws in her
husbands country, she acquires the latters nationality.

Natural-Born Citizens

It is true that our laws do not say that only male alien may seek
naturalization but we must take into account other provisions on
citizenship. The granting of citizenship by naturalization to a female
alien applicant while her marriage to another alien is subsisting would
be inoperative because she would still be a citizen of the country of her
husand. The result would be dual citizenship which is believed not
contemplated under our laws.

Tecson vs. COMELEC

Denaturalization

The 3 consolidated cases challenge the qualification of FPJ to hold the


highest office of the land on account of his allegedly not being a natural
born citizen. His mother was an American and his father a Spanish
national. Petitioner averred that assuming that his father was a Filipino
citizen, this would not be transmitted to him because he is an
illegitimate child of an alien mother.

Chan Tek Lao v. Republic


55 SCRA 1 (1974)

Citizenship is a treasured right conferred to those whom the state


believes are deserving of the privilege. It is a precious heritage, as well
as an inestimable acquisition, that cannot be taken lightly by anyone
either by those who enjoy it or by those who dispute it.
The evidence may not establish conclusively that respondent FPJ is a
natural born citizen, the evidence on hand still would preponderate in
his favor enough to hold that he cannot be held guilty having made
material misrepresentation in his certificate of candidacy.

Facts: Petitioners application for naturalization was denied by the trial


court but the same was reversed by the SC. More than 10 yrs. later, the
OSG filed a petition for the cancellation of the certificate of
naturalization, invoking a subsequent issue in Tan Ten Koc v. Republic,
since there was no proof that the Nueva Era was a newspaper of
general circulation in the province of Tarlac, where the petitioner then
resided. It must be proved that the petition was published in a
newspaper of general circulation in the province where the proceedings
were held. The SC noted that although it reversed the decision of the
trial court, it has no jurisdiction to hear the application if Chan Tek Lao
and therefore ordered the cancellation of his certificate of
naturalization.
Issue: WON petitioners certificate of naturalization should be cancelled

Naturalization
Qualifications and Disqualifications
Procedure
Effect of Naturalization on Spouse and Children
BURCA vs republic
Petitioner, a Chinese woman filed a petition alleging that she Is married
to a Filipino citizen and possesses all the qualifications and none of the
disqualifications for naturalization. The SG opposed because it lacks
essential requisites such as formen resisdenc etc. the rtc granted
naturalization. SC agreed with SG and reversed rtc judgement.
The only means by which an alien wife of Filipino be declared as a
Filipino is by full compliance with the procedure in naturalization law.
She must also prove that she has all the qualifications and none of
disqualifications. She must file petition for naturalization in court,
allege and prove all the requisites such as continuous residency of
atleast 10 years, lucrative income, etc.
We declare it to be a sound rule that where the citizenship of a party in
a case is resolved by a court or by an administratie agency, as a material
issue to the controversy, after full blown hearing, with the sg of his
authorized representative, and this finding is affirmed by court, the
decision is conclusive f citizenship.
Po vs republic
Betty po lim a native born Chinese woman married lim son hue, alsoa
Chinese, filed with the lower court a petition for admission as citizen of
the Philippines under the commonwealth act no. 473. She was also

Held: As decided in Gan Tsitung v. Republic, no retroactive effect is to


be given a judicial pronouncement that would impose on a party
proceeded against in a denaturalization proceeding a requirement not
in existence at a time that his application was heard and favorably
acted on. It would be unfair to set aside a decision that has become
final and has led to the grant of citizenship. Moreover, the status of the
petitioner as a national of this country for 13 yrs. has remained
undisturbed. The decision of the lower court ordering the cancellation
of the certificate of naturalization of the petitioner is set aside and
reversed.

Loss and Reacquisition of Citizenship


Loss of Citizenship
Yu v. Defensor-Santiago
169 SCRA 364 (1989)
Facts: The case originated from a petition for habeas corpus filed with
the SC on July 4, 1988 seeking for the release from detention of herein
petitioner. Upon its denial of the petition for habeas corpus, in its
November 10, 1988 resolution the SC disposed of the pending issues of
(1) jurisdiction of the Commission on Immigration and Deportation
(CID) over a naturalized Filipino citizen and (2) validity of warrantless
arrest and detention of the same person. SC gave petitioner Yu a nonextendible period of 3 days from notice to explain and prove why he
should still be considered a citizen of the Philippines despite his
acquisition and use of a Portuguese passport.
Issue: WON petitioners claim to continued Philippine citizenship is
meritorious
Held: Petitioners motion for release from detention is denied. As
petitioner was originally issued a Portuguese passport which he
152 | P

LATON

renewed, and despite of his naturalization as a Philippine citizen on Feb


10, 1978, he still applied for and was issued a Portuguese passport by
the Consular Section of the Portugal Embassy in Tokyo. Also, while still
a Philippine citizen who had renounced upon his naturalization,
absolutely and for all allegiance and fidelity to any foreign prince,
potentate state or sovereignty and pledged to maintain true faith
and allegiance to the Rep. of the Phils., he declared his nationality as
Portuguese in commercial documents. Such acts were considered by
the SC amount to an express renunciation of petitioners Philippine
citizenship acquired through naturalization. Express renunciation
means a renunciation that is made known distinctly, and explicitly and
not left to inference or implication (Board of Immigration
Commissioners v. Go Gallano).

repatriated and ordered the cancellation of her alien certificate of


registration. The Provincial Fiscal, however, appealed.

Aznar v. COMELEC
185 SCRA 703 (1990)

Angat v. Republic
314 SCRA 438 (1999)

Facts: Private respondent Emilio Lito Osmena filed his certificate of


candidacy for the January 1988 local elections. However, the Cebu PDPLaban Provincial Council (Cebu-PDP Laban), represented by petitioner
Jose B. Aznar, the incumbent Provincial Chairman, filed with the
COMELEC, a petition for disqualification of private respondent alleging
that that he is not a citizen of the Philippines, but of USA. Aznar
submitted then a certificate issued by the CID certifying the private
respondent is an American and holder of Alien Certificate of
Registration. The COMELEC resolved to order the Board of Canvassers
to continue canvassing but to suspend the proclamation. At the hearing
before the COMELEC, private respondent claims that he is a Filipino
citizen since he is a legitimate child of Dr. Emilio D. Osmena, a Filipino
and son of late President Sergio Osmena, Sr.; that he is a holder of a
valid and subsisting Phil. Passport; that he has been continuously
residing in the Philippines since birth; that he has not gone out of the
country for more than 6 months, and that he has been a registered
voter in the Phils. since 1965. Having obtained the highest no. of votes,
private respondent was proclaimed the Provincial Governor of Cebu.
COMELEC dismissed the petition for disqualification because it was not
timely filed and it lacks of sufficient proof.

Facts: Petitioner Gerardo Angat was a natural born Filipino citizen who
lost such until his citizenship by naturalization in USA. On Mar. 11,
1996, he filed before the RTC of Marikina City a petition to regain his
status as a Filipino citizen under CA No. 63, RA No. 965 and RA No. 2630
stating that he was born in 1954 in Tondo, Manila; he lost his Philippine
citizenship when he got naturalized as an American; he returned to the
Phils. in 1991; he has the qualifications required in CA No. 63, and RA
Nos. 965 and 2639 to reacquire Philippine citizenship; he possesses
none of the disqualification prescribed in CA No. 473 and he has reside
in the Phils. at least 6 mos. immediately preceding the date of the
petition. He sought to be allowed to take his oath of allegiance to the
RP but the motion was denied. He filed another motion to have the
denial reconsidered and he was allowed by the court a quo. The court
also ordered him to take his oath of allegiance to the RP. Thereafter,
petitioner was declared as repatriated and a citizen of the Phils.
However, the OSG filed a Manifestation and Motion for reconsideration
asserting that the petition should have been dismissed since the court a
quo lacks jurisdiction because the proper forum for it was the Special
Committee on Naturalization. The court a quo found the petition
meritorious and declared that it has no jurisdiction and considered the
granting of the petition null and void. Petitioner filed a motion for
reconsideration since his petition was filed on Mar 14 1996 or months
before the Special Committee on Naturalization was constituted by the
Pres. under AO 285 on Aug. 22, 1996, and stated that the court a quo
had the authority over the case. The RTC denied the motion and
petitioner assails the lower courts dismissal of the petition by giving
retroactive effect to AO 285.

Issues: WON private respondent is a Filipino citizen


WON he is allowed to run for and being elected to the office of
Provincial Governor of Cebu
Held: Petitioner failed to present proof that private respondent had lost
Filipino citizenship by any of the modes provided for under C.A. No. 63
which includes (1) by naturalization in a foreign country; (2) by express
renunciation of citizenship; and (3) by subscribing to an oath of
allegiance to support the Constitution or laws of a foreign country.
From the evidence, it is clear that private respondent did not lose his
Philippine citizenship by any of the aforementioned modes or any other
mode of losing Philippine citizenship. There is neither an express or
implied renunciation of Philippine citizenship by private respondent; I It
must be noted that in order to lose Philippine citizenship, renunciation
must be express. Petition for certiorari is DISMISSED and Resolution of
the COMELEC is AFFIRMED.

Reacquisition of Citizenship
Repatriation
Jao v. Republic
121 SCRA 358 (1983)
Facts: Jao filed a petition in the CFI of Davao for repatriation alleging
that although her father was Chinese, she was a Filipino citizen because
her mother was a Filipina who was not legally married to her Chinese
husband; that she lost her Philippine citizenship when she married a
Chinese with whom she had 3 children and that he died in 1962; that
her illiterate mother erroneously registered her as an alien with the
Bureau of Immigration which issued her an Alien Certificate of
Registration. Petition was not published but notice was served to the
Provincial Fiscal. The CFI subsequently, declared petitioner as judicially

Issue: WON petitioner, as judicially repatriated, is already considered a


Filipino citizen
Held: There is no law requiring or authorizing that repatriation should
be effected by a judicial proceeding. All that is required for a female
citizen of the Philippines who lost her citizenship to an alien to
reacquire her Philippine citizenship, upon the termination of her
marital status, is for her to take necessary oath of allegiance to the
Republic of the Philippines and to register the said oath in the proper
civil registry. Decision appealed from is hereby revoked and set aside.

Issue: WON the court a quo had the jurisdiction over the petitioners
case
Held: Under PD No. 725, dated June 5, 1975, amending CA No. 63, an
application for repatriation could be filed by Filipino women who lost
their Philippine citizenship by marriage to aliens, as well as by natural
born Filipinos who lost their Philippine citizenship with the Special
Committee on Naturalization. The OSG was right in contending that the
petition should have been filed with the Committee and not with the
RTC which has no jurisdiction. It was also incorrect for the petitioner to
invoke RA Nos. 965 and 2639 since these laws could only apply to
persons who had lost their citizenship by rendering service to or
accepting commission in the armed forces of an allied foreign country
or of the USA. Also, a person who desires to reacquire Philippine
citizenship would not even be required to file a petition in court. All he
has to do is to take an oath of allegiance to the RP and to register that
fact with the civil registry in the place of his residence or where he had
last resided in the Phils. Petition for review is DENIED.
Frivaldo v. COMELEC
174 SCRA 245 (1989)
Facts: Petitioner Juan G. Frivaldo was proclaimed to be the governorelect of Sorsogon on Jan 22, 1988. However, the League of
Municipalities, represented by its President filed with the COMELEC a
petition for the annulment of Frivaldos election and proclamation since
he was not a Filipino citizen having been naturalized in the USA on Jan
23 1983. Frivaldo admitted that he was naturalized but only due to his
153 | P

LATON

protection against Pres. Marcos. He asserted that he also returned to


the Phils. after the EDSA revolution. He also claims that the League was
not the proper party to file the petition because it is not a voter and
could not sue him. The COMELEC denied Frivaldos motion for
reconsideration. He then filed a petition for certiorari and prohibition
before the SC. He claimed that his oath in his certificate of candidacy
that he was a natural-born citizen should be sufficient act of
repatriation and that his active participation in the 1987 congressional
elections had divested him of American citizenship under the laws of
the USA , thus restoring his Philippine citizenship.
Issue: WON Frivaldo is a citizen of the Philippines under our own laws,
regardless of other nationality laws
Held: If he really wanted to disavow his American citizenship and
reacquire Philippine citizenship, Frivaldo should have done such in
accordance the Philippine laws. Under CA no. 63 as amended by CA No.
43 and PD No. 725, Philippine citizenship may be reacquired by direct
act of Congress, by naturalization or by repatriation. Since Frivaldo did
not invoke either of the first two methods, he still claims that he has
reacquired Philippine citizenship by virtue of a valid repatriation as he
actively participated in the 1987 election. However, such act might
have forfeited his American citizenship but it does not necessarily mean
that he has automatically reacquired Philippine citizenship. Petition is
DISMISSED and petitioner is declared not a citizen of the Philippines
and therefore, disqualified from serving as Governor of Sorsogon.
Republic v. De La Rosa
232 SCRA 785 (1994)
It involves three consolidated petitions. The private respondent filed a
petition for naturalization. The judge granted the petition and
readmitted Frivaldo as a citizen of the Philippines by naturalization.
Frivaldo ran for governor in the May 1992 elections. Raul Lee also ran
for the same position. The former won and Lee filed a petition with a
petition to the COMELEC to annul the proclamation on the ground that
the private respondent is not a duly registered voter due to the pending
case. The COMELEC dismissed the petition.
The COMELEC failed to resolve the more serious issue the
disqualification of private respondent to be proclaimed Governor on
grounds of lack of Filipino citizenship. Both the Local Government Code
and the Constitution require that only Filipino citizens can run and be
elected to public office.
Private respondent is declared NOT a citizen of the Philippines and
therefore DISQUALIFIED from continuing to serve as GOVERNOR of the
Province of Sorsogon. He is ordered to VACATE his office and to
SURRENDER the same to the Vice-Governor of the Province of
Sorsogon.
Frivaldo v. COMELEC
257 SCRA 727 (1996)
Frivaldo filed his Certificate of Candidacy for the office of Governor of
Sorsogon while Raul Lee, another candidate, filed a petition praying
that Frivaldo be disqualified from seeking or holding any public office
or position by reason not yet being a citizen of the Philippines.
Who should be declared the rightful governor of Sorsogon?
The Local Government Code of 1991 expressly requires Philippine
citizenship as a qualification for elective local officials, including the
provincial governor. Under Philippine law, citizenship may be
reacquired by direct act of Congress, by naturalization or by
repatriation.
Despite his lack of Philippine citizenship, Frivaldo was overwhelmingly
elected governor by the electorate of Sorsogon. Twice, he was judicially
declared a non Filipino and thus twice disqualified from holding and
discharging his popular mandate.

elective public office, and the purpose of the citizenship qualification is


none other than to ensure that no alien, i.e., no person owing
allegiance to another nation, shall govern our people and our country
or a unit of territory thereof. Now, an official begins to govern or to
discharge his functions only upon his proclamation and on the day the
law mandates his term of office begin.
Being a former Filipino who has served the people repeatedly, Frivaldo
deserves a liberal interpretation of Philippine laws and whatever
defects there were in his nationality should now be deemed mooted by
his repatriation.
In sum, we rule that the citizenship requirement in the Local
Government Code is to be possessed by an elective official at the latest
as of the time he is proclaimed and at the start of the term of office to
which he has been elected.
Laws governing election contest must be liberally construed to the end
that the will of the people in the choice of public officials may not be
defeated by mere technical objections.
Labo, Jr. vs. COMELEC
176 SCRA 1 (1989)
Ponente: Cruz, J.
FACTS: Ramon Labo, Jr., married, an Australian citizen in the
Philippines. He was granted Australian citizenship in 1976. In 1980, the
marriage was declared void for being bigamous. Labo returned to the
Philippines in 1980, using an Australian passport, and obtained an Alien
Certificate of Registration (ACR). He later applied for a change in status
from immigrant to returning Filipino citizen. However, the Commission
on Immigration and Deportation denied his application for the
cancellation of his ACR since he has not applied for reacquisition of his
Filipino citizenship. According to the records of the Australian Embassy
(as certified by the Australian Consul), Labo was still an Australian
citizen as of April 12, 1984. Although no direct evidence was presented
to prove that he took an oath of allegiance as a naturalized Australian
citizen, the laws of Australia at the time required any person over the
age of 16 years who is granted Australian citizenship to take an oath of
allegiance. The wording/text of this oath includes a renunciation of all
other allegiance. Labo ran and won as Mayor of Baguio City in the local
elections held on January 18, 1988. The second-placer, Luis Lardizabal,
filed a petition for quo warranto, alleging that Labo is disqualified from
holding public office on the grounds of alienage, and asking that the
latter's proclamation as Mayor be annulled.
ISSUES: *The original issue raised before the Supreme Court concerned
only the COMELEC's jurisdiction over Lardizabal's petition. Labo
contended that the petition for quo warranto was not filed on time,
hence the COMELEC lacks the jurisdiction to conduct an inquiry
regarding his citizenship. However, the SC decided to rule on the merits
of the case, given that the issue is also of considerable importance (a
foreign citizen holding public office in the Philippines), and in the
interest of the speedy administration of justice.
1. Does the COMELEC have the jurisdiction to inquire into Labo's
citizenship?
2. Is Ramon Labo, Jr. a Filipino citizen?
3. Is he qualified to hold public office in the Philippines?
4. If Labo is not eligible to serve as Mayor, can Lardizabal, as the
runner-up in the elections, replace him?
HELD/RATIO:
1. Yes. Contrary to Labo's claim, the petition for quo warranto was filed
on time. Lardizabal did not immediately pay the filing fee because the
COMELEC had at first considered the petition as a pre-proclamation
proceeding, which does not require the payment of such a fee. When
the COMELEC reclassified the petition, Lardizabal immediately paid the
filing fee -- thus, he still complied with the prescribed 10-day period.
Furthermore, the Court held that such technicalities should not hinder
judicial decisions on significant issues, such as the one being decided in
this case.

Philippine citizenship is an indispensable requirement for holding an


154 | P

LATON

2. Labo is not a Filipino citizen. He had lost his Philippine citizenship by


all 3 modes specified in the Constitution: (1) naturalization in a foreign
country, (2) express renunciation of citizenship, and (3) subscribing to
an oath of allegiance to support the Constitution or laws of a foreign
country. He has not reacquired Philippine citizenship by any of the 3
methods prescribed in the Constitution: (1) direct act of Congress, (2)
naturalization, and (3) repatriation.

proceeding established by the law, or the Rules for the judicial


declaration of the citizenship of an individual. Hence, a judicial
declaration that a person is a Filipino citizen cannot be made in a
petition for naturalization because under our laws there can be no
action or proceeding for the judicial declaration of the citizenship of an
individual. Such a declaration or pronouncement is beyond the courts
jurisdiction.

- Contrary to Labo's claim, his naturalization in Australia did not confer


him with dual citizenship. The Constitution explicitly states that dual
citizenship is inimical to national interest.

The appealed decision is AFFIRMED and the Commissioner of


Immigration and Deportation is ordered to CANCEL applicants alien
certificate of registration.

BENGSON III V. HRET


357 SCRA 545 (2001)

CHIAO BEN LIM V. ZOSA


146 SCRA 366 (1986)

Facts: Respondent Teodoro Cruz was a natural-born citizen of the


Philippines. He was born in San Clemente, Tarlac, on April 27, 1960, of
Filipino parents. The fundamental law then applicable was the 1935
Constitution. On November 5, 1985, however, respondent Cruz enlisted
in the United States Marine Corps and without the consent of the
Republic of the Philippines, took an oath of allegiance to the United
States. As a Consequence, he lost his Filipino citizenship for under
Commonwealth Act No. 63, section 1(4), a Filipino citizen may lose his
citizenship by, among other, "rendering service to or accepting
commission in the armed forces of a foreign country. He was
naturalized in US in 1990. On March 17, 1994, respondent Cruz
reacquired his Philippine citizenship through repatriation under
Republic Act No. 2630. He ran for and was elected as the
Representative of the Second District of Pangasinan in the May 11,
1998 elections. He won over petitioner Antonio Bengson III, who was
then running for reelection.

FACTS: The petitioner filed a petition for the correction of an alleged


wrong entry on the birth records of Kim Joseph describing him as a
Chinese national instead of a Filipino citizen. He sought to prove the
error through several pieces of evidence. The respondent judge
dismissed the petition and sustained the contention that only clerical
errors were allowed to be corrected. Substantial issues like citizenship
were not covered. In effect, it was held the petition was for a judicial
declaration of citizenship, which was not allowed under existing rules.

Issue: Whether or Not respondent Cruz is a natural born citizen of the


Philippines in view of the constitutional requirement that "no person
shall be a Member of the House of Representative unless he is a
natural-born
citizen.
Held: Respondent is a natural born citizen of the Philippines. As
distinguished from the lengthy process of naturalization, repatriation
simply consists of the taking of an oath of allegiance to the Republic of
the Philippine and registering said oath in the Local Civil Registry of the
place where the person concerned resides or last resided. This means
that a naturalized Filipino who lost his citizenship will be restored to his
prior status as a naturalized Filipino citizen. On the other hand, if he
was originally a natural-born citizen before he lost his Philippine
citizenship, he will be restored to his former status as a natural-born
Filipino.
JUDICIAL DECLARATION AND CORRECTION OF CITIZENSHIP
The general rule is that there is no judicial proceeding for declaration of
citizenship. Thus, in the past, corrections of entries in birth certificates
with regard to citizenship were considered not possible as this would
involve making a declaration that a person is a Philippine citizen or not.
However, the Court has relaxed the rules.

Judicial Declaration and Correction of Citizenship


Yung Uan Chu V. Republic
159 SCRA 593 (1988)
FACTS: Appellee married to a native born citizen of the Philippines.
Out of said wedlock and at the time of the filing of the petition for
naturalization, the couple had six (6) children, with ages ranging from 6
to 15 years old. All said children were registered as natural born Filipino
citizens. Find by the trial court that the appellee possessed of all the
qualifications and none of the disqualifications of a Filipino citizen and
therefore authorized to take her oath of allegiance to the Republic of
the Philippines and to register the same in the proper civil registrar.
RULING: A careful examination of the records shows that the sole and
only purpose of the petitioner is to have the petitioner declared a
Filipino citizen. This Court has consistently ruled that there is no

RULING: Article 412 of the Civil Code simply provides: No entry in the
civil registry shall be changed or corrected without a judicial order. In a
number of earlier cases, the Court has ruled that the birth entry
regarding a persons citizenship could not be changed under Rule 108
as this would involve substantive rights that the rules of court could not
diminish, increase, or modify under the Constitution. Rule 108 of the
Rules of Court provides only the procedure or mechanism for the
proper enforcement of the substantive law embodied in Article 412 of
the Civil Code and so does not violate the Constitution.

Dual Citizenship and Allegiance

Additional Cases
(S) Citizenship [5]

S2. JOEVANIE ARELLANO TABASA vs. HON. COURT OF APPEALS,


BUREAU OF IMMIGRATION and DEPORTATION and WILSON SOLUREN.
petitioner Joevanie Arellano Tabasa was a natural-born citizen of the
Philippines. By derivative naturalization (citizenship derived from that
of another as from a person who holds citizenship by virtue of
naturalization), petitioner also acquired American citizenship.
His passport was revoked by the US because he is the subject of an
outstanding federal warrant of arrest for violation of Section 1073,
"Unlawful Flight to Avoid Prosecution," of Title 18 of the United States
Code. He is charged with one count of a felon in possession of a
firearm, in violation of California Penal Code, Section 12021(A)(1), and
one count of sexual battery, in violation of California Penal Code,
Section 243.4 (D).
The BID ordered petitioners deportation to his country of origin, the
United States, on May 29, 1996.
Tabasa alleged that he was not afforded due process; that no warrant
of arrest for deportation may be issued by immigration authorities
before a final order of deportation is made; that no notice of the
cancellation of his passport was made by the U.S. Embassy; that he is
entitled to admission or to a change of his immigration status as a nonquota immigrant because he is married to a Filipino citizen as provided
in Section 13, paragraph (a) of the Philippine Immigration Act of 1940;
and that he was a natural-born citizen of the Philippines prior to his
derivative naturalization when he was seven years old due to the
naturalization of his father, Rodolfo Tabasa, in 1968.
155 | P

LATON

Petitioner is alleging that he had acquired Filipino citizenship by


repatriation in accordance with Republic Act No. 8171 (RA 8171), and
that because he is now a Filipino citizen, he cannot be deported or
detained by the respondent Bureau.
Issue:
Whether petitioner has validly reacquired Philippine citizenship under
RA 8171. If there is no valid repatriation, then he can be summarily
deported for his being an undocumented alien.
No. it provides repatriation of only two (2) classes of persons
Filipino women who have lost their Philippine citizenship by marriage
to aliens and natural-born Filipinos who have lost their Philippine
citizenship, including their minor children, on account of political or
economic necessity, may reacquire Philippine citizenship through
repatriation in the manner provided in Section 4 of Commonwealth Act
No. 63, as amended: Provided, That the applicant is not a:
(1) Person opposed to organized government or affiliated with any
association or group of persons who uphold and teach doctrines
opposing organized government;
(2) Person defending or teaching the necessity or propriety of violence,
personal assault, or association for the predominance of their ideas;
(3) Person convicted of crimes involving moral turpitude; or
(4) Person suffering from mental alienation or incurable contagious
diseases. (Emphasis supplied.)
To reiterate, the only persons entitled to repatriation under RA 8171
are the following:
a. Filipino women who lost their Philippine citizenship by marriage to
aliens; and
b. Natural-born Filipinos including their minor children who lost their
Philippine citizenship on account of political or economic necessity.
Petitioner is not entitled to automatic repatriation because he was no
longer a minor at the time of his "repatriation" on June 13, 1996. The
privilege under RA 8171 belongs to children who are of minor age at
the time of the filing of the petition for repatriation.
Neither can petitioner be a natural-born Filipino who left the country
due to political or economic necessity. Clearly, he lost his Philippine
citizenship by operation of law and not due to political or economic
exigencies.
In sum, petitioner is not qualified to avail himself of repatriation under
RA 8171. However, he can possibly reacquire Philippine citizenship by
availing of the Citizenship Retention and Re-acquisition Act of 2003
(Republic Act No. 9225) by simply taking an oath of allegiance to the
Republic of the Philippines.
Even if petitionernow of legal agecan still apply for repatriation
under RA 8171, he nevertheless failed to prove that his parents
relinquished their Philippine citizenship on account of political or
economic necessity as provided for in the law. It is notable that under
the Amended Rules and Regulations Implementing RA 8171, the SCN
requires a petitioner for repatriation to set forth, among others, "the
reason/s why petitioner lost his/her Filipino citizenship, whether by
marriage in case of Filipino woman, or whether by political or economic
necessity in case of [a] natural-born Filipino citizen who lost his/her
Filipino citizenship. In case of the latter, such political or economic
necessity should be specified."
Petitioner contends it is not necessary to prove his political or
economic reasons since the act of renouncing allegiance to ones native
country constitutes a "necessary and unavoidable shifting of his
political allegiance," and his fathers loss of Philippine citizenship
through naturalization "cannot therefore be said to be for any reason
other than political or economic necessity."
Repatriation is not a matter of right, but it is a privilege granted by the
State. This is mandated by the 1987 Constitution under Section 3,
Article IV, which provides that citizenship may be lost or reacquired in
the manner provided by law. The State has the power to prescribe by
law the qualifications, procedure, and requirements for repatriation. It

has the power to determine if an applicant for repatriation meets the


requirements of the law for it is an inherent power of the State to
choose who will be its citizens, and who can reacquire citizenship once
it is lost. If the applicant, like petitioner Tabasa, fails to comply with said
requirements, the State is justified in rejecting the petition for
repatriation.
Petitioner Tabasa, whose passport was cancelled after his admission
into the country, became an undocumented alien who can be
summarily deported. His subsequent "repatriation" cannot bar such
deportation especially considering that he has no legal and valid
reacquisition of Philippine citizenship.
WHEREFORE, this petition for review is DISMISSED, and the August 7,
1996 Decision of the Court of Appeals is AFFIRMED. No costs to the
petitioner.
S3. EDISON SO,vs.REPUBLIC OF THE PHILIPPINES
Edison So filed in the RTC a Petition for Naturalization. RTC Granted the
petition. Respondent Republic of the Philippines, through the Office of
the Solicitor General (OSG), appealed the decision to the CA. CA
Reversed the RTC hence this petition for review on certiorari.
Edison So was born on February 17, 1982, in Manila; he is a Chinese
citizen who has lived in No. 528 Lavezares St., Binondo, Manila, since
birth; as an employee, he derives an average annual income of
around P100,000.00 with free board and lodging and other benefits; he
is single, able to speak and write English, Chinese and Tagalog.
He was denied by the CA because, according to the CA, petitioners two
(2) witnesses were not credible because they failed to mention specific
details of petitioners life or character to show how well they knew him;
they merely "parroted" the provisions of the Naturalization Act without
clearly explaining their applicability to petitioners case. The appellate
court likewise ruled that petitioner failed to comply with the
requirement of the law that the applicant must not be less than 21
years of age on the day of the hearing of the petition; during the first
hearing on December 12, 2002, petitioner was only twenty (20) years,
nine (9) months, and twenty five (25) days old, falling short of the
requirement. The CA stated, however, that it was not its intention to
forever close the door to any future application for naturalization which
petitioner would file, and that it believes that he would make a good
Filipino citizen in due time, a decided asset to this country.
WHETHER OR NOT THE HONORABLE COURT OF APPEALS COMMITTED
REVERSIBLE ERROR WHEN IT REVERSED THE DECISION OF THE
REGIONAL TRIAL COURT OF MANILA.
In its Comment on the petition, respondent countered that R.A. No.
9139 (which took effect on August 8, 2001 and where the applicants
age requirement was lowered to eighteen (18) years old), refers only to
administrative naturalization filed with the Special Committee on
Naturalization; it does not apply to judicial naturalization before the
court, as in the present case.
The petition is denied for lack of merit.
Naturalization signifies the act of formally adopting a foreigner into the
political body of a nation by clothing him or her with the privileges of a
citizen. Under current and existing laws, there are three ways by which
an alien may become a citizen by naturalization: (a) administrative
naturalization pursuant to R.A. No. 9139; (b) judicial naturalization
pursuant to C.A. No. 473, as amended; and (c) legislative naturalization
in the form of a law enacted by Congress bestowing Philippine
citizenship to an alien.
Indeed, R.A. No. 9139 was enacted as a remedial measure intended to
make the process of acquiring Philippine citizenship less tedious, less
technical and more encouraging. It likewise addresses the concerns of
degree holders who, by reason of lack of citizenship requirement,
cannot practice their profession, thus promoting "brain gain" for the
Philippines. These however, do not justify petitioners contention that
156 | P

LATON

the qualifications set forth in said law apply even to applications for
naturalization by judicial act.
There is nothing from which it can be inferred that C.A. No. 473 was
intended to be amended or repealed by R.A. No. 9139. What the
legislature had in mind was merely to prescribe another mode of
acquiring Philippine citizenship which may be availed of by native born
aliens. The only implication is that, a native born alien has the choice to
apply for judicial or administrative naturalization, subject to the
prescribed qualifications and disqualifications.
In the instant case, petitioner applied for naturalization by judicial act,
though at the time of the filing of his petition, administrative
naturalization under R.A. No. 9139 was already available. Consequently,
his application should be governed by C.A. No. 473.
Second. If the qualifications prescribed in R.A. No. 9139 would be made
applicable even to judicial naturalization, the coverage of the law would
be broadened since it would then apply even to aliens who are not
native born. It must be stressed that R.A. No. 9139 applies only to
aliens who were born in the Philippines and have been residing here.
Third. Applying the provisions of R.A. No. 9139 to judicial naturalization
is contrary to the intention of the legislature to liberalize the
naturalization procedure in the country. One of the qualifications set
forth in R.A. No. 9139 is that the applicant was born in the
Philippines and should have been residing herein since birth. Thus, one
who was born here but left the country, though resided for more than
ten (10) years from the filing of the application is also disqualified. On
the other hand, if we maintain the distinct qualifications under each of
the two laws, an alien who is not qualified under R.A. No. 9139 may still
be naturalized under C.A. No. 473.
In any event, petitioner failed to prove that the witnesses he presented
were competent to vouch for his good moral character, and are
themselves possessed of good moral character. It must be stressed that
character witnesses in naturalization proceedings stand as insurers of
the applicants conduct and character. Thus, they ought to testify on
specific facts and events justifying the inference that the applicant
possesses all the qualifications and none of the disqualifications
provided by law.
Petitioners witnesses, Atty. Adasa and Salcedo, did not testify on his
specific acts; they did not elaborate on his traits. Their testimonies do
not convince the Court that they personally know petitioner well and
are therefore in a position to vouch for his qualifications. As correctly
found by the CA, the witnesses testimonies consisted mainly of general
statements in answer to the leading questions propounded by his
counsel. What they conveniently did was to enumerate the
qualifications as set forth in the law without giving specific details. The
pertinent portion of Atty. Adasas testimony follows:
In sum, petitioners witnesses clearly did not personally know him well
enough; their testimonies do not satisfactorily establish that petitioner
has all the qualifications and none of the disqualifications prescribed by
law.
In naturalization proceedings, it is the burden of the applicant to prove
not only his own good moral character but also the good moral
character of his/her witnesses, who must be credible persons. Within
the purview of the naturalization law, a "credible person" is not only an
individual who has not been previously convicted of a crime; who is not
a police character and has no police record; who has not perjured in the
past; or whose affidavit or testimony is not incredible. What must be
credible is not the declaration made but the person making it. This
implies that such person must have a good standing in the community;
that he is known to be honest and upright; that he is reputed to be
trustworthy and reliable; and that his word may be taken on its face
value, as a good warranty of the applicants worthiness.
The records likewise do not show that the character witnesses of
petitioner are persons of good standing in the community; that they are
honest and upright, or reputed to be trustworthy and reliable. The

most that was established was the educational attainment of the


witnesses; however, this cannot be equated with their credibility. In
fine, petitioner focused on presenting evidence tending to build his
own good moral character and neglected to establish the credibility and
good moral character of his witnesses.
We do not agree with petitioners argument that respondent is
precluded from questioning the RTC decision because of its failure to
oppose the petition. A naturalization proceeding is not a judicial
adversary proceeding, and the decision rendered therein does not
constitute res judicata. A certificate of naturalization may be cancelled
if it is subsequently discovered that the applicant obtained it by
misleading the court upon any material fact. Law and jurisprudence
even authorize the cancellation of a certificate of naturalization upon
grounds or conditions arising subsequent to the granting of the
certificate. If the government can challenge a final grant of citizenship,
with more reason can it appeal the decision of the RTC within the
reglementary period despite its failure to oppose the petition before
the lower court.
Thus, petitioner failed to show full and complete compliance with the
requirements of naturalization law. For this reason, we affirm the
decision of the CA denying the petition for naturalization without
prejudice.
It must be stressed that admission to citizenship is one of the highest
privileges that the Republic of the Philippines can confer upon an alien.
It is a privilege that should not be conferred except upon persons fully
qualified for it, and upon strict compliance with the law.
IN LIGHT OF ALL THE FOREGOING, the petition is DENIED for lack of
merit.
SO ORDERED.
Qualifications and Disqualifications according to:
C.A. No. 473 (Judicial Act of acquiring Philippine Citizenship)
Section 2. Qualifications. Subject to section four of this Act, any
person having the following qualifications may become a citizen of the
Philippines by naturalization:
First. He must be not less than twenty-one years of age on the day of
the hearing of the petition;
Second. He must have resided in the Philippines for a continuous period
of not less than ten years;
Third. He must be of good moral character and believes in the
principles underlying the Philippine Constitution, and must have
conducted himself in a proper and irreproachable manner during the
entire period of his residence in the Philippines in his relation with the
constituted government as well as with the community in which he is
living;
Fourth. He must own real estate in the Philippines worth not less than
five thousand pesos, Philippine currency, or must have some known
lucrative trade, profession, or lawful occupation;
Fifth. He must be able to speak and write English or Spanish and any
one of the principal Philippine languages; and
Sixth. He must have enrolled his minor children of school age, in any of
the public schools recognized by the Office of Private Education of the
Philippines (now the Department of Education, Culture and Sports),
where Philippine history, government and civics are taught or
prescribed as part of the school curriculum, during the entire period of
residence in the Philippines required of him prior to the hearing of this
petition for naturalization as Philippine citizen.
Section 4. Who are disqualified. The following cannot be naturalized
as Philippine citizens:
(a) Persons opposed to organized government or affiliated with any
association or group of persons who uphold and teach doctrines
opposing all organized governments;
(b) Persons defending or teaching the necessity or propriety of
violence, personal assault, or assassination of the success and
157 | P

LATON

predominance of their ideas;


(c) Polygamist or believers in the practice of polygamy;
(d) Persons convicted of crimes involving moral turpitude;
(e) Persons suffering from mental alienation or incurable contagious
diseases;
(f) Persons who, during the period of their residence in the Philippines,
have not mingled socially with the Filipinos, or who have not evinced a
sincere desire to learn and embrace the customs, traditions, and ideals
of the Filipinos;
(g) Citizens or subjects of nations with whom the United States and the
Philippines are at war, during the period of such war;
(h) Citizens or subject of a foreign country other than United States,
whose laws do not grant Filipinos the right to become naturalized
citizens or subjects thereof.
R.A. No. 9139 (administrative act of acquiring Philippine Citizenship)
Section 3. Qualifications. Subject to the provisions of the succeeding
section, any person desiring to avail of the benefits of this Act must
meet the following qualifications:
(a) The applicant must be born in the Philippines and residing therein
since birth;
(b) The applicant must not be less than eighteen (18) years of age, at
the time of filing of his/her petition;
(c) The applicant must be of good moral character and believes in the
underlying principles of the Constitution, and must have conducted
himself/herself in a proper and irreproachable manner during his/her
entire period of residence in the Philippines in his relation with the duly
constituted government as well as with the community in which he/she
is living;
(d) The applicant must have received hid/her primary and secondary
education in any public school or private educational institution duly
recognized by the Department of Education Culture and Sports, where
Philippine history, government and civics are taught and prescribed as
part of the school curriculum and whose enrollment is not limited to
any race or nationality; Provided, That should he/she have minor
children of school age, he/she must have enrolled them in similar
schools;
(e) The applicant must have a known trade, business, profession or
lawful occupation, from which he/she derives income sufficient for
his/her support and if he/she is married and/or has dependents, also
that of his/her family; Provided, however, That this shall not apply to
applicants who are college degree holders but are unable to practice
their profession because they are disqualified to do so by reason of
their citizenship;
(f) The applicant must be able to read, write and speak Filipino or any of
the dialects of the Philippines; and
(g) The applicant must have mingled with the Filipinos and evinced a
sincere desire to learn and embrace the customs, traditions and ideals
of the Filipino people.
Section 4. Who are disqualified. The following cannot be naturalized
as Philippine citizens:
(a) Those opposed to organized government or affiliated with any
association or group of persons who uphold and teach doctrines
opposing all organized governments;
(b) Those defending or teaching the necessity or propriety of violence,
personal assault, or assassination of the success and predominance of
their ideas;
(c) Polygamists or believers in the practice of polygamy;
(d) Those convicted of crimes involving moral turpitude;
(e) Those suffering from mental alienation or incurable contagious
diseases;
(f) Those who, during the period of their residence in the Philippines,
have not mingled socially with the Filipinos, or who have not evinced a
sincere desire to learn and embrace the customs, traditions, and ideals
of the Filipinos;
(g) Citizens or subjects with whom the Philippines is at war, during the
period of such war;
(h) Citizens or subjects whose laws do not grant Filipinos the right to
become naturalized citizens or subjects thereof.

Chapter 25
Suffrage
Suffrage may be exercised by all citizens of the Philippines not
otherwise disqualified by law, who are at least eighteen years
of age, and who shall have resided in the Philippines for at least
one year, and in the place wherein they propose to vote, for at
least six months immediately preceding the election. No
literacy, property, or other substantive requirement shall be
37
imposed on the exercise of suffrage.
Corollary to the idea of a republic, or a representative
government, must necessarily go the attendant presence of a
mechanism by which the people can express and manifest their
sovereign will. That is through the exercise of their power of
suffrage. The holding of periodic elections constitutes the
very foundation of a republican form of government such as
ours. Through elections, the people can install who they
choose to momentarily represent them in the governance of
their affairs. By means of elections, too, the people can cleanse
the government of people that they do not think deserving of
their trust, or by their lethargy and inattention retain or elect
people who may not be deserving of the honor and the office.
At the same time, suffrage also gives them a say in regard to
certain issues that may be presented to them for their approval
or rejection, as exemplified in the conduct of plebiscites,
referenda and initiative. Whatever direction the people take
through the exercise of their right, it is the choice they make
would not live up their expectations, then it would simply be a
38
mater of getting what they deserve
Also, if only to underscore the importance of the political rights
of the people, it has been held that among those that may not
be the subject matter or object of contracts are certain rights of
individuals, which the law and public policy have deemed wise
to exclude from the commerce of man, such as the political
rights conferred upon citizens, including, but not limited to,
ones right to vote, the right to present ones candidacy to the
people and to be voted to public office, provided, however,
that all qualifications prescribed by law obtain
Suffrage, A Historical Background
While the right of suffrage may be taken for granted now, it has
not always been available, much less to everyone. The
Constitution now guarantees the right to all citizens not
otherwise disqualified by law, eighteen years of age, residents
of the Philippines for at least one year, and of the place where
he or she intends to vote for at least six months immediately
preceding the election. He or she need not be literate, not must
he show any property, or comply with other substantive
requirements before he or she may cast his or her vote
The 1935 Constitution provided: Suffrage may be exercised by male
citizens of the Philippines not otherwise disqualified by law, who are
twenty-one years of age or over and are able to read and write, and
who shall have resided in the Philippines for one year and in the
municipality wherein they propose to vote for at least six months
preceding the election. The National Assembly shall extend the right of
CONSTITUTION, Art. V, 1
Absolute freedom of choice of the parties and men by whom we shall be
governed, even if only among varying evils, is of the very essence in the
concept of democracy consecrated in the fundamental law of our land.
Gonzales v. COMELEC, 27 SCRA 835 (1969)
37
38

158 | P

LATON

suffrage to women, if in a plebiscite which shall be held for that


purpose within two years after the adoption of this Constitution, not
less than three hundred thousand women possessing the necessary
qualifications shall vote affirmatively on the question.

The modern conception of the suffrage is that voting is a


function of government. The right to vote is not a natural right
but is a right created by law. Suffrage is a privilege granted by
the State to such persons or classes as are most likely to
exercise it for the public good. In the early stages of the
evolution of the representative system of government, the
exercise of the right of suffrage was limited to a small portion
of the inhabitants. But with the spread of democratic ideas, the
enjoyment of the franchise in the modern states has come to
embrace the mass of the audit classes of persons are excluded
from the franchise. Among the generally excluded classes are
minors, idiots, paupers, and convicts.
Regulation of Suffrage
Just like any other right, simply because it exists does not mean
that it can be exercised in any manner a person might want it
done. It is also subject to regulation for the benefit of everyone.
In a representative democracy such as ours, the right of
suffrage, although accorded a prime niche in the hierarchy of
rights embodied in the fundamental law, ought to be exercised
within the proper bounds and framework of the Constitution
and must properly yield to pertinent laws skillfully enacted by
the Legislature, which statutes for all intents and purposes, are
crafted to effectively insulate such so cherished right from
ravishment and preserve the democratic institutions our people
have, for so long, guarded against the spoils of opportunism,
debauchery and abuse. Thus, one may not simply go to the
polling precinct and cast his or her vote one election day, even
if he or she did not bother registering earlier
The act of registration is an indispensable precondition to the right of
suffrage. For registration is part and parcel of the right to vote and an
indispensable element in the election process...Proceeding from the
significance of registration as a necessary requisite to the right to vote,
the State undoubtedly, in the exercise of its inherent police power, may
then enact laws to safeguard and regulate the act of voters registration
for the ultimate purpose of conducting honest, orderly and peaceful
election, to the incidental yet generally important end, that even preelection activities could be performed by the duly constituted
authorities in a realistic and orderly manner--one which is not
indifferent and so far removed from the pressing order of the day and
the prevalent circumstances of the times.

Of course, it may not be lost sight of that registration is merely


a means, not the end. One must not be mistaken for the other.
Registration regulates the exercise of the right of suffrage. It is
not a qualification for such right
One of the limitations on the right to exercise suffrage is
conviction of a crime punishable with a penalty that includes
disqualification from the exercise of the right to vote
The right of the State to deprive persons to the right of suffrage by
reason of their having been convicted of crime, is beyond question.
"The manifest purpose of such restrictions upon this right is to preserve
the purity of elections. The presumption is that one rendered infamous
by conviction of felony, or other base offense indicative of moral
turpitude, is unfit to exercise the privilege of suffrage or to hold office.
The exclusion must for this reason be adjudged a mere disqualification,
imposed for protection and not for punishment, the withholding of a
privilege and not the denial of a personal right.

Elections and Representation


The exercise of suffrage would have to find expression through
periodic elections where men and women compete for the
electorates votes in order to gain the right to represent them
momentarily. In this regard, questions may arise every now and
then with respect to the determination of what really is the will
of the people as shown in connection with ballots which may go
wither way for or against certain candidates. On this score, the
Court said:
As long as popular government is an end to be achieved and
safeguarded, suffrage, whatever may be the modality and form
devised, must continue to be the manes by which the great reservoir of
power must be emptied into the receptacular agencies wrought by the
people through their Constitution in the interest of good government
and the common weal. Republicanism, in so far as it implies the
adoption of a representative type of government, necessarily points to
the enfranchised citizen as a particle of popular sovereignty and as the
ultimate source of the established authority. He has a voice in his
Government and whenever called upon to act in justifiable cases, to
give it efficacy and not to stifle it. This, fundamentally, is the reason for
the rule that ballots should be read and appreciated, if not with utmost,
with reasonable, liberality. Counsel for both parties have called our
attention to the different and divergent rules laid down by this Court on
the appreciation of ballots. It will serve no good and useful purpose for
us to engage in the task of reconciliation or harmonization of these
rules, although this may perhaps be undertaken, as no two cases will be
found to be exactly the same in factual or legal environment. It is
sufficient to observe, however, in this connection that whatever might
have been said in cases heretofore decided, no technical rule or rules
should be permitted to defeat the intention of the voter, if that
intention is discoverable from the ballot itself, not from
evidence aliunde.

To prevent the establishment of political dynasties is not the


only policy embodied in the constitutional provision in
question. The other policy is that of enhancing the freedom of
choice of the people. To consider, therefore, only stay in office
regardless of how the official concerned came to that office
whether by election or by succession by operation of law
would be to disregard one of the purposes of the constitutional
provision in question
Related to the provisions on term limits is the constitutional
directive with regard to political dynasties: The State shall
guarantee equal access to opportunities for public service, and
prohibit political dynasties as may be defined by law. This has
been apparently consciously ignored by Congress, however
Moya v. Del Fierro
69 Phil. 199 (1939)
Petitioner Moya and respondent Del Fierro were both candidates for
the office of mayor of Paracale, Camarines Norte. Petitioner was
proclaimed as the elected mayor by 91 votes which was contested by
the respondent. On appeal, the CA declared the respondent the winner
by three votes
WON the appellate court erred in its act of admitting and counting
certain ballots in favor of respondent
Held: No. As long as popular government is an end to be achieved and
safeguarded, suffrage, whatever may be the modality and form
devised, must continue to be the manes by which the great reservoir of
power must be emptied into the receptacular agencies wrought by the
people through their Constitution in the interest of good government
and the common weal. (supra)
Maquera v. Borra
159 | P

LATON

15 SCRA 7 (1965)
RA 4421s (which requires all candidates for national, provincial, city
and municipal offices...to post surety bond equivalent to one-year
salary...if the candidate fails to obtain at least 10% of the votes)
constitutionality is in question on the ground that the same is
undemocratic and contrary to the letter and spirit of the Constitution.
Moreover, it has the effect of imposing property qualifications in order
that a person could run...
The avowed purpose of RA 4421 in requiring a candidate to post a bond
equal to a years salary of the office for which he will run is to curb the
practice of so-called nuisance candidates

promise to reside in the Philippines in the future to be qualified


to voted immediately--a class of promissory note voters. Even
as they might be considered residents-to-be n the promised
future, the Court has upheld the validity of the law, considering
it within the power of the legislature, pursuant to the
Constitution itself, to determine who are qualified
Macalintal v. COMELEC
405 SCRA 614 (2003)

It is within the power of Congress, however, to prescribe the manner of


exercising political rights so long as it does not run counter to the
Constitution

Petitioner, as a taxpayer and a lawyer, challenges the constitutionality


of certain provisions of RA 9189 (The Overseas Absentee Voting Act of
2003), among others, Section 5(d) which allows the registration of
immigrant or permanent residents...who execute, upon registration, an
affidavit declaring that they shall resume actual physical permanent
residence in the Philippines not later than three (3) years from approval
of their registration, contrary to Section 1. Art. V of the Constitution
which requires that the voter must be a resident in the Philippines for
at least one year and in the place where he proposes to vote for at least
six months immediately preceding an election

The imposition of unwarranted restrictions and hindrances precluding


qualified candidates from running, is, therefore, violative of the
constitutional guaranty of freedom in the exercise of elective franchise

The seed of the present controversy is the interpretation that is given


to the phrase, qualified citizens of the Philippines abroad as it
appears in RA 9189

Nuisance candidates, as an evil to be remedied, do not justify the


adoption of measures that would bar poor candidates from running for
office

Held: No. RA 9189 was enacted in obeisance to the mandate of the first
paragraph of Section 2, Article V of the Constitution that Congress shall
provide a system for voting by qualified Filipinos abroad...In the
absence of restrictions, Congress is presumed to have duly exercised its
function as defined in Article VI (The Legislative Department) of the
Constitution

Held: Yes. A democratic form of government requires that political


rights be enjoyed by the citizens regardless of social or economic
distinctions

Elections, Ballot Sanctity and the Peoples Will


The Congress shall provide a system for securing the secrecy
and sanctity of the ballot as well as a system for absentee
voting by qualified Filipinos abroad.
The Congress shall also design a procedure for the disabled
and the illiterates to vote without the assistance of other
persons. Until then, they shall be allowed to vote under existing
laws and such rules as the Commission on Elections may
39
promulgate to protect the secrecy of the ballot.
The object of elections being the selection of persons who are
really chosen by the people to be their representatives, it is
imperative that there be an accurate and trustworthy
determination of the peoples will. If not, then the citizenry
may simply end up having in power persons or pretenders who
were not really their choice. One way of ensuring that the
people are able to really express their choice is to see to it that
the secrecy and sanctity of the ballot are kept intact

Ordinarily, an absentee is not a resident and vice versa; a person


cannot be at the same time, both a resident and an absentee. However,
under our election laws and the countless pronouncements of the
Court pertaining to elections, an absentee remains attached to his
residence in the Philippines as residence is considered synonymous with
domicile

Additional Cases
(T) Suffrage [1]

Education is what remains after one has forgotten everything


one learned in school. -Albert Einstein

The ballot, as an expression of the voters will, should also be


inviolate just like the persons right to privacy
Absentee Voting
An acknowledgment of the fact that quite a number of Filipinos
have gone abroad for work or otherwise, the Constitution has
also thought it advisable and desirable that they not be
deprived of the chance to express themselves in the political
field simply because they might be physically outside the
Philippines. Thus, the Charter has directed Congress has
directed to provide a system of absentee voting for qualified
overseas Filipinos. Congress, in so complying, however, appears
to have gone overbroad by including even those who would not
otherwise be considered residents. This Congress did by coming
up with Republic Act No. 9189 which allows even those who
39

CONSTITUTION, Art. V, 2

160 | P

LATON

You might also like